Orthopedics PDF

You might also like

Download as pdf or txt
Download as pdf or txt
You are on page 1of 193

 

 
 
 
 
 
 
 
 

ORTHOPEDICS
TRANSCRIPTIONS
BATCH 2016
_

ORTHOPEDIC HISTORY AND PHYSICAL EXAMINATION


Jose Ramon C. Pascual, MD, FPOA
May 27, 2014; 1:00 – 2:00 PM
Orthopedics

OUTLINE ANATOMIC CONCEPTS TO BE REVIEWED


 Anatomical terms, planes and positions  What joints are involved?
 GALS Screening History and PE  Cardinal motions possible in a particular joint
 Context Based History and Regional Examination  What are the static stabilizers of the joint?
of the Musculoskeletal System  What muscles are involved?
 Origins/insertions/function of muscles
ANATOMICAL TERMS, PLANES AND  What neurovascular structures are involved?
 Course of neurovascular structures
POSITIONS  Area supplied by neurovascular structures
 Anatomic Zero
 Position in which a cadaver presents itself
NOTE: Review all concepts listed above, they will be
 Supine position with the palmar side of the
asked in the exam
arms facing forward
 Axial Plane
 A plane that cuts the body into top and GALS SCREENING HISTORY AND
bottom portion
 Coronal Plane
PHYSICAL EXAMINATION
 Plane that slices the body to a front and back
portion GALS SCREENING
 Sagittal Plane  Screening examination used to detect locomotor
 Plane that sliced the body into a right and abnormalities and functional disabilities
left half
 Joint Motions GALS SCREENING HISTORY
 Shoulder flexion/extension  Have you any pain or stiffness?
 Occurs in the sagittal plane  Can you dress yourself without difficulty?
 Abduction  Can you walk up and down the stairs without
 Movement away from the midline difficulty?
along the coronal plane
 Shoulder rotation (external) GALS SCREENING
 Movement away from the midline on  Gait
the axial plane  Arms
 Asking the patient to place his hands  Legs
on his head  Spine
 Shoulder rotation (internal)
 Movement towards the midline on the GAIT
axial plane
 Asking the patient to place his hands
on the back

NOTE 1: you have to know the movements before you


are able to describe the deficiency or inability.

Please do review the other cardinal movements of


every joint in the body. Read on the chapter on Moore
for more information. NOTE: Gait can be characterized by the position of the
involved extremity whether they are in the swing
phase or stance phase. Each of these phases has sub-
NOTE 2: phases: the new gait terms and the classic gait
Pronation of the foot – combination of abduction, terms. Each phase has corresponding gait pathology
eversion and dorsiflexion along with its manifestation and possible causes.
Supination of the foot – combination of forefoot
adduction, hind foot inversion and plantar flexion

Transcriber/s: Anthea Marie Cornejo, Jan Cynric Cacao, Bernard Kevin Elipane
Formatting: Jan Cynric Cacao DLSHS I M e d i c i n e B a t c h 2 0 1 6 | 1 of 5
Editor/s: Jan Cynric Cacao
Phase Manifestation Possible Cause
Foot strike to foot flat Foot slap Moderately weak dorsiflexors
Weak, short, or spastic quadriceps;
compensated hamstring weakness;
Genu recurvatum
Achilles tendon contracture; plantar
flexor spasticity
Compensated forefoot valgus
Foot strike through mid-stance deformity; pes cavus; short limb;
Excessive foot supination
uncompensated external rotation of
tibia or femur
Weak hip extensor or flexor; hip pain;
Excessive trunk extension
decreased knee ROM
Excessive trunk flexion Weak gluteus maximus and quadriceps
Hamstring contracture; increased
Excessive knee flexion ankle dorsiflexion; weak plantar flexor;
long limb; hip flexion contracture
Tight medial hamstrings; anteverted
Excessive medial femur rotation femoral shaft. Weakness of opposite
muscle group
Tight hamstrings; retroverted femoral
Foot strike through toe off
Excessive lateral femur rotation shaft; weakness of opposite muscle
group
Abductor muscle contracture;
Increased base of support instability; genu valgum; leg length
discrepancy
Adductor muscle contracture; genu
Decreased base of support
varum
Ipsilateral gluteus medius weakness;
Excessive trunk lateral flexion
hip pain (Compensated
(Trendelenburg gait)
Foot flat through heel off Trendelenburg)
Pelvic drop Contracted gluteus medius weakness
Waddling gait Bilateral gluteus medius weakness

NOTE 1: In the foot flat to heel off phase, there is NOTE 2: Without the function of the gluteus medius,
excessive trunk level flexion known as the the trunk would tend to lift. Some patients would
Trendelendburg gait. This might be due to an compensate by using the trunk muscles to shift the
ipsilateral gluteus medius weakness, or hip pain. gravity towards the side of the involved muscle.
Sometimes there might be a compensating lurch. This When a normal individual walks, the gluteus medius
is one of the more common gait patterns seen in must also spire to keep the spine align.
orthopedic patients both young and old. In pediatric In some cases where there is a pathology of the hip, the
patients, we often find a dysplastic hip. The pain or the pain in the involved hip might be so much that the
abnormality of the hip joint may cause this kind of gait patient would try to avoid contracting the gluteus
in patients with arthritic hip. Also in patients who medius. Further contraction of the muscle would
have undergone hip surgery who acquired infections in produce more pain. And so, they relax the muscle and
the innervation of the gluteus muscles. then over-compensate with a lurch.
In cases of bilateral paralysis of the gluteus medius,
During the foot flat, the gluteus medius contracts to we sometimes end up with a waddling gait. The
keep the trunk in the neutral position. patient does not feel any pain but there is paralysis of
the hip joint. Instead of a lurch, you will see a pelvic
drop.

The gluteus maximus is responsible for hip extension.


If you lose the function of this muscle the lurch will
occur front to back.

Transcriber/s: Anthea Marie Cornejo, Jan Cynric Cacao, Bernard Kevin Elipane
Formatting: Jan Cynric Cacao DLSHS I M e d i c i n e B a t c h 2 0 1 6 | 2 of 5
Editor/s: Jan Cynric Cacao
Phase Manifestation Possible Cause
Compensated forefoot or rear foot
varus deformity; uncompensated
forefoot valgus deformity; pes planus;
Excessive foot pronation decreased ankle dorsiflexion;
increased tibial varum; long limb;
uncompensated internal rotation of
tibia or femur; weak tibialis posterior
Mid-stance through toe off
Bouncing or exaggerated plantar Achilles tendon contracture; gastroc-
flexion soleus spasticity
Gastroc-soleus weakness; Achilles
Insufficient push-off tendon rupture; metatarsalgia; hallus
rigidus
Hip flexor contracture; weak hip
Inadequate hip extension
extensor
Severely weak dorsiflexors; equinus
Steppage gait
deformity; plantar flexor spasticity
Long limb; abductor muscle
Swing phase Circumduction
shortening or overuse
Long limb; weak hamstring; quadratus
Hip hiking
lumborum shortening

ARMS  Evaluate patient on the side


 “Hands behind your head"  Check for hip and knee extension
 Sternoclavicular, acromioclavicular,  Joint swelling
glenohumeral, scapulothoracic articulation  Knee joint swelling, erythema, effusion
function  Patellar tap/ bulge test
 Elbow flexion  Full range of motion of hip, knee and ankle
 Flex hip
NOTE: When you ask the patient to flex his elbow and  Neutral rotation
put his fingers together behind his head, the IP and  External rotation of hip
MCP joints are extended. In effect, the function of the  Pain – early sign of osteoarthritis
entire upper extremity is evaluated.  Internal rotation of the hip
 Early signs of osteoporosis

 "Elbow straight, palms down”


 Forearm pronation NOTE: Squeeze test of the hands and feet are done to
 Elbow extension evaluate arthritic conditions.
 Finger extension
 Joint swelling  Inspect feet for lesions and callosities
 “Palms up”  Check arches and bony prominences
 Forearm supination
 Thenar/hypothenar atrophy
Note: We can also perform range of motion of the
 “Make a fist”
lower extremities in standing, siting or even in prone
 Finger flexion (IP, MCP joints)
position.
 “Make an OK sign”
 Opposition
 "Grab my fingers" SPINE
 Grip strength  For spine assessment the patient is ideally placed
 Squeeze test metacarpals in standing position
 Arthritic process  Check for cervical motions like flexion,
extension, rotation and lateral flexion.
LEGS  Assess thoracic and lumbar motions by
 "Please lie down" (anatomic position) rotation, flexion, extension, lateral flexion
 Joint swelling and rotation.
 Muscle bulk and symmetry  Note for any deformities in the spine like
 True and apparent leg length scoliosis by doing Forward bending test.
 Assess for the muscle bulk or any masses.
NOTE:  We can also measure the degree of
True length - measurement from the ASIS to the separation (Schober’s test) of the spinous
medial malleolus ('direct measurement') process wherein a decrease spine motion
Apparent length - measurement from the umbilicus may suggest Ankylosing spondylitis.
to the medial malleolus

Transcriber/s: Anthea Marie Cornejo, Jan Cynric Cacao, Bernard Kevin Elipane
Formatting: Jan Cynric Cacao DLSHS I M e d i c i n e B a t c h 2 0 1 6 | 3 of 5
Editor/s: Jan Cynric Cacao
CONTEXT BASED HISTORY AND
REGIONAL EXAMINATION OF THE
MUSCULOSKELETAL SYSTEM
 Advantages of GALS
 Easy and quick to perform
 Easily understood by undergraduates and
trainees
 Helps trainees to adjust to more complex
systems or techniques of evaluation
CASE 2: A 20 year old female, playing ultimate freebie
 Limitation of GALS falls on her right shoulder. This may result in
 Only a screening exam, does not always give
acromioclavicular separation
a definite diagnosis
 Optimized for rheumatic conditions,
because musculoskeletal conditions are not
limited by rheumatic diseases like trauma,
congenital anomalies, tumors and infections

COLD ORTHOPEDIC HISTORY


 Non-traumatic
 For orthopedic conditions that are 6 weeks and
older. The history taking resembles more of the
medicine history which includes:
 Primary Symptom and Accompanying
Symptoms
 Duration and Development
 Triggers
 Relieving Factors
 Birth and Maternal History especially in
pediatric patient
CASE 3: 40 year old male involved in a dashboard
ORTHOPEDIC TRAUMA (HOT ORTHOPEDICS) injury, comes in with hip slightly flexed and internally
HISTORY rotated and leg length is relatively short, this might be
 Trauma a posterior hip dislocation
 Date of Injury, Time of Injury, Place of Injury,
Mechanism of Injury
 MOI and POI are the most important to
elicit since we want to know how the
injury had happened. Because of
knowing the MOI we can determine
how the patient is badly injured, where
or up to what extent the injury is
 Trauma history also consists of any
potential allergies, medications, last
input or output and events leading to
injury as well as past medical history
since this may have an impact in our
treatment.
 AMPLE (Allergies, Medications, Past Medical
History, Last Input or Output, Events
Leading to the Injury)
 Important if you are planning to do CASE 4: A baseball player stabs his finger against the
surgery on the patient ball comes in with a mallet finger deformity, most
probably a tendon rupture

COLD ORTHOPEDICS PE
 Inspection
 Palpation
 Evaluation of joint stability
 Range of motion
 Vascular evaluation
CASE 1: A pediatric patient falls on an outstretched  Neurologic evaluation
extended elbow would most likely receive a  Auscultation
supracondylar fracture  Special maneuvers

Transcriber/s: Anthea Marie Cornejo, Jan Cynric Cacao, Bernard Kevin Elipane
Formatting: Jan Cynric Cacao DLSHS I M e d i c i n e B a t c h 2 0 1 6 | 4 of 5
Editor/s: Jan Cynric Cacao
NEUROLOGIC EXAMINATION

ORTHOPEDIC TRAUMA PE
 Location of injury
 Presence of fractures/dislocations
 Identification of complications
 Skin and soft tissue injuries
 Open factures
 Compartment syndrome
 Vascular evaluation
 Neurologic examination

SAMPLE CASE
 20 year old male
 CC: right knee swelling
 2 months ago patient twisted right knee while
playing basketball
 Since then, there has been swelling and
NOTE: occasional pain over the right knee
Finkelstein test: a maneuver that tries to replicate the
pain associated with De Quervain’s Tenosynovitis GALS
 Pain and difficulty in ambulation
 Swelling right knee
 (+) bulge test

DIAGNOSIS
 Anterior Cruciate ligament tear
 Meniscal tear
 Collateral ligament tear

“Live in the ward. Do not waste the hours of daylight in listening to that which you may read at night. But when you have seen, read. And
when you can, read the original descriptions of the masters who, with crude methods of study, saw so clearly.” – Sir William Osler

-END-

TRANSCRIPTION DETAILS
BASIS Latest PPT RECORDINGS + NOTES + DEVIATIONS 3-5% CREDITS -
Please study the other special maneuvers and the origins, insertions and innervation of the different muscle groups. LEARN and
REMARKS
MASTER the BASICS - Spidey
-BATCH 2016 Transcribers’ Guild Transcriptions. Version 1.0.0.0.0 Build 3101-

Transcriber/s: Anthea Marie Cornejo, Jan Cynric Cacao, Bernard Kevin Elipane
Formatting: Jan Cynric Cacao DLSHS I M e d i c i n e B a t c h 2 0 1 6 | 5 of 5
Editor/s: Jan Cynric Cacao
INJURIES OF THE LOWER EXTREMITIES
Jonathan C. Ronquillo, MD, FPOA
June 9, 2014; 1:00 – 2:00 PM
Orthopedics

OVERVIEW
• “Bony Orientation” • Diagnostics
• Treatment Principles • Others: Dislocations and Soft Tissue Injuries
• Fracture Overview • Management
• Clinical Evaluation & Initial Management • Complications

BONY ORIENTATION
 Orthopedics – in general, a diagnosis of
Anatomy: where it happens, what bone, how it
looks like?

Hip

REVIEW: Bony Anatomy of the Lower Extremities

Pointed arrow above shows the Femoral Head (blue)

*Femoral Head Fracture – fracture of the head of the


femur; rare, almost always caused by hip dislocations

Proximal Femur

*Subcapital Fracture (red) – fracture located below the


head of the femur (caput – head, sub – below)
X-ray of the Pelvis (AP view):

Pointed arrow above shows the Acetabulum (red), the *Transcervical Fracture (gray) – a femoral neck fracture
concave surface of the pelvis; it articulates with the head (cervix – neck, trans – entire)
of the femur (blue), forming the hip joint

Transcriber/s: AJ Panlilio, Leanne Ramos, Paolo Ramirez


Formatting: Leanne Ramos D L S H S I M e d i c i n e B a t c h 2 0 1 6 | 1 of 14
Editor/s: Jan David Monzon
Proximal Shaft of the Femur

*Basi Fracture (white) – fracture located at the base of


the femoral neck

Shaft of the Femur: Diaphyseal area (yellow)


*Diaphyseal Fractures are described as:
(1) Proximal Third (2) Middle Third & (3) Distal Third.

Why? You have to take into consideration the muscle attachments /


insertions in that particular part of the bone to be able to characterize
fractures and their functional consequences.

Knee

X-Ray of the Knee (AP and Lateral Views):


*Intertrochanteric Fracture (orange) – a fracture that
crosses the area between the greater trochanter up to
the lesser trochanter; there can also be an isolated
fracture of the greater trochanter, or of the lesser
trochanter

*Intercondylar Fracture – Any fracture running in


between the medial and lateral condyles

Landmarks:
MC = Medial Condyle T = Tibia (medial)
LC = Lateral Condyle F = Fibula (lateral)
P = Patella

*Subtrochanteric Fracture (blue) – relatively avascular


as compared to the intertrochanteric area, and to the
metaphyseal region where there is ‘good potential
healing’; located 5-7cm below the lesser trochanter

REVIEW: Parts of the Long Bone

*Supracondylar Fracture (blue) – a fracture above the


condylar area

*Tibial spines (red) = there are only 2 tibial spines

Transcriber/s: AJ Panlilio, Leanne Ramos, Paolo Ramirez


Formatting: Leanne Ramos D L S H S I M e d i c i n e B a t c h 2 0 1 6 | 2 of 14
Editor/s: Jan David Monzon
*Tibial Plafond – the distal articulation of the tibia;
Fracture in this area is called a Pilon Fracture

There are a total of 4 malleoli as shown on the previous


photo (Yellow) => (1) anterior, (2) posterior, (3) lateral, and
(4) medial malleoli

Note: If you are suspecting an ankle fracture, always do an


X-ray first to assess visually what structures are affected

Foot
*Tibial plateau (blue)
X-Ray of the Foot (AP, Oblique, and Lateral Views):

Take note of the bones of the foot:


 Tarsus: talus, calcaneus, (3) cuneiforms, cuboid,
and navicular
 Metatarsus: first, second, third, fourth and fifth
metatarsal bones
 Phalanges
*Tibial Tuberosity (red) Note: There are 2 sesamoids in the foot, which function
like the patella of the knee
X-Ray of the Tibia / Fibula:

*Tibial Fracture – may be classified as: In this X-ray, you will see a hallux valgus, a bunion, and a
(1) Proximal, (2) Middle & (3) Distal 3rd, just like the femur bunionette (yellow arrows)

Ankle Fractures & Dislocations: Treatment


X-Ray of the Ankle (Lateral and AP views): Principles
1. Reduce
C 2. Maintain reduction – until union is achieved
 O 3. Rehabilitate – to restore extremity function
N 4. Prevent / treat complications
S
T I

Transcriber/s: AJ Panlilio, Leanne Ramos, Paolo Ramirez


Formatting: Leanne Ramos D L S H S I M e d i c i n e B a t c h 2 0 1 6 | 3 of 14
Editor/s: Jan David Monzon
Definition of Terms Range of Motion
• Strains = injury to the muscle or tendons, causing
pulling or tearing due to overstretching (e.g. quadriceps
muscle strain where the quadriceps muscles or tendons
are stretched beyond their normal limits; can be seen in
vigorous kicking/running)
• Sprains = torn or over-stretched ligament (e.g. ankle
sprain usually seen in sports like basketball)
• Dislocations = complete separation of 2 joint surfaces
(that normally form a joint)
• Subluxations = a partial separation of joint surfaces
• Fracture = a disruption in the integrity of a living bone,
involving injury to the bone marrow, periosteum, and
adjacent soft tissues; IN SHORT, Fracture is a soft
tissue injury complicated by bony involvement

Note: When you hear the word fracture, try not to


concentrate on the bone alone because the bone is
surrounded by other soft tissue structures which contribute
to bony healing so the management of fractures involve
the soft tissues as well, and to a significant extent.

Frequency
• Trauma – leading cause of death 1-34 y.o.
 <65 y.o. CAD + cancer + stroke
• Incidence is multifactorial
 Age, sex, comorbidities, lifestyles and
occupation
• Bimodal incidence in males
Basically, just a review of our Intro to Anatomy during first
 Men are “more involved” in physical activities;
year! Recall and master by heart!
but this is already changing since more women
are now getting more active in this aspect
Single-Limb Injury
Etiology
Thorough Complete History and Associated Events
• Force that exceeds the strength of Bone
• Mechanism of Injury (MOI) (Ground fall? Fall from a
• Factors Influencing Fracture Occurrence
height?)
(REMEMBER THIS)
• Previous fractures
 Extrinsic: rate of mechanical load, duration,
• PMHx, SHx, P/SHx (especially smoking, because that
direction & magnitude of forces
 Intrinsic: bone’s energy-absorbing capacity, affects healing a lot, as well as comorbidities such as
MOE (modulus of elasticity), fatigue, strength & diabetes mellitus)
density • Occupation
• Direct or Indirect
Physical Examination
• High or Low Energy
• Thorough inspection & documentation
• Neurovascular status (very important!)
Clinical Evaluation • Range of Motion (ROMs) (check for the stability as
well)
Anatomical Terms of Location • Ligamentous or tendon injuries

Multiple Traumatic Injuries


• ATLS protocols (*this is another topic)
• Identification & treatment of life-threatening injuries
 ABC’s (it has been changed to CAB =
“circulation” is now more important in the
trauma life support protocol)
 Immediate intubation & rapid fluid
administration
 Spinal precautions
 Secondary survey once haemodynamically
stable

Life-Saving Measures
A = Airway and cervical spine immobilisation
B = Breathing
C = Circulation (treatment and diagnosis of cause)
D = Disability (head injury)
E = Exposure (musculo-skeletal injury)

Transcriber/s: AJ Panlilio, Leanne Ramos, Paolo Ramirez


Formatting: Leanne Ramos D L S H S I M e d i c i n e B a t c h 2 0 1 6 | 4 of 14
Editor/s: Jan David Monzon
Clinical Findings 5. Early stabilization
6. Early bone grafting
Constitutional Signs and Symptoms of Fracture 7. Early rehabilitation
• Pain and tenderness
• Swelling Diagnostic Work Up
• LROM, loss of power, inability to WB
• Bruise, contusion, ecchymosis (bleeding unto the Laboratory Studies
subcutaneous tissue) • Depends on age, extent of injuries & medical condition
• +/- Wound, bleeding • Trauma requires ATLS work-up
• Pre-operative tests, but NOT mandatory
Note: A bleeding not proportionate to what you really see  FBC
can either be a (1) vascular injury or (2) an open fracture; in  ELFTs
an open fracture, we see a “Halo Sign” wherein there is  Urinalysis
presence of fat that surrounds the blood  Coagulation profile
 Cross-matching & typing
• Haemarthrosis (blood within the joint)  Alcohol & toxicology screening
• Deformity
• Crepitus Imaging Studies
• Neurocirculatory disturbances • Rule of 2’s
 2 views = AP and lateral
Initial Management  2 joints = joints above and below
• Thorough history  2 limbs = for comparison purposes, especially
• Thorough physical examination for pediatric patients
• Stabilize v/s  2 times = before and after reduction
• Temporary stabilization
 Reduction Note: Identify where the X-ray should be done; when you
 Splint / traction have articular involvement, you would want to try to
localize that area as well and describe the angulation
Note: In stabilizing a limb, if you are not that experienced,
just place a splint above and below the joint; no need to • CT scan = is requested to identify or describe the
reduce it, EXCEPT if limb is already turning black => just characteristics of the bone itself
put gentle traction on that limb • MRI = look into the soft tissues
• Description, Rule of 6’s
 Anatomy  Angulation 
 Articular  Apex 
 Alignment  Apposition 

Imaging Study: Pelvis

Open / Compound Fractures


1. Treat life threatening injuries before limb
threatening injuries
2. Control obvious haemorrhage – direct pressure +
fluid replacement
3. Analgesia – incremental IV opioids
4. Correct obvious deformities with gentle traction and
splint. Remove obvious contaminants if possible.
5. Take photos
6. Irrigate wound with saline. Cover wound with sterile
moist dressing. Immobilise limb.
7. IV antibiotics (eg. cefuroxime +/- metronidazole or
gentamicin)
8. Tetanus prophylaxis
9. Check distal neurovascular status
10. Re-assess
 Get the AP and lateral views
7 Principles / “Rules” of Open Fracture  Frog leg lateral view = you get want to establish
(REMEMBER THIS: MUST KNOW) bilateral integrity of the hip joint
1. All open fractures are emergencies  Traumatized limbs = cannot externally rotate
2. Early antibiotic treatment (not as prophylaxis but as and abduct the hip (as in the case of vehicular
treatment) accident patients) and thus would require a
3. Early debridement different approach to visualize the hip laterally.
4. Early wound closure

Transcriber/s: AJ Panlilio, Leanne Ramos, Paolo Ramirez


Formatting: Leanne Ramos D L S H S I M e d i c i n e B a t c h 2 0 1 6 | 5 of 14
Editor/s: Jan David Monzon
 Cross table lateral view = an improvised way to
visualize the hip for trauma patients who can’t
move the injured limb
o The patient’s uninvolved limb is
elevated to allow a lateral view of the
injured limb
o The patient is supine  you get a
decubitus board to support the hip 
place the image receptor on the medial
side and the x-ray tube on the lateral
side  you get a “flipped” image of the
hip where your imaging tools are the
ones that “moved” to adopt to the Iliopectineal line (blue) = inner margin of the pelvic ring
immobilized / traumatized hip (border of the eminence); any fracture along this area is
considered a fractue of the anterior column of the
Anatomic Landmarks acetabulum
Again, just a review of Anatomy!

 Posterior Column = comprised of the ischium,


the posterior part of the ilium
 Anterior Column = iliac bone

Ilioischial line (orange) = demarcates the posterior


column. If you have a fracture there, consider a posterior
column injury as well and perform an x-ray to
distinguish where the other injuries are located

 Iliopectineal eminence = ridge above the


acetabulum
 Quadrilateral space = the wall of the true pelvic
cavity

Landmarks:
 Acetabular lines and the Ischio-medial wall
 Inferiorly: Ischio-posterior lip and the anterior
lip of the acetabulum
 Dome of acetabulum = the weight bearing
force of the acetabulum

Radiographic Markers of Hip Injuries

Shenton’s Line (orange) = if you follow this line, if you


see any beaks, consider a hip fracture

Transcriber/s: AJ Panlilio, Leanne Ramos, Paolo Ramirez


Formatting: Leanne Ramos D L S H S I M e d i c i n e B a t c h 2 0 1 6 | 6 of 14
Editor/s: Jan David Monzon
 A.k.a. Sunrise, sunset, or merchant view
 Important to do if you are considering a
longitudinal fracture or a dislocation of the
patella, which cannot be normally viewed in the
AP / Lateral View

X-Ray of the Knee: Tunnel View

Teardrop (green) = part of the quadrilateral space

Considerations: Frature of the pelvic wall; injuries to


the medial acetabular wall. May indicate bleeding,
sinuvitis within the hip joint (you will have to measure if
one side is wider than the other).
The tunnel view may be indicated if you would want to
Imaging Studies: Femur and Tibia assess the articular surface of the tibial plateau

X-Ray of the Femur and Tibia (AP + Lateral Views):

Imaging Studies: Knee


Imaging Studies: Ankle and Foot
 Indicated if there is tenderness at the lateral
X-Ray of the Knee (AP + Lateral Views):
and medial malleolus, the distal fibula, non-
weight bearing or if the patient cannot walk 3
steps or more

Ottawa Rules:
- A set of guidelines for clinicians to help
decide if a patient with foot or ankle pain
should be offered X-rays to diagnose a
possible bone fracture
- The Ottawa Rules indicate whether a foot
X-ray series is required on the basis of any
This X-ray shows an arthritic knee. In such cases, it is pain in the midfoot zone AND any of the
preferably done while the patient is weight-bearing (if following:
she/he can assume it) 1. Bone tenderness at the base of the
5th metatarsal
X-Ray of the Patella / Knee Cap: Axial View 2. Bone tenderness at the navicular
bone
3. Inability to bear weight both
immediately and in the
emergency department for four
steps

Transcriber/s: AJ Panlilio, Leanne Ramos, Paolo Ramirez


Formatting: Leanne Ramos D L S H S I M e d i c i n e B a t c h 2 0 1 6 | 7 of 14
Editor/s: Jan David Monzon
 Request for foot x-rays if there is mid-foot tenderness or tenderness at the 5th metatarsal and if it is non-weight bearing
as well (as stated in the Ottawa rules)
 It may be in APL + mortise + oblique views
 ANKLE X-rays:
 APL + Mortise
 Oblique views

MORTISE ANKLE VIEW

 FOOT XRAYS: APL + Oblique

Transcriber/s: AJ Panlilio, Leanne Ramos, Paolo Ramirez


Formatting: Leanne Ramos D L S H S I M e d i c i n e B a t c h 2 0 1 6 | 8 of 14
Editor/s: Jan David Monzon
Diagnosis: Calcaneal Fracture

RADIOGRAPHIC INTERPRETATION
 In interpreting fractures, one must evaluate Thus…
midfoot tenderness or tenderness at the 5th Fracture, open, displaced, comminuted, proximal third,
metatarsal and if it’s non-weight bearing as well. tibia-fibula, right
 It may be in APL+mortise+oblique view.
 Fracture
 Closed or Open
 Complete or incomplete
 Displacement
 Geometry / Fragmentation
 Level or Articular involvement
 Name of bone
 Laterality (left or right)

BIG NOTES: If the radiograph presents with


incomplete fracture then obviously there won’t
be any displacement (but there could be
angulation)

Dislocations and Soft Tissue Injuries of the Lower Extremity

HIP DISLOCATIONS
 Posterior  Anterior
 Flexion + adduction  Hyperabduction + extension
 Shortened, internally rotated & adducted  Shortened & externally rotated

Transcriber/s: AJ Panlilio, Leanne Ramos, Paolo Ramirez


Formatting: Leanne Ramos D L S H S I M e d i c i n e B a t c h 2 0 1 6 | 9 of 14
Editor/s: Jan David Monzon
Iliac View (Left)

Diagnosis: Right posterior hip dislocation

BIG NOTES: Femoral head usually displaces Obturator View (Right)


UPWARD on x-ray in posterior hip
dislocation.
MNEMONICS: “Posterior, Pataas”
 Posterior dislocation MORE
COMMON than Anterior
dislocation

Diagnosis: Left anterior hip dislocation

OSTEONECROSIS
 Subchondral sclerosis
 Narrowed joint space
 Collapse

BIG NOTES: Femoral head usually displaces


DOWNWARD on x-ray in anterior hip dislocation.

BIGGER NOTES: Femoral head displacement in


hip dislocation is synonymous with humeral head
displacement in shoulder dislocation meaning,
posterior dislocation will have both upward
movement of femoral and humeral head while
anterior dislocation will have downward Diagnosis (right picture): Left femoral head
movement of both femoral and humeral head. necrosis; this usually happens in 15% of dislocations
(Nice2Know for this lecture but Need2Know on
the next. )  Associated injuries:
 Neurovascular
 Femoral head / neck fractures
 Acetabular fractures
Diagnosis: Right femoral head fracture
 Pelvic ring fractures
*Appreciate the Shenton’s line on the left
 Spine injuries
 Femoral / leg shaft fractures
Anterior View  Knee injuries: bones and soft tissues

KNEE DISLOCATIONS
 True orthopaedic emergency! (because of the
involvement of the popliteal artery)
 0.2% of all orthopaedic cases
 MVA > 50%
 Vascular injury: 4.8% vs. 65%
 Classifications:
 Direction-based, temporal, open or closed &
extent of injury

Transcriber/s: AJ Panlilio, Leanne Ramos, Paolo Ramirez


Formatting: Leanne Ramos D L S H S I M e d i c i n e B a t c h 2 0 1 6 | 10 of 14
Editor/s: Jan David Monzon
 Clinical evaluation:
 Pedal pulses (100% specificity & sensitivity),
changes in color & temperature
 Expanding haematoma
 History of abnormal PE prior to presentation
in ED

The Axial View is helpful for vertical


fractures and patellar dislocations

ACHILLES TENDON RUPTURE


 History: “pop” sound on jump
 PE:
 Palpable gap
 Decreased plantarflexion strength
 Inc. passive ankle dorsiflexion
OPEN KNEE INJURY  Thompson test
 53% to 91% of all open joint injuries  Diagnostics:
 3 types:  X-rays, MRI, USS (inconclusive)
 Associated with fractures  Management: non/operative
 No fractures
 Surgical precautions:
 GSW (Gunshot Wounds)
 DM, neuropathy, immunocompromised
 Lead poisoning: 2d – 52yrs states, >65 yo, tobacco use, sedentary
• Mechanical damage, FB reactions  lifestyle, obesity (>30), PVD or systemic
proliferative synovitis  destructive
dermatologic disorder
arthritis
• Microcytic anemia + GSW
*Chelation + debridement & irrigation

Thompson Test: The patient lies face down with feet


hanging off the edge of the bed. If the test is positive,
there is no movement of the foot (normally
plantarflexion) on squeezing the corresponding calf,
signifying likely rupture of the achilles tendon

PATELLAR DISLOCATION
SUBTALAR DISLOCATIONS
 Lateral
 Medial
 Easily reduced
 Extensor retinaculum
 Immobilization
 Extensor digitorum brevis
 3-6/52
 Lateral approach
 arthrofibrosis
 Radiographs pre/post
 MRI

Transcriber/s: AJ Panlilio, Leanne Ramos, Paolo Ramirez


Formatting: Leanne Ramos D L S H S I M e d i c i n e B a t c h 2 0 1 6 | 11 of 14
Editor/s: Jan David Monzon
CONSERVATIVE MANAGEMENT
 Closed reduction
 Cast

TRACTION
 Skin – temporary measure as the skin can only
MUST KNOW: hold 10% of the body weight
Subtalar joint comprises of  Skeletal – definitive because it can hold as much
-Talocalcaneal joint as 100 pounds
-Talonavicular joint
-Calcaneocuboid joint

 Lateral
 Posterior tibial
tendon (common
impingement)
 Osteochondral
fracture
 Medial approach
 Poorer prognosis

TREATMENT GOALS FOR SURGICAL MANAGEMENT,


ASIF 1958
 Anatomic reduction
 Diaphyseal
 Intra-articular
 Stable fixation ~ biomechanical demands
 Preservation of blood supply
 Active, pain-free mobilization to prevent “fracture
disease”

 Complications:
 Arthritis and stiffness
 Worse with open injuries
 tibial nerve
 Posterior tibial tendon ruptures
 articular fracture
 ON INDICATIONS FOR SURGICAL TREATMENT
 Open fractures
MANAGEMENT  Failed closed methods
General Types of Treatment  Multiple traumatic injuries
 Conservative  Fractures known to heal poorly (Ex. Femoral neck
 Traction fractures)
 Casting / Splinting  Pathologic fractures
 Operative  Large avulsions / tension injuries (Ex. Patellar
fractures)
 Displaced articular fractures (articular
incongruency is more than 1-2mm)

Transcriber/s: AJ Panlilio, Leanne Ramos, Paolo Ramirez


Formatting: Leanne Ramos D L S H S I M e d i c i n e B a t c h 2 0 1 6 | 12 of 14
Editor/s: Jan David Monzon
 Poor candidates for non-operative management
 Children’s fractures which have a propensity to Nails
lead to deformities

CONTRAINDICATIONS TO SURGICAL
STABILIZATION
 Active infection
 Soft tissue compromise
 Medical (who are unable to tolerate the surgical
procedure
 Amputation

Salter – Harris Classification Pins

Thurston-Holland Sign

Tension bond wire is a combination of pins and wires.


Tension on the knee compresses the fracture.

 Normal–No fracture
 Type +1 Proximal and distal separation
 Type +2 You have a metaphyseal fragment
called The Thurston-Holland Sign
 Type +3 Goes through epiphysis and diaphysis
 Type +4 All the way from the epiphysis to the
metaphysis
 Type +5 Crushing fracture that destroys the
growth plate in affected areas

TYPES OF SURGICAL MANAGEMENT


 Open / Closed Reduction + External or Internal
Fixation

HOW DO WE FIX FRACTURES?


 Pins
 Plates
 Nails
 Screws

Transcriber/s: AJ Panlilio, Leanne Ramos, Paolo Ramirez


Formatting: Leanne Ramos D L S H S I M e d i c i n e B a t c h 2 0 1 6 | 13 of 14
Editor/s: Jan David Monzon
REHABILITATION
 Restore pre-injury functional level:
 Pragmatic targets
 Multidisciplinary
 Physiotherapy, OT, district nurse, GP
 Social worker – housing, disability
grants
 Rest, Elevation, Mobilisation (active/passive)
 Drugs – analgesia
 Work assessment and re-employment

COMPLICATIONS OF FRACTURES & DISLOCATIONS


 Shock
 ARDS
 Fat embolism
 Head, chest, abdomen and pelvic injuries
 Crush syndrome
 Tetanus
 Gas gangrene
 DVT/PE
 Bed sores
CONSERVATIVE VS. OPERATIVE  Depression/PTSD
 Infections – UTI, Chest
Operative Non-operative
Rehabilitation Rapid Slow Early Late
Risk of joint Low Present  Visceral injury  Delayed union
stiffness
 Vascular injury  Non-union
Risk of mal-union Low Present
 Compartment  Mal-union
Risk of non-union Present Present syndrome  Tendon rupture
Speed of healing Slow Rapid (Volkmann  Myositis ossificans
Risk of infection Present Low contracture)  Osteonecrosis
Cost ? ?  Nerve injury  Algodystrophy
 Haemarthrosis  Osteoarthritis and
 Infection joint stiffness

Algodystrophy - A local disturbance of growth,


particularly in bone and cartilage, that causes pain

SUMMARY
 Know your damn bones
 Orientation
 Associated structures
 Thorough history and PE
 Initial assessment and management
 Identify potential problems
 Request proper imaging
 Proper treatment

-END-

TRANSCRIPTION DETAILS
BASIS Latest PPT RECORDINGS + NOTES + DEVIATIONS 5-10% CREDITS HB Notes, Google (Images), Aibhen N.
A very photo-heavy transcription, some of which were lifted from the previous HB Notes Transcriptions. You may want to print this in
REMARKS COLORED to see the labeling properly (or just review it on your tablets / laptops to save ink! ). I highly suggest reviewing your
anatomy too, to get a better grasp & perhaps, appreciate this topic more. :)
-BATCH 2016 Transcribers’ Guild Transcriptions. Version 1.0.0.0.0 Build 3101-

Transcriber/s: AJ Panlilio, Leanne Ramos, Paolo Ramirez


Formatting: Leanne Ramos D L S H S I M e d i c i n e B a t c h 2 0 1 6 | 14 of 14
Editor/s: Jan David Monzon
OPEN FRACTURES
Joaquin C. Pandanan, MD, FPOA
June 17, 2014; 1:00 – 2:00 PM
Orthopedics

OUTLINE GOAL OF POLYTRAUMA MANAGEMENT


 Principles in Open Fracture  Bone is not a primary importance, LIFE is
 Polytrauma  Therefore assessment in of the polytrauma
 Open Fractures patient is according to ATLS (advanced trauma
 Indications for Amputation vs. Limb Salvage life support) protocol
 Current Antibiotic Therapy for Open Fractures
ADVANTAGES OF SPLINTING
 Wound debridement
 Prevents further soft tissue injury
 Skeletal stabilization
 Relieves pain
 Soft tissue coverage
 Facilitates transport and radiographic studies
 Reduces incidence of fat embolism
PRINCIPLES IN OPEN FRACTURE
 Emergency – everything ASAP
NOTE: A splint stabilizes the joint above and below
 Life threatening injuries
the fracture. For example, in a tibial fracture, a splint
 Wound debridement
will stabilize the ankle joint below and the knee joint
 Wound cleaning
above
 Removal of devitalized tissues
 Removal of foreign objects
 Antibiotics
 Stabilization
 Soft tissue coverage
 Complications
 Early rehabilitation

MULTIPLY INJURED PATIENT


 30% of open fractures are polytrauma
 75% died from head injury

POLYTRAUMA
 Syndrome of multiple injuries exceeding a
defined severity (ISS>17)
 Sequential systemic traumatic reactions
 Dysfunction or failure of remote organs and vital NOTE:
systems, not directly injured Periosteum – outer covering of the bone; provides
blood supply for the outer 1/3 of the bone
INITIAL EMERGENCY MANAGEMENT The remaining 2/3 is supplied by the medullary canal
 Identify life- threatening conditions
 ABC’s
OPEN FRACTURES
 Initial compressive dressings
 Application of splints
DEFINITION OF OPEN FRACTURE
 History
 Soft tissue injury + cortical break
 X-rays/ CT scan – cervical spine, chest x-ray,
 Break in the skin and underlying soft tissue and
pelvic AP
communicates with the outside environment
PRINCIPLES OF RESUSCITATION (ATLS)
CLUES FOR OPEN FRACTURES
 Phases of management
 Exposed bones
 Primary Survey
 Presence of fat droplets
 Resuscitation
 Radiolucent on x-ray
 Secondary Survey
 Definitive care
 Priorities in treatment
 Airway
 Breathing
 Circulation/CNS
 Digestive system
 Excretory Tracts
 Fractures

Transcriber/Formatting/Editor: Jan Cynric Cacao


D L S H S I M e d i c i n e B a t c h 2 0 1 6 | 1 of 5
NOTE:
In-out mechanism – bone is fractured in came out
after an injury
Out-in mechanism – a force outside causes the bone
to come out

Exposed ulnar bone

NOTE: ulna is on the MEDIAL side of the forearm


while the radius is on the LATERAL side. Always base
the position of the bones in the ANATOMICAL
ZERO/POSITION of the body OPEN FRACTURE TYPE II
 Wound: >1 cm. but <10 cm
 Level of contamination: Moderate
CLASSIFICATION
 Soft tissue injury: Moderate, some muscle
 Old classification damage
 Simple
 Bone injury: Moderate comminution
 Compound
 Mechanism of injury: out-in
 New Classification (Gustilo Classification)
 Open Fracture Type I
 Open Fracture Type II
 Open Fracture Type III
 Type III-A
 Type III-B
 Type III-C

NOTE:
Why do we classify?
1. To know what kind of fracture you are dealing
with
2. For prognostication
3. For research purposes

In the Gustilo Classification, we have to take note of


these parameters:
1. Size
2. Soft tissue injury
3. Level of contamination
4. Bone injury Open Fracture Type III
 Open Fracture Type III-A
OPEN FRACTURE TYPE I  Wound: >10 cm
 Wound: Small laceration <1 cm  Level of contamination: High
 Level of contamination: Clean  Soft tissue injury: Severe with crushing
 Soft tissue injury: Minimal  Bone injury: Usually comminuted, soft
 Bone injury: Simple, minimal comminution tissue coverage possible
 Mechanism of injury: out-in
 Mechanism of injury: in- out
 Vascular status: intact
 No crushing
 Simple fracture

Transcriber/Formatting/Editor: Jan Cynric Cacao


D L S H S I M e d i c i n e B a t c h 2 0 1 6 | 2 of 5
 Open Fracture Type III-B
 Wound: >10 cm SPECIAL OPEN TYPE III FRACTURES
 Level of contamination: High  Segmental fractures
 Soft tissue injury: Severe loss of coverage,  Farmyard injuries
moderate periosteal stripping
 Fractures occurring in a highly contaminated
 Bone injury: Usually comminuted, poor
environment
bone coverage, requires soft tissue
 Shotgun wounds
reconstruction
 High-velocity gunshot wounds
 Mechanism of injury: out-in
 >8 hours untreated
 Cannot be closed primarily. Requires
external fixator and a flap to cover it
IIIA (29) IIIB (31) IIIC (8)
 Open Fracture Type III-C
 Wound: >10 cm Infection 17% (5) 26% (8) 13%
 Level of contamination: High Amputation 9% (3) 19% (6) 62% (5)
 Soft tissue injury: Very severe loss of
coverage, severe periosteal stripping. INDICATIONS FOR AMPUTATION VS
VASCULAR INJURY REQUIRING REPAIR LIMB SALVAGE
 Bone injury: Usually comminuted, poor  Non-viable limb
bone coverage, requires soft tissue  Non viability even after revascularization
reconstruction
 In a presence of severely debilitating or chronic
 Mechanism of injury: out-in
diseases
 Demand several operative procedures
 Mass casualty situation
 Severe multi system injuries

MANGLED EXTREMITY SEVERITY SCORE


 Skeletal / soft tissue injury
 Low energy (1)
 Medium energy (2)
 High energy (3)
 Very high energy (4)
 Limb ischemia
 Pulsed reduced or absent (1)
 Paresthesia decrease capillary refill (2)
Open fracture type IIIC  Cool, paralyzed, insensate limb (3)
 Shock
 Systolic pressure >90 mm/hg (0)
NOTE: to check for vascular injury, you have to check
 Hypotensive transiently (1)
for the pulses and capillary refill
 Persistent hypotension (2)
 Age
 <30 (0)
 30-50 (1)
 >50 (2)

NOTE: if the score is 7 or more – predictive of


amputation

Transcriber/Formatting/Editor: Jan Cynric Cacao


D L S H S I M e d i c i n e B a t c h 2 0 1 6 | 3 of 5
AMPUTATE OR SAVE? SKELETAL STABILIZATION
 Better to do early amputation  Close reduction and casting
 Not costly  External fixation
 May have higher level of amputation  Internal fixation (plates and screws and
 Psychologic impact intramedullary nailing)
 Better acceptance
 Early recovery to work

MANAGEMENT
 Initial management
 Classification
 Antibiotic therapy
 Immunization therapy
 Soft tissue coverage
 Type of fixation

CURRENT ANTIBIOTIC THERAPY


IN OPEN FRACTURES
 Type 1 – Cefazolin 1.0 gm / 8HRS x 2-3 days.
 Type 2 & 3 – Combination Cefazolin &
aminoglycoside
 For type 3 open fracture and with severe
contamination you have to add Penicillin 10-20
million or Metronidazole (for penicillin sensitive
Casting and splint for stable type 1 open fractures
patients) units daily must be included for farm
injuries

NOTE: repeat antibiotic coverage after major surgical


procedure

Most common pathogen for type 1 are the gram


positive organisms while types 2 and 3 is a
combination of both gram positive and gram
negative organisms

ANY first generation cephalosporin will do as antibiotic


therapy in open fractures

WOUND DEBRIDEMENT
 Removal of foreign material Intramedullary nailing for type 1 and 2 open fractures
 Hay, dirty clothing bullets and pellets
 Removal of nonviable tissue
 Muscles, devitalized bone
 Reduction of bacterial contamination
 1-2 liters of NSS for type I
 5-10 liters of NSS for type II and III

ORDER OF DEBRIDEMENT
 Skin and subcutaneous fat
 Fascia
 Muscle
 Tendon
 Bone
External fixation for some open type 2 and all type 3
MUSCLE VIABILITY (4C’S)
 Color- poor guide to viability
 Consistency
 Capacity to bleed
 Contractility- establishes viability (most
important)

Transcriber/Formatting/Editor: Jan Cynric Cacao


D L S H S I M e d i c i n e B a t c h 2 0 1 6 | 4 of 5
SOFT TISSUE COVERAGE COMPARTMENT SYNDROME (5P’S)
 Primary closure  Pain
 Secondary (Flaps, skin grafting)  Paralysis
 Pulselessness
 Pallor
NOTE: do not directly suture wounds of open fractures,
 Paresthesia
do debridement first at the OR

NOTE: mostly seen in type 1 and rarely in types 2 and


CRITERIA FOR PRIMARY CLOSURE
3. There are 4 compartments in the leg: anterior,
 Clean wound lateral, posterior (superficial and deep)
 Necrotic tissue have been removed
 Intact circulation
 Intact nerve supply FASCIOTOMY
 Satisfactory condition of patient  Patient absorbed high energy injury causes
 No skin tension muscle to swell
 Won’t create dead space  Removal of fascia and all compartments prevents
muscles from dying because fascia does not
DELAYED SECONDARY CLOSURE expand
 Done before 5th day
 Minimize risk of (anaerobic) infection

EARLY COMPLICATIONS
 Infection
 gangrene
 Compartment syndrome
 Fat embolism syndrome

FAT EMBOLISM SYNDROME


 High energy long bone fracture in 2nd or 3rd
decade of life
 Usually occurs on 2nd or 3rd day
 Symptoms: shortness of breath
 Sudden onset of restlessness, disorientation,
LATE COMPLICATIONS
marked confusion or coma
 Nonunion- distal 3rd
 Hallmark: arterial hypoxemia
 Delayed union
 Petechiae: chest, axilla, root of neck and
conjunctivae  Chronic osteomyelitis
 Chest x-ray: snow-storm pattern
 ECG: non-specific SUMMARY
 Treatment: prevent hypoxemia, immediate  Treat all open fractures as emergency
stabilization of fracture  Treat first all life threatening conditions
 Immediate wound debridement
 Appropriate antibiotics
 Stabilization
 Early soft tissue coverage
 Identify and treat complications
 Early rehabilitation

-END-

TRANSCRIPTION DETAILS
BASIS Latest PPT RECORDINGS + NOTES + DEVIATIONS 3-5% CREDITS -
WARNING: Gore and bloody pictures included. This topic is not for the faint of heart; you may skip the pictures but be sure to know all the
REMARKS
important details of the different types of open fractures. Use your imagination and you’ll master this topic very fast - Spidey
-BATCH 2016 Transcribers’ Guild Transcriptions. Version 1.0.0.0.0 Build 3103-

Transcriber/Formatting/Editor: Jan Cynric Cacao


D L S H S I M e d i c i n e B a t c h 2 0 1 6 | 5 of 5
FRACTURES & DISLOCATION
Ricardo C. Liwag, MD
June 30, 2014; 1:00 – 2:30 PM
Orthopedics

LECTURE OUTLINE:
 Definition of Terms  Local (degree of trauma, etc.)
 Classification of Fractures  Systemic (age, hormones, etc.)
 Open vs. Closed  Common Complications in Patients with
 Configuration (Transverse, oblique, etc.) Orthopedic Injuries
 Location  Orthopedic‐Related Complications
 Displacement  Diagnosis
 Signs and Symptoms  Management
 Fracture Healing  Tractions, Splints, Casts
 3 Stages  Tractions
 Histologic Picture  Splints
 Factors Affecting Healing  Casts

DEFINITION OF TERMS
Fractures Dislocation Subluxation Sprain Strain
 Break in the  Complete  Incomplete  Injury of a ligament,  Injury to the
continuity of a bone disruption of a disruption of the where there is muscles and tendon,
joint (joint = union joint or a partial stretching or tearing where there is tearing
between two dislocation of a joiint of the ligament of the muscle fibers
articulating bones) Ex. Glenohumeral joint Ex. Ankle sprain Ex. 1 Lumbar strain = an
subluxation (a commonly torn ligament injury involving the back
for the ankle is the anterior muscles
talofibular Ex. 2 Achilles tendon
ligament) strain = an injury to the
tendon

Note: A ligament is a fibrous connective tissue that connects a bone to another bone; while a tendon is a fibrous connective tissue that
binds a muscle to a bone.
MNEMONIC: STrain - Tendon

CLASSIFICATION OF FRACTURES
OPEN VS. CLOSED
 Open Fracture  B - severe loss of coverage;
 Soft tissue injury + cortical break moderate periosteal
 Any break in the bone with continuity to stripping
the outside environment  C - very severe loss of
 Infection is a common complication coverage; vascular injury
 Needs more aggressive management requiring repair
 Clues
 Exposed bones  Closed Fracture
 Presence of fat droplets  Has intact overlying skin
 Example:  Does NOT communicate with the external
 Pelvic fracture with rectal environment
involvement  Antibiotics are NOT given except if there are
 Types: lacerations present
 Type I - small laceration < 1 cm.
 Type II ‐ wound > 1 cm but < 10 cm
 Type III ‐ > 10 cm
 Subtypes:
 A - possible soft tissue
coverage

BIG NOTES:
Observe lacerations or open wounds for bone communication
to the environment to establish diagnosis of open or closed.
Treatment is different for both cases.

Transcriber/s: Paolo Ramirez


Formatting: Paolo Ramirez D L S H S I M e d i c i n e B a t c h 2 0 1 6 | 1 of 7
Editor/s: Leanne Ramos & Jan Cynric Cacao
CONFIGURATION
 Refers to the pattern of the fracture line as seen in the x-ray
 Used for determining the mechanism of injury

Transverse Oblique Spiral


 The fracture line is directly  The fracture line is inclined or  Somewhat similar to oblique but it is
perpendicular to the long axis of bone angulated to the horizontal line more three dimensional
 Usually occurs in a direct force or  It can be short oblique or long oblique  Has element of rotation (“twisting
slightly angular  Produced by direct torsional or injury” resulting to pointed ends on
 More stable than oblique rotational force with upward thrust x-ray)
 Most surface area for healing, ergo
better prognosis

Comminuted/Butterfly fragment Compression/Crushed Greenstick Torus


 When there are MORE THAN  Mostly seen in  One side is broken while  One side is broken while there
2 FRAGMENTS cancellous bone lengthening on the other is a crushing fracture on the
 *Butterfly fragment – 3 large (spine, distal radius, side (elongation/curving due other side
fragments usually proximal, metaphysis, pelvis) to malleability of pediatric  Also seen mostly in children
distal and another large  May be caused by bones) but can be seen in adults too
fracture in the area osteoporosis seen in  Seen mostly in pediatric
older adults patients
(compression fracture
of the vertebral body)

BASED ON LOCATION  Others


 3 halves of a long bone (proximal, middle and distal  Minimal, Partial, Undisplaced
third)
 Anatomic orientation (anterior, posterior, proximal,
distal)
 Anatomic landmarks (head, neck, body/shaft, base
etc.)
 Segment of long bones (epiphysis, metaphysis,
diaphysis)
DISPLACEMENT
 Complete
 Fragments are completely separated
 Incomplete
 Not fractured all the way through

Transcriber/s: Paolo Ramirez


Formatting: Paolo Ramirez D L S H S I M e d i c i n e B a t c h 2 0 1 6 | 2 of 7
Editor/s: Leanne Ramos & Jan Cynric Cacao
SIGNS & SYMPTOMS
 Note the cardinal signs of inflammation:
 Swelling
 Erythema
 Pain
 Heat
 Loss of function
 Also take note of the presence of deformity

PHASES OF FRACTURE HEALING


Inflammatory Reparative Remodeling
10% 40% 70%
 Bleeding, inflammatory cell  Begins when hematoma is  Fracture healing is completed
migration to injured site (including organizing (2 weeks) during this stage
the macrophages/ neutrophils/  Call on progenitor cells, callus  Greatly depends on stressors like
platelets and cytokines) formation, revascularization, weight bearing or implants;
 Infiltration of fibroblasts prominent osteoclastic activity with therefore an injured bone in healing
 Presence of exudation, swelling, eventual bridging of bone  new process or if already healed may be
erythema with tenderness at the bone formation subjected to different stimuli and
affected area  *Note: it is very important to will react accordingly
assume proper immobilization,  *Wolff’s law: formation and
otherwise, ossification of the callus resorption of bone is dependent on
may not occur, and an unstable the mechanical stress applied to it
fibrous union may develop instead  This requires strict and good
immobilization for the injury not to
recur
 Persists for days to weeks  Several weeks  Occurs for a long time (may even
take 2 years or beyond for the bone
to remodel)

NOTES:
(1) Taking note of the percentages, there is an overlap among the phases of healing which is why the percentage exceeds 100%. Meaning, as
the inflammatory phase starts to wean off, the reparative phase will already begin and while the remodelling phase may begin even at
reparative phase.
(2) The most critical period of bone healing is the first 1 to 2 weeks in which inflammation and revascularization occur

HISTOLOGIC PICTURE  From there, the osteoblasts and


 Early new bone formation chondroblasts come in
 Cartilage  Hence, there is new bone
 Revascularizing cortical bone formation and neovascularization,
 Organized hematoma since blood is needed for bone to grow
 Fiber bone  Union: should be regarded as an incomplete repair
 Persistent cartilage  Non‐union: there is still movement
 There is increased temperature in a present in the fracture (6 mos)
fractured bone  Delayed union: Prolongation of
 Once the hematoma is organized, this is the time to fracture union
start of the reparative phase

Transcriber/s: Paolo Ramirez


Formatting: Paolo Ramirez D L S H S I M e d i c i n e B a t c h 2 0 1 6 | 3 of 7
Editor/s: Leanne Ramos & Jan Cynric Cacao
FACTORS AFFECTING FRACTURE HEALING
LOCAL  Presence of Avascular Necrosis
 Degree of Local Trauma  Femoral head and scapula are common
 The more severe the trauma is or the more  Also the talus
injury the patient has, the likelihood  Intra‐articular Fractures
problem in late union will be seen  One of the more difficult fractures to treat
 Presence of Bone Loss are those involving articular cartilages
 One of the problems as regards to necrotic  Very important because these are the areas
bones where the joints are
 Leads to delayed fracture healing  Requires almost perfect anatomic
 The more comminuted or the more bones structure to allow motion
are affected, the worse the healing will be  Hence, any destruction in articular
 E.g. congruency will be a problem; it is a difficult
 If you have to connect the proximal area to treat
and distal bones  Make sure to restore joint congruity
 Use of bone grafts  Synovial fluid (bathing the fracture)
 Type of Bone Involved prevents natural healing
 Some bones are more predisposed to poor  Infections
healing  Impair bone healing
 Bones in cancellous areas heal better, even
without any surgical intervention (E.g. SYSTEMIC
scapular or pelvic fractures)  Age
 Metaphysis heals more rapidly  Younger patients heal better
because it is cancellous  Depends on bone maturity
 Cortical bones are more prone to non-union  Once the bone has already
 E.g. Femoral neck fracture matured, the age would not matter
 Fractures that involve the diaphysis  Ex. A femoral fracture of a 25 year old
 Scaphoid & Talus = also prone to non-union will heal just the same as that of a 60‐
 Degree of Immobilization year old. The difference is not
 One of the principles in fracture healing is to that significant.
allow healing to proceed as natural as  Growth hormones
possible  Thyroid hormone, insulin, calcitonin
 Purpose is to avoid disruption of the healing  These can serve as a supplement to
process good bone healing
 In orthopedics, control motion in the bone  Corticosteroids delay healing by markedly
is being practiced to help avoid disruption of inhibiting capillary budding, fibroblast
the healing process proliferation, and the rate of
 Put nails, screws, metals epithelialization
 Depending on the degree of immobilization,  Physiologic doses of Vitamin A & D help in
the fracture may have a problem fracture healing
 Presence of Local Malignancy  Exercise
 This will render the bone to be weak, leading  Important because the bone is sensitive to
to pathologic fractures stimulus applied
 Other Pathologic Conditions  Movement will result in stronger bone
 Inherent  Electrical currents
 If the patient has Ricketts, or other bone  Good for fractured healing
problems  Smoking
 Radiation Necrosis  Bones heal slower with smokers
 For those with tumors undergoing soft tissue
radiotherapy, bone becomes weak, even if
it’s not part of the tumor

COMPLICATIONS
NON-ORTHOPEDIC COMPLICATIONS ORTHOPEDIC COMPLICATIONS
 Shock  Fat Embolism Syndrome
 Can be neurogenic or cardiogenic  Bone marrow in the inflammatory site
 Common in patients with multiple injuries results in embolism
 Hemorrhagic from bleeding and pain  Due to fat globules in fractured area (closed)
 Cardiac arrest  Classical signs include presence of:
 Hemorrhagic complications  Petechiae
 Especially when it involves bones that bleed  Change in sensorium of patient
too much when injured (e.g. pelvis)  With history of fractured bone
 Thromboembolism  Gas gangrene
 DIC  Seen in open fractures only
 Seen also in other organ injuries  Clostridium perfringens
 Mortality rate is high

Transcriber/s: Paolo Ramirez


Formatting: Paolo Ramirez D L S H S I M e d i c i n e B a t c h 2 0 1 6 | 4 of 7
Editor/s: Leanne Ramos & Jan Cynric Cacao
 To prevent this, open fractures are managed  Patients present with:
as emergencies by doing early debridement  Swollen extremity after a fracture
and cleaning  Stiffness in the fingers
 Tetanus  The cause is still unknown
 Any break in tissue would result into  Can still occur despite proper
probable tetanus management of the fractures
 Cause: Clostridium tetani  Compartment Syndrome
 Osteomyelitis  One of the dreaded complications
 Acute infection  One must have a high index of suspicion
 Infection of the bone  This usually happens in the leg and forearm
 S. aureus  There are 4 compartments in the leg:
 Post‐traumatic Reflex Dystrophy Anterior, lateral, superficial posterior
 Nowadays known as “Regional Pain and deep posterior
Syndrome”  A painful condition that occurs when
 “Sudeck’s atrophy” pressure within the muscles builds to
dangerous levels; this pressure can decrease
the blood flow, which prevents nourishment
and oxygen from reaching the nerve and
muscle cells

DIAGNOSIS
 History
 Physical examination
 Imaging (X-rays)
 Get at least 2 views
 One view perpendicular to the other one
 Most of the time it is AP and LATERAL
 If you need special views you can request for oblique views, or even orthopedic‐specific special views

MANAGEMENT
 Emergency  Conventional
 Initial treatment/First Aid  Wood, carton, plastic, aluminum
 Splints, tractions, casts
 Definitive  Definitive
 Closed reduction (usually in Children)  The splint itself
 Align the fracture without visualization  Use splints for definitive treatment
 Non‐surgical, noninvasive,  Examples
conservative  Volar splints for fractures of radial
 Open reduction (usually in Adults) bone
 Align the fracture with visualization
 Surgical or invasive
 Placement of metals, pins, plates,
implants
 Under guidance of imaging technology
 Surgery makes a closed fracture an
open fracture, so be cautious
 Rehabilitation  Thomas splint for lower extremities
 Restore previous function and strength
 Heal the bone, muscles, tendons, ligaments
 Different bones = different healing

SPLINTING
 Indications
 Decreased soft tissue injuries (prevention of
simple fractures from becoming compound)
 Decreased pain (immobilization of injured
extremity promotes decrease or absence of
pain)
 Decreased fat embolism
 Transport
 Types
 Improvised
 Any rigid object used
 Umbrella, patient's own body
 Normal thigh can be used to splint a
fractured thigh

Transcriber/s: Paolo Ramirez


Formatting: Paolo Ramirez D L S H S I M e d i c i n e B a t c h 2 0 1 6 | 5 of 7
Editor/s: Leanne Ramos & Jan Cynric Cacao
TRACTION
 Indications (PAID)  Skeletal (invasive)
 Prevention of deformity  You can apply even patient’s own
 Alignment weight
 Immobilization  Invasive because metals or pins are
 Decrease pain (relaxed muscles surrounding applied through the bone
injury)  Effective versus internal fixation
 Principles of Traction  Allows minimal motion vs. no
 Traction on distal fragment (distal part is movement  good healing
more controllable)  Skull traction ‐ pin pierced through
 Don’t put too much weight skull only up to cortices
 Must not overstretch  Proximal tibial pin
 Forces – constant amount & direction  Skin and skeletal traction may be combined
 Counter‐traction  Things to monitor
 Types  Color
 Skin (non‐invasive)  Motion
 Adhesive straps, encircling devices  Temperature
 Not invasive; and cannot be applied in  Sensation
a patient with wounds  Pulses
 Cannot apply a greater amount of
weight (REMEMBER: 10 lbs. in a 10
year old for 10 weeks)

EXAMPLES OF TRACTIONS
Skin Traction Skeletal Traction
Buck’s Traction Balanced Skeletal Traction

Dunlop’s Traction Cervical Spine Traction (Halter)

CASTING
 Materials  Upper Extremity
 Plaster of Paris (Calcium Sulfate)  Short Arm Cast (SAC)
 Plaster + water = exothermic reaction  Proximal hand to forearm
(release of heat which promotes  Does not involve the elbow
hardening of cast)  Long Arm Cast (LAC)
 Heavier, less durable, cheaper  Metacarpophalangeals to humerus
 Fiberglass or shoulder
 Array of colors  Shoulder Spica
 Radiolucent (easier for x-rays)  Long arm cast with inclusion of
 Stronger, more durable, expensive trunk
 Involves shoulder, with portion of
the torso

 Types

Transcriber/s: Paolo Ramirez


Formatting: Paolo Ramirez D L S H S I M e d i c i n e B a t c h 2 0 1 6 | 6 of 7
Editor/s: Leanne Ramos & Jan Cynric Cacao
 Lower Extremity  Cylinder Cast
 Long Leg Cast  Like LLC without covering on the
 Foot to proximal femur (knee foot
included)  Spine
 Short Leg Cast  Minerva Cast
 Foot to proximal tibia (knee NOT  Cervical and thoracic fractures
included)  Usually in scoliotic patients
 Patellar Tendon-Bearing Cast  Plaster Jacket
 For tibial shock fractures  Trunk fractures
 Knee is allowed to flex and extend  Hip Spica
but NOT to rotate  Femoral shaft fractures, lower
lumbar spine fractures, Pott’s
disease
 Trunk immobilized, lower lumbar
vertebra management

 Hip Spica
 For fixation in the lower
extremities and lumbar spine
 Also used for femoral shaft
fixation
 Not allowed to walk

-END-

TRANSCRIPTION DETAILS
BASIS Latest PPT, 2012 Past Transcriptions RECORDINGS + NOTES + DEVIATIONS 8-10% CREDITS JCacao for PPT pictures
REMARKS Much words. So wow. Take note of the TABLES and all those in BOLD ;)
-BATCH 2016 Transcribers’ Guild Transcriptions. Version 1.0.0.0.0 Build 3101-

Transcriber/s: Paolo Ramirez


Formatting: Paolo Ramirez D L S H S I M e d i c i n e B a t c h 2 0 1 6 | 7 of 7
Editor/s: Leanne Ramos & Jan Cynric Cacao
FRACTURES AND DISLOCATIONS OF THE UPPER EXTREMITIES
Ricardo C. Liwag, MD
July 8, 2014; 1:00 – 2:00 PM
Orthopedics

OUTLINE  Respiratory function – accessory rib


 Fractures of the Clavicle  Cosmesis (especially for women)
 Shoulder Dislocation
 Fractures of the Proximal Humerus CLASSIFICATION (ALLMAN)
 Humeral Shaft Fractures  Group 1 – fractures of the middle third
 Supracondylar Fractures of the Humerus  Group 2 – fractures of the distal third
 Elbow Dislocation  Group 3 - fractures of the medial third
 Olecranon Fractures
 Fractures of the Radius and Ulna
 Distal Radial Fractures

FRACTURES OF THE CLAVICLE


 Most frequent fracture in children

ANATOMY
 Connecting bone between the central and the
appendicular bones
 Connects the upper limb (superior appendicular
skeleton) to the trunk (axial skeleton)
 Bony strut NOTE (RECALL): Remember in our Anatomy that the
 S-shaped weakest part of the clavicle is the junction of its middle and
 Readily visible – one of the superficial bones in lateral thirds? Shocks received by the upper limb (especially
the body the shoulder) are transmitted through the clavicle, resulting
 Important structure; but one can live without a in a fracture that most commonly occurs between its middle
clavicle and lateral thirds
 Congenital deformities – cleidocranial
dysostosis MECHANISM OF INJURY
 Fall on shoulder – direct injury
 Fall on outstretched hand
 Due to the instinct to protect the body
 Hyperextension of the wrist, elbow
extension
 Transmits the impact of injury from the
most distal portion going upward
 May result in the fracture of the radius, ulna,
humerus and the clavicle

SIGNS AND SYMPTOMS


 Swelling/tenderness
 Due to the inflammatory phase of bone
healing
 Presence of exudation and edema →
swelling
 Crepitus – two fragments of fracture are moved / rubbed
against each other (take note that this type of crepitus is
FUNCTION different from a joint crepitus)
 Power and stability of the arm  No need to elicit (intentional)
 Motion of the shoulder girdle  If present, suspect of fracture
 Muscle attachment
 Pectoralis major TREATMENT
 Sternocleidomastoid  Conservative (non-surgical) – immobilization
 Deltoid [general management]
 Trapezius  Surgery – rarely
 Subclavius  Neurovascular involvement – ruptured
 Protection of neurovascular structures subclavian vessel
 Subclavian vessels  Soft tissue interposition – removal of
 Nerve components muscle obstructions

Transcriber/s: Jan Cynric Cacao


Formatting: Jan Cynric Cacao D L S H S I M e d i c i n e B a t c h 2 0 1 6 | 1 of 6
Editor/s: Leanne Ramos & Paolo Ramirez
 Open fracture – for wound cleaning and NOTE: the humeral head is not completely encased by the
stabilization of fracture glenoid, only 1/3 is covered by the articular surface of the
 Multiply-injured – more than one systemic glenoid. This would explain why the shoulder joint is very
injury prone to dislocation.
 Distal 3rd displaced fractures – near the Other structures that will provide stability are the capsule
joint and the surrounding muscles (rotator cuff muscles).
Muscles involved are considered in the rehabilitation of the
joint when there is dislocation
HEALING
 Generally well
 About 4-8 weeks MECHANISM OF INJURY
 4-6 weeks in children; takes longer in  Direct
adolescents and adults
 Indirect – most common

NOTE: according to Dr. Liwag, the most common mechanism is


a direct blow but on the slide, it says indirect

TYPES
 Anterior – most common type of dislocation
(90% of of shoulder dislocations)
 Subcoracoid – most common subtype
 Subglenoid
 Subclavicular
 Intrathoracic – rare; poorest prognosis
 Posterior

NOTE: you will hardly tell whether the dislocation is anterior


or posterior on an AP view. Lateral view would differentiate
an anterior dislocation from a posterior dislocation
You can see a Y-shape structure of the bone in a lateral
view. If the head of the humerus is within the intersection of
the ”Y”, it is NOT dislocated

NOTE: in adults, it can be easy seen that there is a clavicular


fracture due to the presence of scar in the area but in
children, you will hardly see any because of the high
remodeling capacity of their bones

SHOULDER DISLOCATION
 Dislocation vs. subluxation
 Major joint involved:
 Glenohumeral joint
 Other joints
 Acromioclavicular joint
 Coracoclavicular joint

NOTE: the shoulder is the most commonly dislocated major


joint of the body, accounting for up to 45% of dislocations

Subcoracoid dislocation
ANATOMY

Subglenoid dislocation

Transcriber/s: Jan Cynric Cacao


Formatting: Jan Cynric Cacao D L S H S I M e d i c i n e B a t c h 2 0 1 6 | 2 of 6
Editor/s: Leanne Ramos & Paolo Ramirez
TREATMENT
 Immediate reduction
 Hippocratic technique (dirty socks
technique)
 Stimson’s technique
 Kocher’s technique
 Traction/counter traction technique – safest
 Open – done if there is difficulty in close
reduction
 Immobilization
 Duration
 Recurrence – the younger the patient, the
higher the risk of recurrence of a shoulder
dislocation
 Rehabilitation – to strengthen the shoulder stabilizers
(rotator cuff muscles) and normalize range of motion; as
well as to prevent recurrence of shoulder dislocation

Intrathoracic

Hippocratic technique (dirty socks technique)

FRACTURES OF THE PROXIMAL


HUMERUS
 Usually in elderly patients after a minor fall

ANATOMY

SIGNS AND SYMPTOMS


 Pain and lack of mobility
 (+) Lump – not appreciated in late inspection due
to swelling
 Roundedness lost – the deltoids muscle is responsible
for the ‘roundedness’ of the shoulder, considering its
insertion in the proximal humerus and attachment to the
scapula

 Important parts: humeral head, anatomical neck,


greater and lesser tuberosity (trochanter) and the
surgical neck
 Muscles attached to the structures mentioned
(rotator cuff muscles) would be significant in the
displacement of the fracture

Lump on the anterior aspect of the shoulder NOTE: muscles attached to the greater trochanter are the
supraspinatus, infraspinatus and the teres minor; muscle
attached to the lesser trochanter is the subscapularis muscle.
Collectively, they are termed as the rotator cuff muscles.
Just remember the mnemonic SITS

Transcriber/s: Jan Cynric Cacao


Formatting: Jan Cynric Cacao D L S H S I M e d i c i n e B a t c h 2 0 1 6 | 3 of 6
Editor/s: Leanne Ramos & Paolo Ramirez
MECHANISM OF INJURY HUMERAL SHAFT FRACTURE
 Fall on outstretched arm
 Direct fall on to shoulder MECHANISM OF INJURY
 Direct violence/force – most common
FOUR MAJOR FRAGMENTS  Indirect force
 Anatomical neck  E.g. fall, severe and sudden muscle
 Greater tuberosity/trochanter contraction
 Lesser tuberosity/trochanter
 Surgical neck LOCATION OF FRACTURE
 Important to know the fracture displacement
SIGNS AND SYMPTOMS related to anatomy
 Tenderness  Muscles to consider: deltoid and pectoralis
 Swelling muscle
 LROM (limited range of motion)
 Crepitation TREATMENT
 Closed reduction and casting
CLASSIFICATION (NEER)  Traction and casting
 One part fracture – no displacement fragment  Open reduction, internal fixation (ORIF)
 Two part fracture – 1 displaced fragment
 Three part fracture – 2 displaced fragment NOTE: radial nerve involvement – wrist drop deformity
 Four part fracture – 3 displaced fragment *The radial nerve passes anterior to the long head of triceps;
and between the medial and lateral heads of the triceps
brachii; and cross onto the posterior shaft of the humerus
that is why it can get damaged in a humeral shaft fracture

SUPRACONDYLAR FRACTURES OF THE


HUMERUS
 Common among children and adolescents
 Two types
 Extension type
 More common (98% of supracondylar
fractures in children)
 Posterior displacement
 Mechanism of Injury: hypertension occurs
during fall onto an outstretched hand with or
without varus/valgus force
 Flexion type
 2% of supracondylar fractures in children
 Mechanism of Injury: direct trauma/fall
onto a flexed elbow

SIGNS AND SYMPTOMS


 Swelling and deformity of the elbow

X-RAYS
NOTE: most of the time, fractures are managed especially in  AP/lateral as well as the contralateral side
the elderly as non-surgical. But if you have a young patient
with the same fracture condition, it is most probably
TREATMENT
sustained from a very high impact injury (vehicular accident).
Management would be different for this age group. We
 Closed reduction/posterior splint
have to fix the displaced fragment so that the patient would  Closed reduction and pinning – gold standard
be able to function as normal as possible. The only  Open reduction and pinning
complication of a non-surgical approach to the elderly is
limited range of motion. COMPLICATIONS
 Early
TREATMENT  Nerve injuries – may be caused by a traction
injury during reduction owing to tenting or
 Early motion
entrapment at the fracture site
 Healed despite exercises
 Vascular injuries – direct injury to the brachial
 Main goal: restoration of shoulder function and artery or may be secondary to ante-cubital swelling
prevention of adhesion  Volkmann’s ischemia – lack of blood flow
(ischemia to the forearm); occurs when there is
COMPLICATIONS increased pressure due to swelling, called
 Joint stiffness compartment syndrome
 Avascular necrosis
 Mal-union
 Myositis ossificans

Transcriber/s: Jan Cynric Cacao


Formatting: Jan Cynric Cacao D L S H S I M e d i c i n e B a t c h 2 0 1 6 | 4 of 6
Editor/s: Leanne Ramos & Paolo Ramirez
 Late MECHANISM OF INJURY
 Cubitus varus/valgus  Direct – A fall on the point of the elbow/ direct trauma to
 Loss of elbow motion the olecranon typically results in a comminuted olecranon
 Myositis ossificans fracture
 Progressive ulnar nerve palsy  Indirect – A fall onto the outstretched upper extremity
accompanied by a strong, sudden, contraction of the
triceps, typically results in a transverse or oblique fracture
ELBOW DISLOCATION
 Easy to reduce TREATMENT
 The elbow joint is a modified hinge type of joint –  Immobilization in extended position
high degree of intrinsic stability owing to joint congruity,
 ORIF – screw, TBW, excision (2/3)
opposing tension of triceps and flexors, and ligamentous
constraints
 Three joints FRACTURES OF THE RADIUS AND ULNA
 Ulnohumeral joint – major joint  Fractures of necessity – requires surgical
 Radiohumeral joint intervention
 Proximal radioulnar joint  Monteggia’s fracture – fracture of the ulna with
radial head dislocation
TYPES  Galleazzi’s fracture – fracture of the distal radius
 Anterior – rare and involvement of the distal radio-ulnar joint
 Posterior – most common; this is a combination of  Management: ORIF
elbow hyperextension, valgus stress, arm abduction and
forearm supination
 Medial
 Lateral

MECHANISM OF INJURY
 Extended/hyperextended arm

NOTE: Most commonly, injury is caused by a fall onto an


outstretched hand / elbow resulting in a levering force to
Monteggia’s fracture
unlock the olecranon from the trochlea, combined with
translation of the articular surfaces to produce the
dislocation

X-RAYS
 To rule out associated fractures
 May be associated with fractures of the
capitellum, trochlea and radial head

TREATMENT
 Immediate closed reduction
 Early gentle active motion

OLECRANON FRACTURES
 Requires fixation Galleazzi’s fracture
 Difficult in the maintenance of reduction
NOTE: Monteggia’s and Galleazzi’s fracture are considered
 Triceps attaches to the olecranon fractures of necessity because they are “unstable” and needs
immediate reduction and fixation
NOTE: Epidemiology  Bimodal distribution is seen, with MNEMONICS: “MUGR”
younger individuals as a result of high-energy trauma and (Monteggia:Ulnar Fx :: Galleazzi: Radial Fx)
older individuals as a result of a simple fall
ANATOMY

CLASSIFICATION
 Undisplaced
 Displaced
 Avulsion
 Oblique
 Comminuted
 Fracture-dislocation

Transcriber/s: Jan Cynric Cacao


Formatting: Jan Cynric Cacao D L S H S I M e d i c i n e B a t c h 2 0 1 6 | 5 of 6
Editor/s: Leanne Ramos & Paolo Ramirez
DEFORMING MUSCLE FORCES
 Pronator teres
 Biceps brachi
 Supinator brevis
 Pronator quadratus

FRACTURE LOCATION
 Proximal
 Middle
 Distal SIGNS AND SYMPTOMS
 Silverfork deformity (Colle’s Fracture)
 Swelling
DISTAL RADIAL FRACTURES
 Tenderness
 First described by Abraham Colles in 1814
 Common in elderly women
 Usual mechanism of injury: fall on outstretched
hand

COLLE’S VERSUS SMITH’S FRACTURE

Colles’ Fracture
Fracture of distal 2 cm of radius and/or ulna with dorsal Silverfork deformity
angulation of distal fragment
“Silverfork Deformity” / Dinnerfork deformity / Bayonet CLASSIFICATION
deformity  Frykman – based on the pattern of intra-articular
involvement

TREATMENT
 Classical – closed manipulation/cast
 Pins and plaster, external fixation
 ORIF

COMPLICATIONS
 Median nerve palsy – carpal tunnel compression
symptoms are common, owing to traction during forced
Smith’s Fracture hyperextension of the wrist, direct trauma from fracture
A fracture with volar angulation of the distal radius, fragments, hematoma formation, or increased
with volar displacement of the hand and distal radius compartment pressure
“Garden spade deformity”  Stiffness
 Sudeck’s atrophy
 Volkmann’s ischemia

BIG NOTES:
*Colles’ fracture is a fracture of the distal radius; arises upon
falling on an outstretched hand.
*Smith’s fracture is a fracture of the distal radius; arises
when a person falls on a flexed wrist

-END-

TRANSCRIPTION DETAILS
Anonymous guy/girl and Xmark for the
BASIS Latest PPT RECORDINGS + NOTES + DEVIATIONS 3-5% CREDITS
recordings
Are you an aspiring Orthopedic Surgeon like me? Then study very well the different fractures discussed in this lecture. As for everyone
else, know by heart the different muscle groups associated with each kind of fracture, their insertions and origins for they play a vital
role in classifying and understanding further the mechanisms of injury of each type of fracture. This, by far is the most complete
REMARKS
transcription for Orthopedics with two layers of recordings. Pictures are lifted from the internet. – Spidey
Spread the #ortholove
Editor’s Note: References: Handbook of Fractures, 3rd Ed., Clinical Oriented Anatomy Moore, 6th Ed.
-BATCH 2016 Transcribers’ Guild Transcriptions. Version 1.0.0.0.0 Build 3105-

Transcriber/s: Jan Cynric Cacao


Formatting: Jan Cynric Cacao D L S H S I M e d i c i n e B a t c h 2 0 1 6 | 6 of 6
Editor/s: Leanne Ramos & Paolo Ramirez
INJURIES TO THE SPINE
Paul M. Atienza, MD, FPOA, FPSS, PTRP
July 28, 2014; 1:00 – 2:00 PM
Orthopedics

OUTLINE
 Spine anatomy  Key muscles
 Spine motion  Neurological classification
 Goals of spine care  Spinal cord syndrome
 Spine immobilization  Cervical fracture
 Transport  Lower cervical fracture
 Cervical trauma  Hospital care
 Radiographic evaluation  Principles of treatment
 Spinal cord injury
 Thoracic spine
SPINE ANATOMY  Relatively stiff due to costal articulations
 33 vertebra  Thoracolumbar segment
 7 cervical  T12-L1: transitional vertebra from rigid
 12 thoracic (thoracic) to mobile (lumbar segment)
 5 lumbar  Lumbar spine
 5 sacral (fused)  Largest vertebra
 1 coccyx  Common site for back pain and sciatica
 Normal spinal curvatures curvatures  L4/L5/S1 facet – trunk flexion
 Cervical lordosis  L1/L2 – filum terminale
 Thoracic kyphosis  Sacral spine
 Lumbar lordosis  No motion
 Sacral kyphosis  Sacro-iliac joint – attachment of upper body
to pelvis

GOALS OF SPINE CARE


 Protect further injury during evaluation and
management
 Put sand bags, neck collar
 Protect the spine – make the spine straight
at all times
 Identify spine injury or document absence of
spine injury
 Every patient that comes to the emergency
room is a potential spine injury patient,
otherwise, you have to rule it out
 Optimize conditions for maximal neurologic
recovery

SPINE IMMOBILIZATION
 Rigid cervical collar
 Neutral position
 Hard backboard
 Lateral support (sand bag)
SPINE MOTION
 Cervical spine
TRANSPORT
 Most movable segment of the vertebra  Remove the helmet
 Occiput/C1 joint (C0/C1)/Atlanto-occipital  Two person maneuver
joint: flexion/extension of the head  Unlock first the strap
 Mnemonic: Atlanto-Occipital (“AO” joint =  Before pulling it out, somebody should
Ah Oo! (as if you are agreeing, shake you hold/support the neck and keep it straight
head up and down = flex and extend)  Two-man lift
 Atlanto-axial joint (C1/C2): rotation of the  Three-man lift
head  There is a neck collar
 Mnemonic: Atlanto-Axial (“AA” joint” = A-A/  Patient is kept straight
NA-AH! (as if you are disagreeing, shaking
your head to the left and right = rotation)
 C6/7-C7/T1: most mobile subaxial segments

Transcriber/s: Sarah Livelo, Jan Cynric Cacao


Formatting: Jan Cynric Cacao D L S H S I M e d i c i n e B a t c h 2 0 1 6 | 1 of 10
Editor/s: Paolo Ramirez, Leanne Ramos
CERVICAL TRAUMA
 Seatbelt injury – commonly encountered are whiplash injuries
 Neck trauma secondary to a fall from a height
 Sports injury

RADIOGRAPHIC EVALUATION
VIEW IMAGING
 To assess cervical spine trauma,
request for open mouth odontoid
view, AP and lateral
 Lateral Atlantodens Interval or
LADI (blue lines)
Left and right side of the dens
must be equidistant
If the dens moves laterally on
one side, the other side is
A. OPEN MOUTH fractured
ODONTOID  Lateral Mass Overhang (red
VIEW lines)
If the atlas moves laterally
(hindi na siya tumapat sa
axis), it is fractured
If the axis moves beyond the
line as well, it is fractured
 Joint space (wink sign) (yellow
arrows)
If one side of the space is
absent, suspect a fracture
 Wackenheim’s line (blue line)
From the posterior part of the
dens to the basion
Not much use
 Spinolaminar line (red line)
Connect the spinolamina of
the C1, C2 & C3, which should
B. LATERAL
bisect the basion
VIEW: UPPER
 Basion Dens Interval (BDI) (red
CERVICAL
arrow)
 Space Available for the Cord
(SAC) (orange line)
 Posterior Axis Line-Basion
(PAL-B) (green line)
 Atlantodens Interval (ADI)
(yellow line)
 McRae line (blue line)
A line drawn between the
occiput and the basion
If the tip of the dens migrates
above this line, it indicates
presence of basilar
BASILAR invagination
INVAGINATION  Chamberlain line
(dens protrudes From the posterior arc of the
inside the occiput jaw to the opisthion
and may stab the Helps recognize basilar
cerebellum, or invagination, which is said to
the occiput goes be present if the tip of the
down to the dens) dens is >3mm above this line
 McGregor line
Is a modification of the
chamberlain line
 Used in the evaluation of basilar
invagination when the opisthion
could not be identified on plain
radiographs

Transcriber/s: Sarah Livelo, Jan Cynric Cacao


Formatting: Jan Cynric Cacao D L S H S I M e d i c i n e B a t c h 2 0 1 6 | 2 of 10
Editor/s: Paolo Ramirez, Leanne Ramos
Refers to a line connecting
posterior edge of the hard
palate to the most caudal
point of the occipital curve
 If the dens protrudes inside the
occiput, impingement of the
cerebellum and medulla
oblongata may occur and you
will lose breathing
 Other lines:
Interspinous distance
 Should be the same for all
levels
 If one is shorter than the
other, a dislocation may
be present
Disc space distance
Atlanto-Dens Interval (ADI)
 Anterior prevertebral soft
tissue line (orange line)
Always around 5mm in
length from C2 or C3 (Doc
mentioned the normal value
as 5-8mm, then a few
seconds later, 3-5mm)
In injuries, swelling occurs,
and soft tissue will be pushed
anteriorly and the line will
5 CERVICAL
become wider
LINES
Indicates injury to cervical
spine
 Anterior vertebral line (blue
line)
Almost parallel and lordotic
 Posterior vertebral line (red
line)
 Spinolaminar line (yellow line)
 Spinous process line (green
line)

SPINAL CORD INJURY

 Cervical injury = high risk of being quadriplegic


 Thoracic and/or lumbar injury = high risk of being paraplegic

Transcriber/s: Sarah Livelo, Jan Cynric Cacao


Formatting: Jan Cynric Cacao D L S H S I M e d i c i n e B a t c h 2 0 1 6 | 3 of 10
Editor/s: Paolo Ramirez, Leanne Ramos
KEY MUSCLES
 Breathing = C3-5
 Elbow flexors = mainly C5,
supported by C6
 Wrist extensors = mainly C6,
supported by C7
 Hand flexors = C8
 Hand abductors = T1

NOTE: Doc always emphasized the


first key muscles listed for each one
(ex: for knee extensors, L3; for elbow
extensors, C7)

NEUROLOGICAL CLASSIFICATION

Transcriber/s: Sarah Livelo, Jan Cynric Cacao


Formatting: Jan Cynric Cacao D L S H S I M e d i c i n e B a t c h 2 0 1 6 | 4 of 10
Editor/s: Paolo Ramirez, Leanne Ramos
 ASIA Classification = “ASIA” stands for American Spinal Injury Association
 It is the most commonly used classification of spinal cord injuries
 Should be provided in every hospital because it directly assesses the injury of the patient
 Sensory Level (normal)
 Most caudal 2/2 sensation for both pinprick and light touch
 Motor Level (normal)
 Lowest 3/5 key muscle grade with 5/5 key muscle grade above it
 As long as the patient can use the muscle against gravity (resistance is not necessary)
 Neurologic Level of Injury (NLI)
 Most caudal level with both motor and sensory intact
 Total scores (motor and sensory) are taken to monitor and determine if patient is improving (check the
image above)
 Examinations repeated at intervals between 72 hours and 10 days
 Why 72 hours? In the first 72 hours of spinal injury, the patient is in spinal shock, so we cannot assess
the motor and sensory level of injury

SPINAL CORD SYNDROMES

SYNDROME IMAGING
 Central cord syndrome is most often
caused by hyperextension of the
cervical region of the spine
 The cord is pressed on anteriorly by the
vertebral bodies and posteriorly by the
bulging of the ligamentum flavum,
causing damage to the central region
of the spinal cord
 Radiographs of these injuries often
appear normal because no fracture or
dislocation has occurred
 It is the most common syndrome and
is seen in cervical spine injuries
 Upper extremities with almost
complete involvement
 The lower limb fibers are affected less than the
CENTRAL CORD upper limb fibers because the descending fibers
in the lateral corticospinal tracts are
SYNDROME laminated, with the upper limb fibers located
medially and the lower limb fibers located
laterally
 Lateral Corticospinal Tract:
descending motor fibers (crosses in
medulla)
 Posterior Columns: position and
vibration (crosses in medulla)
 Spinothalamic Tract: pain and
temperature (crosses near level of
entry to spinal cord)
 Bladder dysfunction and varying
sensory dysfunction because this
affects the center of the cord, where
the sacral and lumbar segments are
found
 Fair prognosis (50-60% recovery)

Transcriber/s: Sarah Livelo, Jan Cynric Cacao


Formatting: Jan Cynric Cacao D L S H S I M e d i c i n e B a t c h 2 0 1 6 | 5 of 10
Editor/s: Paolo Ramirez, Leanne Ramos
 Cervicomedullary junction fractures
(yellow arrow shows the junction)
 Upper extremity paralysis (patchy and
irregular involvement, similar to central
cord syndrome)
 Upper extremities decussate at the
medulla
CRUCIATE  This area is also known as the
PARALYSIS cruciate
 Minimal or no lower extremity
involvement
 Lower extremity tracts decussate lower
between C1 and C2 (highlighted in
blue)
 25% respiratory insufficiency; in C1 and
C2, thus patients are intubated

 Hemisection of the spinal cord can be


caused by fracture dislocation of the
vertebral column, by a bullet or stab
wound, or by an expanding tumor
 Rarely seen in spinal trauma; more
common in gunshot wounds and stab
wounds
 LEVEL of the lesion (where the injury
occurred):
 Ipsilateral sensory loss
 Destruction of the posterior root and its
entrance into the spinal cord at the level
of the lesion
 Ipsilateral motor loss
 Damage to the neurons on the anterior
BROWN gray column and possibly by damage to
SEQUARD the nerve roots of the same segment
SYNDROME  BELOW the LEVEL of the lesion:
 Ipsilateral loss of position &
vibration sense
 Destruction of the ascending tracts in
the posterior white column on the
same side of the lesion
 Ipsilateral motor loss
 Contralateral pain & temperature
sense
 Destruction of the crossed lateral
spinothalamic tracts on the same
side of the lesion. Because the
tracts cross obliquely, the sensory
loss occurs 2-3 segments below
the lesion distally.
 Good prognosis (90% recovery)

Transcriber/s: Sarah Livelo, Jan Cynric Cacao


Formatting: Jan Cynric Cacao D L S H S I M e d i c i n e B a t c h 2 0 1 6 | 6 of 10
Editor/s: Paolo Ramirez, Leanne Ramos
 Involves loss of neural function in the
rd
anterior 2/3 of the spinal cord
 Patients with these injuries experience
ANTERIOR complete loss of motor function and
CORD pain (sharp) and temperature sensation
SYNDROME but there is relative preservation of light
touch, proprioception & deep pressure
sensation
 Poor prognosis (10% recovery)

 Least common and is very rare


 Can be caused by an interruption to the
posterior spinal artery
 Loss of proprioception, deep pain and
deep pressure ONLY
POSTERIOR
 Motor function and gross touch
CORD
sensation are typically spared due to the
SYNDROME
ventral location of the descending motor
tracts and spinothalamic tracts,
respectively
 The prognosis is fair

CONUS MEDULLARIS SYNDROME


 (+) Saddle anesthesia
 Bowel and bladder dysfunction
 Only spinal cord syndrome with both
UMN & LMN lesions

EPIDURAL CAUDA EQUINA SYNDROME


COMPRESSION  Is a rare disorder affecting the
SYNDROMES bundle of nerve roots (cauda
(both usually equina) at the lower (lumbar) end of
happen at the the spinal cord and is considered a
lumbosacral area surgical emergency
of the spine – S2  The nerve roots in the lumbar spine are
& S4) compressed, cutting off sensation and
movement
 Nerve roots that control the function
of the bladder and bowel are especially
vulnerable to damage
 (+) Saddle anesthesia
 Only spinal cord syndrome with
LMN lesion (all others cause UMN
lesions)
 Depressed deep tendon reflexes
(DTR)

Saddle anesthesia

Transcriber/s: Sarah Livelo, Jan Cynric Cacao


Formatting: Jan Cynric Cacao D L S H S I M e d i c i n e B a t c h 2 0 1 6 | 7 of 10
Editor/s: Paolo Ramirez, Leanne Ramos
CERVICAL FRACTURE C2 PEDICLE FRACTURE (HANGMAN’S FRACTURE)
 Associated with C2/3 facet dislocation
C1 FRACTURE (JEFFERSON’S)  Mechanism of Injury (MOI): Classic injury is a
 Overhang fracture bilateral fracture passing through the posterior part
 Sudden load on the top of head of the lateral masses or pars inter-articularis of the
axis and into the intervertebral notch; the body of
 Usually no neurologic damage
the axis is then SUBLUXATED or DISLOCATED in
 Mechanism of Injury (MOI): The superior articular
relation to the body of the C3.
process of the atlas face upward, inward and
 Needs surgical stabilization
slightly backward. A vertical compression can
thrust the lateral masses outward, and disrupt the
ring of the atlas, producing a C1 ring fracture.

The lines found on the lateral view is broken in Hangman’s


fracture

LOWER CERVICAL FRACTURE


 In this region of the spine, dislocations without
fractures are common, but fractures and
fracture-dislocations do occur
 The spinal cord and nerve roots frequently are
involved, in addition to displaced bony elements,
the IV disc can become displaced and function as
the leading impact force against the spinal cord

LOWER CERVICAL INJURY (C3-C7)


 Wedge fracture
 Posterior ligament injury
C1/Jefferson’s Fracture  Hyperextension injury
 Burst fracture
C2 FRACTURE (ODONTOID FRACTURE)  Most important and considered an
 Mechanism of Injury (MOI): Flexion injury after emergency case
high velocity or severe fall  Bones are beyond the margins and the
 Neurological symptoms occur in about 20% of posterior segment goes beyond the
cases posterior vertebral line – almost 100% will
cut the spine
 Fracture-dislocation
 Emergency case
 Tear drop injury

HIGHLY UNSTABLE CERVICAL FRACTURES


 Fracture-dislocation
 Burst fracture

THORACIC/LUMBAR SPINE
 Compression fracture – a wedge compression of the
vertebral body is produced by a flexion force, but the
posterior ligament complex remains intact

The atlantodens interval is WIDER on one side in odontoid


fracture

Transcriber/s: Sarah Livelo, Jan Cynric Cacao


Formatting: Jan Cynric Cacao D L S H S I M e d i c i n e B a t c h 2 0 1 6 | 8 of 10
Editor/s: Paolo Ramirez, Leanne Ramos
 Flexion-distraction
 Result from failure of the posterior elements in
tension while the anterior and middle column are
compressed
 The most common mechanism is lap belt in a motor
vehicle accident

Compression fracture Burst Fracture

 Burst fracture
 Stable burst – posterior column uninjured, and
patients remain neurologically intact A flexion-distraction injury
 Unstable burst – posterior column disrupted and
amount of neurologic injury varies based more on  Fracture-dislocation
the level of the injury than the degree of canal
compromise by bone fragments
 Chance facture (seat belt)
 These fractures result from tension failure of all
spinal bony elements as a result of hyperflexion
over a secured lap belt
 Commonly occurs in back seat passengers and seen
frequently in children
 Bowel injuries occur in up to 65% of these patients
because the lap belt provides the fulcrum against
the abdominal wall

A fracture-dislocation of the thoracic vertebra

DENIS 3 COLUMN THEORY

Anterior column (Denis 1) Middle column (Denis 2) Posterior column (Denis 3)


 Anterior longitudinal ligament  Posterior halt of vertebral body  Spinous process
 Anterior half of vertebral body  Posterior annulus/posterior disc  Laminae
 Anterior annulus fibrosis  Facets
 Anterior disc  Posterior longitudinal ligament  Pedicles
 Posterior ligamentous structures
 Ligamentum flavum
 Intraspinous ligaments
 Supraspinous ligaments

NOTE: if you have a break in two out of three segments, the fracture is UNSTABLE

Transcriber/s: Sarah Livelo, Jan Cynric Cacao


Formatting: Jan Cynric Cacao D L S H S I M e d i c i n e B a t c h 2 0 1 6 | 9 of 10
Editor/s: Paolo Ramirez, Leanne Ramos
HOSPITAL CARE NOTE:
 Proper positioning/turning schedule Spinal Shock - involves a spinal cord concussion which
 Bowel and bladder care usually involves 24-72 hour period of paralysis,
 Postural drainage/assisted coughing hypotonia, & areflexia, and at its conclusion there may
be hyperreflexia, hypertonicity, and clonus. [non-
circulatory collapse]
PRINCIPLES OF TREATMENT Neurologic Shock - a falling blood pressure with
 Decompression
bradycardia in a cervically injured patient
 Stabilization
 Correction of deformity
MEDICAL TREATMENT
GENERAL PRECAUTIONS  Still controversial
 Strict spine precautions (immobilization)  NASCIS I, II & III (currently used)
 Emergency resuscitation (ABC)  If < 1-3 MPSS (methylprednisolone sodium
 Comprehensive approach succinate) 30 mg bolus the 5.4 mg/kg for 24
 Neurological and radiological assessment hours
 Always expect multiple trauma (neuroexam,  If 3-8 hours. MPSS 30 mg bolus then 5.4
chest, abdomen, musculoskeletal) mg/kg for 48 hours
 If > 8 hours, not indicated
 Differentiate hypovolemic from neurologic shock
 Others
 Naloxone
 Lazaroids
 Ganglioside GM-1
 Erythropoietin
 Gacyclidine GX-11
 Cethrin BA-210
 External immobilization
 Internal fixation

PRE-QUIZ
1. Measures the basilar invagination or a. Posterior disc a. C7
encroachment of the dens odontoid b. Posterior cortex b. C8
process to the occipital fossa EXCEPT c. Posterior longitudinal c. T1
a. McRae ligament d. None of the above
b. Power’s ration d. Posterior facet
c. Chamberlain 8. Radiographic line seen on the lateral
d. Atlantodens interval 5. Upper extremity is more involved in view EXCEPT
the following syndrome a. Lateral atlantodens interval
2. Which of the following has an upper a. Conus medularis syndrome b. Anterior vertebral line
and lower motor neuron lesion? b. Cauda equina syndrome c. Posterior vertebral line
a. Conus medularis syndrome c. Brown-sequard syndrome d. Spinolaminar line
b. Cauda equina syndrome d. Cruciate paralysis
c. Brown-sequard syndrome 9. Jefferson’s fracture involves the
d. Cruciate paralysis 6. ASIA scale level for big toe extensor a. Axis pedicle fracture
a. L3 b. Odontoid fracture
3. In ASIA scoring, the key motor level b. L4 c. Atlas ring fracture
for wrist extensors is c. L5 d. None of the above
a. C4 d. S1
b. C5 10. Hangman’s fracture involves the
c. C6 7. A patient has a C7 level sensory of a. Axis pedicel fracture
d. C7 2/2 and a motor of 5/5, C8 level b. Odontoid fracture
sensory of 2/2 and a motor of 3/5 and a c. Atlas ring fracture
4. Denis type 2 fracture involve the T1 level sensory of 1/2 and a motor of d. None of the above
following EXCEPT 3/5. The neurologic level of injury is

-END-

TRANSCRIPTION DETAILS
BASIS Latest PPT RECORDINGS + NOTES + DEVIATIONS 3-5% CREDITS -
Editor’s note: You may want to print in colored for better appreciation of important structures pointed in the images. Other images
REMARKS embedded and additional information were lifted from Clinical Neuroanatomy by Snell, 7th Ed, Manual of Orthopaedics, 6th Ed., The
Netter collection of Medical Illustrations – Musculoskeletal – The Spine Vol. 6 & other internet sources.
-BATCH 2016 Transcribers’ Guild Transcriptions. Version 1.0.0.0.0 Build 3107-

Transcriber/s: Sarah Livelo, Jan Cynric Cacao


Formatting: Jan Cynric Cacao D L S H S I M e d i c i n e B a t c h 2 0 1 6 | 10 of 10
Editor/s: Paolo Ramirez, Leanne Ramos
FRACTURES IN CHILDREN
Ricardo C. Liwag, MD
July 21, 2014; 1:00-2:30 PM
Orthopedics

LECTURE OUTLINE:
 Definition of Terms  Salter Harris Classification Of Epiphyseal Fracture
 Pediatric Bone vs. Adult Bone  Remodelling
 Review of previous lectures  Management of fractures in children
 4 zones of the Physis  Specific fractures in children

DEFINITION OF TERMS
Epiphysis Periosteum Epiphyseal Plate/Physis Epiphyseal Line
 Part of the bone  Tissue layer of the  Part of the PEDIATRIC bone  Part of the ADULT bone
attached to both bone that surrounds (children and adolescents)  Remnant of the physis of the
ends of the diaphysis, the cortex  Hyaline cartilage plate in the epiphyseal cartilage when the bone
that ossifies  The membrane of metaphysis at each end of a long stops growing
separately and later connective tissue bone
becomes ankylosed that closely invests  Site where longitudinal growth
to the main part of all bones except at
occurs
the bone the articular surfaces
 Will become the epiphyseal line

PEDIATRIC BONE VS ADULT BONE


PEDIATRIC ADULT
Growth Plate Present Absent
Periosteum Thicker, stronger Thin
Bone Remodelling Greater potential Lesser potential
Bone Overgrowth Possible Not possible
Plasticity More plastic/pliable/flexible Less plastic, more prone to breaking
Healing Rate Faster Longer
Risk of Non-union Uncommon More common
Stiffness Less after cast removal Very stiff after cast removal

BIGGEST NOTES:
 Pediatric bones have a growth plate, thicker periosteum, greater remodelling, prone to overgrowth, more
plastic, faster healing rate, uncommon non-union, and less stiff
 Deformity and fracture is more prone to pediatric bones while brittleness or comminuted fractures are
more prone to adult bones in relation to their periosteum.
 For healing rate, For femoral shaft fracture healing:
 Neonates: 2-3 weeks
 <5 years old: 4 weeks
 >6 years old: 6 weeks
 Adolescents: 6-8 weeks
 Adults: 2-3 months
 Pediatric elbows are prone to stiffness, requiring early ambulation and management

Editor’s note:
 Growth plate is the source of bone length
 Periosteum in children is whitish, ligamentous in character and it would be of help in the reduction of
fractures.
 End-to-end reduction in bone fractures of children is prone to overgrowth
 Early ambulation is needed for adults than in children due to stiffness

Transcriber/s: Paolo M. Ramirez


Formatting: Paolo M. Ramirez D L S H S I M e d i c i n e B a t c h 2 0 1 6 | 1 of 5
Editor/s: Jan Cynric Cacao & Leanne Gale Ramos
REVIEW OF PREVIOUS LECTURES
 Note that we can use epiphysis, diaphysis, and
FRACTURES BASED ON PATTERN: metaphysis in children as basis of location
 Transverse  In adults, we use: proximal 3rd, middle 3rd, or distal
 Oblique
rd
3
 Spiral (s-shaped)
 Comminuted SIGNS AND SYMPTOMS OF FRACTURES:
 Greenstick  Pain - most common complaint
 Exclusively in children  Tenderness
 Fractured in cortex of the bone but no fracture  Deformity - attitude of extension upon inspection
in the opposite cortex (bone is incompletely  Crepitus - absent in children
fractured)  Swelling
 The fracture may need to be completed to obtain  Warmth
an adequate reduction  Shortening
 Torus  Fever - not commonly observed, more
 Mostly seen in children, but can also happen in on children
adults
 Break in one cortex and a compression of the COMPLICATIONS:
other cortex  For open fractures: gas gangrene, osteomyelitis
 Rarely cause physeal injury, but may result in  For close fractures: compartment syndrome, fat
acute angular deformity embolism
 They are stable, and rarely require manipulative
reduction; If manipulated, they usually regain X-RAY VIEWS:
the original fracture deformity as swelling  2 basic views: AP and Lateral view
subsides  Additional view: oblique
 Also seen at the metaphysis fracture in adults  Contralateral extremity view: used to compare the
injured extremity from the normal extremity. In
FRACTURES BASED ON LOCATION: children, epiphyseal line may be mistaken as a
 Same locations as to adult fractures fracture so we request for this view to confirm
 Epiphyseal fractures are seen only in children.

FOUR ZONES OF THE PHYSIS


Zone of Provisional
Zone of Undifferentiated Zone of Proliferating Zone of Vacuolization or
Calcifications or Cell
or Resting Cartilage cells Cartilage Hypertrophy
Degenerations
[ZONE 1] [ZONE 2] [ZONE 3]
[ZONE 4]
 Germinal layer  Area of active cell  Weakest part  Cartilage becomes bone
 Zone nearest epiphysis growth, where  There is no actual cell (transitional part)
 Injury in this layer causes multiplication occurs formation, only  Nearest to the ossified bone
cessation of growth and  Responsible for hypertrophy  Nearest to metaphysis
100% deformity lengthening the bone  No additional cells
 *Growth arrest will be produced
present if this is damaged  No growth arrest if
damaged
 The larger the cells, the
weaker it becomes
 MOST FRACTURES
OCCUR IN THIS SITE

CLASSIFICATION OF EPIPHYSEAL FRACTURE:


1. Foucher
2. Poland
3. Aitken
4. Ogden
5. Salter-Harris – most preferred

Transcriber/s: Paolo M. Ramirez


Formatting: Paolo M. Ramirez D L S H S I M e d i c i n e B a t c h 2 0 1 6 | 2 of 5
Editor/s: Jan Cynric Cacao & Leanne Gale Ramos
SALTER HARRIS CLASSIFICATION OF EPIPHYSEAL FRACTURE
TYPE 1 TYPE 2 TYPE 3
 Injury to the zone 3, which  MOST COMMON TYPE  Fracture separation of physis with the
is the weakest part  Injury and fracture line goes into the involvement of epiphysis (zone 3)
 Fracture separation along metaphysis (physis to metaphysis)  Involvement of the articular surface
the physis  THURSTON-HOLLAND SIGN – (intraarticular fracture)
 No displacement of bone pathognomonic of epiphyseal fracture  Restoration/ reduction of configuration
 Manage conservatively  Manage conservatively of joints
 Fracture heals properly  No need for reduction (seldom usage of  Surgical management is needed
ORIF for type 2) 
 BEST PROGNOSIS Restore anatomic intraarticular surface
 Good prognosis  Open reduction/ internal fixation (ORIF)
is required

TYPE 4 TYPE 5
 Fracture combination of type 2 and type 3  Compression fracture of the physis
 Epiphysis passing thru the physis and metaphysis  Involves the physis and the germinal layer
 Separation of the metaphysis and epiphysis  Most severe crushing injury
 Surgery is the sole management  Injury of zone 1 which leads to growth arrest with
subsequent deformity
 POOREST PROGNOSIS
 Same management as type 1

Editor’s note:
*Thurston-Holland sign is seen on BOTH type 2 and type 4 epiphyseal fractures
*Minor residual deformity in Salter-Harris I and II injuries correct themselves with subsequent growth, so open reduction is not indicated
because the operation itself may just cause more trauma!
*The epiphyseal growth plate is weakest at the site of cell degeneration and provisional calcification (grown plate zones of calcification
and hypertrophy)  children who have undergone a rapid growth spurt, and in those who are excessively heavy for their skeletal maturity
are particularly vulnerable to such growth plate injuries

REMODELLING  Bayonnet apposition


 There is good remodeling potential in pediatric  Overlap of fractured segments is present
fractures  No need to reduce the bone from end to end
 Connection of bone misalignment is done in because there’s still a chance for bone
fracture management overgrowth
 Biometaposition is the treatment
CONDITIONS FOR BONE MODELING TO OCCUR  During the process of remodeling, there is
 2 years of remaining growth deposition of the bone in the concave side
 There is a greater potential for remodeling (fractured side) and osetoblastic activity in
in children if it occurs within the remaining 2 the convex side (plastic deformity)
years of bone growth  Takes several years to see the progress of
 Girls- 12 to 14 y/o remodeling
 Boys- 14 to 16 y/o  The Deformity should be in the plane of motion
 Metaphyseal of the joint
 Fracture should be in the metaphyseal area.  Varus cubitus = deformity is found medially
Remodelling is less when it occurs in and laterally against the plane
diaphysis or farther away  Rotational deformity = varus cubitus +
 *REMEMBER that the metaphysis is deformity in AP plane
cancellous and has a better blood supply
hence better remodelling

Transcriber/s: Paolo M. Ramirez


Formatting: Paolo M. Ramirez D L S H S I M e d i c i n e B a t c h 2 0 1 6 | 3 of 5
Editor/s: Jan Cynric Cacao & Leanne Gale Ramos
MANAGEMENT OF FRACTURES IN  Care must be taken especially in zone 1
 Splint and metal screws seldom used
CHILDREN  Internal implants are small, smooth and non-
 Conservative management – closed reduction threated
and cast (Majority of the time)  May use external fixator
 Open reduction is seldom done  Displaced intraarticular fracture
 As a rule, the younger the patient, the greater the
 Most prone to non-union
remodeling potential; thus, absolute anatomic
reduction in a child is less important than in a  Management is open reduction fixed with
comparable injury in an adult smooth pins
 Emergency management, debridement,  Open reduction maintains congruence of the
antibiotics, and stabilization of the bone for open joint
fractures  restore the anatomic configuration of the
articular surface of the bone
INDICATIONS FOR OPEN REDUCTION  Types 3 and 4 Salter Harris Classification of
 Failed closed reduction epiphyseal fracture
 Seen in older children  Needs anatomic restoration/reduction of the
 When good acceptable reduction cannot be physis and epiphysis
achieved, proceed to open reduction  Head injury and multiple fractures

SPECIFIC FRACTURES IN CHILDREN


SUPRACONDYLAR FRACTURE OF THE HUMERUS
 Fracture along the elbow
 Common in children and adolescents (these
comprise 55% to 75% of all elbow fractures)
 2 TYPES:
 Extension – triceps which is an extensor
muscle will pull the distal fragment of the
elbow posteriorly
 Natural defense of the body when
falling is to extend the elbow
 More common (90%)
 Flexion – biceps which is a flexor muscle will
pull the distal fragment of the elbow
anteriorly
 Fall on a flexed elbow
 Less common (10%)
 S/Sx: swelling and deformity if the elbow (Patients
typically present with a swollen, tender elbow with
Treatment:
painful range of motion)
 Closed reduction and posterior splinting
 X-ray: AP, Lateral, contralateral
 Closed reduction and percutaneous pinning –
 UTZ: for diagnosis
GOLD STANDARD
 Open reduction and pinning are used for unstable
supracondylar fracture and those that can’t be
managed by closed reduction

Transcriber/s: Paolo M. Ramirez


Formatting: Paolo M. Ramirez D L S H S I M e d i c i n e B a t c h 2 0 1 6 | 4 of 5
Editor/s: Jan Cynric Cacao & Leanne Gale Ramos
Complications:
 Early
 Nerve injury (ulnar and radial nerve)
 Ulnar nerve: This is most common in flexion-
type supracondylar fractures
 Vascular injury leading to Volkmann’s
ischemia (development of compartment
syndrome) which is the most dreaded
complication (usually brachial artery is
affected)
 Compartment syndrome can be exacerbated
by elbow hyperflexion when excessive
swelling is present in the cubital fossa
 Late
 Cubitus valgus/varus (varus more frequent than
valgus)
 Treated by corrective osteotomy
 Loss of elbow motion
 Myositis ossificans
CLAVICULAR FRACTURE
 Progressive ulnar nerve palsy (at the medial
 Most commonly fractured bone in children
condyle)
 Volkmann’s contracture – flexor muscles  Considered a birth injury
are contracted  Management involves closed reduction
BIRTH INJURIES
 Seen in breech vaginal spontaneous delivery or
difficult labor
 In decreasing order of frequency:
 Clavicle
 Humeral shaft
 Femoral shaft
 Upper and lower humeral epiphysis
 Upper and lower femoral epiphysis
 Nerve injuries
 Skull depressed deformities
 Intrauterine fractures
 Fractures below or distal the elbow and knees are
uncommon. If it is present, think of a pathologic
MONTEGGIA’S FRACTURE
fracture (e.g. osteogenesis imperfecta)
 Fracture of ulna associated with (proximal)
radial head dislocation. Composed of two PATHOLOGIC FRACTURES
compartments: head of radius and misdshaft of
 Bone has disease and is prone to fracture
ulna
 Pseudoarthritis, osteomyelitis, rickets, vitamin
 Seen in both children and adults but differs in
deficiency, tumors, infection
management
 For adults, this is a fracture with necessity;
CHILD ABUSE
fracture is treated with open reduction and
 Children <3yo or less than 6months (2/3 will be
internal fixation
<3 y/o, the other 1/3 is < 6 months)
 In children, treated with closed reduction
 Battered children syndrome with multiple
 Diagnosis: X-ray of whole arm to see ulnar
fractures: deliberate, non-accidental physical
compartment
injury to a child who is under the care of someone
 Very unstable
else
 Multiple fractures in different healing stages
 Male > Female
 A history (mechanism of injury) that is
INCONSISTENT with the fracture pattern
 Skin stigmata suggestive of abuse: Multiple bruises
in various stages of resolution, cigarette burns, etc.

-END-
TRANSCRIPTION DETAILS
BASIS Latest PPT, HB 108 tranx RECORDINGS + NOTES + DEVIATIONS 8-10% CREDITS Xmark’s Recordings
100% passing guys 
Source: The Netter Collection of Medical Illustration, Musculoskeletal – Upper Limbs Vol6
REMARKS
Editor’s note: Highlighted some of the important things to remember in this lecture. Most are recall from our previous lectures.
Additional source: Egol MD, Kenneth. “Handbook of Fractures.”
-BATCH 2016 Transcribers’ Guild Transcriptions. Version 1.0.0.0.0 Build 3108-

Transcriber/s: Paolo M. Ramirez


Formatting: Paolo M. Ramirez D L S H S I M e d i c i n e B a t c h 2 0 1 6 | 5 of 5
Editor/s: Jan Cynric Cacao & Leanne Gale Ramos
ORTHOPEDIC RADIOLOGY
Dr. Frederick Hipol Jr., MD, FPOA, FPCR, FCT-MRISP
August 11, 2014; 1:00 – 2:30 PM
Orthopedics

LECTURE OUTLINE:
 Choice of imaging modality  Myelography
 Conventional Radiography  Diskography
 Scanogram  Ultrasound
 Digital Radiography  Scintigraphy
 Computed Tomography  Factors that affect choice of modality
 CT with contrast  Proper sequencing of procedures
 Arthography  Role of the orthopaedic radiologist
 Tenography & Bursography  Musculoskeletal imaging of specific body parts
 Angiography

CHOICE OF IMAGING MODALITY


 What modality should I use for this particular problem?
 Most of the time, the choice of imaging technique is dictated by the type of suspected abnormality
 Restrictions may be imposed by the needs of individual patients
 Plain film radiograph should be available for comparison

*TAKEN FROM THE HANDOUT PROVIDED


IMAGING ADVANTAGES / INDICATIONS DISADVANTAGES
MODALITY
Plain  Most commonly used imaging modality  Radiation exposure
Radiography  Relatively inexpensive  Not effective in soft-tissue imaging
 Real-time radiographic imaging, or because of poor contrast resolution
fluoroscopy, allows instantaneous  Magnification of the images. Measurement
feedback on stress radiographs, “standards” can be placed with the object
angiography, and orthopedic interventions to allow for determination of
magnification.
 Although most medical x-ray beams do not
pose a risk to a fetus, there is a small
possibility that serious illness and
developmental problems can occur. The
actual risk depends on the type of imaging
study and the trimester of pregnancy.
CT Scan  Tomographic nature of the images with  Much higher radiation exposure than plain
high contrast resolution radiographs
 Images are processed digitally; images in  Subject to motion artifacts because each
plane other than the one imaged can be slice can take acquisition time up to 1 sec.
reconstructed to give a different  There is artifact with metal objects and it is
perspective of the object/tissue of interest difficult to image around metal prosthesis.
 Direct measurements can be performed on  Scans have weight limit because patients
the scans because there is no need to lie on a scanner table
magnification  Contraindicated for pregnant patients,
 Can be combined with arthography or except in life-threatening circumstances
myelography to evaluate specific joint or  Note:
spinal abnormalities CT ARTIFACTS: There are many CT
 Used for injections and biopsy to allow artifacts that can occur for various
precise location of structures reasons. Knowledge of these artifacts
 Provides better delineation of bone is important because they can mimic
structures than MRI pathology (e.g. partial volume
artefact) or can degrade image
quality to non-diagnostic levels.
MRI  Superior images of soft tissues, such as  Motion blurring and metal artifacts are
ligaments, tendons, fibrocartilage, poorly tolerated.
cartilage, muscle, bone marrow and fat  The examination time is much longer than
 Has an advantage over plain radiography CT scans: patients need to remain still
on obtaining tomographic images of the throughout the scanning process.
object of interest.  Sedation is often needed for pediatric

Transcriber/s: Paolo Ramirez, Chrismark Reyes


Formatting: Paolo Ramirez D L S H S I M e d i c i n e B a t c h 2 0 1 6 | 1 of 11
Editor/s: Leanne Gale Ramos, Jan Cynric Cacao
 More effective than CT at detecting patients younger than age 7 years.
changes in intensity within the bone  Metal screws, pellets, prostheses and
marrow to diagnose osteomyelitis, foreign bodies can produce significant
malignancy, contusions, occult fractures artifact obscuring anatomic structures.
and stress fractures Metal suppression sequences can be used
 MRI contrast (gadolinium) has less allergic but with loss of resolution.
reaction than iodine-based media  Although MRI does not use radiation, the
 There is no radiation exposure to the effect of radiofrequency and magnetic field
patient on the fetus is unknown. It is usually
recommended that a pregnant woman not
have an MRI.
Ultrasound  Noninvasive at the frequencies used for  Image quality and interpretation depend
diagnostic imaging. on the experience of the ultrasonographer
 Commonly used in the imaging of children and radiologists
and pregnant women  Cannot detect image inside bone; bone
 Shows non-ossified structures, such as cortex reflects almost all of the sound
femoral head, to diagnose hip dysplasia waves.
and dislocation.  Internal joint structures are not well
 Equipment is portable and inexpensive visualized unless in a superficial location.
compared with MRI and CT equipment.
 Highly echogenic structures, such as a
foreign body that may not be visible on
radiographs, can be easily detected using
ultrasound
 Can be used for targeted therapy, such as
injections and ablations
 Useful for injections and aspirations of fluid
collections
 Provides dynamic assessment of structures
(ie tendon and nerve subluxation)
Nuclear  Scintigraphy allows imaging of metabolic  Lack of detail and spatial resolution.
Medicine activity.  Limited early sensitivity to detect acute
 Most metabolic processes involving bone fractures in patients with slow bone
have slow metabolic activity compared metabolism; it may take several days for
with soft-tissue organs, such as the kidney the bone scan to be positive to diagnose an
and the liver. Fortunately, most occult femoral neck fracture.
radioisotopes are relatively long lived.  Low sensitivity can occur with lytic bone
 White blood cell scintigraphy can be used lesions, such as multiple myeloma and
to diagnose osteomyelitis. some metastases.
 Scintigraphy can be used to diagnose  Avoid in breastfeeding mothers. Nuclear
metastasis, stress fracture, or occult agent can pass from the mother’s milk to
fractures. the child.

CONVENTIONAL RADIOGRAPHY
 Used for bone and joint disorders
 At least two views of the bone involved at 90 degree
angles to each other [orthogonal]
 In children, it is frequently necessary to obtain a
radiograph of the normal unaffected limb for
comparison [contralateral view]
 Usually the standard films comprise the
anteroposterior and lateral views.
 Occasionally oblique and special views are necessary.
 Higher spatial resolution than digital radiography

Transcriber/s: Paolo Ramirez, Chrismark Reyes


Formatting: Paolo Ramirez D L S H S I M e d i c i n e B a t c h 2 0 1 6 | 2 of 11
Editor/s: Leanne Gale Ramos, Jan Cynric Cacao
NOTES: (*This was not mentioned in the lecture, but for learning purposes and for future references, this might come in handy )
X-Ray Physics:
 Cortex, trabecullae is WHITE
 Medulla, intertrabecular space is GRAY
 The endosteum lines the inner wall of the cortex but because it is microscopic, it is not seen on x-ray
 The periosteum lines the outer wall of the cortex and this is also not normally seen
 Skin is GRAY, muscles are GRAY, and fat tissue is BLACK
 These different shades of gray and black and white are brought about by different absorption of x-ray by these tissues

STRESS VIEWS DIGITAL RADIOGRAPHY


 Stress views are special x-ray views in which  Digital image data is readily manipulated to produce
radiograph of the part examined is done after applying alternative renderings
requisite stress  Contrast and brightness optimization
 Important in evaluating ligamentous tears and joint  Digital subtraction radiography
stability (note area of tenderness to detect area  Vascular
affected)  Arthrography
 Abduction stress [MCL injury]  Advantages: Efficient storage, retrieval and
 Inversion transmission
 Anterior drawer [Anterior tibiofibular joint  Commonly used to improve diagnostic efficiency and
attachement] the quality of images

*Medial Fibular Calcaneal ligament BIGGEST NOTES:


The scientific unit of the measurement of radiation dose,
commonly referred to as the “effective dose”, is the
millisievert (mSv)

COMPUTED TOMOGRAPHY
 X-ray source, detectors and a computer data
processing system
 Circular scanning gantry, table, x-ray generator,
computerized data processing unit
SCANOGRAM  CT number (Hounsfields units)
 The most widely used method Water: 0HU
for Limb-Length measurement Air: -1000HU
 Motorized radiographic Bone/Fat: 1000HU
tube can traverse the whole  Trauma
length of a long film  Define the presence and extent of fracture or
 3 separate exposures over dislocation
hip joints, knees and ankles  Evaluate various intraarticular abnormalities
 Also known as  Evaluate the adjacent soft tissues
Orthoroengenogram  Detection of small bony fragments displaced
into joints
 Detection of small displaced fragments of
fractures vertebral body

Transcriber/s: Paolo Ramirez, Chrismark Reyes


Formatting: Paolo Ramirez D L S H S I M e d i c i n e B a t c h 2 0 1 6 | 3 of 11
Editor/s: Leanne Gale Ramos, Jan Cynric Cacao
 Assessment of concomitant injury to the cord or
thecal sac

BIGGEST NOTES:
Types of CT scans (Were not discussed but were in past exams)
 Transpiral/Hypocyloidal
 Superior over conventional radiography for
visualization of subtle fractures; delineating the
extent of the fracture line, evaluating healing
process, evaluating non-union
 Helpful in evaluating tumor and tumor-like
lesions
COMPUTED TOMOGRAPHY WITH CONTRAST
 Increases the distance of excursion of the tube
 Can aid in identifying a
and create a varying angle of projection of the x –
ray beam during exposure suspected soft tissue mass
 Produce even greater burring outside the field of when initial CT’s are
interest and yield the sharpest focal plane images unremarkable
 Helical/Spiral  Assess the vascularity
 Permits three – dimensional reconstruction for
analysis of regions with complex anatomical
structures
 Face, pelvis, vertebral column, wrist, foot and
ankle
 Plastic models of the area of interest , thus
facilitate operative planning and allow reversal of
complex reconstructive procedures ARTHOGRAPHY
 A medical imaging to evaluation conditions of the
joints, which either be direct or indirect with the
ntroduction of contrast agent
 Iodide solution
 Air
 Combination
 Direct – gadolinium-containing solution
injected in the JOINT
 Indirect – gadolinium-containing solution
injected INTRAVENOUSLY
 Ligamentous tears, adhesive capsulitis,
osteochondritis dissecans, osteochondral bodies,
subtle abnormalities in the articular cartilage
 Few absolute contraindications [Renal insufficiency,
dialysis, and GFR <30]
 Commonly used to diagnose labral tears in the
shoulder and the hip joints, triangular fibrocartilage
and ligament tears of the wrist, collateral ligament
evaluation in the elbow, and post-operative evaluation
of repaired meniscus
 Nice to know : When you hear the prefix “arthro” it
means = joint; as in arthropathy, and arthroscopic.
Before a vowel, it becomes arthr-, as in arthalgia, or
arthritis. It comes from the greek word = ‘arthron’ for
joint!

TENOGRAPHY AND BURSOGRAPHY


 Relatively limited clinical use
*Encroachment of the spinal cord  Mainly to evaluate traumatic or inflammatory
conditions
 Bone and soft-tissue tumors
 Rarely helpful in making a specific diagnosis
 Precisely evaluate the extent of the bone lesion
 Demonstrate break through the cortex and
involvement of surrounding soft tissues
 Helpful in delineating a tumor in bones having
complex anatomic structures
 Determine the extent and spread of a tumor in
the bone
 Demonstrate intra-osseus extension of a tumor
or extra-osseus involvement of soft tissues
 Guided aspiration or biopsy

Transcriber/s: Paolo Ramirez, Chrismark Reyes


Formatting: Paolo Ramirez D L S H S I M e d i c i n e B a t c h 2 0 1 6 | 4 of 11
Editor/s: Leanne Gale Ramos, Jan Cynric Cacao
DISCOGRAPHY
 A discography, or discogram is an interventional
diagnostic imaging test that helps determine whether
a specific interveterbral disc may be the source of back
pain
 It involves injection of contrast material into the
nucleus pulposus

ANGIOGRAPHY
 A type of X-ray used to examine blood vessels
 The images created during an angiography are called
angiograms
 Blood vessels don’t show up clearly on ordinary x-rays, ULTRASOUND
so a special dye (or contrast material) is injected into
 Interaction of propagated sound waves with tissue
the area being examined
interfaces with the body
 The dye highlights the blood vessels as it moves
 Whenever the directed pushing of sound waves
through them and appears white on the angiogram
encounters an interface between tissues of different
 Arteriography
acoustic impedance, reflection or refraction occurs
 Venography
 Advantages:
 Relatively inexpensive
 Allows comparisons with the opposite normal
side
 No ionizing radiation
 Can be performed at bedside or in the
operating room
 Can visualize: rotator cuff muscles [e.g. if there is
presence of impingement), tendons, soft tissue tumors
and infant hip

MYELOGRAPHY
 Myelography is an imaging examination that involves
the introduction of spinal needle into the spinal canal
*Femoral head assessment
and injection of water-soluble contrast agents
injected into the subarachnoid space, mixing freely
with the CSF to produce a column of opacified fluid

Transcriber/s: Paolo Ramirez, Chrismark Reyes


Formatting: Paolo Ramirez D L S H S I M e d i c i n e B a t c h 2 0 1 6 | 5 of 11
Editor/s: Leanne Gale Ramos, Jan Cynric Cacao
SCINTIGRAPHY
 Bone scans
 Detects the distribution in the body of a radioactive
agent introduced into the vascular system [TC 99m]
 Interaction of the gamma rays in the patient’s body
with sodium iodide crystals in the head of the camera
 “Metabolic picture”
 Indicator of mineral turnover
 Detects: Fibrous dysplasia, langerhans cell
histiocytosis, and metastatic cancer

*Las Vegas conference story by Dr. Hipol: infusion of water to


the common peroneal nerve [hydrodissection] which is only
done with UTZ, to treat the patient’s noted unnecessary
movement of the leg

FACTORS THAT WOULD AFFECT CHOICE


OF MODALITY
 Equipment availability
THE NERVES THAT CAN BE TREATED WITH  Physician expertise
ULTRASOUND [High-end UTZ]  Cost
 Dorsal scapular nerve  Individual patient restrictions
 Intercostal nerve [Pacemaker + MRI = Nope]
 Lateral femoral cutaneous nerve
 Ilioinguinal nerve PROPER SEQUENCING OF PROCEDURES
 Iliohypogastric nerve  Depends to a greater extent on the pertinent clinical
 Obturator nerve information provided by the referring physician
 Lumbar medial branch
 Sciatic nerve REACHING A CORRECT DIAGNOSIS DOES
 Posterior curtaneous nerve of thigh NOT END THE PROCESS OF RADIOLOGIC
 Superior cluneal nerve
INVESTIGATION
 Pudendal nerve
 Identification of distinguishing features of a particular
 Saphenous nerve
disorder
 Common fibular nerve
 Crucial features of a tumor must be identified:
 Malignant transformation
 Extent
 Localization

Transcriber/s: Paolo Ramirez, Chrismark Reyes


Formatting: Paolo Ramirez D L S H S I M e d i c i n e B a t c h 2 0 1 6 | 6 of 11
Editor/s: Leanne Gale Ramos, Jan Cynric Cacao
 Distribution  Ways of positioning the patient
 Progress of treatment and healing  Size, shape, configuration, and density of a lesion
 Complications  ALARA principle (“As Low As Reasonably
Achievable”) of dosing for pregnant and children
ROLE OF THE ORTHOPEDIC RADIOLOGIST  CT delivers the highest amount of radiation dose
 Diagnose a disorder using standard (5-15 mSv versus the 0.1-2.0 mSv for plain
projections/conventional radiography before radiography)
employing more sophisticated modalities  When obtaining radiologic studies, sensitive organs
 Perform examination in the prior sequence and to such as gonads should be shielded.
know what should be done next “You gotta always protect the Mcnuggets” –
 Demonstrate the determining radiologic features of a Cheeseburger Eddie 
known disorder, the distribution of alesion and its
location
 Occasionally assume a more active role in therapeutic
management

REMEMBER
 Clinical data are important to the radiologist in
correctly interpreting a radiograph
 Where to begin and what to do next
 Until conventional radiography fails to provide the
radiographiv findings necessary for correct diagnosis
and precise evaluation of an abnormality, the
radiologist need not turn to more costly procedures

*TAKEN FROM THE HANDOUT

MUSCULOSKELETAL IMAGING OF SPECIFIC BODY PARTS


SHOULDER  Ultrasound – can allow dynamic evaluation of
rotator cuff injuries and biceps mobility; however,
FRACTURES resolution is lower an evaluation is more localized.
 AP and axillary lateral or scapula Y views –  Goutalier method – evaluating the quality of the
commonly used to rule out shoulder fractures rotator cuff muscles
 Clavicle – AP and serendipity view (45 degree  CT scan – estimate the amount of fat present in
tilted view) the muscle
 Bony Bankart injury – West point axillary lateral  Modified Goutalier classification system –
view grades the muscle of the rotator cuff on the
 Hill-Sachs lesions – Stryker notch view amount of fat present relative to muscle volume
 CT scan is used to evaluate the complexity of the  ABER (abduction and external rotation) sequence
fracture pattern and any comminution of proximal – evaluate the anterior inferior labral structures
humerus and scapular fractures and also the posterosuperior rotator cuff tendons
when in contact with the posterior labrum.
ARTHRITIS
 Shoulder arthritis – AP view of glenohumeral SHOULDER INSTABILITY
joint or Grashey view and axillary lateral views of  Plain radiograph – evaluates bony deformities,
the shoulder to evaluate amount of joint space such as the Hill-Sachs lesion and, anterior inferior
narrowing glenoid bone loss.
 Primary osteoarthritis of the shoulder
 AP view of glenohumeral joint has large NEUROPATHY
inferior osteophytes  Ganglion cysts in the shoulder can lead to nerve
 Axillary lateral view has posterior wear compression and muscle weakness
 MRI is used for the evaluation of rotator cuff tendon  Compression of the nerve at the:
for surgical planning - Suprascapular notch can lead to weakness in
 Ultrasound guided glenohumeral joint infection both supraspinatus and infraspinatus muscles
- Spinoglenoid notch can lead to weakness in
ROTATOR CUFF TEARS the infraspinatus muscle only
 MRI is the method of choice for Rotator Cuff  Most common compressed nerve in the shoulder
injuries. It can confirm the diagnosis of anterior region: Suprascapular nerve
inferior labral tears and internal impingement  Direct Magnetic Resonance arthography –
syndrome. evaluate communication between cyst and the
joint

Transcriber/s: Paolo Ramirez, Chrismark Reyes


Formatting: Paolo Ramirez D L S H S I M e d i c i n e B a t c h 2 0 1 6 | 7 of 11
Editor/s: Leanne Gale Ramos, Jan Cynric Cacao
 Indirect Magnetic Resonance arthography –  Evaluation of the integrity of the articular cartilage
evaluate presence of dilated vein or aneurysm within the wrist remains a challenge.
that can lead to nerve compression  MDCT artrhography proved more sensitive than
1.5T magnetic resonance arthrography
ELBOW Tendinopathy and Neuropathy
 MRI
FRACTURES  Commonly used to evaluate tendinopathy
 AP extended and lateral flexion view – standard of the wrist (De Quervain tenosynovitis or
radiograph views “Baby Wrist”, rheumatoid arthritis,
 Oblique lateral view – for trauma imaging of the tuberculosis)
radial head  Median nerve pathology in Carpal Tunnel
 Sail sign – lifting of the inferior edge of the fat Syndrome and Ulnar nerve pathology in
contained within the coronoid fossa due to the Guyon Canal
presence of a joint effusion or hemarthrosis  Sonography – extensor carpi ulnaris instability and
 Posterior fat pad sign – displacement of the fat impingement by volar plate fixation screws on
contained within the olecranon fossa of the distal extensor tendons
humerus allows this fat to be visualized posterior
to the distal humeral condyles Tumor
 MRI – can detect occult fractures and bone bruises  MRI & Ultrasonography – Solid masses
not seen in standard radiographs
 ST with multiplanar reformations – used for Hip
treatment planning for complex fractures of the
distal humerus and the radial head Fracture
 AP and Lateral view – hip fractures
ARTHRITIS  MRI
 CT – can better detect intra-articular bodies and  Osteonecrosis or muscle injury
osteophytes that conventional radiographs  Equally accurate to bone scans in fracture
 MRI – useful in evaluating for intraarticular bodies detection but provides additional diagnoses
and has the advantage of disclosing soft-tissue
abnormalities that may also produce locking, such Osteonecrosis
as a thickened synovial fringe or synovial plicae  MRI – double line sign
 Detects abnormalities of the proximal femur,
INSTABILITY AND TENDINOPATHY including transient marrow edema of the femoral
 MRI head and subchondral insufficiency fractures
 Reliable tool in assessment of the ulnar  Distinguishing osteonecrosis from a subchondral
collateral ligament complex, the radial fracture can be done by observing that in
collateral ligament, the annular ligament, and osteonecrosis, the double line forms a
the lateral ulnar collateral ligament continuous ring
 Can evaluate biceps or triceps tendon tears or
medial or lateral epicondylitis Developmental Dysplasia of the Hip
 Can also be used to screen Little Leaguer’s  Dynamic and Static Ultrasound – examination of
Elbow and Tennis Elbow the hip for developmental dysplasia in the
pediatric population
NEUROPATHY  Plain film evaluation – assessment of the hips in
 MRI - can evaluate Cubital Tunnel Syndrome AP projection with analysis of Shenton line,
acetabular angles, the position of the genreiner
WRIST and Perkins lines

Fractures Arthritis
 PA, oblique and lateral views – standard plain  Plain films – joint space narrowing, osteophytes
film evaluation at the femoral head neck junction or acetabular
 Ulnar-deviated PA view – specific evaluation of rim, subchondral cysts, subchondral sclerosis and
occult fractures of the scaphoid eventually loss of femoral head sphericity from
 Semisupinated oblique view – occult radial subchondral collapse
fractures  MRI – often discloses joint effusion, diffuse bone
 Multidetector CT – often used for complex distal marrow edema in the femoral head and neck,
radius fractures femoral head flattening, and cyst-like subchondral
defects
Instability  Magnetic resonance arthrography or specialized
 MDCT arthrography and Magnetic Resonance sequences may be helpful in assessing articular
Arthrography – for evaluation of the intrinsic cartilage damage in the hip
scapholunate and lunotriquetrial ligaments of the
wrist and evaluation of the triangular Femoral Acetabular Impingement and Labral Tear
fibrocartilage.  Presence of a mismatch between the femoral head
 Plain film Lateral view – distal radioulnar joint and neck with acetabulum resulting in abutment of
the proximal femur with the acetabulum is
Arthritis

Transcriber/s: Paolo Ramirez, Chrismark Reyes


Formatting: Paolo Ramirez D L S H S I M e d i c i n e B a t c h 2 0 1 6 | 8 of 11
Editor/s: Leanne Gale Ramos, Jan Cynric Cacao
termed Femoral Acetabular Impingement,  Merchant/skyline view – evaluate patella-
which may lead to premature osteoarthritis femoral joint arthritis; important to obtain these
 Plain Films views at accurate knee flexion angles
 Fibrocystic changes are seen on plain films in  MRI – evaluates focal cartilage injuries as well as
the anterosuperior femoral neck in 33% of the the quality of the cartilage
patients with FAI in one study  New image modalities that allow to directly probe
 Rim ossification in the pincer type FAI and the the biochemical composition of cartilage
pistol grip deformity of the femoral neck in  Delayed gadolinium-enhanced MRI of
the cam type cartilage (dGEMRIC)
 MRI – can assess for abnormal epiphyseal torsion  Gadolinium is injected IV and allowed
angles, labral tears, cartilage delamination in this to diffuse to the joint
entity  Amount of diffusion if inversely
proportional to the amount of
Total Hip Arthroplasty proteoglycan in the articular cartilage
 Plain films – levaluate lucency between  T2 mapping
prosthesis and bone. May also disclose stress  Measures organization of cartilage
shielding, insufficiency fractures near the femoral matrix and water content
stem, component wear, component dislocation of  T1rho mapping
the liner or dislocation.  Measures amount of proteoglycan
 Arthrography plus aspiration and synovial biopsy within the cartilage
to exclude infection can also disclose sinus tract or
extravasation below the intertrochanteric line Total Knee Replacement
 Nuclear scintigraphy with indium-labeled white  Plain radiographs are used
blood cells, immunoglobulin G scanning, or  Loosening  change in the position of the
gallium scanning can also help in the diagnosis of prosthesis
peroprosthetic infection  Diagnosis of component loosening not
as difficult as total hip arthroplasty
Infection because (a) more superficial joint; and
 Aspiration and synovial biopsy under either (b) patient complain of localized pain
fluoroscopic guidance or ultrasonography – to along the loose prosthesis
diagnose septic hip (UTZ and CT not reliable)  Infected knee prosthesis  similar to total
hip arthroplasty
Muscles and Tendons
 MRI – abductor gluteus medius, gluteus minimus, Ligament and Meniscus injuries
hip adductor and rectus tendinopathies, bursitis,  MRI is the main modality used, especially for
groin hernias, aponeurosis avulsions, athletic anterior cruciate ligament (ACL) injuries and
pubalgia, stress fractures, ischiofemoral meniscus injuries
impingement, and snapping hip  Radiographic abnormalities does not entail
symptomatic patients
KNEE  Meniscus tears are present in advancing age
 Surgery on these will not be beneficial
Fractures compared to placebo treatment
 Commonly evaluated using orthogonal views of  MRI is not as accurate when compared to clinical
the tibiofemoral joint and patellar joint examinations in the diagnosis of chronic posterior
 AP and lateral views to rule out cruciate ligament (PCL) and posterolateral corner
 Merchant/skyline view – rule out dislocations or injuries
evaluate longitudinal fractures of the patella  Chronic PCL injuries  continuity of fibers
 Oblique view – evaluate out of plane fractures in MRI but have clinical instability
 CT Scan with coronal and sagittal reformations  Posterolateral corner injuries  needs
– comminuted and displaced tibial plateau dedicated oblique views to diagnose
fractures or distal femoral fractures
 Three-dimensional reconstruction – better FOOT AND ANKLE
visualization of depressed articular fragments and  Standard radiographs for the ankle
identify the fracture plane  AP
 MRI – evaluate soft-tissue injuries (meniscus or  Mortise
ligament injuries)  Lateral view
 Standard radiographs for the foot
Arthritis  AP
 Important to perform weight bearing views to  Oblique
allow an indirect views of evaluating cartilage  Lateral
thickness  Other common views
 Bilateral flexion weight-bearing PA view  Harris Beath axial views  calcaneus
(Rosenberg view) – posterior weight bearing  Gravity stress views  integrity of ankle
surface of the tibia ligaments
 Flexed PA view – evaluates joint space  Weight bearing views  acquired or
congenital deformities and interrogation of
the Lisfranc joint

Transcriber/s: Paolo Ramirez, Chrismark Reyes


Formatting: Paolo Ramirez D L S H S I M e d i c i n e B a t c h 2 0 1 6 | 9 of 11
Editor/s: Leanne Gale Ramos, Jan Cynric Cacao
 Foreign bodies
Fractures  Nuclear scintigraphy with combined triple-
 Ottawa rules  used in acute trauma setting phase Tc-99m bone
 Tibiotalar joint effusion is seen as a  used to rule out infection
teardrop sign in the anterior aspect of the Tagged white blood cell scans
joint on lateral radiograph  occult  used to rule out infection
fractures  MRI
 Occult fracture sites: talar dome, tibial  T1-weighted images  loss of normal fat
overhang, posterior rims of the distal signal
tibia (Tillaux fracture), medial and  Fat saturated T2-weighted images  high
lateral malleoli, tibial plafond and signal from bone marrow edema
anterior process of the calcaneus  Findings:
 MRI  assess occult subchondral lesions  Uninfected neuropathic bone 
confined to the immediate subcortical
Instability bone in a thin rim pattern
 Direct magnetic resonance arthography or  Infected bone  confluent deep
indirect arthrography marrow signal abscesses

Impingement Tumors
 MRI  MRI is used to specifically diagnose:
 Soft tissue masses in the anterolateral gutter  Ganglion cysts
 arthroscopically confirmed anterolateral  Plantar fibromas
impingement  Morton neuromas
 Sonographic arthography and MDCT  Lipomas
arthography  anterolateral  Sonography can also be used
impingement
Neuropathy
Tenosynovitis  MRI is used to examine specific nerve
 Confirmed via MRI if present with the following: neuropathies
 Signal abnormalities within the tendons  Neuropathies affecting the branches of the
 Fluid in the tendon sheaths posterior tibial nerve
 Secondary edema in subjacent bones  Medial and lateral nerves of the tarsal
 MRI  used to evaluate tendon abnormalities tunnel
because of its superior tissue contrast  Baxter neuropathy  inferior
 Sonography  used to evaluate tendon calcaneal nerve
subluxation, tears or impingement  Jogger’s foot  entrapment of the
medial plantar nerve by the abductor
Infection hallucis muscle
 Plain films  Master knot of Henry (chiasma
 Suspected osteomyelitis plantaris)  where the flexor hallucis
 Periostitis longus and flexor digitorum longus
 Focal osteopenia cross
 Cortical erosion  MRI and ultrasound can be used to examine the
 Soft tissue edema sural and peroneal nerve
 Soft-tissue gas

PAST-E QUESTIONS (BATCH 2014)

1. The most sensitive to detect osteonecrosis is: 4. The type of CT scan which can delineate tumors
a. UTZ in bones having complex anatomical structures
b. CT is:
c. MRI a. Helical
d. Conventional radiograph b. Transpiral
2. If a patient is suspected to be suffering from a c. Circular
partial tear of the rotator cuff, the most d. Linear
effective modality to request is: 5. The best procedure to evaluate kinematics of
a. UTZ joints is:
b. CT a. CT
c. MRI b. MRI
d. Conventional radiograph c. Fluroscopy
3. To delineate the extent of a fracture line, the d. UTZ
best type of CT to request is: 6. One of the following will show a decreased
a. Helical uptake in a radionuclide bone scan:
b. Transpiral a. Fracture
c. Circular b. Neoplasm
d. Linear c. Focus of osteomyelitis
d. Early stage of osteonecrosis

Transcriber/s: Paolo Ramirez, Chrismark Reyes


Formatting: Paolo Ramirez D L S H S I M e d i c i n e B a t c h 2 0 1 6 | 10 of 11
Editor/s: Leanne Gale Ramos, Jan Cynric Cacao
7. In an MRI, one of the following will show high 13. Choice of radiologic technique will depend on
signal intensity on T2 W sequence: the following except:
a. Hyaline cartilage a. Expertise
b. Cortical bone (low signal) b. Clinical impression
c. Tendon (low signa) c. Patient’s choice
d. Muscle (intermediate signal) d. Cost
8. The best view to request when a patient is 14. When conventional radiograph showed no
suspected to have a tear of the medial collateral evidence of tissue extension, the best modality
is: to detect subtle cortical erosion and determine
a. AP intraosseus extension is:
b. AP-lateral a. UTZ
c. Stress view b. CT
d. Weight bearing view c. MRI
9. For a trauma to an extremity, a digital d. Nuclear
radiograph can evaluate: 15. A radiogpraph of the normal, unaffected limb is
a. Arterial occlusion usually requested among:
b. AV fistulas a. Elderlies
c. Transaction of arteries b. Adolescents
d. All of the above c. Children
10. A very safe and effective modality for pregnant d. A and B only
women to evaluate dysplasia of the hip is: 16. The initial view/s of a conventional radiograph
a. Plain radiograph is/are:
b. UTZ a. Oblique
c. CT b. Antero-posterior (AP)
d. MRI c. Lateral
11. . To demonstrate bone metastases of the entire d. B and C only
skeleton at once, the best modality is: 17. In T1 W sequence in MRI, synovial fluid and
a. CT cerebro-spinal fluid appears:
b. MRI a. Dark or hypointense
c. Skeletal scintigraphy b. Bright or hyperintense
d. UTZ c. Intermediate
12. . Initial diagnostic procedure for any bone d. B and C
tumor is: 18. The cost effective way to demonstrate a Baker’s
a. UTZ cyst is:
b. Plain film a. X-ray
c. Nuclear b. UTZ
d. CT c. CT
d. MRI
19. If a patient has rheumatoid arthritis and he has
carpal tunnel syndrome, the best way to
demonstrate thickening of the synovium is:
a. X-ray
b. Scintigraphy
c. MRI
d. CT

-END-

TRANSCRIPTION DETAILS
BASIS Latest PPT & Handout RECORDINGS + NOTES + DEVIATIONS 8-10% CREDITS Xmark’s recordings
We have incorporated the handout of Orthopedic Imaging provided by Dr. Hipol para cool 
Orangefont – handout based
REMARKS Please also study the pictures in the powerpoint
Editor’s note: Don’t just rely on the past-e. Understand the topic. READ and DIGEST. Apply your basic knowledge on radiography to this
lecture.
-BATCH 2016 Transcribers’ Guild Transcriptions. Version 1.0.0.0.0 Build 3109-

Transcriber/s: Paolo Ramirez, Chrismark Reyes


Formatting: Paolo Ramirez D L S H S I M e d i c i n e B a t c h 2 0 1 6 | 11 of 11
Editor/s: Leanne Gale Ramos, Jan Cynric Cacao
HAND INJURIES
Frederick Hipol Jr., MD, FPOA, FPCR, FCT-MRISP
August 19, 2014; 1:00 – 2:00 PM
Orthopedics

OUTLINE
 Anatomy of the Hand
 Anatomy of the Fingertip
 Fractures and dislocations of the hand
 Distal Interphalangeal Joint Injuries
 Middle Phalanx Fractures
 Proximal Phalanx Fractures
 Proximal Interphalangeal Joint fractures
 Metacarpophalangeal Joint Fractures
 Metacarpal Fractures Sensory nerve distribution of the hand
 Thumb fractures
 IP dislocations
 Amputations
 Dorsal extensor compartments of the hand
 Wrist Fractures

ANATOMY

Anatomy of the finger

 Median nerve – majorly supplies the hand (it


enters the hand via the carpal tunnel, located
centrally) to supply motor function to the thenar
aspect of the hand and lateral 2 lumbrical
muscles; it also gives sensory innervation to
lateral part of the palm, and the lateral 3 and a
half fingers of the palmar surface of the hand
 Radial nerve – in the forearm, the deep branch
innervates the muscles in the posterior
compartment; these muscles also extend the wrist
and finger joints, meanwhile, the superficial
branch of the radial nerve provides sensory
innervation to much of the back of the hand,
including the web of skin between the thumb and
index finger
 Ulnar nerve – it enters the hand via the ulnar
canal/guyon’s canal, to supply the intrinsic
muscles (hypothenar muscles – a group of
muscles associated with the little finger), flexor Extensor mechanism
carpi ulnaris, and medial half of flexor digitorum
profundus; the sensory function involves
innervation to the anterior and posterior surfaces
of the medial one and half fingers, and the
associated palm area

Transcriber/s: Jan Cynric Cacao


Formatting: Jan Cynric Cacao D L S H S I M e d i c i n e B a t c h 2 0 1 6 | 1 of 9
Editor/s: Paolo Ramirez, Leanne Gale Ramos
 The extensor mechanism of the hand is very NOTE:
complex wherein the interossei contributes to the Important things to remember regarding hand injuries:
function of the extensor mechanism  How a particular fracture is displaced
 The interossei proximally, the volar and  How to detect particular bone involved
dorsal interossei and the lumbricals
contributes to the dorsal apponeurosis of
the MCP joints HAND FRACTURES & DISLOCATIONS
 In the PIP, the lumbricals is attached to the
lateral band. PHALANGEAL FRACTURES
 Distally, the lateral band contributes to  Mechanism of Injury: Distal phalanx fractures
the dorsal mechanism at the distal most commonly occur after a crush injury → tuft
interphalangeal joint and extends the fracture
DIP joint  Associated with a nail bed injury and are
 Volarly, the profundus is attached the DIP while theoretically considered open fractures
the superficialis is attached to the volar aspect of  Goals of treatment
the middle phalanx  Achieve adequate length and alignment
 Initiate early mobilization
NOTE: knowing the anatomy, you’ll be able to  Realize maximum hand function
understand how to treat injuries of the hand  Non-displaced, stable fractures can be buddy-
Review of terms: taped and patients sent to occupational therapy
“Volar” - relating to the palm of the hand or the sole of  Displaced fractures can be reduced
the foot.  If reduction is successful, displaced fractures
can be splinted for 4 weeks, after which the
splint is discontinued and therapy started
PRIORITIES
 Unstable injuries with mal-alignment may need
 Always think of the function of the hand
surgical intervention
 Generally, you should think of what is the
 The objective is to achieve reduction and
best way possible to treat the patient
stable fixation with the least invasive means
 Must be able to test sensory, motor and tendon
to minimize adhesions and stiffness
function
 After closed reduction, manually or with
 Two-point discrimination for sensory of the
tenaculum-type bone clamps, K-wires can
distal part of the finger
be placed in a variety of patterns
 Should be able to draw a cross-section of wrist
 If closed techniques fail, a small incision can be
 If tendon, nerve or vascular injury, consult
made to mobilize the fracture

ANATOMY OF THE FINGERTIP DISTAL INTERPHALANGEAL JOINT


INJURIES
MALLET FINGER
 Mechanism of Injury: Volar displacement of the
fingertip
 Stretched or ruptured distal attachment of
the tendon (extensor pollicis longus
tendon)
 Dorsal lip fracture of the attachment of the
extensor tendon
 Consists of the nail, nail bed and proximally, the  It can be an epiphyseal fracture in a young child
germinal matrix  Injuries of the extensor mechanism of the DIP
 If you have a fingertip injury (e.g. a finger that was joint that can involve the terminal tendon (Doyle
impinged on a door), it may present as having Ia) or have a bony fragment (Doyle Ib)
hematoma underneath the nail bed  Most can be treated with splinting in full
 If the hematoma (subungual) is 50%, most extension for 6 to 8 weeks with no flexion of the
likely the nail bed is not destroyed. It can be DIP joint
drained by placing a needle into the nail  Treatment is started immediately but good
(irrigation and debridement) outcomes have been reported even with delayed
 After debridement and repair, the nail or treatment
foil be placed underneath the nail fold to  Indications for Surgery
keep it open  Open injuries
 If the hematoma is more than 50%, most  Fractures that involve more than 30% of the
likely the nail bed is destroyed (significant joint surface
nail bed laceration), hence, a repair of the  Joint subluxation
nail bed is necessary  Surgical options include
 If there is significant displacement of a  Dorsal block splinting (DIP joint is flexed
transverse fracture, reduction and and the pin is placed in the head of the
Kirschner wire (K-wire) pinning can be middle phalanx, then the joint is extended)
performed  Internal fixation with a screw or a plate

Transcriber/s: Jan Cynric Cacao


Formatting: Jan Cynric Cacao D L S H S I M e d i c i n e B a t c h 2 0 1 6 | 2 of 9
Editor/s: Paolo Ramirez, Leanne Gale Ramos
 If the test is positive, full-time PIP joint
splinting in full extension is initiated for 6 to
8 weeks along with flexion exercise of the
DIP joint

VOLAR FRACTURE-DISLOCATIONS
 Mechanism of Injury: Results in fracture of the
dorsal, proximal aspect of the middle phalanx
 Fixation of the fracture may be performed with
percutaneous K-wires or internal fixation

DORSAL FRACTURE-DISLOCATIONS
JERSEY FINGER  Have varying degrees of involvement of the
 Mechanism of Injury: Hyperextension of the proximal volar aspect of the middle phalanx
attachment of the finger (DIP)  If the fracture is small volar plate avulsion fracture
 Rupture or avulsion of the flexor digitorum from the middle phalanx, then treatment is the
profundus (FDP) tendon same as for simple dislocation
 If 40% of the joint surface is involved, the dorsal
portion of the collateral ligaments may be
attached to the middle phalanx maintaining joint
stability
 Treated with dorsal block splinting for 3
weeks
 If more than 40% of the joint surface is involved,
most of the collateral ligaments will be attached
to the fracture fragment, rendering the joint
unstable
 Surgical intervention is needed

PILON FRACTURES
 Mechanism of Injury: Involve complete joint
involvement of the proximal middle phalanx
 Axial compressive forces cause central joint
MIDDLE PHALANX FRACTURES depression and dorsal and volar fragment
 The attachment of the tendon of the flexor displacement
digitorum superficialis is on the middle portion of  Surgical fixation is almost always necessary
the middle phalanx  Open reduction and internal fixation (ORIF) is
 Mechanism of Injury: challenging because the fracture fragments are
 If the fracture line is distal to the small and swelling and stiffness from the injury,
attachment, the superficialis will pull the the incision and the approach ensue
proximal fragment into a volar position
 If the fracture line is proximal to the
attachment, the superficialis tendon will pull
the distal portion portion and displace the
proximal portion dorsally

PROXIMAL PHALANX FRACTURES


 Mechanism of Injury: Fractures occurring in the
proximal phalanx will displace the segment
volarly due to the attachment of the lumbrical
muscle tendon to the base of the phalanx
ARTICULAR FRACTURES OF THE PROXIMAL
PHALANGEAL HEAD
PROXIMAL INTERPHALANGEAL JOINT
 Surgically treated if there is articular incongruity
FRACTURES of more than 1 mm finger mal-alignment or
 Challenging to treat because of the joint’s malrotation
proclivity to stiffness  Closed reduction and percutaneous pinning are
 Simple dorsal dislocations without fracture are attempted first
usually stable after reduction  ORIF is performed if close reduction is
 Splinting should be limited to a few days unsuccessful
duration
 Buddy taping can be used in lieu of splinting
 Early motion should be initiated with therapy
 Mild residual flexion contracture is common
 Volar dislocations need to be evaluated for central
slip injury with Elson test

Transcriber/s: Jan Cynric Cacao


Formatting: Jan Cynric Cacao D L S H S I M e d i c i n e B a t c h 2 0 1 6 | 3 of 9
Editor/s: Paolo Ramirez, Leanne Gale Ramos
BOUTONNIERE’S FINGER  Suture anchors are the contemporary
 Mechanism of Injury: There is rupture of the choice for fixation of the ligament to bone
attachment of the central tendinous slip of the  Laceration at the dorsal hood of the MCP without
extensor hood and is most common after trauma treatment of broad-spectrum antibiotics may lead
or in rheumatoid arthritis to a septic arthritis
 Results in volar subluxation or displacement
of the lateral band PRIMARY STABILIZERS OF THE THUMB MCP JOINT
 Swelling of the PIP joint after jabbing of the finger  Ulnar collateral ligament
to a hard surface, resulting in the extension of the  Radial collateral ligament
MCP and DIP joints and flexion of the PIP joint
(primary deformity) NOTE: injuries to the mentioned structures above can
 Clinically, palpation for tenderness will be be easily missed in an emergency.
appreciated at the dorsal aspect of the PIP joint
 Can be also diagnosed using the Nelson’s test
 Patient grabs the end of the table and asked GAMEKEEPER’S THUMB
to hyperextend the affected finger  Mechanism of Injury: Hyperabduction of the
 Can be also diagnosed using Haines-Zancolli test thumb causing fracture or rupture of the ulnar
 Positive if unable to passively flex the DIP collateral ligament at the base of the thumb
with an extended PIP  Ulnar collateral ligament is displaced dorsally
 Treatment is done by placing a volar splint  Diagnosed by x-ray and doing hyperabduction of
extending up to the DIP and is maintained for 3 the thumb and is compared to the other hand
weeks  If there is more than 30º angulation, then it
 If fracture is present, it is treated by placing a pin is positive of having a gamekeeper’s thumb
on the fracture site  Ultrasound can be also used in the diagnosis
 Widening of the joint can be appreciated
with the detachment of the ulnar collateral
ligament

METACARPOPHALANGEAL JOINT
FRACTURES
 Relatively uncommon
 Dislocation may be simple (closed reduction
possible) or complex (requires open treatment)
 Dorsal dislocations are more common than volar
dislocations
 Mechanism of Injury:
 Dorsal dislocations are caused by a
hyperextension injury
 Volar dislocations are caused by either a
hyperextension or hyperflexion injury
 Longitudinal traction can transform a simple
dislocation into a complex dislocation
 Irreducible dorsal dislocations are a result of
interposition of the volar plate in the joint or the
metacarpal head buttonholing between the flexor
tendons and the radial lumbrical
 Irreducible volar dislocations are caused by
interposed dorsal capsule, the distal insertion of
the volar plate and/or the collaterals
 Complete ulnar collateral ligament tears are
diagnosed by more than 30 to 35 degrees of the STENER’S LESION
thumb MCP joint angulation  The ulnar collateral ligament is detached and is
 Complete tears on the ulnar side commonly between the attachment of the abductor pollicis
have a Sterner lesion, where the distal aponeurosis
ligament is displaced superficial and  Treatment is surgical
proximal to the adductor aponeurosis  Torn ulnar collateral ligament stump comes to
 This rarely occurs on the radial side lie dorsal to the aponeurosis and is thus prevented
 Partial injuries can be splinted from healing to its anatomic insertion on the
 Complete tears are repaired surgically volar, ulnar base of the proximal phalanx

Transcriber/s: Jan Cynric Cacao


Formatting: Jan Cynric Cacao D L S H S I M e d i c i n e B a t c h 2 0 1 6 | 4 of 9
Editor/s: Paolo Ramirez, Leanne Gale Ramos
BIG NOTE: Invest in
quality handwraps
and boxing gloves if
you’re planning or
currently boxing to
prevent injuries of
your hands. And
learn proper
handwrapping 

METACARPAL FRACTURES
 The hand is consist of a radial post (thumb) and
two pillars, the central post and ulnar post
 The radial post and the ulnar post are very mobile
 Fractures that have wide angles are candidates for PSEUDOCLAWING
reduction  Mechanism of Injury: Too much angulation due
 If you have a fracture of the first metacarpal to a fracture at the neck of the metacarpal bone
bone, you are allowed to have a wide  Can result due to a Boxer’s Fracture
angulation, but you have to maintain the
length THUMB FRACTURES
 In the central post (2nd and 3rd metacarpal  Extra-articular fractures
bone), you treat them anatomically as  Usually transverse or oblique
possible  Most can be held by closed reduction and
 10º for the index finger (2nd casting
metacarpal bone)  Some unstable fractures require closed
 20º for the middle finger (3rd reduction and percutaneous pinning
metacarpal bone)  Intra-articular fractures
 In the ring finger and digiti minimi (ulnar  Bennett’s fracture
post), it can be allowed to have greater  Rolando’s fracture
angulation  Treatment: closed reduction and
 30º for the ring finger (4th metacarpal percutaneous pinning or ORIF
bone)
 40-40º for the small finger/digiti BENNETT’S FRACTURE
minimi (5th metacarpal bone)  Mechanism of Injury: Axial compression at the
base of the first metacarpal bone
NOTE: if you make a fist, all the fingers will point to the  Intra-articular fracture (partial)
navicular bone (scaphoid). If you have a fracture of a  Bone is displaced proximally because of the pull of
digit and there’s slight rotation (5º malrotation) it will the abductor pollicis longus and extensor pollicis
be oriented in a different manner and result to brevis
overlapping fingers  Treated surgically by pinning

METACARPAL SHAFT FRACTURES


 Mechanism of Injury: Usually displaced dorsally
due to the pull of the lumbricals
 Flexes the metacarpal shaft

BOXER’S FRACTURE
 Depics a common result of pugilistic activities
 Fracture of the neck of the metacarpal bone
 Fracture can also occur at the midshaft or the
base

Transcriber/s: Jan Cynric Cacao


Formatting: Jan Cynric Cacao D L S H S I M e d i c i n e B a t c h 2 0 1 6 | 5 of 9
Editor/s: Paolo Ramirez, Leanne Gale Ramos
ROLANDO’S FRACTURE
 It requires a greater force than Bennett fracture;
presently used to describe a comminuted Bennett
fracture; a “Y” or “T” fracture, or a fracture with
dorsal and palmar fragments
 Intra-articular fracture of the base of the thumb
(complete)
 Appreciated by a Robert’s view (PA)

DORSAL EXTENSOR COMPARTMENTS


OF THE HAND
 Composed of six compartments
 Patients with lacerations in any part of the dorsal
IP DISLOCATIONS compartment can still perform normally
 Dorsal dislocations  This is because of the presence of an inter-
 Usually at the DIP or PIP tendinous connection (juncturae tendinum)
 Treated with closed reduction (pulling the joint  During assessment, you should apply
back) and placing a buddy splint for 3 to 6 weeks resistance to other fingers to be able to
 Complex MCP dislocations is usually seen with detect what structure is affected
skin dimpling
 Traction is not advised in simple dislocations PRINCIPLES OF EVALUATION (PROFUNDUS AND
 Bayonette apposition of the bone is also SUPERFICIALIS)
considered as a complex dislocation  Both tendons have one common belly, hence, it
is hard to pinpoint what structure is affected if
there is no resistance applied to the affected
NOTES: finger
Generally, there are only three areas where you have  FDP is tested
to suture a laceration  To test for an intact profundus tendon, the
 Ulnar side of the thumb examiner maintains the digit in extension
 Radial side of the index finger while the patient attempts to flex the
 Ulnar side of the fifth digit terminal phalanx

If you suspect a flexor tendon injury, you should know


how to examine which tendon is involved. It can be the
profundus or the superficialis. One way to know which
tendon is involved is by restraining the other tendon
and ask the patient to flex the finger, if the patient
cannot flex the DIP, then the profundus is injured since
the attachment of the ligament is at the base of the
distal phalanx. If the patient cannot flex the PIP, then
the superficialis is injured since the attachment of the
ligament is at the base of the proximal phalanx

AMPUTATIONS  FDS is tested


 Amputations of the thumb requires length  Same principle is applied as to FDP
restoration for apprehension examination
 In terms of function, the thumb contributes 5o%,
index is 20% and the rest are 10%
 If you cannot restore the length, you have to
transfer the big toe to function as a thumb

Transcriber/s: Jan Cynric Cacao


Formatting: Jan Cynric Cacao D L S H S I M e d i c i n e B a t c h 2 0 1 6 | 6 of 9
Editor/s: Paolo Ramirez, Leanne Gale Ramos
NOTE: generally, if you take an X-ray of the wrist,
there is a co-linear relationship the distal radius, lunate
and capitate. If this is disrupted, then most likely, you
have a problem somewhere

WRIST FRACTURES
SCAPHOID FRACTURE
 Mechanism of Injury: Fall on the radial side with a
pronated position
 Common and accounts for about 50-80% of carpal
injuries
 The major blood supply is derived from scaphoid
branches of the radial artery, entering the dorsal
ridge, and supplying 70-80% of the scaphoid,
NOTE: on an AP view, the proximal carpal bone, if you
including the proximal pole; the remaining distal
draw a curve line, they should form a smooth curve
aspect is supplied through branches entering the
and each bone is practically “kissing” each other. If
tubercle
there is a widening (particularly between the scaphoid
 If you palpate for the anatomical snuff box and
and the lunate) of more than 3 mm, it is positive for
tenderness is present, expect a fracture of the
Terry Thomas Sign
scaphoid
 X-ray with ulnar deviation position (30º) is
diagnostic of the fracture
 Treatment
 Suspected fracture with normal plain films
 Short arm thumb spica (splint or cast)
 Follow up in 2 weeks
 Consider bone scan
 Refer to Orthopedics
 Angulated or displaced (1mm)
 Non-union or AVN
 Scapholunate dissociation
 Proximal fractures
 Late presentation
 Early return to play (95% will heal with
vascularized surgery)

SCAPHOID FRACTURE IMAGING


 Initial plain films often normal
 Bone scan 100% sensitive and 92% specific at 4
days
 MRI, CT scan

Transcriber/s: Jan Cynric Cacao


Formatting: Jan Cynric Cacao D L S H S I M e d i c i n e B a t c h 2 0 1 6 | 7 of 9
Editor/s: Paolo Ramirez, Leanne Gale Ramos
TRIANGULAR FIBROCARTILAGE COMPLEX (TFCC)  Distal lateral border of Guyon’s Canal
TEAR  Patients typically present with pain and
 In between the ulnar styloid and the pisiform tenderness over the hamate; ulnar and median
 Mechanism of Injury: Fall on dorsiflexed and neuropathy can also be seen as well as rare
ulnar deviated wrist injuries to the ulnar artery
 Axial load with forearm in hyperpronation  CT scan is the best radiographic test to visualize
the fracture
TFCC TEAR PHYSICAL EXAMINATION  High rate of non-union
 Press test  May consider early operative treatment
 TFCC grind test
 Check for Distal Radioulnar Joint (DRUJ) injury

TFCC TEAR IMAGING


 Plain films may show positive ulnar variance
 Assess for fracture or ulnar subluxation
 MRI (the best way to demonstrate a TFC injury) or
Arthrography

Golfer’s Fracture

GOLFER’S FRACTURE
 Mechanism of Injury: Fracture of the hook of Carpal tunnel view
hamate is a frequent injury sustained when the
palm of the hand is struct by an object
 Swing of golf club, bat
 2% of all carpal fractures
 1/3 of all hamate fractures = golf related
-END-

TRANSCRIPTION DETAILS
Latest PPT, Handout, Netter’s, Moore,
BASIS RECORDINGS + NOTES + DEVIATIONS 30-50% CREDITS -
Handbook of Fractures
Majority of the contents are lifted from the handout. Some inputs are from the Handbook of Fractures, Netter’s and Moore. Basically, you
REMARKS have to study the anatomy of the hand to further understand the mechanisms of fractures in the different segments.
All rights reserved to the owner of the images
-BATCH 2016 Transcribers’ Guild Transcriptions. Version 1.0.0.0.0 Build 3110-

PAST E QUESTIONS 2015

1. The following statement is true for scaphoid fracture: 3. In a gamekeepers thumb there is:
a. Dorsal radial artery supplies the bone on a proximal to the distal a. Radial collateral ligament is ruptured
manner b. STEINER LESION IS PRESENT
b. THE PROXIMAL PORTION IS PRONE TO AVASCULAR c. The hyperextension injury of the 1st MCP joint
NECROSIS d. Fracture of the lip of the radial base of the proximal phalanx
c. Positive tenderness on the volar radial aspect of the wrist
d. AP xray with wrist in radial position is the best way to 4. The following is true for a complex dislocation of the MCP joint:
demonstrate the fracture a. Skin dimpling is present
b. X-RAY SHOWS THE PROXIMAL PHALANX IS
2. The following deformity is not acceptable in a fracture of a PERPENDICULAR TO METACARPAL BONE
metacarpal bone: c. Proximal phalanx is parallel to metacarpal bone on x-ray
a. Mild varus angulation d. Can be reduced by closed reduction
b. Mild dorsal angulation 5. The following statement/s is/are true for Bennette’s fracture:
c. ROTATION a. Comminuted fracture of the base of the 1st metacarpal bone
d. Minimal shortening b. ABDUCTOR POLLICIS LONGUS IS THE MAIN DEFORMING
MUSCLE

Transcriber/s: Jan Cynric Cacao


Formatting: Jan Cynric Cacao D L S H S I M e d i c i n e B a t c h 2 0 1 6 | 8 of 9
Editor/s: Paolo Ramirez, Leanne Gale Ramos
c. Oblique nnon-articular fracture of the base 13. In boxer’s injury, the fracture is seen in what part of the
metacarpal bone:
6. The following statement is true for tendon injury of the hand: a. NECK
a. No man’s land refers to the carpal tunnel area b. Base
b. Tendon heals intrinsically c. Head
c. Inability to flex the distal interphalangeal joint shows a d. Diaphysis
transected flexor digitorum superficialis tendon
d. TRANSECTION OF THE EXTENSOR TENDON AT THE MCP 14. A patient presents to you with a lacerated irregularly shaped
JOINT MAY PRESENT WITH FALSE POSITIVE EXTENSOR wound at the dorsum of the metacarpophalangeal joint. What
TENDON TRANSACTION ON PHYSICAL EXAM should one do, EXCEPT:
a. Xray
7. The following statement/s is/are true for Boutonniere’s b. SUTURE THE WOUND
deformity: c. Give antitetanus
a. TRANSECTED CENTRAL SLIP OF EXTENSOR TENDON AT d. Debridement and irrigation of the wound
THE PROXIMAL INTERPHALANGEAL JOINT
b. There is extension of the PIP & flexion of the DIP joint 15. A twenty year old had a trap door injury of the distal part of his
c. There is dorsal subluxation of lateral slip of extensor tendon index finger. If on exam you see a swollen finger with a subungal
d. All of the above hematoma, what should you do:
a. Remove the nail
8. The main deforming muscle in a fracture of the middle phalanx b. PUNCTURE THE NAIL WITH HEATED NEEDLE
is: c. Xray of the distal finger
a. Interossei muscle d. Pin the tufted part of the distal if with fracture
b. FLEXOR DIGITORUM SUBLIMIS
c. Extensor muscles 16. Following a wrist injury one sees disruption of the proximal
d. Flexor digitorum profundus muscles Gigula’s arches. Most likely there is a:
a. SCAPHOLUNATE DISLOCATION
9. A patient falls on an outstretched extended forearm. On b. Triangular fibrocartilage complex injury
physical exam he has tenderness of the volar ulnar aspect of the c. Perilunate dislocation
wrist. The patient has pain on pressing on both radius and ulna and d. Scaphocapitate dissociation
when pronating. What is the most possible injury that he has:
a. Scaphoid fracture 17. The following are components of the triangular fibrocartilage
b. Fracture of the hook of the hamate complex:
c. TRIANGULAR FIBROCARTILAGE COMPLEX INJURY a. Radiostyloid ligament
d. Fracture of the metacarpal neck b. Ulnostyloid lunate ligament
c. ULNOSTYLOID CAPITATES LIGAMENT
10. If in the above patient our consideration is a fracture of the d. Meniscus homologue
hook of the hamate, the best way of demonstrating this is:
a. AP view of the wrist on xray 18. Which of the following metacarpal bones should have a near
b. Lateral view of the wrist on xray perfect anatomical alignment when fractured:
c. Ultrasound a. First
d. CARPAL TUNNEL VIEW b. SECOND
c. Third
11. A 3 year old patient tried to catch a ball, however the ball hit his d. Fourth
finger in an extended position. He now presents a mallet finger. e. Fifth
The most likely pathology is:
a. Stretched distal extensor injury 19. The best way to demonstrate a triangular fibrocartilage
b. EPIPHYSEAL FRACTURE OF THE DISTAL PHALANX complex injury is:
c. Fracture of the dorsal lip of the distal phalanx a. X-ray
d. Distal extensor tendon transaction b. Ultrasound
c. CT Scan
12. The following principles are noted in amputation of the hands d. MRI
EXCEPT:
a. Always maintain the length of the thumb 20. A patient presents to you with a wrist lacerated wound. Which
b. COSMESIS IS EMPHASIZED MORE THAN FUNCTION of these will confirm that he has an ulnar nerve injury? He cannot
c. Amputation of the proximal phalanx may be corrected by doing do this maneuver:
ray amputation a. Thumb opposition
d. The basic hand consists of radial and ulnar post b. Fifth digit adduction
c. EXTENSOR LAG
d. Thumb adduction

Additional Notes 
(Taken from Past E Rationalization)

Hand Fractures/Injuries with associated structures/affectation

Middle Phalanx = Flexor digitorum superficialis


Proximal Phalanx = No associated tendon or muscle
Distal Phalanx = Epiphyseal fracture (volar displacement)
Mallet Finger = Extensor Digitorum Tendon (forced flexion of DIP)
Jersey Finger = Flexor Digitorum Profundus Tendon (forced extension of DIP)
GameKeeper’s Thumb = 1st MCP joint (hyperabduction of 1st digit)
Golfer’s Fracture = Hook of Hamate
Boxer’s Fracture = Metacarpals (Loss of Grip Strength)
Boutonniere’s Deformity = DIP extended, PIP flexed
Bennett’s Fracture = Abductor Policis Longus tendon (avulsion of 1st MCP +
subluxation at CMC)
Rolando’s Fracture = Base of 1st MCP (Comminuted)
IP Dislocation = PIP and DIP

Transcriber/s: Jan Cynric Cacao


Formatting: Jan Cynric Cacao D L S H S I M e d i c i n e B a t c h 2 0 1 6 | 9 of 9
Editor/s: Paolo Ramirez, Leanne Gale Ramos
ARTHRITIDES
Michael Muñoz, MD, FPOA
September 1, 2014; 1:00-2:30 PM
Orthopedics

LECTURE OUTLINE:
 Introduction
 Normal Joint
 General Pathology of Arthritis
 Classification
 Osteoarthritis
 Rheumatoid Arthritis
 Gouty Arthritis

INTRODUCTION
 “ARTHRITIS”  Signs and Symptoms
 Arthro – Joint (Greek)  Pain
 -itis – Inflammation  Swelling
 Stiffness/limitation of motion
So what’s Arthritides? = Plural form of Arthritis 

NORMAL JOINT
 Articular Cartilage [end part of the bone]  Chondrocyte Metabolism
 Distributes load and decreases friction  Modulated by mechanical stimulation
 Water = 65-80% [movement]
 Collagen = 10-20% wet weight, >50% dry  Proteoglycan = 10-15% wet weight
weight, 95% type II

- BIGGEST NOTES:
Joints function also as shock absorbers, distributes Role of the Meniscus - deepens articular surface
load and decreases friction during movement area and distributes the mechanical load

AGING JOINT OSTEOARTHRITIC JOINT


Loss of Proteoglycans Loss of Collagen > Loss of Proteoglycan
Decreased Water Content Increased Water Content

GENERAL PATHOLOGY OF ARTHRITIS


 Joint Derangement
 Synovitis & Soft Tissue Swelling
 Joint Space Narrowing
 Soft Tissue Thickening
 Subchondral Erosion
 Osteophyte Formation
 Deformity

Transcriber/s: Paolo M. Ramirez


Formatting: Paolo M. Ramirez D L S H S I M e d i c i n e B a t c h 2 0 1 6 | 1 of 10
Editor/s: Leanne Gale Ramos & Jan Cynric Cacao
CLASSIFICATION OF ARTHRITIS
Osteoarthritis
Non-
Osteonecrosis
inflammatory
Osteochondritis Dissecans (OCD)
Rheumatoid Arthritis
SLE
Inflammatory
AS
Gouty Arthritis
Pyogenic
Infectious TB
Fungal
Pigmented Villonodular Synovitis
Hemorrhagic Hemophilic Arthropathy
Sickel Cell Arthropathy
“Rayuma” – misnomer of arthritis and is actually
Rheumatoid Arthritis

OSTEOARTHRITIS
 Primary OA
 Etiology [still undetermined]
 Aging process (Degenerative)
 Wear & Tear (Mechanical)
 Hereditary
 Articular Cartilage (Biochemical)
 Increased water content (thereby
decrease in strength and elasticity)
 Collagen abnormality (tensile *Erosion of the cartilage
strength)
 Proteoglycan alteration (compressive) CLINICAL MANIFESTATION
 Inflammatory cascade (enzymatic  Joint Pain (mechanical; prolonged standing or
degradation) activity)
 Secondary OA  Swelling
 Trauma  Stiffness (inactivity) [Theatre Sign - bilateral
 Infection [septic arthritis] knee stiffness and pain which occurs after
 Congenital condition [Joint Dysplasia prolonged sitting in patients with patellofemoral
presenting as hyper/hypoplasia of the joint = joint pain]
incongruity of the joint]  Deformity - varus or valgus
 Avascular Necrosis/Osteonecrosis [usually  Contracture/Limitation of Motion
the hip, in the subchondral area]
 X-ray
 Osterochondritis Dissecans (OCD)

PATHOPHYSIOLOGY OF OA
 Mechanical & Inflammation
 Mechanical imbalance
 Synovitis/inflammatory process  Joint Space Narrowing = BLUE
 Articular cartilage softening/damage  Cartilage & Bone Erosion = RED
 Sclerosis = GREEN
 Soft tissue swelling
 Osteophyte Formation = ORANGE
 Derangement  Deformity (Varus/Valgus)
 Degradation, erosion (loss of synovial fluid)
 Narrowing of joint space(localized ) BIGGEST NOTES:
 Bony changes (osteophytes, sclerosis, cyst Review of terms:
formation) VALGUS – joint moves medially
 Soft tissue thickening (capsule, ligaments) VARUS – joint moves laterally

Transcriber/s: Paolo M. Ramirez


Formatting: Paolo M. Ramirez D L S H S I M e d i c i n e B a t c h 2 0 1 6 | 2 of 10
Editor/s: Leanne Gale Ramos & Jan Cynric Cacao
TREATMENT Editor’s Notes:
 Goal:  The most common type of osteotomy performed on
 Alleviation of Pain & Swelling arthritic knees is a high tibial osteotomy, which
 Minimize Disability addresses cartilage damage on the inside (medial)
 Maintain Quality of Life portion of the knee. The procedure usually takes 60 to
 Non Pharmacologic 90 minutes to perform.
 Exercise Strengthening of Muscles  During a high tibial osteotomy, surgeons remove a
wedge of bone from the outside of the knee, which
 Improve Flexibility
causes the leg to bend slightly inward. This resembles
 Weight Loss
the realigning of a bowlegged knee to a knock-kneed
 Activity Modification position. The patient's weight is transferred to the
 Pharmacologic outside (lateral) portion of the knee, where the cartilage
 NSAID – oral or topical (pain & is still healthy.
inflammation)
 Steroid – oral or injection
 Unicondylar Joint Replacement – “New joint”
 Opiod-like substance – centrally acting
analgesic [pain reliever only]
 Viscosupplementation – Hyaluronic Acid
(intra-articular injection) [acts as lubricant
and shock absorbers]
 Glucosamine & Chondroitin Sulfate – oral
supplementation [postulated to slow
cartilage breakdown]

Surgery
 Osteotomy – “cutting of the bone”. An
unloading procedure. Distributing load of
affected side of the joint to the unaffected side

Editor’s Notes:
 In a unicompartmental knee replacement, only the
damaged compartment is replaced with metal and
plastic. The healthy cartilage and bone in the rest of the
knee is left alone.
 Multiple studies have shown that modern
unicompartmental knee replacement performs very well
in the vast majority of patients who are appropriate
candidates.

Editor’s Notes:
 Osteotomy ("bone cutting") is a procedure in which a
surgeon removes a wedge of bone near a damaged joint.
This shifts weight from an area where there is
damaged cartilage to an area where there is more or
healthier cartilage.
 Osteotomy may be effective for hip and knee joints.
Doctors often do an osteotomy to correct certain knee
deformities such as bowleg (varus) and knock-knee
(valgus) deformities of the knees. Hip osteotomy
involves removing bone from the upper thighbone
(femur).

 HTO (High Tibial Osteotomy)  Patellofemoral Joint Replacement

Transcriber/s: Paolo M. Ramirez


Formatting: Paolo M. Ramirez D L S H S I M e d i c i n e B a t c h 2 0 1 6 | 3 of 10
Editor/s: Leanne Gale Ramos & Jan Cynric Cacao
 Total Hip Replacement

Editor’s Notes:
 Patellofemoral arthritis occurs when the articular
cartilage along the trochlear groove and on the
underside of the patella wears down and becomes Editor’s Notes:
inflamed. When cartilage wears away, it becomes In a total hip replacement (also called total hip arthroplasty),
frayed, and when the wear is severe, the underlying the damaged bone and cartilage is removed and replaced
bone may become exposed. Moving the bones along this with prosthetic components.
rough surface is painful.
 During this "partial" knee replacement”, worn down
bone and cartilage surfaces are removed and replaced
with metal and polyethylene (plastic) implants. The
trochlear groove is covered by a thin metallic shield and
a dome-like plastic implant is used on the patella. Both
components are held in place by bone cement.

 Total Knee Replacement – if joint is severely


eroded

RHEUMATOID ARTHRITIS
 Systematic Autoimmune Disease (Symmetric
Inflammatory Polyarthropathy)
 Articular & Less commonly extra-articular
 Female > Male
 Variable but progressive clinical course:
 Chronic ≥ 75%,
 “waxing & waning”
 Severe Crippling = 15%
 Erosive
 Deforming & Disabling
 Morning Stiffness (>1 hour) [brought about
Editor’s Notes: by Interleukin-6]
Total knee replacement indications: (according to American
Academy of Orthopaedic Surgeons/AAOS)
 Severe knee pain or stiffness that limits your everyday
activities, including walking, climbing stairs, and getting
in and out of chairs. You may find it hard to walk more
than a few blocks without significant pain and you may
need to use a cane or walker
 Moderate or severe knee pain while resting, either day or
night
 Chronic knee inflammation and swelling that does not
improve with rest or medications
 Knee deformity — a bowing in or out of your knee
 Failure to substantially improve with other treatments
such as anti-inflammatory medications, cortisone
injections, lubricating injections, physical therapy, or
other surgeries
JOINT INVOLVEMENT (SYMMETRIC
POLYARTICULAR)
 Wrist
 Hand (MCP & PIP joints)
 Foot (MTP joint)
 Spine (Cervical)
 Hip
 Knee

Transcriber/s: Paolo M. Ramirez


Formatting: Paolo M. Ramirez D L S H S I M e d i c i n e B a t c h 2 0 1 6 | 4 of 10
Editor/s: Leanne Gale Ramos & Jan Cynric Cacao
PATHOBIOLOGY OF RHEUMATOID JOINT

BIGGEST NOTES:
RA usually attacks the small joints of the
wrists, hands, and feet while OA usually
attacks the large weight bearing joints like the
hip and knees

LABORATORY TEST
 Anticyclic Citrullinated Peptide Antibodies (anti
CCP)
 2/3 of patients , 90% sensitivity, 98%
specificity
 Confirms the diagnosis
 Confers the higher risk of erosive &
progressive dse.
 Present in 30% of RF seronegative
 Rheumatoid Factor (Ig M vs Ig G) – 90% of
patients
 Less specific but more sensitive
 OTHERS (compatible but not diagnostic)
 ESR
 CRP
 Decrease. Hematocrit
 Elevated Platelet

CLASSIC JOINT MANIFESTATIONS


 Polyarthritis: swelling with pain even at rest &
tenderness
 Symmetrical
 Morning stiffness (>1 hour), Limitation of motion
 Malalignment of Small joints – hands & feet
 Chronic - > 6 weeks
 Recurrent – flare
WAKE UP!
 Tri-compartmental Arthritis

Transcriber/s: Paolo M. Ramirez


Formatting: Paolo M. Ramirez D L S H S I M e d i c i n e B a t c h 2 0 1 6 | 5 of 10
Editor/s: Leanne Gale Ramos & Jan Cynric Cacao
Editor’s Notes:
 Most RA patients have foot involvement (80%)
 Foot is the initial site of RA manifestation in 17% of
cases
 Forefoot is the most common site
 Midfoot and hindfoot involvement are less common
 X-Ray : Bone destruction and joint erosions are only
seen in advanced cases

 Knee
 Valgus Deformity

6 points – DEFINITE RA
3-4 points – POSSIBLE RA
0-3 points – UNLIKELY RA

Editor’s Notes:
 Valgus deformity is a term for outward angulation of
the distal segment of a bone or joint. The opposite
deformation, medial deviation of the distal bone, is
called varus.
 Valgus knee in adults is generally caused by arthritis or
trauma. It is more common with rheumatoid arthritis
than with osteoarthris.

IMAGING  Hips
 Hand  Protrusio Acetabuli – intrapelvic
displacement of the medial wall of the
acetabulum

BIGGEST NOTES:
Different nodes of Arthritis
HEBERDEN’S NODE – Distal Interphalangeal joint
BOUCHARD’S NODE – Proximal Interphalangeal joint
TREATMENT
 Goal
 Foot  Control Synovitis
 Control Pain
 Maintain Joint Function
 Prevent Deformity
 EARLY DIAGNOSIS , AGGRESSIVE
TREATMENT ESSENTIAL
 Education is important [erosion would
indicate irreversibility]
 Suppress Inflammation

Transcriber/s: Paolo M. Ramirez


Formatting: Paolo M. Ramirez D L S H S I M e d i c i n e B a t c h 2 0 1 6 | 6 of 10
Editor/s: Leanne Gale Ramos & Jan Cynric Cacao
 DMARDs (disease-modifying anti-  30-50yo / Males
rheumatic drugs)  MTP = 50%
 Combination Therapy is the new
paradigm [Anticytokine + DMARD’s] CLINICAL MANIFESTATION
 Steroids- bridging therapy  Acute Recurrent
 NSAID – still useful in variable dse  Great Toe (podagra)
course  Joint Pain (night)
 Prevent Musculoskeletal Dysfunction  Redness
 Exercise, PT, OT, Diet  Tenderness
 Correction Deformity early in to preserve  Swelling
function  Hot (warm)
 Orthopedic Surgery
 Usually after intake of high purine diet
SURGERY
CAUSES OF HYPERURICEMIA
 Osteotomy
 Diet
 Genetic Predisposition
 Underexcretion of urate – primary cause, seen in
90% of cases

 Total Knee Replacement for Severely Damaged


Rheumatoid Arthritic Joint

 Underexcretion of urate from the Kidney

 Other organs
 Kidney = 2nd most affected ( kidney stone)
 Crystal deposit = “tophi”
 Ear helix
 Eyelids
GOUTY ARTHRITIS  Olecranon
 Inflammatory arthritis  Achilles tendon
 Disorder of nucleic acid metabolism
 Hyperurecemia - monosodium urate crystals
deposition in joints
 Crystals activate inflammatory mediators

Transcriber/s: Paolo M. Ramirez


Formatting: Paolo M. Ramirez D L S H S I M e d i c i n e B a t c h 2 0 1 6 | 7 of 10
Editor/s: Leanne Gale Ramos & Jan Cynric Cacao
 Monosodium Urate (MSU)
 Thin, tapered intracellular crystals
 Strongly Negative (yellow) Birefringent
 Elevated Ser. Uric Acid = NOT diagnostic of
gout

*REMEMBER XANTHINE OXIDASE

TREATMENT
 Patient Education
 Diet
 Pharmacologic
 Surgery – tophi excision, debridement, joint
fusion
Tophus

“punched-out” periarticular erosion & sclerosis

Transcriber/s: Paolo M. Ramirez


Formatting: Paolo M. Ramirez D L S H S I M e d i c i n e B a t c h 2 0 1 6 | 8 of 10
Editor/s: Leanne Gale Ramos & Jan Cynric Cacao
 Severely Eroded Joint  Excision Arthroplasty & Fusion

SUMMARY

-END-
TRANSCRIPTION DETAILS
BASIS Latest PPT RECORDINGS + NOTES + DEVIATIONS 8-10% CREDITS Xmark’s the spot Recordings
REMARKS Sources: American Academy of Orthopedic Surgeons (AAOS) Website: http://orthoinfo.aaos.org
-BATCH 2016 Transcribers’ Guild Transcriptions. Version 1.0.0.0.0 Build 3111-

PAST-E 2013

The articular mechanical strength affected by the


deformation of proteoglycan architecture: Water content in an osteoarthritic joint is:
a. Tensile a. Decreased
b. COMPRESSIVE b. INCREASED
c. Shear c. Not variable
d. Torsional d. The same as elderly joint

Transcriber/s: Paolo M. Ramirez


Formatting: Paolo M. Ramirez D L S H S I M e d i c i n e B a t c h 2 0 1 6 | 9 of 10
Editor/s: Leanne Gale Ramos & Jan Cynric Cacao
In joint replacement, what is the most appropriate for
The water composition of an aging articular cartilage is: rheumatoid knee?
a. DECREASED a. Unicompartmental joint replacement
b. Increased b. Patellofemoral joint replacement
c. Not variable c. TOTAL JOINT REPLACEMENT
d. The same as arthritic cartilage d. High tibial osteotomy

Role of meniscus: The new paradigm in the treatment of rheumatoid


a. Production of synovial fluid arthritis:
b. Stabilizes the knee joint during anterior-posterior a. Non-steroidal Anti-Inflammatory plus Steroid
translation b. DMARDs plus Steroid
c. DEEPENS ARTICULAR SURFACE AREA AND c. ANTICYTOKINES PLUS DMARDS
DISTRIBUTES THE MECHANICAL LOAD d. Anticytokines plus Non-Steroidal Anti-Inflammatory
d. Protects the knee in varus stress and protects the
cartilage In Diagnosis Rheumatoid Arthritis: 1-2 small joints
involved, negative RF and CCP, duration of symptoms
Secondary osteoarthritis can be brought about by: >6 weeks and abnormal CRP:
a. Obesity a. Unlikely rheumatoid arthritis
b. Age b. PROBABLE RHEUMATOID ARTHRITIS
c. JOINT DYSPLASIA c. Definite rheumatoid arthritis
d. Heavy intake of alcoholic beverage d. Undefined rheumatoid arthritis

An Immunoglobulin (Ig M) serum marker in rheumatoid


Tri-compartamental arthritis is usually observed in: arthritis that is directed against another
a. Primary osteoarthritis immunoglobulin (Ig G):
b. Secondary osteoarthritis a. Anti-cyclic cirtullinated peptide antibody
c. RHEUMATOID ARTHRITIS b. Anti-nuclear antibody
d. Gouty arthritis c. RHEUMATOID FACTOR
d. C-reactive protein
Uni-compartamental arthritis is commonly seen in:
a. Gouty arthritis Clinical suspicion of rheumatoid arthritis and early
b. Rheumatoid arthritis referral to rheumatologist is essential if:
c. OSTEOARTHRITIS a. ≥ 2 swollen joints
d. Infectious arthritis b. positive squeeze test on CMC joint
c. morning stiffness ≤ 30 mins
Joint stiffness is due to cytokine increase in early d. POSITIVE ANTI-CCP
morning. What cytokine is attributed to morning
stiffness in rheumatoid arthritis? Diagnosis of gouty arthritis is confirmed when a crystal
a. IL-1 seen in microscopy is:
b. IL-6 a. positive blue birefringence
c. IL-10 b. positive yellow birefringence
d. IL-4 c. negative blue birefringence
d. NEGATIVE YELLOW BIREFRINGENCE
Subchondral erosion is most prominent early in the
disease process of: The rate of limiting enzyme in uric acid formation
a. Primary osteoarthritis a. Hypoxantine oxidase
b. Secondary arthritis b. XANTHINE OXIDASE
c. RHEUMATOID ARTHRITIS c. Monosodium urase
d. Gouty arthritis d. Monosodium uricase
A serum marker in rheumatoid arthritis that signify
The production of joint fluid is facilitated by: progressiveness:
a. Prolonged immobilization a. IL-4
b. Cartilage regeneration b. IFN
c. Joint replacement c. ANTI-CCP
d. MOBILITY d. RF

Unloading surgical procedure is a variable operative The main role of surgery in the treatment of arthritis:
procedure for a knee with: a. To restore the length of the limb and preserve the
a. PRIMARY OSTEOARTHRITIS cartilage
b. Rheumatoid arthritis b. To relieve contracture and prevent arthritis
c. Gouty arthritis c. TO PRESERVE FUNCTION AND MAINTAIN QUALITY
d. All of the above OF LIFE
d. To salvage the limb and avoid dreadful amputation

Transcriber/s: Paolo M. Ramirez


Formatting: Paolo M. Ramirez D L S H S I M e d i c i n e B a t c h 2 0 1 6 | 10 of 10
Editor/s: Leanne Gale Ramos & Jan Cynric Cacao
SPORTS INJURIES
Rene B. Rivera, MD, FPOA
September 9, 2014; 1:00 – 2:00 PM
Orthopedics

OUTLINE SLIDE NOTE:


 Sports Injuries - an overview Sports injuries come in two basic types, acute and
 Soft Tissue Injuries overuse. Acute injuries are those that involve sudden
 Fractures trauma, such as sprains, strains, bruises, and
 Joint Injuries fractures. On the other hand, overuse injuries result
 Overuse Syndromes from the cumulative result of “micro trauma”, that is,
 Prevention of Sports Injuries a series of repeated smaller stresses or injuries.
 Principles of treatment
 Rest, Immobilization, Compress, Elevation CHILDREN & SPORTS
(RICE)
 Increasing awareness
SLIDE NOTES: Sports, as all other physical activities,
involve some risks. However, when it comes to children
WHAT DO ORTHOPAEDIC SURGEONS and sports, these risks can be minimized. It is
DO? important to remember that young athletes are NOT
 Diagnose small adults. Their young bones, muscles, tendons,
 Treat and ligaments are growing, making them very
 Medication vulnerable to injury.
 Physical/Occupational Therapy
 Exercise CHILDREN’S SPORTS INJURIES, 2007
 Brace
 Surgery (if necessary) Number of Injuries requiring
 Prevent Sport
treatment per year
Football 556,928
SLIDE NOTE: Orthopaedic surgeons use the most Basketball 412,075
effective and efficient diagnostic tools and experience Baseball/Softball 276,733
in musculoskeletal treatment to determine the best Soccer 220.162
course of treatment for patients. Playground
410,634
injuries
SPORTS INJURIES
SLIDE NOTE: Each year, nearly 1,900,000 children
SLIDE NOTES: under the age of 15 are treated in hospital emergency
Every day, millions of people participate in sports rooms in the United States for sports-related injuries.
activities. Regardless of age, exercise is an important The National Electronic Injury Surveillance System of
part of a healthy lifestyle and participation in sports the United States Consumer Product Safety
activities is a great way to improve one’s physical Commission also collects information about whether
fitness. Taking a few precautions can make it safe to the injury was sustained in organized or unorganized,
reap the many benefits of sports participation. informal activities. This table highlights several
However, each year, thousands of children and adults popular sports and their incidence of injury for children
are injured during sporting activities, taking them out under age 15.
of the game. When sports injuries happen,
orthopaedists have the expertise to diagnose and treat
the athletes. As the team doctors for every major
professional sport, orthopaedic surgeons specialize in
getting you back in the game.

ACUTE VS. OVERUSE INJURIES


 Acute - sudden trauma such as sprains, strains,
bruises & fractures
 Overuse - series of repeated small injuries

Left: pediatric bone; Right: adult bone - Growth plates are


weaker than ligaments.

Transcriber/s: Jan Cynric Cacao


Formatting: Jan Cynric Cacao D L S H S I M e d i c i n e B a t c h 2 0 1 6 | 1 of 13
Editor/s: Paolo Ramirez, Leanne Gale Ramos
SLIDE NOTE: At the ends of their bones, children have COMMON SOFT TISSUE INJURIES
growth plates, which are areas of developing
cartilage where bone growth occurs. In young growing ANKLE SPRAIN
children, growth plates are weaker than the nearby  Ligament injury
ligaments and tendons. As a result, an injury that  REVIEW: Ligaments are fibrous connective
might result in a bruise or sprain for an adult can be a tissues that connects the bones together to
much more serious issue for the young athlete. form your joints
 Ankle pain, tenderness, swelling
 Ankle is the most common area for sprain
 Youths of same age can differ tremendously in
 Most common ligament affected is the
size and physical maturity.
anterior talofibular ligament (Mnemonic:
 Even though they have the same age and
ATAFI)
same mechanism of injury, they can have
 MOI: Supination and External Rotation
differing severity of injury
(most common)

SLIDE NOTE: Further marking the difference between


adults and children in athletics is the reality that
children mature at sometimes vastly different rates.
Athletic youths of the same age can differ
tremendously in size and physical maturity, increasing
the opportunity for injuries.

BOOMERITIS®
 Born 1946 - 1964
 More than 1 million sports-related injuries each
year
 Bike accidents prevail
 Mortality > children
 < 50% wear helmets
 Most common injuries in this age group:
 Bursitis SLIDE NOTES:
 Tendinitis  In the treatment of musculoskeletal concerns,
 Sprains certain injuries and conditions are most
 Strains commonly related to sports activities.
 Ankle sprains are one of the most common
SLIDE NOTES (TRIVIA): injuries seen in sports. They occur when the ankle
 Children are by no means the only population at suddenly twists and there is an overstretching or
risk for athletic injuries. Baby boomers, born injury of the ligaments that support the ankle.
between 1946 and 1964, who sustain sports- The ligaments on the outside of the ankle are
related injuries, are dealing with “Boomeritis.” most commonly injured when the foot is turned
This nickname, coined by an orthopaedic surgeon, inward on an awkward step. Ankle sprains are
has been given to a group of orthopaedic injuries, acute injuries. Symptoms of ankle sprain include a
ailments and musculoskeletal vulnerabilities tender and swollen ankle on the outside below and
commonly seen in this age group. The U.S. in front of the ankle bone.
Consumer Product Safety Commission (CPSC)
estimates that in 1998, Baby Boomers had more GRADING AND SEVERITY
than one million sports-related injuries that  Type 1 – Mild/ Slight stretching
required medical treatment.  Slight swelling and tenderness
 Exercise is a key component of a healthy lifestyle,  Difficulty hopping
but Baby Boomers need to remember that their  Recovery 2-10 days
bodies are aging, and certain exercise and  Type 2 – Moderate/ Partial tear
fitness programs bear greater risk for injury,  Hematoma formation and ecchymosis
particularly as we age.  Unable to heel raise, hop, run
 The many injuries common to Baby Boomers are  Decreased Achilles’ definition
most often the result of years of overuse. The  Recovery 10-30 days
cumulative effect of this assault on the  Type 3 – Severe/ Complete tear
musculoskeletal system of old injuries that occur  The ankle is swollen GLOBALLY (No
again added to the normal “wear and tear” of Achilles’ definition)
tendons and joints, muscle loss associated with  Recovery 30-90 days
aging, and the rest of the aging process all add up
to common overuse ailments such as tendonitis,
bursitis, rotator cuff tears, and sprains and strains.

Transcriber/s: Jan Cynric Cacao


Formatting: Jan Cynric Cacao D L S H S I M e d i c i n e B a t c h 2 0 1 6 | 2 of 13
Editor/s: Paolo Ramirez, Leanne Gale Ramos
OTHER SITES OF SPRAIN
 Knee
 Middle and lateral collateral ligaments
 Valgus/varus stress
 Wrist
 Palmar ligament
 Shoulder
 Elbow

STRAINS
 An injury to either a muscle or tendon, more
common in the leg or foot
 May be a simple stretch, or it may be a partial or
complete tear

HAMSTRING MUSCLE STRAIN


 “pulled hamstring”
TREATMENT  Common among athletes that require sprinting
 R.I.C.E. (Rest, Ice, Compression, Elevation)  Common injury in the muscle-tendon attachment
 Rehabilitation  “eccentric contraction”
 Range of motion [Achilles tendon stretching  A lengthened muscle (not contracting)
is a key component to preserving range of suddenly (an external force) made it to
motion] contract → overload
 Strengthening (muscles)
 Flexibility
 Balance (weight-bearing joints) REVIEW: Hamstring muscles
 Semitendinosus
 Semimembranosus
SLIDE NOTES:
 Biceps femoris
 Treatment of ankle sprains features that old
standby, R.I.C.E., with ice (not directly applied to
the skin) recommended for 20-30 minutes each
hour. Be especially careful with freezer gel packs
which are much more likely to cause frostbite
than regular ice. In addition, your doctor may
suggest rehabilitation to begin a few days after
the injury, when the swelling has gone down. The
goals of this rehabilitation are to restore motion
and flexibility, and to restore strength and
balance.
 Generally, your physician will give you the go
ahead to return to sports after you have full range
of motion in all directions, good strength in the
muscles all around the ankle, good balance, and
no pain or swelling with exercise or activity. He or
Editor’s Notes:
she may recommend, or you may find it is just
CONCENTRIC VERSUS ECCENTRIC Contraction
more comfortable, to wear a brace or tape your
ankle when first returning to activity. However,
Concentric Eccentric
braces and taping are meant to lend support to
Muscle Actively Muscle Actively Lengthening
strong muscles, not serve as a substitute for them. Shortening
Most common type of Less common and usually involves the
PHASES OF RECOVERY muscle contraction, control or deceleration of a movement
 Phase 1 – resting and protecting, reducing and occur frequently in being initiated by the eccentric
daily and sporting muscles agonist
swelling
activities
 Lasts for 3-5 days
Examples: Walking, when the
 Splinting quadriceps (knee extensors) are active
 Phase 2 – restoring ROM, strength and flexibility Example: Raising of a
just after heel strike while the knee
 Intermittent removal of splint weight during a bicep
flexes, or setting an object down
curl
 Phase 3 – gradual return to activity/sports gently (the arm flexors must be active
 Sometimes patients wear braces, depending to control the fall of the object)
on the severity of the injury EXAMPLE: When kicking a football, the Quadriceps muscle
contracts concentrically to straighten the knee and the
Hamstrings contract eccentrically to decelerate the motion of
the lower limb.

Transcriber/s: Jan Cynric Cacao


Formatting: Jan Cynric Cacao D L S H S I M e d i c i n e B a t c h 2 0 1 6 | 3 of 13
Editor/s: Paolo Ramirez, Leanne Gale Ramos
RISK FACTORS  Pain that develops gradually, increases with
 Muscle tightness weight-bearing and relieved by rest
 No warm-up  Swelling/tenderness to touch
 Muscle imbalance  Difficult to see on X-rays until they have actually
 If the quadriceps is more powerful than the started to heal → callus formation
hamstring muscles
 Bone scan or MRI
 Poor conditioning
 Muscle fatigue STRESS FRACTURES OF THE FOOT
 Choice of activity  Commonly seen on football players, runners,
soldiers
SIGNS AND SYMPTOMS
 Swelling
 Bruising or discoloration
 Weakness of the hamstring muscle

IMAGING
 X-rays
 To rule out any fractures
 MRI
 For diagnosing a tear (muscle belly)

TREATMENT
 Goal is return to activity or sport (once muscle SLIDE NOTES
strength is 90% of the uninjured side)  Our body makes new bone daily to replace loss
 Non-surgical due to stress. Usually this process is balanced, but
 RICE lack of recovery after exercise sessions,
 Immobilization inadequate calcium, and rapid increase of
 Physical therapy activity levels can create an imbalance that
 Return to sport if strength is approximately 90% results in micro-cracks. These micro-cracks, or
of opposite side stress fractures, can produce pain in a limited
 Surgical area directly over the point of the bone where
 For detached muscles or tears the fracture has occurred. The pain is worse with
activity and improves with rest. Female athletes
MUSCLES IN THE THIGH are more vulnerable to stress fractures, especially
 Quadriceps endurance athletes, gymnasts and dancers.
 Adductor muscles Nutrition and hormonal factors are important to
assess in female athletes with stress fractures,
MUSCLE CONTUSION/HEMATOMA especially if they are recurrent.
 Contact sports (boxing, wrestling)  Your physician will generally diagnose a stress
 Bruising/pool of blood forming or hematoma fracture with a physical exam. X-rays do not
 Intracompartmental bleeding – beware! typically show early stress fractures or stress
reactions. MRI scans and sometimes bone scans
COMPARTMENT SYNDROME are needed to make the diagnosis. Casts are not
 Intracompartmental pressure build up usually required for stress fractures, although
 Surgical emergency your doctor will likely recommend a period out of
 Can have muscle necrosis and eventually death of your sport or activity and a slow progression of
a limb return to activity and intensity to allow your body
to heal.
TREATMENT
 Control pain, bleeding, and inflammation – gentle
stretch position CAUSES
 RICE  Increasing high-impact activity
 Frequency increases
 NSAIDs
 Duration becomes longer
 “do not massage” – may increase hematoma
 Intensity becomes harder
formation
 Improper conditioning
ACUTE COMPARTMENT SYNDROME  Wrong technique
 Pain is more intense than expected, especially  Equipment and Environment
when muscles in the compartment are stretched  Bone Insufficiency
 Surgical emergency!
TREATMENT
 Fasciotomy – treatment of choice
 Stop activity/rest typically 6-8 weeks for fracture
STRESS FRACTURES to heal
 Imbalance bone loss vs. replacement  Cross training
 “Micro-cracks”  Protective footwear/cast
 Pain  Surgical (bone grafting)

Transcriber/s: Jan Cynric Cacao


Formatting: Jan Cynric Cacao D L S H S I M e d i c i n e B a t c h 2 0 1 6 | 4 of 13
Editor/s: Paolo Ramirez, Leanne Gale Ramos
GROWTH PLATE FRACTURES IN CHILDREN #ORTHOFACTS: Did you know?
 Growth plates and epiphyseal fractures are most
French physician Nicholas Andry coined the term "orthopedia" in his 1741
vulnerable in this age group book on the prevention and correction of muscular and skeletal
 Always have a high index of suspicion that there deformities in children. He united the Greek word "orthos," meaning
could be a growth plate injury straight, with "pais," meaning child. The term orthopedics has remained
in use, though the specialty has broadened beyond the care of children.
 Most commonly injured are the wrist and ankles
SLIDE NOTE:
 Your shoulder has the greatest range of motion
of all of your joints. Dislocations occur when the
shoulder receives a strong force that pulls the
arm in an extreme direction, during a fall or a
sports injury, for example. The shoulder joint can
dislocate forward, backward, or downward.
Sometimes your muscles can be used to pull it
back in, but usually the trauma causes spasms and
it will come back out.
 When your orthopaedic surgeon treats you for
Traumatic Shoulder Dislocation, he or she will
usually give you medication to decrease the pain
and the spasms and then use traction to put it
back into place, recommending the use of a sling
to protect it as it heals.
SLIDE NOTE:  Following a dislocated shoulder, there are several
 Always be suspicious for a growth plate fracture exercises you can do to increase the strength and
when there is pain and swelling in a younger control of the rotator cuff and shoulder blade
athlete. What looks like a simple sprain in a young muscles to reduce the chances of a repeat
athlete could be a growth plate fracture. dislocation.
 Children are not just little adults, they get  Closed grip pull downs, rowing on a machine
different injuries. The growth plate is often the and shrugs all help increase shoulder strength.
“weakest link” in their musculoskeletal chain. In  Rotation exercises with your arm down at the
general, younger athletes do not get sprains but side can promote rotator cuff strength.
more often will get growth plate fractures. The Resistant tubing can also be used.
two most common growth plate fractures occur  If shoulder instability becomes a recurrent
in the ankle and the wrist in the distal radius. disability and a conservative exercise program has
failed, surgery to have the stretched ligaments
DISLOCATIONS tightened and/or repaired has a 90-95% success
 Joint disruption/injury to capsule, ligaments and rate.
articular surface
 Partial (Subluxation) SIGNS AND SYMPTOMS
 Complete  Pain/swelling/tenderness
 Most commonly involved  Loss of roundness of shoulder
 Shoulder  Unusual attitude of extremity
 Has the greatest range of motion
 There is pain and tenderness TREATMENT
 Loss of roundedness of the shoulder  Gentle close reduction
 Acromion process readily palpable  Give sedatives
 Elbow  For the muscles to be relaxed and not
 Finger contracting
 Patella  Immobilize with an arm sling
 Hip  Younger individuals needs longer
immobilization (4-6 weeks)
 For older individuals, the goal is to have an
early ambulation to prevent joint stiffness
 Rehabilitation (exercises)
 Closed grip pull downs
 Rotation exercises
 Resistance exercises
 Surgery (for recurrent disclocations)
 Arthoscopic
 Open
 Bankart procedure

Transcriber/s: Jan Cynric Cacao


Formatting: Jan Cynric Cacao D L S H S I M e d i c i n e B a t c h 2 0 1 6 | 5 of 13
Editor/s: Paolo Ramirez, Leanne Gale Ramos
NOTES: diagnostic modality for dislocations especially
to the shoulders is still x-ray with special views. An
AP-lateral view of the chest WILL NOT give you a
TRUE AP-LATEARAL VIEW. A Y-scapular view can
be requested instead. The Y-scapular view will tell you
if the dislocation is anterior or posterior. It is “Y”
because the spine is downward then the coracoid and
acromion process is on the sides and the glenoid
fossa is in the middle. Normally, the head of the
humerus is in the middle of the “Y”, if it is on the
anterior, then it is an anterior dislocation and vice
versa. The most common dislocation of the shoulder is
ANTERIOR.
SLIDE NOTE:
 If you play sports, there are many different
structures you can injure in your knee; two of the
most common are the ACL injury and the
meniscus tear. Your ACL, or Anterior Cruciate
Ligament, is one of the most important
stabilizing ligaments in the knee. It connects the
front part of your shin bone to the back part of
your thigh bone and keeps your shin bone from
sliding forward. The ACL is usually injured by
either a direct blow to the knee or through a
non-contact injury resulting from planting your
foot and cutting, landing on a straight leg, or
making an abrupt stop. In skiers, it frequently
occurs when skiing in the “back seat”, with one’s
weight too far back on one’s heels. Some signs of
an ACL injury are feeling your knee give way
OTHER DISLOCATIONS (sometimes accompanied by an audible “pop”),
 Patella moderate pain that makes it impossible to
 Lateral patellar dislocation
continue the activity, and over several hours
 Usual attitude is flexed
swelling is present and it is difficult to walk. The
 Gentle extension will slowly put the displaced
patella in place swelling and pain are usually the worst during the
 Inter phalangeal first two days following the injury.
 PIP dislocations
 Simple or complex dislocation ACL TEAR

ANTERIOR CRUCIATE LIGAMENT INJURY


 Anterior cruciate ligament
 Most important stabilizer of the knee
 Paired with the posterior ligament Left: normal knee; Right: hyperextension of the knee
 MOI: causing the femur to slip and tear the ACL
 Direct blow to knee
 Non-contact injury, with foot planted
 Landing on straight leg
 Making abrupt stops
 “Back seat” skiing

Transcriber/s: Jan Cynric Cacao


Formatting: Jan Cynric Cacao D L S H S I M e d i c i n e B a t c h 2 0 1 6 | 6 of 13
Editor/s: Paolo Ramirez, Leanne Gale Ramos
SLIDE NOTE:  Lachmann’s test – most sensitive; a variant of
 ACL injuries usually make you uncomfortable anterior drawer test
enough to seek out medical attention. Through a  Knee is flexed 20-30º
physical exam, your orthopaedic surgeon can  Distal femur is stabilized
usually tell which ligaments are injured. Further  Shin bone is gently pulled
tests such as an MRI or arthroscopy may be  Positive test: increased forward translation
called for to properly diagnose your ACL injury. without a decisive 'end-point' - a soft or
Treatment of ACL can include surgery and mushy endpoint indicative of a positive test
rehabilitation. You and your orthopaedic surgeon  Pivot/reverse pivot
will want to consider your activity level, your  Patient in supine position
expectations of your recovery, any associated  Extend and hold the leg
injuries and the amount of abnormal loosening of  Do a valgus stress test and flexion extension
your knee laxity as you decide whether or not  Positive test: dimpling on the lateral aspect
surgery is right for you. If you choose not to have
surgery, you will want to start rehabilitating
immediately with exercises designed to restore
your full range of motion.
 If you have surgery, your orthopaedic surgeon will
reconstruct your ACL by building a new
ligament with your own tissue or a ligament
from a tissue bank. With time, rehabilitation and
hard work ACL surgery results in a 90 percent
success rate. Following ACL injury, you should
return to sports, with or without a brace, only after
your leg strength, balance and coordination are
near normal.

ACL TEAR - WOMEN ATHLETES


 Incidence of ACL much higher among female
athletes
 Combination of causal factors
 Neuromuscular prevention programs

SLIDE NOTE:
 The incidence of ACL injuries among female
athletes is significantly higher than among DIAGNOSIS AND TREATMENT
males, for some sports even five to seven times as  MRI
high! There are several theories as to why this is  Rehabilitation
so, but it is most likely to be a combination of  Surgery/Arthroscopic ACL Reconstruction
factors related to female and male anatomy,  Hamstring tendon is used
hormones, and brain/muscle interaction and
biomechanics. MENISCUS INJURIES
 These prevention programs are important for both  Most common
males and females including those that have  Meniscus:
already had ACL reconstruction to prevent re-  2 wedge-shaped cartilages
injury. There is emerging research that  “shock absorbers”
neuromuscular and biomechanics-related  Helps knee joint carry weight, glide,
conditioning programs can lower the incidence of and turn
an ACL tear.  Tears

PHYSICAL EXAMINATION
 Anterior/Posterior Drawer’s test
 Anterior Drawer’s Test
 Knee flexed on a 90º angle
 Stabilize the foot by sitting on it
 Shin bone is gently pulled
 Positive test: excessive anterior
dislocation of the shin bone
 Posterior Drawer’s Test
 Knee flexed on a 90º angle
 Stabilize the foot by sitting on it
 Shin bone is gently pushed
 Positive test: excessive posterior
dislocation of the shin bone

Transcriber/s: Jan Cynric Cacao


Formatting: Jan Cynric Cacao D L S H S I M e d i c i n e B a t c h 2 0 1 6 | 7 of 13
Editor/s: Paolo Ramirez, Leanne Gale Ramos
MENISCUS TEAR PREDISPOSING FACTORS
 MOI:  Intrinsic
 Twisting injury  Malalignment of leg
 Football and other contact sports  Muscle imbalance
 Signs/Symptoms: Pain, giving way, locking,  Extrinsic
clicking, and swelling  Training errors
 Faulty techniques
SLIDE NOTE:  Incorrect equipments
 The other common type of knee injury is called a  Poor environment
meniscus tear. The meniscus is a triangular, C-
shaped piece of cartilage that improves shock TENNIS ELBOW & GOLFER’S ELBOW
absorption in your joint. It helps the knee joint  Tennis elbow
carry weight, glide, and turn in many directions. It  AKA Lateral epicondylitis
also keeps your femur (or your thighbone) and  Involves the wrist extensors
tibia (shinbone) from grinding against each  Contraction of the extensors will pull the
other. Football players and others in contact insertion of the muscles and this may result
sports may tear the meniscus by twisting the knee, to micro-tears causing pain in that particular
pivoting, cutting, or suddenly decelerating. You area
can also tear your meniscus with deep squatting  Tendon usually involved: Extensor Carpi
type maneuvers as in gardening. Radialis Brevis (ECRB)
 You might experience a “popping” sensation when
your tear the meniscus. Most people can still walk
on the injured knee and many athletes keep
playing. Symptoms of a torn meniscus include
knee pain, swelling, clicking, locking, and
catching.

DIAGNOSIS
 McMurray Test
 Lateral – internal rotate palpating the PL
aspect with ROM
 Medial – external rotate palpating PM aspect
 Positive test: a snap or click often
accompanied with PAIN
 MRI SLIDE NOTE:
 The term “tennis elbow” is used to describe
TREATMENT localized pain over the bony prominence, called
 Arthroscopy/Surgery the lateral epicondyle, on the outside of the
 Rehabilitation elbow. Tennis elbow is caused by repetitive stress
on the muscles connected to the lateral
epicondyle, which result in micro tears causing
NOTE:
inflammation and pain. It is particularly common
 There are two areas in the meniscus. The outer 1/3
to tennis players because of the side-side-side
of the meniscus is VASCULAR so we call it RED
motion required to swing a racket, combined with
area. The inner 2/3 of the meniscus is
the repetitive impact of hitting the ball.
AVASCULAR or the WHITE area.
 If the tear is in the outer 1/3, we repair. Since it is
vascular, it will heal but surgery is done to  Golfer’s elbow
stabilize it. We put sutures using arthroscopy or  AKA Medial epicondylitis
keyhole surgery  Same principle as with tennis elbow except
 If it is in the inner 2/3, we trim or shave or do this would now involve the wrist flexors
partial meniscectomy so that it won’t hinder
motion of the knee joint

OVERUSE SYNDROMES
 Overload or repeated microscopic injuries
 25-50% of athletes visiting clinics
 80% occur in endurance sports

Transcriber/s: Jan Cynric Cacao


Formatting: Jan Cynric Cacao D L S H S I M e d i c i n e B a t c h 2 0 1 6 | 8 of 13
Editor/s: Paolo Ramirez, Leanne Gale Ramos
SLIDE NOTE:
 There is no guaranteed treatment, but your
physician can recommend stretching and
strengthening exercises that should help. You can
prevent becoming a victim of tennis elbow by
heeding four simple tips:
 Warm up well before you play and then do
forearm, wrist and elbow stretching
exercises.
 Choose appropriate equipment and
maintain it properly.
 Condition for the activity you are going
to engage in by stretching and
strengthening all of the muscles your sport
will use.
 Evaluate your playing techniques to be
SLIDE NOTE: sure you are not irritating this condition.
 Golfer’s elbow is a similar ailment affecting the Sometimes a lesson with a tennis or golf
medial or inner side of the elbow. pro will be equally important as seeing an
 This pain generally increases with any activity orthopaedic surgeon since technical
that requires contraction of these muscles, like factors are often the culprit.
shaking hands or turning doorknobs. There are no  Your treatment plan will depend upon the
special tests to diagnose tennis elbow, so your specific diagnosis, but is likely to include cutting
orthopaedic surgeon will most likely rely upon your back in your intensity, duration and frequency of
history and a physical exam, although x-rays can sports activities. You may also be encouraged to
sometimes be helpful. warm up before exercising and ice the injured area
 You don’t have to golf or play tennis to get these after, adopt a schedule that alternates hard and
ailments. They can occur with any repetitive, easy workouts, cross train, and work with a coach
overuse activity. or teacher to gain training on proper techniques to
prevent further injury.
TREATMENT
 Warm up, stretch before play SIGNS AND SYMPTOMS
 Correct, maintained equipment  Pain on muscle-tendon attachment to bone
 Condition beforehand  Swelling/tenderness
 Evaluate play techniques to minimize irritation  Limitation of motion
 Take a lesson
 Reduce intensity DIAGNOSIS
 Warm up before  Thorough history & physical exam
 Ice afterwards  X-rays, MRI might be needed
 Work with a coach
SLIDE NOTE:
STUDY BREAK!  Some individuals are more prone to overuse
injuries due to anatomic or biomechanical factors.
A thorough history and physical exam are your
orthopaedist’s primary diagnostic tools for overuse
injuries. In some instances, x-rays or an MRI might
also be called for. Seeking medical attention from
a specialist with specific interest in and knowledge
of your sport or activity is often a wise choice.

OVERUSE INJURIES: 10% RULE


 Do not increase activity by more than 10% per
week

SLIDE NOTE:
 Overuse injuries are largely preventable through
thorough warm ups and stretching before sports
activities and following the 10% rule: Don’t
increase your activity or intensity by more than
Nom. Nom. Nooom.
10% per week. Also, try to mix up your routine—
avoid the same workout or activity on consecutive
days. If you do the same sport daily you are more
likely to be injured, while doing running one day
and swimming the next, you stay in good shape
and are less likely for overuse injuries.

Transcriber/s: Jan Cynric Cacao


Formatting: Jan Cynric Cacao D L S H S I M e d i c i n e B a t c h 2 0 1 6 | 9 of 13
Editor/s: Paolo Ramirez, Leanne Gale Ramos
SHOULDER OVERUSE INJURIES  Bursitis - the bursa can become inflamed and
 Tendinitis/Bursitis swell with more fluid causing pain
 Throwing sports  Causes swelling and irritation
 Weight lifting

SLIDE NOTE:
 Shoulders are also prone to overuse injuries, such
as tendonitis. Throwing sports and weight-lifting
can cause sore shoulders, so it is important to use
proper technique in these sports. A supervised
program can help keep participants injury-free.

ROTATOR CUFF TEARS


 Involves acute injuries, or may be due to overuse
injuries such as sports and occupations that
involve repetitive overhead motions

REVIEW: Editor’s Notes:


 Rotator cuff muscles  When you raise your arm to shoulder height, the space
between the acromion and rotator cuff narrows. The
 Supraspinatus
acromion can rub against (or "impinge" on) the tendon
 Infraspinatus and the bursa, causing irritation and pain.
 Teres minor
 Subscapularis
 The first three muscles mentioned above inserts at BURSITIS
the greater tuberosity while the Subscapularis  Inflammation of a bursa, small sac located
inserts at the lesser tuberosity between bone, muscle, skin or tendon
 The bursa normally allows smooth gliding
 Subacromial bursitis – impingement/calcium
deposits
 Other areas:
 Pre-patellar (housemaid’s knee)
 Olecranon (prudent medical student elbow
:p)
 Retrocalcaneal

SLIDE NOTE:
 Another common cause of shoulder pain,
especially in the over-40 set, is a rotator cuff tear.
The rotator cuff is composed of the four tendons Editor’s Notes:
 There are four bursae located around the knee joint.
that merge at the top of the shoulder to move and
They are all prone to inflammation, or bursitis.
rotate the upper arm. A rotator cuff tear may However, the prepatellar bursa (the bursa in between
result suddenly from a single traumatic event the skin and the kneecap) is most commonly affected.
but, is more likely to develop gradually because
of repetitive overhead activities.
 People who are especially at risk for overuse are PLANTAR FASCIITIS
those who engage in repetitive overhead motions.  Micro tears of plantar fascia
These include participants in sports such as  Painful heel
baseball, tennis, weight lifting, and rowing.  Commonly seen in dancers and gymnast
Occupations that make people vulnerable include
house painting and jobs that involve heavy lifting.

ROTATOR CUFF/SHOULDER IMPINGEMENT


 Common source of pain
 Tendinitis - the rotator cuff tendons can be
irritated or damaged

Transcriber/s: Jan Cynric Cacao


Formatting: Jan Cynric Cacao D L S H S I M e d i c i n e B a t c h 2 0 1 6 | 10 of 13
Editor/s: Paolo Ramirez, Leanne Gale Ramos
SLIDE NOTE:
 Surgery is rarely needed to treat plantar fasciitis.
Taping the heel and arch and wearing gel
inserts in a supportive shoe or sneaker are helpful.
Some individuals need custom-made shoe inserts
called orthotics.
 Regular stretching that works to increase the
flexibility of the plantar fascia and calf can help.
No special equipment is needed.
 You need two stretches to improve calf flexibility
(one for the upper calf and one for the lower calf)
and one additional stretch for the plantar fascia
itself.
 For the upper calf, you can stand on a step with
only the front half of your foot, then lower your
heel and hold it there for 10 seconds. Repeating
this stretch 10 times is recommended.
SLIDE NOTE:  For the lower calf, lean forward onto a
 Plantar fasciitis is pain in the bottom of the heel, countertop, spreading your feet apart with one
usually felt on the first step out of bed in the foot in front of the other. Flex your knees and
morning or when walking again after resting squat down, keeping your heels on the ground as
from activity. If it persists, this pain can long as possible. Your heel cords and foot arches
eventually be felt during all weight-bearing will stretch as the heels come up in the stretch.
activity. It is not uncommon for the pain to Hold for 10 seconds, relax and straighten up.
radiate down the bottom of the foot toward the Repeat 20 times.
toes.  The plantar fascia stretching exercise requires
 The plantar fascia is dense band of tissue deep sitting with one leg crossed over the other, and
below the skin in the foot. Plantar fasciitis is stretching the arch of the foot by taking one
thought to be the result of the repetitive hand and pulling the toes back toward the shin
stretching that causes micro tears in these dense for a count of 10. The exercise must be repeated 10
bands of tissue as they arise from the heel. The times, and performed at least three times a day,
tears build up over time, making plantar fasciitis including before taking the first step in the
an overuse injury, which left unchecked can easily morning and before standing after a prolonged
become chronic. Sudden weight gain may be a period of sitting.
contributing factor in some instances. Sometimes  If you suffer from plantar fasciitis, you should be
it is due to a change in shoewear. It should be particularly careful to warm up well before
noted that plantar fasciitis can result in bone stretching and to ice your heel after stretching
spurs. for 20-30 minutes at the point of maximum
tenderness. Arch massage can provide relief and
can be done by rolling your foot and arch area
TREATMENT over a tennis ball. Taking oral anti-
 Tape heel, arch inflammatory medications, trying a night splint
 Orthotics prescribed to keep the fascia stretched while you
 Stretch (calf and plantar fascia) sleep, and massaging your heel with sports
 Against wall cream may also be helpful.
 On a step  There are quite a few things you can do to avoid
 Plantar fascia stretch the painful experience of plantar fasciitis in the
 Massage may be helpful first place:
 Warm up well before stretching  Don’t run to lose weight after rapid
 Ice heel, 20-30 minutes weight gain—start with walking, and be
 Anti-inflammatories sure to stretch your foot and calf
 Night splint thoroughly before and after.
 Wear good, supporting shoes  Wear good, supporting shoes for athletic
 Arch support activities.
 Avoid activities that cause heel pain  Keep the muscles of your feet and
 Reduce pain and inflammation ankles strong to support your arch.
 Prevent disability and recurrence  Rest from activities that cause you pain
 RICE in the heel.
 Medications/NSAIDS  And as always, see your physician if your
 Physical/Occupational therapy pain persists.

Transcriber/s: Jan Cynric Cacao


Formatting: Jan Cynric Cacao D L S H S I M e d i c i n e B a t c h 2 0 1 6 | 11 of 13
Editor/s: Paolo Ramirez, Leanne Gale Ramos
PREVENTING SPORTS INJURIES SLIDE NOTE 1:
 Kids should be well coached & supervised  Whether you are a child, a baby boomer, or a
 Wear properly fitted, maintained & appropriate senior citizen, stretching before engaging in
protective gear physical activity is one of the best things you can
 Play on safe surfaces do to protect yourself from injury. However, the
 Children should train for their sport prior to way you stretch is key to reaping its benefits.
playing  Before you stretch, warm up your muscles.
 Know and abide by rules Remember, warming up and stretching are not the
 Wear appropriate protective gear same thing. An efficient warm-up can include
 Know how to use equipment marching, walking in place while swinging your
 Never “play through pain” arms, jumping jacks, or mimicking the sport you
 Skilled instruction are about to do. The point is to get your heart rate
 Wear safety gear! up and your muscles warm and flexible, as cold
 Year-round conditioning muscles are more prone to injury. “Warming up
usually involves breaking a light sweat. It prepares
your body for the activity to follow. It is especially
SLIDE NOTE: important in aging athletes and individuals who
 Many sports-related injuries, for younger and older have had injuries or who have tight muscles.
athletes alike, are preventable. As adults  If you have especially tight muscles, stretching
responsible for young athletes, we owe it to them after the activity is also advised.
to ensure that they are well coached and  After warming up, your basic stretching routine
supervised, wearing protective gear that is well should cover all of the major muscle groups. The
fitted and in good condition, and playing on safe order of the stretches is not critical, but rather
surfaces. stretching thoroughly should be your focus.
 Many of us falsely assume that by virtue of their Depending upon the sport you choose, there are
youth, children are in peak physical condition. This also specialized stretches you will benefit from.
is not a given. Children should train for the Stretching before sports activities promotes better
particular sport they wish to participate in prior to performance and results in fewer injuries.
playing the sport, rather than expecting the sport Investigate the best stretches for your particular
to get them into shape. Careful well-structured sport and add them to your basic stretching
and closely supervised weight training may routine for optimum results.
modestly assist some youngsters in preparing for
athletic activities.
SLIDE NOTE 2:
STRETCHING  Keep your shoulders, hands and feet relaxed at all
 Warm- up: times during flexibility and stretching exercises.
 Break a sweat  Don’t forget to breathe slowly, using deep,
 Marching rhythmic breathing.
 Walk in place  Avoid bouncing, but rather focus on slowly
 Jumping jacks lengthening into your stretches.
 Mimic the sport you are about to do  Be sure to stretch both the right and left sides of
 Stretching: an area to maintain balance and symmetry.
 You should NOT feel pain
 Hold stretch 30 seconds
 Relax into the stretch
 Breathe deeply & slowly
 Avoid bouncing
 Stretch both right & left sides

-END-

TRANSCRIPTION DETAILS
BASIS Latest PPT RECORDINGS + NOTES + DEVIATIONS 3-8% CREDITS -
REMARKS Sources: Gulick (2005) Ortho notes clinical examination pocket guide; Busconi & Stevenson (2009) Sports Medicine Consult.
-BATCH 2016 Transcribers’ Guild Transcriptions. Version 1.0.0.0.0 Build 3112-

Transcriber/s: Jan Cynric Cacao


Formatting: Jan Cynric Cacao D L S H S I M e d i c i n e B a t c h 2 0 1 6 | 12 of 13
Editor/s: Paolo Ramirez, Leanne Gale Ramos
PAST E 2013
Most commonly affected joint in Complete joint disruptions with injury A 25 year old female tennis player
sprains: to capsule, ligaments, and articular noted pain and swelling at her right
a. Knee surfaces are: elbow after practice. On PE, there is
b. Wrist a. Joint subluxation point tenderness over the right
c. Elbow b. JOINT DISLOCATION lateral elbow and pain on resisted
d. ANKLE c. Bursitis dorsiflexion of the wrist. The most
e. Shoulder d. Muscle strains likely diagnosis is:
e. Fractures a. Elbow dislocation
The medical care of athletes involves b. Medial epicondylitis
the following, EXCEPT: Intrinsic factors that causes overuse c. LATERAL EPICONDYLITIS
a. ON-FIELD SURGICAL syndrome in sports: d. Olecranon bursitis
MANAGEMENT OF INJURIES a. Incorrect equipment e. Lateral condylar fracture
b. Pre-participation assessment b. Training error
c. Rehabilitation c. Faulty technique A 20 year old basketball player noted
d. Return to sports d. MALALIGNMENT OF LEG pain on his anterior right thigh after a
e. Poor environment rebound play. He is limping with
Most commonly affected ligaments in severe tenderness and a palpable
ankle sprains: lump on his proximal thigh. The most
Treatment modalities for overuse
a. Anterior tibo-fibular ligament likely diagnosis is:
syndromes include the following
b. Anterior talo-calcaneal ligament a. Hamstring muscle strain
EXCEPT:
c. ANTERIOR TALO-FIBULAR a. Analysis and correction of b. Patellar tendinitis
LIGAMENT causative factors c. QUADRICEPS MUSCLE TEAR
d. Subtalar ligament b. Surgery d. Patellar tendon rupture
e. Posterior talo-fibular ligament c. Physical modalities e. Anterior cruciate ligament rupture
d. Drugs
e. NONE OF THE ABOVE On site management for No. 15 is:
The most sensitive physical
a. Stretching
examination/test for acute anterior
b. Ultrasound
cruciate ligament injury? In hamstring muscle strains, return to
c. ICE COMPRESS
a. Apprehension test sports can be safely advised once
d. Warm compress
b. Apley’s test hamstring muscle strength is:
e. None
c. LACHMANN’S TEST a. 20-30% compared to contralateral
d. Anterior drawer test side
e. Posterior drawer test b. 5% of the uninjured side Plantar fasciitis is said to be caused
c. 75% of the contralateral side by:
d. 90% OF THE UNINJURED SIDE a. PRONATION OF THE FOOT AND
Dense connective tissues that link OVER STRETCHING OF THE FASCIA
bone to bone and gives stability to e. None of the above
AT HEELSTRIKE
joints:
b. Supination of the foot at heelstrike
a. Tendon A 10 year old sprinter experiences a
c. Subtalar inversion
b. Fascia severe pain on the posterior thigh as
d. Dorsiflexion of the ankle
c. LIGAMENTS soon as he starts his run. Physical
e. None of the above
d. Bursa examination showed swelling and
e. Muscles tenderness at posterolateral aspect
of left thigh. The most likely diagnosis Treatment modalities for plantar
is: fasciitis is/are:
Muscle injuries in sports can be a. Stretching exercises
characterized as follows EXCEPT: a. Quadriceps muscle strain
b. Anterior cruciate ligament tear b. NSAID
a. Can be caused by direct trauma
c. Lateral meniscal tear c. Ultrasound
b. 10-30% of all sport injuries
d. Paterlla tendon rupture d. RICE
c. CANNOT BE CAUSED BY INDIRECT
e. HAMSTING MUSCLE STRAIN e. ALL OF THE ABOVE
TRAUMA OR OVERLOAD
d. Can occur at origin, belly, muscle-
tendon junction After an hour, the same patient noted The most common site of muscle
e. None of the above increase in severity of pain and rupture is:
swelling of left thigh. Physical a. Tendon insertion to bone
examination revealed pain on range b. Belly
The following are predisposing
of motion of left knee. The most likely c. MYOTENDINOUS UNIT
factors to muscle rupture in sports
consideration is: d. Avulsion fracture
EXCEPT:
a. Lack of flexibility a. Femoral fracture e. Tendon
b. Inadequate warm-up b. Knee dislocation
c. Fatigued muscles c. Paterllar dislocation Test for meniscal pathology is:
d. Previous injury d. COMPARTMENT SYNDROME a. Lachmann’s test
e. NONE OF THE ABOVE e. Posterior hip dislocation b. Anterior drawer’s test
c. Thompson test
d. MCMURRAY TEST
e. Apprehension test

Transcriber/s: Jan Cynric Cacao


Formatting: Jan Cynric Cacao D L S H S I M e d i c i n e B a t c h 2 0 1 6 | 13 of 13
Editor/s: Paolo Ramirez, Leanne Gale Ramos
OSTEOPOROSIS
Jose Ramon C. Pascual, MD, FPOA, MD
September 29, 2014; 1:00 – 2:30 PM
Orthopedics

LECTURE OUTLINE:
 Definition  Physical Examination
 Natural History  Laboratory Findings
 Epidemiology  Diagnosis
 Risk Factors  Treatment
 Past Medical History  Conclusion
 Medications

DEFINITION EPIDEMIOLOGY
 Orthopedic definition: A metabolic bone disorder  Mortality after Vertebral Fractures
of INCREASED BONE RESORPTION RELATIVE  The mortality rate during the 22 years
TO BONE DEPOSITION leading to DECREASED following the diagnosis of a vertebral
BONE MASS & POROUS fracture according to sex
MICROARCHITECTURE predisposing the person  Male patients was 111.7 per 1,000
to FRAGILITY FRACTURES person-years as compared with 73.4
 IM definition: Defined as a REDUCTION IN THE per 1,000 person-years among the
STRENGTH OF BONE that leads to an male population at risk.
INCREASED RISK OF FRACTURES. Loss of bone  Female patients was 95.1 per 1,000
tissue is associated with DETERIORATION IN person-years as compared with 62.0
SKELETAL MICROARCHITECTURE per 1,000 person-years among the
 WHO definition: Bone density that FALLS 2.5 female population at risk
STANDARD DEVIATIONS BELOW THE MEAN for  Mortality after Hip Fractures
young healthy adults of the same sex (also  Older adults have a 5- to 8-fold increased
referred to as a T-score of -2.5). risk for all-cause mortality during the first 3
months after hip fracture.
 Excess annual mortality persists over time
for both women and men, but at any given
age, excess annual mortality after hip
fracture is higher in men than in women
 Mortality associated with All low trauma
fractures
 All low trauma fractures were associated with
increased mortality for 5-10 years
 Subsequent fracture associated with increased
mortality risk for an additional 5 years
 Prevalence of Osteoporosis in the Philippines
 2003 – 1 million
 2020 – 4 million
 2050 – 10 million
*Electron microscope view of a normal bone and an  Burden
osteoporotic bone.  By 2020, the number of hip fractures will be
65,000
NATURAL HISTORY  By 2050 the number of hip fractures will be
Osteoporosis  Fragility Fracture/Subsequent fracture 175, 000
Death
BIGGEST NOTES:
FRAGILITY FRACTURE The most common fragility fractures:
 A fracture that occurs spontaneously or following - Spine - Lumbosacral
minor trauma - Distal Radius
 Fall from a standing height - Hip region – Proximal Femur, Femoral Neck
 Fall from a sitting position - Proximal Humerus
 Fall from laying down on a bed or reclining
deck chair from less than a meter high
 Fall after having missed 1-3 steps in a
staircase
 Fall after movement from outside of the
typical plane of motion
 Coughing

Transcriber/s: Paolo M. Ramirez


Formatting: Paolo M. Ramirez D L S H S I M e d i c i n e B a t c h 2 0 1 6 | 1 of 8
Editor/s: Leanne Gale Ramos & Jan Cynric Cacao
SLIDE NOTES  Ankylosing spondylitis
 Systemic lupus erythematosus
(Analogy on the burden of Osteoporosis):
 Polyarticular juvenile idiopathic arthritis
 The Air Philippines Airbus A320 can carry 180
 Renal insufficiency
people in an all Coach setup. The larger PAL
B747 can carry between 400 to 500 people.  Hematologic disorders
 Multiple myeloma and other monoclonal
 By 2020, there will be one planeload (Airbus
gammopathies
A320) of hip fracture cases PER DAY.
 Lymphoma and leukemia
 By 2050 that plane will be a Boeing 747. Each
 Mastocytosis
surgery will cost no less than USD 2000. It is  Haemophilia
not clear if all patients will be able to afford  Sickle-cell disease
surgery.  Thalassemia
 Several inherited disorders
RISK FACTORS  Osteogenesis imperfect
 Marfan syndrome
 Hemochromatosis
Modifiable Non-Modifiable
 Hypophosphatasia
 Chronic Alcoholism (Intake  Advanced age (both  Glycogen storage diseases
greater than 3 units/day) sexes)  Homocystinuria
 Vitamin D Deficiency  Female sex  Ehlers-Danlos syndrome
 Tobacco use  European or Asian  Porphyria
 Malnutrition ancestry  Menkes’ syndrome
 Excess/Inadequate physical  Family history (fracture  Epidermolysis bullosa
activity & osteoporosis)  Gaucher’s disease
 Heavy Metals \m/  Past medical history of
 Soft drinks osteoporotic fracture MEDICATIONS
 Estrogen deficiency as an adult  Steroid-induced osteoporosis (SIOP) arises due
 Low body weight [<58kg  Dementia to use of glucocorticoids
(127 lbs)]  Analogous to Cushing’s syndrome and involving
 Low calcium intake mainly the axial skeleton
 Recurrent falls  Prednisone is a main candidate after prolonged
*RED – lifted from Harrison’s intake
 Guidelines recommend prophylaxis in patients who
take the equivalent of more than 30 mg
PAST MEDICAL HISTORY MAY REVEAL hydrocortisone (7.5 mg of prednisolone), especially
ASSOCIATED DISEASES when this is in excess of 3 months. Alternate day
use may not prevent this complication.
DISCLAIMER: this part was discussed only briefly, but it would  Barbiturates, phenytoin and some other
help to understand the specific etiology of the diseases unto enzyme-inducing antiepileptics – these probably
why they bring about osteoporosis accelerate the metabolism of vitamin D.
 L-Thyroxine over-replacement may contribute to
 Prolonged immobilization
osteoporosis, in similar fashion as thyrotoxicosis
 Hypogonadal states
does. This can be relevant in subclinical
 Endocrine disorders hypothyroidism.
 Cushing’s syndrome
 Several drugs induce hypogonadism, for example
 Hyperparathyroidism
aromatase inhibitors used in breast cancer,
 Thyrotoxicosis
 Hypothyroidism methrotrexate and other anti-metabolite drugs,
 DM type 1 & 2 depot progesterone and gonadotropin-releasing
 Acromegaly hormone agonists
 Adrenal insufficiency  Anticoagulants – long term use of heparin is
 Malnutrition, parenteral nutrition and associated with a decrease in bone density, and
malabsorption can lead to osteoporosis warfarin (and related coumarins) have linked with
 Coeliac’ disease an increased risk in osteoporotic fracture in long-
 Crohn’s disease term use.
 Lactose intolerance  Proton pump inhibitors – these drugs inhibit the
 Surgery (after gastrectomy, intestinal bypass production of stomach acid; it is thought that this
surgery or bowel resection) interferes with calcium absorption chronic
 Severe liver disease (especially primary biliary phosphate binding may also occur with
cirrhosis) aluminium-containing antacids.
 Patients with bulimia  Thiazolidinediones (for diabetes) – rosiglitazone
 Those with an otherwise adequate calcium intake and possibly pioglitazone, inhibitors of PPARy,
can develop osteoporosis due to the inability to have been linked with an increased risk of
absorb calcium and/or vitamin D osteoporosis and fracture
 Other micro-nutrients such as contribute  Chronic lithium therapy has been associated with
 Rheumatologic disorders either as part of their osteoporosis
disease or because of other risk factors (notably
corticosteroid therapy)
 Rheumatoid arthritis

Transcriber/s: Paolo M. Ramirez


Formatting: Paolo M. Ramirez D L S H S I M e d i c i n e B a t c h 2 0 1 6 | 2 of 8
Editor/s: Leanne Gale Ramos & Jan Cynric Cacao
PERTINENT PHYSICAL EXAM FINDINGS
 Osteoporosis Screening tool for Asians [OSTA]
OF FRAGILITY (ACUTE)  A free, simple validated tool using only the
 The patient might present acutely with fractures weight and age of the patient.
in areas of cancellous bone namely: the vertebral
bodies, the hip and the wrist. The patient may
also have fractures in the ribs and proximal
humerus

IMAGING STUDIES
 Obtain radiographs of the affected area in
patients who are symptomatic. Lateral spine
radiographs are obtained in patients who are
asymptomatic and at risk for detection of
vertebral fracture.
 Radiographs may show fractures or other
conditions, such as osteoarthritis, disk
disease, or spondylolisthesis.
 Osteopenia (low bone density) may be
apparent as radiographic lucency BUT is not
always noticeable until 30% of bone mineral
is lost.
 Plain radiography is not as accurate as bone
mineral density (BMD) testing using DEXA
 The patient may present after several weeks or and should not be used as a method of bone
months with a Dowager’s hump – a spine densitometry
rendered kyphotic by one or more vertebral body
fractures, and some loss in height
BIGGEST NOTES:
The following are observed in an osteoporotic patient:
 History of loss of height
 Low body weight (BMI <19 kg/m2)
 Kyphosis, cervical lordosis (dowager hump), point
tenderness over a vertebrae or other fracture site
 Signs suggestive of existing osteoporosis
 Exaggerated cervical lordosis
 Thoracic kyphosis
 Loss of lumbar lordosis

LABORATORY FINDINGS
Disease Calcium Phosphate Alkaline Phosphatase PTH
Osteoporosis N N N (elevated in fractures) N Decreased bone mass
Osteomalacia Low Low Variable High Soft bone; decreased mineralization
Osteitis Fibrosa Cystica High Low High High Brown tumors
Osteopetrosis N N Elevated N Thickened bone/ Marble bone
Paget’s Disease of Bone N N Variable N Abnormal bone architecture

 Markers of Bone Turnover may be elevated


 Some of these biochemical measures include the following:
 Bone-specific alkaline phosphatase (bone formation)
 Osteocalcin (bone formation)
 Type I procollagen peptides (bone formation)
 Urinary deoxypyridinoline and cross-linked N- and C-telopeptide of type I collagen (bone resorption)
 Further study is needed to determine their clinical utility in osteoporosis management

Transcriber/s: Paolo M. Ramirez


Formatting: Paolo M. Ramirez D L S H S I M e d i c i n e B a t c h 2 0 1 6 | 3 of 8
Editor/s: Leanne Gale Ramos & Jan Cynric Cacao
DUAL ENERGY X-RAY ABSORPTIOMETRY
 Gold standard for noninvasive evaluation of bone density and best predictor of fracture risk. Although
measurement at any site can be used to assess overall fracture risk, measurement at a particular site is the best
predictor of fracture risk at that site (Lumbar & Pelvis)

 BMD should be measured in the following people:


 Postmenopausal women older than 65 years because,
although preventive measures may no longer be
effective, these women are at risk and should be
treated if they have osteoporosis
 Postmenopausal women younger than 65 years who
have 1 or more risk factor
 Postmenopausal women who present with fragility
fractures
 Women who are considering therapy in which BMD
will affect that decision
 Women who have been on hormone replacement
therapy (HRT) for prolonged periods
 Men who experience fractures after minimal trauma
 People with evidence of osteopenia on radiographs or a disease known to place them at risk for osteoporosis.
 Advantages of DEXA
 Dual-energy x-ray absorptiometry (DEXA) requires less radiation, is less expensive, and has better reproducibility than
quantitative computed tomography.
 This method can be used in both adults and children.
 It can also measure bone density at the spine and the hip. It has become the standard method for determining bone density.
 Limitations of DEXA
 Differences in calibration have been noted. Whenever possible, the same technologist should perform subsequent
measurements on the same patient using the same machine.
 Confounding factors in DEXA results interpretation (falsely high bone density) include spinal fractures, osteophytosis, and
extraspinal (eg, aortic) calcification, use of drugs like strontium ranelate
 Monitoring bone mineral density in postmenopausal women in the first three years after starting treatment with a potent
bisphosphonate is unnecessary and may be misleading. Routine monitoring should be avoided in this early period after
bisphosphonate treatment is commenced.

DIAGNOSIS
 WHO Criteria for Diagnosing Osteoporosis Based on BMD [MUST KNOW]

BMD BIGGEST NOTES:


x≥-1 SD Normal OSTA – For assessment of risk of OSTEOPOROSIS
-2.5<x<-1 SD Osteopenia FRAX – For assessment of risk of FRACTURES
x≤-2.5 SD Osteporosis
With Fragility Fracture Severe Osteoporosis

 Fracture Risk Assessment with FRAX


 All postmenopausal women with at least one WHO risk factor should have fracture probability assessed using the Fracture Risk
Assessment Tool (FRAX) tool before undergoing central DXA

EDITOR’S NOTES: (ADDITIONAL INFO REGARDING OSTA AND FRAX)


OSTA
 A simple risk index called the Osteoporosis Screening Tool for Asians (OSTA), based only on two variables, age (in years)
and body weight (kg/lbs), is a tool used in identifying the risk of osteoporosis among postmenopausal women.
 A useful tool in identifying individuals who will require Bone Mineral Density (BMD). BMD measurement will be relevant to
the definitive diagnosis, fracture risk prediction and treatment decisions in high-risk patients.
 This index has been validated in Japan, Korea and other Caucasian populations as a useful tool in identifying individuals who
will require BMD measurement.
FRAX
 The Fracture Risk Assessment Tool (FRAX) was developed by the World Health Organization (WHO) task force in 2008 to
provide a prediction tool for assessing an individual’s risk of fracture in order to provide general clinical guidance for
treatment decisions.
 Algorithm that incorporates risk factors including previous fractures, smoking, alcohol, glucocorticoids, rheumatoid arthritis,
and bone mineral density, age, weight, sex, and height.
 Based on individual models that integrate the risks associated with clinical risk factors as well as bone mineral density
(BMD) at the femoral neck.

Transcriber/s: Paolo M. Ramirez


Formatting: Paolo M. Ramirez D L S H S I M e d i c i n e B a t c h 2 0 1 6 | 4 of 8
Editor/s: Leanne Gale Ramos & Jan Cynric Cacao
TREATMENT
IF THE PATIENT HAS BMD, START TREATMENT IF FOR THOSE WITHOUT BMD MEASUREMENT, IT IS
THERE IS: SUGGESTED THAT TREATMENT BE STARTED IF THE
 Vertebral compression fracture/s evident on VFA PATIENT:
or confirmed through radiograph (clinical  Belongs to the high risk category based on OSTA
osteoporosis); tool where central BMD cannot be done or not
 BMD T-score of < or equal to -2.5; available
 BMD T-score between -1 and -2.5 SD with any of  Has a 10-year probability of hip fracture > or equal
the following: to 3% or any major osteoporosis related fracture
 History of previous fracture of greater than or equal to 20% based on the
 Secondary causes associated with high risk for FRAX estimates
fracture
 10-year probability of hip fracture > or equal to NON-PHARMACOLOGIC TREATMENT: EXERCISE
3% or any major osteoporosis related fracture of  Weight-bearing and resistance training exercises
> or equal to 20% based on the FRAX have been shown to have a positive effect on
BMD [to reduce atrophy]
 Exercise also improves agility and balance,
thereby reducing the risk of falls (Tai Chi Chuan)

DIETARY REQUIREMENTS
 Calcium Requirement for Post-Menopausal
Women is 800mg/day
 Vit D Requirement for PMW is 10-15ug/day

Strontium
Alendronate Denosumab Raloxifene Teriparatide
Ranelate
Osteoporosis Osteoporosis Osteoporosis; Osteoporosis, risk Osteoporosis
st
[1 line drug] osteoarthritis of reduction of breast
[Bisphosphonate] hip and knee cancer in
Indication
postmenopausal
women with
osteoporosis
Reduces bone RANK-L Decreases Selective estrogen Biosynthetic peptide
resorption by inhibitor osteoclast activity receptor modulator fragment of the biologically
inhibiting and increases active region of human PTH
MOA
osteoclasts osteoblast activity preferentially stimulates
osteoblastic activity over
osteoclastic activity
Antiresorptive? Yes Yes Yes Yes No
Promotes Bone
No No Yes Yes Yes
Deposition?
70mg Once Weekly 60mg SC once 1 2gm sachet daily 1 60mg tab daily 20ug SC daily for 24 months
Dosage every 6
months
Oral, taken with SC Oral Oral SC
one full glass of
water at least
30min before first
Mode of
food/drink of the
Administration
day; remain in
sitting position for
at least 30min,
swallowed whole
GI Irritation Cataracts, skin Venous Venous Osteosarcoma (in rats)
[can cause erosive infections, thromboembolism Thromboembolism
gastritis hence the osteonecrosis
need to be sitted of jaw in
upright for at least patients with
Adverse Effects
30 mins] advanced
cancer,
atypical
femoral
fractures
Correct Correct Contraindicated in Do not use in Exclusion of other disease
Special hypocalcemia, d/c if hypocalcemia patients with CVS patients at risk of apart from osteoporosis,
Precautions GI irritation occurs disease; severe VTE patients at risk for
renal disease osteosarcoma
US FDA Yes Yes No Yes Yes
Approved

Transcriber/s: Paolo M. Ramirez


Formatting: Paolo M. Ramirez D L S H S I M e d i c i n e B a t c h 2 0 1 6 | 5 of 8
Editor/s: Leanne Gale Ramos & Jan Cynric Cacao
70mg 4’s (PHP PHP 18490/ 28 sachets per box 28’s PHP 87.00/tab USD 7,529 per year
818.18/box) (PHP pre-filled (PHP 3402/box) (PHP 31755 per year)
Cost
9816 per year) syringe (PHP 40,824 per
year)
None Treatment of Enhanced Fracture Healing
Interesting Off
segmental bone
Label Use
defects
What drugs Denosumab/Zolendranate
work in
synergy?

TREATMENT FOR EXTRACAPSULAR HIP FRACTURE  Partial/Hemiarthroplasty (only the femoral


 “Fracture outside of the capsule” head)
 Heal well on their own
 Compression Fixation for a Fractured Hip
[Treatment of Choice]

 Total Hip Arthroplasty (Acetabulum + Femoral


Head)

An extramedullary device. The screw holds and goes up


the femoral head and holds the intertrochanteric fracture
together.

TREATMENT FOR UNDISPLACED INTRACAPSULAR


HIP FRACTURE
 In-situ fixation [Treatment of choice]
TREATMENT FOR VERTEBRAL FRACTURES
 Vertebroplasty
 Can be done in an outpatient basis
 Insert a needle through the pedicles, guided
by an image intensifier, and inject a
POLYMETHYLMETHACRYLATE CEMENT.
 Prevents motion therefore decreasing pain

*Multiple screw fixation

TREATMENT FOR DISPLACED INTRACAPSULAR HIP


FRACTURES
 Does not heal well. Sometimes end up with
avascular necrosis of the femoral head or non-
union of the femoral neck (hence the need for
replacement)

Transcriber/s: Paolo M. Ramirez


Formatting: Paolo M. Ramirez D L S H S I M e d i c i n e B a t c h 2 0 1 6 | 6 of 8
Editor/s: Leanne Gale Ramos & Jan Cynric Cacao
EDITOR’S NOTES:
 Vertebroplasty is a pain treatment for vertebral
compression fractures that fail to respond to
conventional medical therapy, such as minimal or no
pain relief with analgesics or narcotic doses that are
intolerable.
 Vertebroplasty, a nonsurgical treatment performed by
interventional radiologists using imaging guidance,
stabilizes the collapsed vertebra with the injection of
medical-grade bone cement into the spine. This reduces
pain, and can prevent further collapse of the vertebra,
thereby preventing the height loss and spine curvature
commonly seen as a result of osteoporosis.
 Vertebroplasty is an outpatient procedure using X-ray
imaging and conscious sedation. The interventional EDITOR’S NOTES:
radiologist inserts a needle through a nick in the skin in  Balloon kyphoplasty is a minimally invasive
the back, directing it under fluoroscopy (continuous, treatment for spinal compression fractures (also
moving X-ray imaging) into the fractured vertebra. The called vertebral compression fractures of VCFs).
physician then injects the medical-grade bone cement  Kyphoplasty is similar to vertebroplasty; the main
into the vertebra. Vertebroplasty takes from one to two difference is that kyphoplasty uses a small balloon
hours to perform depending on how many bones are that gently lifts bone fragments into their correct
treated. The cement hardens within 15 minutes and position.
stabilizes the fracture, like an internal cast.

CONCLUSION
 Balloon Kyphoplasty
 Osteoporosis is a significant problem
 Correct the kyphosis of the patient
 It is still possible to secure a good quality of life in
 Insert a balloon and is inflated in between
the twilight years
vertebral body and acts like a “jack” like
 Identify patients at risk
lifting a car
 Institute medical therapy early
 Not permanent. There is still a need to give
the anti-osteoporotic meds since the  Apply appropriate surgical techniques
surrounding bones are still osteoporotic.

-END-

TRANSCRIPTION DETAILS
Latest PPT; Harrison’s Internal 8- Xmark’s the Spot
BASIS RECORDINGS + NOTES + DEVIATIONS CREDITS
Medicine 10% Recordings
REMARKS ARAL MABUTI 
-BATCH 2016 Transcribers’ Guild Transcriptions. Version 2.0.0.0.0 Build 3201-

Transcriber/s: Paolo M. Ramirez


Formatting: Paolo M. Ramirez D L S H S I M e d i c i n e B a t c h 2 0 1 6 | 7 of 8
Editor/s: Leanne Gale Ramos & Jan Cynric Cacao
PAST-E (2013)

A patient suffering from osteoporosis Osteopenia or low bone mass occurs if As of 2013, the first line pharmacologic
would be expected to have: the patient’s T-score is: treatment for osteopenia is this class of
a. Joint pain a. Greater than -1 SD drug
b. Scoliosis b. BETWEEN -1 AND -2.5 SD a. Recombinant PTH
c. Joint swelling c. Less than or equal to -2.5 SD b. Selective estrogen receptor
d. NONE OF THE ABOVE d. None of the above modulators
c. Hormone replacement
Multiple osteoporotic compression Osteoporosis occurs if the patient’s T- d. BISPHOSPHONATES
fractures of the thoracic vertebrae will score is :
cause: a. Greater than -1 SD Alendronate
a. Increased scoliosis b. Between -1 and -2.5 SD a. HAS A PRIMARILY ANTIRESORPTIVE
b. INCREASED KYPHOSIS c. LESS THAN OR EQUAL TO -2.5 SD EFFECT
c. Increased lordosis d. None of the above b. Has a primarily osteoanabolic effect
d. None of the above c. Works on both the antiresporptive
9. BMD is normal if the T-score is : and osteoanabolic arms
Based on the Asian Adult Report of a. GREATER THAN -1 SD d. Netiher
2009, the number of hip fractures in the b. Between -1 and -2.5 SD
Philippines is expected to hit _____ by c. Less than or equal to -2.5 SD Teriparatide (Recombinant Parathyroid
2050. d. None of the above Hormone)
a. 28000 a. Has a primarily antiresorptive effect
b. 34000 10. Which screening examination is the b. HAS A PRIMARILY OSTEOANABOLIC
c. 65000 current gold standard? EFFECT
d. 175000 a. CENTRAL (HIP AND SPINE) DEXA c. Works on both the antiresporptive
b. Volumetric CT and osteoanabolic arms
Based on the papers of Haentjens et al c. Peripheral (Wrist and Calcaneal) d. Netiher
and Kado et al, which statement is true DEXA
regarding the clinical significance of d. Single Energy Absorptiometry If Central DEXA BMD is not available,
osteoporosis? treatment should be started in patients
a. An increased risk for fractures is the A 65 year old female is seen at the ER a. Who have a 10 year probability of hip
only clinical significance of with multiple vertebral fractures and an fracture of ≥3% based on FRAX
osteoporosis. intertrochanteric fracture of the left b. Who belong to the high risk category
b. PATIENTS WITH FRAGILITY hip. The patient’s bone has: based on the OSTA tool
FRACTURES OF THE HIP AND THE a. Normal density c. BOTH A AND B
SPINE HAVE HIGHER RISK OF b. Osteopenia d. Neither
MORTALITY. c. ESTABLISHED OSTEOPOROSIS
c. Fixation of hip fagility fractures d. None of the above For patients with a T-score between -2.5
restores mortality risk to that of the and -1.0, treatment should be started
general population within one year What type of exercise address concerns a. If there is a history of previous
after surgery. in osteoporosis? fragility fracture
a. Balance exercises (Tai Chi Chuan) b. 10-year probability of hip fracture ≥3%
A fracture as a result of minor trauma, or any major osteoporosis-related
b. Resistance exercises
in the setting of osteoporosis but in the fracture of ≥20% based on the FRAX
c. BOTH
absence of any other medical condition estimates
is called: d. Neither
c. BOTH
a. Metastatic fracture d. Neither
The OSTA uses the patients_____ and
b. Pathologic fracture
_____ to screen for osteoporosis
c. FRAGILITY FRACTURE Which statement is true regarding
a. Height and weight
d. None of the above treatment with Alendronate?
b. AGE AND WEIGHT
c. Height and age a. The risk of atypical fractures of the
A parental history of hip fracture is an subtrochanteric area of the femur is
example of a: d. None of the above
According to the FNRI, the daily significantly increased when using oral
a. Secondary cause of osteoporosis alendronate for osteoporosis treatment
recommended daily intake of calcium
b. WHO RISK FACTOR b. Osteonecrosis of the jaw is a
for females ≥50 years is_____:
c. Both common complication when using oral
a. 1000mg
d. Neither bisphosphonates
b. 800mg
c. Alendronate is easily absorbed from
c. 500mg
the GIT
d. 200mg
d. POSTMENOPAUSAL WOMEN WHO
ARE OTHERWISE HEALTHY SHOULD
NOT DISCONTINUE ORAL
ALENDRONATE BECAUSE OF THE RISK
OF OSTEONECROSIS

PUSHEEN, THE ORTHO CAT SAYS,


“ALWAYS KEEP IT COOL”

Transcriber/s: Paolo M. Ramirez


Formatting: Paolo M. Ramirez D L S H S I M e d i c i n e B a t c h 2 0 1 6 | 8 of 8
Editor/s: Leanne Gale Ramos & Jan Cynric Cacao
THE DIABETIC FOOT
Jose Ramon C. Pascual, MD, FPOA
October 6, 2014; 1:00 – 2:00 PM
Orthopedics

OUTLINE
 Pathophysiology of diabetic foot  Predictors for amputation
 History  Risk score strata and lower extremity amputation
 Physical examination rates
 Risk classification  Levels of foot amputation
 Specialist referral  Conclusion
 Non-ablative surgical procedures

GENERAL OBJECTIVES  Hyperglycemia, which is one of the complications


 The student should be able to perform a thorough of diabetes, can produce ischemia and
history of a diabetic foot patient neuropathy. These, together with infection,
 The student should be able to perform a physical contribute to the formation of a diabetic foot.
examination of a diabetic foot patient
 The student should be able to assign the patient SLIDE NOTES:
to a foot risk category and refer to the proper  Neuropathy:
specialists if necessary Segmental demyelination, axonal and
neuronal degeneration and decreased
SPECIFIC OBJECTIVES endoneural blood flow produce sensory
neuropathy which causes loss of protective
HISTORY sensation.
 The student should be able to identify a patient Motor neuropathy which causes paralysis of
with a diabetic foot problem based on risk factors muscles leading to joint deformities and
and other features elicited in the history. abnormal pressure distribution
Autonomic denervation of the skin dries the
PHYSICAL EXAMINATION skin and produces cracks and fissures, loss of
 The student should be able to identify abnormal the axon reflex blunting the hyperemic
dermatologic and musculoskeletal conditions that response to injury
may cause problems in a diabetic foot patient. Loss of autonomic control of the blood supply
 The student should be able to properly assess the to the bone can lead to increased blood flow to
neurologic status of a diabetic foot patient. the bone leading to hyperemia in the bone and
 The student should be able to properly assess the a localized osteoporosis
vascular status of a diabetic foot patient.  Vascular pathologies:
 The student should be able to identify potential Non-cocclusive microvascular impairment
problems in the footwear of a diabetic patient. (thickened basement membrane which impairs
nutrient transfer to tissues
RISK ASSESSMENT Atherosclerosis in popliteal and infrapopliteal
 The student should be able to assign the patient vessels (as opposed to iliofemoral involvement
to a foot risk category. in non-diabetic PAD) with the atheromatous
 The student should be able to refer the patient to plaques serving as a physical impediment to
the proper specialist if necessary. blood flow
 The student should be able to identify and Medial calcinosis produces noncompliant
describe the various orthopedic interventions for vessels which although are patent cannot
the diabetic foot patient. respond to increased demand. There is also
increased platelet aggregation and a
PATHOPHYSIOLOGY hypercoagulable state.
 Infection:
Facilitated by neuropathy and ischemia as well
as poor host defenses secondary to
hyperglycemia and hyperinsulinemia

Transcriber/s: Jan Cynric Cacao


Formatting: Jan Cynric Cacao D L S H S I M e d i c i n e B a t c h 2 0 1 6 | 1 of 14
Editor/s: Paolo Ramirez and Leanne Gale Ramos
EDITOR’S NOTES: EDITOR’S NOTES:
 A classical triad of neuropathy, ischemia, and  Intermittent claudication (IC) - pain on walking
infection are the characteristics of the diabetic that resolves at rest - is not a trivial complaint but
foot. is the first recognisable symptom of peripheral
 Neuropathy leads to fissures, bullae, neuropathic arterial disease (PAD).
(Charcot) joint, neuropathic oedema, digital  Diabetes is one of the most important risk factors
necrosis. for PAD and in people with diabetes it tends to be
 Ischaemia leads to pain at rest, ulceration on foot more distal and severe, and more likely to result in
margins digital necrosis and gangrene. gangrene and amputation.
 Differentiating between these two entities is
essential because their complications are different
and they require different therapeutic strategies. OTHER DIABETES COMPLICATIONS
 Renal (dialysis, transplant)
 Visual (impairment, retinal or cataract problems)

HISTORY PHYSICAL EXAMINATION


 Risk Factors
 Essential Features
WAGNER’S CLASSIFICATION
Grade 0 No ulceration in a high risk foot
KNOWN RISK FACTORS FOR ULCERATION
Grade 1 Superficial ulceration
 Previous Foot Ulceration (RR 1.6)
Grade 2 Deep ulceration that penetrates up to
 Prior Lower Extremity Amputation (RR 2.8)
tendon, bone or joint
 Peripheral Neuropathy
Grade 3 Osteomyelitis or deep abscess
 Foot Deformity
Grade 4 Localized gangrene
 Long Duration of Diabetes (>10years) (OR 3.0)
Grade 5 Extensive gangrene requiring major
 Poor Glycemic Control: HbA1c>9% (OR 3.2) amputation
 Impaired Vision (acuity <20/20) (RR 1.9)
 Diabetic Nephropathy FOOTWEAR ASSESSMENT
 Cigarette Smoking  Does it fit well?
 Is there undue pressure on any portion of the
ESSENTIAL FEATURES IN PAST MEDICAL HISTORY foot?
 Ulceration  Is it securely fastened?
 Amputation  Will it allow for use of orthoses?
 Charcot Joint
 Vascular Surgery
 Angioplasty
 Cigarette Smoking

NEUROPATHIC SYMPTOMS
 Positive (Burning/shooting/tingling sensations)
 Negative (numbness)
 This becomes a problem because the patient
does not feel anything at all, they won’t be
able to feel the vascular symptoms as well

EDITOR’S NOTES:
 In diabetic polyneuropathy (sensory, motor and
autonomic), which leaves the foot with loss of IDEAL PROPERTIES IN DIABETIC FOOTWEAR
protective sensations (LOPS), any damaging  Toebox – should be wide enough for the toes
stimuli or external trauma are either perceived as  Vamp – adjustable straps
less or not at all, resulting in an ulcer.
 Insole – customizable orthotics
 Sensory neuropathy is the most important
prerequisite for foot ulcerations.  Heel counter – prevents front-back shearing
motion
 Depth – allows for fitting of orthotics
 Heel – minimal elevation to prevent excessive
VASCULAR SYMPTOMS pressure on the metatarsal heads
 Claudication
 Rest pain
 Non-healing ulcer

NOTE: these may not come out in patients with


neuropathy. Also, a non-healing ulcer suggests that
there is poor blood flow

Ideal footwear for diabetics

Transcriber/s: Jan Cynric Cacao


Formatting: Jan Cynric Cacao D L S H S I M e d i c i n e B a t c h 2 0 1 6 | 2 of 14
Editor/s: Paolo Ramirez and Leanne Gale Ramos
SLIDE NOTES:
 The term “intrinsic minus foot” was first coined by
Heinz I. Lippman, MD, in 1976 . The longest motor
nerves are those that activate the intrinsic muscles of
the foot (lumbricals).
 Since diabetic neuropathies progress from distal to
proximal, the lumbricals are the first muscles to be
affected by neuropathy. The characteristics of the
intrinsic minus foot that can be discerned on simple
physical examination are described below.
 Hammer toes: the lumbricals wrap around the toes
in such a fashion that they extend the
interphalangeal (i-p) joints and flex the metacarpal-
phalangeal (m-p) joints. When they are denervated,
the opposing muscles will cause flexion of the i-p
Left: Supinator; Middle: Normal; Right: Pronator joints and extension of the m-p joints with the
resultant hammer toes.
 Prominent plantar metatarsal heads: extension of
NOTE: Wear pattern in the sole can tell you whether
the m-p joints causes the appearance of visible and
the patient is a supinator or a pronator.
palpable bony prominences on the bottom of the foot,
at the base of each toe. At first, these prominences
INSPECTION: DERMATOLOGIC are found only at the bases of the first and fifth toes,
 Skin status: color, thickness, dryness, cracking but as neuropathy progresses, all of the toes are
 Sweating affected and calluses can be palpated under each
 Infection metatarsal head.
 Ulceration  Wasting of lumbricals: the lumbricals originate on
 Calluses/Blistering the distal metatarsal bones on one side of each toe,
 Check between toes and soles of feet wrap around the toe, and insert in the opposite side of
each metatarsal. When sufficiently wasted by
denervation, visible channels appear on the dorsum
NOTE: These are also the things to look for when you
of the foot between the metatarsals. Contrary to the
instruct the patient to self-inspect
results of the referenced study (1), observations in our
clinic of hundreds of patients with wasted interossei
INSPECTION: MUSCULOSKELETAL have shown that hammer toes are inevitably
 Deformities: Claw toes, Hallux Valgus Deformity , apparent when visible atrophy is present.
Rocker-Bottom Deformity of Charcot foot,
prominent metatarsal joints
HAMMER TOE/ CLAW TOE
 Check if deformity is mobile or rigid
 Metatarsal
 Muscle wasting (guttering between metatarsals)
phalangeal joint
 Lumbricals
infection
 Presence of Infection (Abscess, Wet Gangrene,
 Usually arises
Infected Ulcers, Osteomyelitis, Necrotizing
with footwear
Fasciitis, Cellulitis)
having narrow
toe boxes
PHYSICAL SIGNS OF THE INTRINSIC MINUS FOOT
 Hammer toes Lumbrical denervation
 Prominent Plantar Metatarsal Heads
 Wasting of Lumbricals HALLUX VALGUS
 Upward Rotation of Forefoot  Inward deformity of the metatarsal phalangeal
 Distal Migration of Plantar Metatarsal Pads joint due to a narrow toebox
 Weak Extension of Great Toe  Can develop into ulcers
 Cock-up Deformity of the Great Toe with
Prominent Extensor Tendon
 High Arch
 Dry Skin

Transcriber/s: Jan Cynric Cacao


Formatting: Jan Cynric Cacao D L S H S I M e d i c i n e B a t c h 2 0 1 6 | 3 of 14
Editor/s: Paolo Ramirez and Leanne Gale Ramos
CHARCOT FOOT WITH ROCKER BOTTOM ABSCESS
DEFORMITY AND MIDFOOT ULCER  Might not be obvious
 Occurs in patients with diabetic neuropathy,
localized osteopenia
 Collapse of the midfoot
 Produces abnormal pressure patterns – can
lead to ulceration of the midfoot

EDITOR’S NOTES:
 An abscess is a mass filled with pus (dead tissue,
bacteria, and white blood cells).
 Abscesses can occur if a gland in your skin
becomes blocked; from inflammation of hair
follicles; and also due to breaks in the skin which
can allow germs to get under the skin, causing
inflammation as our bodies try to deal with it.
EDITOR’S NOTES:  Abcesses can occur more frequently in people with
 Neuropathic arthritis, a.k.a., Charcot’s Disease, diabetes, or occur in higher severity because
is a rapidly progressive degeneration in a joint diabetics are both more prone to infection and heal
which lacks position sense and protective pain more slowly.
sensation.
 Neuropathic joint disease occurs in less than 1%
of diabetic patients, yet diabetes is the commonest OSTEOMYELITIS
cause of a neuropathic joint in Europe and North
America.
 The midtarsal joints are the most commonly
affected, followed by the MTP and ankle joints.

SPECIAL TEST
 Brodsky test
 Distinguishes a Charcot process from
infection in patients with associated plantar
ulcers.
 With the patient supine, the involved lower
extremity is elevated for 5-10 minutes.
 If swelling and rubor dissipate, the diagnosis SLIDE NOTES:
of a Charcot process is supported. If the  Wound with exposed bone
swelling and rubor persist, an infectious  Able to probe to bone
process is likely.  Sequestrum seen on x-ray (pathognomonic for
osteomyelitis)
STAGES OF CHARCOT DEFORMITY  Lysis of bone in area of skin ulcer
 MRI evidence of marrow involvement (MRI with
contrast allows early diagnosis of osteomyelitis and
Stage Characteristics Treatment differentiation from soft tissue infection particularly in
Erythema, edema, Limited weight those with intact skin)
increased bearing
Stage 0:
temperature to (possibly TCC
Clinical EDITOR’S NOTES:
foot or PPWB, close
observation  Osteomyelitis is the medical term for an infection
Periarticular TCC, limited of the bone, and can be a complication of foot
ulcers in diabetic patients.
fractures, joint weight bearing
Stage 1:  Osteomyelitis is not a symptom of diabetes, but
dislocation, diabetics are at higher risk for this infection,
Fragmentation
instability, particularly in the feet.
deformed foot  Over time, diabetes damages the nerves in the
Stage 2: Reabsorption of TCC followed feet, causing reduced sensation, and damages
Coalescence bone debris by CROW blood vessels, causing poor circulation. This makes
Stage 3: Stable foot Surgical people more at risk of injuring their foot without
Reparative intervention realizing it, and poor wound healing, both of which
set them up for foot ulcers. These ulcers can
TCC = total contact cast; PPWB = prefabricated pneumatic
become infected, which can then spread to the
walking brace; CROW = Charcot restraint orthotic walker.
underlying bone.
*— Extra-depth shoes and pressure-relieving orthoses also  "Osteomyelitis" of the diabetic's foot is an all too
may be needed. common problem resulting primarily from the
combination of the neuropathy and vasculopathy
which characterize longstanding diabetes.

Transcriber/s: Jan Cynric Cacao


Formatting: Jan Cynric Cacao D L S H S I M e d i c i n e B a t c h 2 0 1 6 | 4 of 14
Editor/s: Paolo Ramirez and Leanne Gale Ramos
WET GANGRENE EDITOR’S NOTES:
 Necrotizing fasciitis is a rapidly progressive
inflammatory infection of the fascia, with secondary
necrosis of the subcutaneous tissues.
 Type I necrotizing fasciitis is more common in patients
with diabetes and is usually due to a mixed aerobic and
anaerobic infection. Common organisms include S.
aureus, Group A streptococci, Escherichia coli, and a
variety of anaerobic organisms. The portal of entry is
usually on a foot scrape or ulcer. Rapid extension can
occur along the fascia into the leg.

EDITOR’S NOTES:
 Gangrene is a potentially life-threatening condition
that arises when a considerable mass of body tissue dies
(necrosis)
 Wet gangrene occurs in naturally moist tissue and
organs such as the mouth, bowel, lungs, cervix, and
vulva.
 This condition is characterized by thriving bacteria and
has a poor prognosis (compared to dry gangrene) due
to septicemia resulting from the free communication
between infected fluid and circulatory fluid. Necrotizing fasciitis before debridement
 In wet gangrene, the tissue is infected by saprogenic
microorganisms (Clostridium perfringens or Bacillus
fusiformis, for example), which cause tissue to swell
and emit a fetid smell.

CELLULITIS
 Superficial infection of the skin
 Erythema, pain
 Must be differentiated from necrotizing fasciitis

After multiple debridement and minimal amputation

EDITOR’S NOTES:
 In patients with diabetes, any foot infection is
potentially serious. Diabetic foot infections range in
EDITOR’S NOTES: severity from superficial paronychia to deep infection
 Cellulitis is a bacterial infection involving the skin. It involving bone.
specifically affects the dermis and subcutaneous fat.  Types of infection include cellulitis, myositis,
 The bacteria most commonly involved are streptococci abscesses, necrotizing fasciitis, septic arthritis,
and Staphylococcus aureus. tendinitis, and osteomyelitis.
 Diabetics are more susceptible to cellulitis than the  Foot infections are among the most common and
general population because of impairment of the serious complications of diabetes mellitus.
immune system; they are especially prone to cellulitis in  They are associated with increased frequency and
the feet, because the disease causes impairment of length of hospitalization and risk of lower extremity
blood circulation in the legs, leading to diabetic foot/foot amputation.
ulcers.  Foot ulceration and infection are the leading risk
 Poor control of blood glucose levels allows bacteria to factors for amputation.
grow more rapidly in the affected tissue, and facilitates  Prevention and prompt diagnosis and treatment are
rapid progression if the infection enters the necessary to prevent morbidity, especially amputation.
bloodstream.
NEUROLOGIC ASSESSMENT
NECROTIZING FASCIITIS  Evaluate Loss of Protective Sensation (LOPS)
 Infection through the plane of the fascia  Semmes-Weinstein 10g monofilament test + 1 of
the following:
 Vibration Perception Testing with 128 Hz Tuning
fork
 Ankle Reflexes
 Pinprick Test
 Vibration Perception Threshold Testing with a
Biothesiometer

Transcriber/s: Jan Cynric Cacao


Formatting: Jan Cynric Cacao D L S H S I M e d i c i n e B a t c h 2 0 1 6 | 5 of 14
Editor/s: Paolo Ramirez and Leanne Gale Ramos
SEMMES-WEINSTEIN MONOFILAMENT (5.07)/ 10G EDITOR’S NOTES:
 Applied on the metatarsal head and ask the  Ankle reflex is a simple screening tool frequently used in
patient if he/she feels anything the detection of peripheral neuropathy.
 Make sure that the patient does not see you  They are typically absent in diabetic patients.
applying the monofilament Past Tranx Ratio:
 Absence of ankle reflexes has been associated with
 Applied on the plantar aspect of the foot and
increased risk of ulceration.
parts of the dorsal foot particularly the first  Done on the Achilles tendon through light tapping.
webbed space

PIN PRICK TEST


 Pin applied against pulp of
toe or finger
 Pin should only deform
skin and not penetrate
 Inability to perceive pin
prick is an abnormal test
result

BIOTHESIOMETER
 Produces uniform vibration

SLIDE NOTES
 Essentially a powered tuning fork
 Placed over the same site, intensity increased
until patient can feel the vibrations (vibration
perception threshold or VPT)
 Mean of 3 readings used
 Abnormal if VPT>25

TUNING FORK
 Tests to determine if the vibration sense of the
foot is intact
 Applied to the bony prominences of the foot
 Must be done on the contralateral foot
(unaffected side) to assess and compare the loss
of vibration sense

VASCULAR ASSESSMENT
 Pulses
 Dorsalis pedis
 Posterior tibial (behind the medial
malleolus)
 Ankle-Brachial Index
 Past Tranx Ratio: In the Vascular Assessment of
128Hz tuning fork; Applied to bony prominences or tips of
Diabetic Foot, a Doppler stethoscope is used to
toes; (+) if patient can no longer perceive vibration
detect the pulses in the following arteries:
femoral, popliteal, dorsalis pedis, posterior tibial
ANKLE REFLEX TESTING
 Assess problems on the reflex arc

Assessment of the dorslis pedis pulse

Transcriber/s: Jan Cynric Cacao


Formatting: Jan Cynric Cacao D L S H S I M e d i c i n e B a t c h 2 0 1 6 | 6 of 14
Editor/s: Paolo Ramirez and Leanne Gale Ramos
ANKLE BRACHIAL INDEX
 Using a Doppler probe take brachial BP
 Using a Doppler probe take ankle BP (post tibial
or dorsalis pedis)
 Divide Ankle Systolic by Brachial Systolic

 ABI >0.9 is normal


 ABI <0.8 associated with claudication
 ABI <0.4 associated with ischemic rest pain and
tissue necrosis
 ABI >1.3 due to false elevation of ankle systolic
pressure: because of calcified arteries
(incompressible arteries); may require
measurement of transcutaneous oxygen tension
or toe pressure

RISK CLASSIFICATION
Risk
Definition Treatment recommendations Suggested follow-up
category
No LOPS, no
 Patient education including  Annually (by generalist and/or
0 PAD, no
advice on appropriate footwear specialist)
deformity
 Consider prescriptive or
accommodative footwear
LOPS with or  Every 3 to 6 months (by generalist
 Consider prophylactic surgery if
1 without or specialist); referral to
deformity is not accommodated
deformity orthopedic surgeon
by footwear
 Continue patient education
 Consider prescriptive or
PAD with or accommodative footwear  Every 2 to 3 months ( by
2
without LOPS  Consider vascular consultation for specialist)
combined follow-up
History of ulcer or  Same as category 1
3  Every 1-2 months (by specialist)
amputation  Consider vascular consultation

SPECIALIST REFERRAL
Specialist Intervention
Interventional Cardiology  Angioplasty (endovascular revascularization procedures)
Vascular Surgery  Open bypass to restore blood flow
Plastic Surgery  Coverage of difficult defects and optimize/facilitate wound healing
 Debridement of infected wounds
 Non-surgical and surgical off-loading of the foot
Orthopedic Surgery
 Arthrodesis procedures
 Ablative treatment (amputation)

Transcriber/s: Jan Cynric Cacao


Formatting: Jan Cynric Cacao D L S H S I M e d i c i n e B a t c h 2 0 1 6 | 7 of 14
Editor/s: Paolo Ramirez and Leanne Gale Ramos
*NPWT – negative pressure wound therapy

TOTAL CONTACT CASTING SLIDE NOTES:


 Transfers the load of an ulcerated foot  The Total Contact Cast is a non-operative
method of offloading a diabetic foot with a
nonhealing wound in the forefoot and
sometimes midfoot.
 The wound is debrided first. Once it has been
rendered clean, a total contact cast is then
applied.
 Mechanisms by which the TCC achieves
forefoot unloading are (1) transfer of
approximately 30% of the load from the leg
directly to the cast wall, (2) greater
proportionate load sharing by the heel, and
(3) removal of a load-bearing surface from the
metatarsal heads because of the "cavity"
created by the soft foam covering the
forefoot.
 These results point out some of the "essential
design features" of the TCC (which are
different from what had been previously
supposed), support the use of the TCC for
healing plantar ulcers in the forefoot, but
raise questions about its utility in the healing
of plantar ulcers on the heel.

Heel Ulcer after 2 months in a TCC


Movie break maybe?

Transcriber/s: Jan Cynric Cacao


Formatting: Jan Cynric Cacao D L S H S I M e d i c i n e B a t c h 2 0 1 6 | 8 of 14
Editor/s: Paolo Ramirez and Leanne Gale Ramos
NON-ABLATIVE ORTHOPEDIC RESECTION ARTHROPLASTY
 One or both ends of a joint are ablated but the
SURGICAL PROCEDURES FOR DIABETIC two ends are not allowed to fuse (pseudarthrosis
FEET or false joint develops)
 Debridement  Pressure over ulcer is removed, deformity is
 Surgical Offloading corrected, joint motion is maintained
 Deformity correction procedures
 Resection arthroplasty
 Soft tissue lengthening procedures
(percutaneous tendon lengthening, plantar
fascia lengthening)
 Arthrodesis techniques

DEBRIDEMENT
 Surgical removal of infected and necrotic tissue

SLIDE NOTES:
 Removal of necrotic and nonviable tissue
 Removal of exposed tendon
 Removal of exposed bone

SURGICAL OFFLOADING PROCEDURES


 Surgical procedures designed to normalize
pressure distribution around the foot

CORRECTIVE OSTEOTOMY RESECTION ARTHROPLASTY OF THE LESSER


• Bone is cut and realigned to counteract the METATARSOPHALANGEAL JOINT FOR CORRECTION
deformity OF THE SEVERE COMPLEX HAMMERTOE

Modified Brunner approach, overlying the affected


metatarsophalangeal joint.

In a different patient dissection of the extensor tendon


complex is shown before sectioning and exposure of the
dorsal capsule.

Deformity Correction – Corrective Osteotomy/Contracture


Release/Tendon Lengthening/ Pinning

Transcriber/s: Jan Cynric Cacao


Formatting: Jan Cynric Cacao D L S H S I M e d i c i n e B a t c h 2 0 1 6 | 9 of 14
Editor/s: Paolo Ramirez and Leanne Gale Ramos
Release of the proximal phalanx from its associated
plantar plate. This figure also shows appropriate control of
the proximal phalanx, with the use of a bone hook placed
at the base of the phalanx.

Exposure of the dorsal capsule of the


metatarsophalangeal joint.

Development of the T-capsular incision, with isolation of


medial and lateral capsular flaps.

Removal of the base of the proximal phalanx.

Isolation of both capsular flaps in a different patient.

Removal of the proximal phalanx from its associated


plantar structures, thus providing direct osseous
decompression of the deformity.

As dissection of the capsule proceeds in a plantar direction,


control of the proximal phalanx can be achieved with the
use of a bone hook.

Transcriber/s: Jan Cynric Cacao


Formatting: Jan Cynric Cacao D L S H S I M e d i c i n e B a t c h 2 0 1 6 | 10 of 14
Editor/s: Paolo Ramirez and Leanne Gale Ramos
Clearly delineates the amount of osseous decompression
A controlled starting point in the metatarsal head ensures
that was achieved by the resection arthroplasty.
proper alignment for the final position of the toe.

Closure of the purse-string suture and apposition of the


After sectioning of the flexor tendons, a generous amount residual proximal phalanx onto the soft tissue interposition
of capsular tissue (dorsal, medial, lateral, and plantar) material.
remains for soft tissue interposition.

Six-week postoperative clinical photograph of a patient


who underwent resection arthroplasty of the right second
metatarsophalangeal joint, for treatment of complex
hammertoe.

Bundling of the residual capsular tissue with a purse-string


technique.

Six-week postoperative radiograph of a patient after the


0.062 K-wire pin was pulled.

Transcriber/s: Jan Cynric Cacao


Formatting: Jan Cynric Cacao D L S H S I M e d i c i n e B a t c h 2 0 1 6 | 11 of 14
Editor/s: Paolo Ramirez and Leanne Gale Ramos
ACHILLES TENDON LENGTHENING
 Percutaneous cuts are made at the sides of the
Achilles tendon
 Lengthening allows greater dorsiflexion
 Plantar pressure over forefoot is reduced
 Risk of increasing ulcer formation over heel

Before and after plantar fascia release

ARTHRODESIS
 Surgical induction of joint ossification between
two bones (joint fusion)

RETROGRADE NAILING ARTHRODESIS FOR


HINDFOOT DEFORMITIES IN CHARCOT FOOT

PERCUTANEOUS FLEXOR TENOTOMY FOR


HAMMERTOE

DEFORMITY CORRECTION IN A CHARCOT FOOT


WITH A TAYLOR SPATIAL FRAME WITH MIDFOOT
FUSION

Flexor tenotomy. A, The toe is flexible and ulcer is at the


tip. B, Lidocaine is injected at the site of entry, and the
patient is asked to dorsiflex the ankle and actively flex all
the toes. This makes the long flexor tendon very taught,
and can easily be tenotomized as shown here with a #61
Beaver blade. No suture is needed. C, Shows all three
middle toes that have had flexor tenotomies. The fourth
toe had resection of osteomyelitis, 4 weeks
postoperatively, done in the office setting.

SELECTIVE PLANTAR FASCIA RELEASE FOR


NONHEALING DIABETIC PLANTAR ULCERATIONS

Transcriber/s: Jan Cynric Cacao


Formatting: Jan Cynric Cacao D L S H S I M e d i c i n e B a t c h 2 0 1 6 | 12 of 14
Editor/s: Paolo Ramirez and Leanne Gale Ramos
PREDICTORS FOR AMPUTATION EDITOR’S NOTES:
 Syme: Includes ankle disarticulation, removal of
Risk factor Weight malleoli & anchoring heel pad to the wt bearing surface
 Chopart: Removes the forefoot and midfoot, saving
Chronic renal disease or creatine >3 mg/dL 1
talus and calcaneus
Male sex 1  Lisfranc: An amputation proximal to the
Temperature < 96F or >100.5F 2 transmetatarsal level
Age ≥ 50 4  Transmetatarsal: A transverse incision is made either
Infected ulcer vs. cellulitis 4 at the middle or proximal third of the metatarsals,
History of amputation 4 extending through skin into the subcutaneous tissue
 Metatarsal phalangeal disarticulation
Albumin <2.8 g/dL 5
History of peripheral vascular disease 5
WBC > 11 7
BIGGEST NOTES:
Surgical site infection vs. cellulitis 10
 Major amputations are indicated in neuroischemic
Transferred from other acute care facility 12 foot where revscularisation is not feasible, in
advanced infections, in medically compromised
RISK SCORE STRATA AND LOWER patient and occasionally in severe Charcot
EXTREMITY AMPUTATION RATES deformities.
 All the patients who have undergone a major
Risk score LEA rate amputation have a high risk of subsequent
1 to 4 <6% (4.2% to 5.7%) contralateral amputation and therefore a
5 to 11 12% (10.7% to 11.9%) surveillance programme, for the remaining foot is
crucial.
12 to 20 29% (25.9% to 29.3%)
21+ 49% (48.6% to 49.2%)
CONCLUSION
BIGGEST NOTES:  Significant morbidity in diabetic foot cases
 Amputation is the end point of diabetic foot  Identify the patient’s problems early
lesions. Major amputations result in considerable  Perform accurate risk assessment
morbidity and at times even mortality. It leads to  Treat and/or refer early to prevent amputation
permanent disability to the patient. However, in
diabetic foot lesions this unfortunate decision has
to be taken in order to save the life. Stressed?
 Amputations are divided into minor amputations
(up to mid-tarsal level) and major amputations
(above midtarsal level).

LEVELS OF FOOT AMPUTATION

Adopting the right attitude can convert a negative stress into a


positive one! 

-END-

TRANSCRIPTION DETAILS
BASIS Latest PPT RECORDINGS + NOTES + DEVIATIONS 5-8% CREDITS -
References: Apley's and Solomans Consise System of Orthopedics and Trauma 4th Ed, Past Transcriptions
REMARKS
Kulkarni (2008) Textbook of Orthopedics and Trauma 2nd edition
-BATCH 2016 Transcribers’ Guild Transcriptions. Version 2.0.0.0.0 Build 3202-

Transcriber/s: Jan Cynric Cacao


Formatting: Jan Cynric Cacao D L S H S I M e d i c i n e B a t c h 2 0 1 6 | 13 of 14
Editor/s: Paolo Ramirez and Leanne Gale Ramos
PAST-E (2013)

Which part of a shoe prevents front- Neurologic assessment technique Which of the following is a non-
back shearing motion? which relies on light taps on the operative method of offloading a
a. Toebox patient’s Achilles tendon diabetic foot with a non-healing
b. Vamp a. Semmes-Weinstein Monofilament wound?
c. Heel Test a. TOTAL CONTACT CASTING
d. HEEL COUNTER b. Tuning Fork Test b. Corrective osteotomy
c. ANKLE REFLEX TEST c. Resection arthroplasty
An elevated heel may produce ulcer- d. Biothesiometry d. Precutaneous tenotomy
causing pressures in which part of the
foot? Proper technique using the pinprick The surgical procedure that fuses a
a. AREA OF THE METATARSAL technique requires that_____. pathologic joint to obtain stability at
HEADS a. The pin be stroked lightly along the the expense of normal joint motion is
b. Heel pulp of the toe called_____.
c. Medial aspect of the b. THE PIN DEFORM THE SKIN OF a. ARTHRODESIS
metatarsophalangeal joint THE DIGIT b. Corrective osteotomy
d. Lateral aspect of the foot c. The pin puncture the skin (hence the c. Resection-interpositional
term prick) arthroplasty
Physical signs of the Intrinsic Minus d. None of the above d. Tenotomy
foot include_____:
a. Flexion of the MTPI and extension of The posterior tibial artery pulse is The surgical procedure that involves
the IPJ located_____. the creation of defined cuts in the
b. WASTING OF THE LUMBRICALS a. BEHIND THE MEDIAL bone and re-angulating the resulting
c. Both MALLEOLUS fragments to correct a bony deformity
d. Neither b. Behind the lateral malleolus is called _____.
c. In front of the medial malleolus a. Arthrodesis
Clawtoe/Hammertoe deformity occurs d. In front of the lateral malleolus b. CORRECTIVE OSTEOTOMY
because of _____: c. Resection-interpositional
a. Dorsalis pedis occlusion The ankle brachial index is arrived at arthroplasty
b. Tibialis anterior rupture by d. Tenotomy
c. LUMBRICAL DENERVATION a. DIVIDING THE ANKLE SYSTOLIC
d. None of the above PRESSURE BY THE BRACHIAL This surgical procedure involves
SYSTOLIC PRESSURE ablation of one or both ends of a joint.
Neurologic assessment technique b. Dividing the brachial systolic However, the two ends are not
which relies on the use of a stiff thread pressure by the ankle systolic pressure allowed to unite and eventually a false
like device, the buckling of which c. Both techniques are valid joint is formed.
indicates application of 10g of force. d. Neither technique is valid a. Arthrodesis
A. SEMMES-WEINSTEIN b. Corrective osteotomy
MONOFILAMENT TEST The ABI is preformed using a/an: c. RESECTION-INTERPOSITIONAL
B. Tuning Fork Test a. Ordinary stethoscope ARTHROPLASTY
C. Pin prick test b. DOPPLER STETHOSCOPE d. Tenotomy
D. Ankle Reflex Test c. Palpation
Which subspecialty offers
The ankle reflex in a patient with ABI value associated with tissue endovascular revasularization
diabetic neuropathy is expected to be: necrosis procedures for the management of
a. HYPOREFLEXIVE a. LESS THAN 0.4 peripheral arterial disease in the
b. Normal b. Greater than 0.9 diabetic patient?
c. Hyperreflexive c. Greater than 1.3 a. Orthopedic surgery
d. None of the above b. Vascular surgery
The tuning fork test utilizes which c. INTERVENTIONAL CARDIOLOGY
frequency? ABI greater than 1.3 may be explained d. Plastic surgery
a. 64Hz by
b. 128Hz a. Increased blood flow into the lower This type of diabetic foot infection is
extremities considered if during the course of
c. 256Hz
examining an ulcer, the physician is able to
d. 512Hz b. Increased systolic ejection fraction probe to bone.
secondary to diabetic neuropathy a. Abscess
c. Decreased renal blood flow with b. Wet gangrene
resultant hypertension c. Necrotizing fasciitis
d. NONCOMPLIANT VESSELS d. OSTEOMYELITIS
SECONDARY TO MEDIAL
CALCINOSIS

Transcriber/s: Jan Cynric Cacao


Formatting: Jan Cynric Cacao D L S H S I M e d i c i n e B a t c h 2 0 1 6 | 14 of 14
Editor/s: Paolo Ramirez and Leanne Gale Ramos
APPROCH TO THE LIMPING CHILD
Jose Ramon C. Pascual, MD, FPOA
October 13, 2014; 1:00 – 2:30 PM
Orthopedics

LECTURE OUTLINE:
 Definition of Limp  Slipped Capital Femoral Epiphysis
 Epidemiology  Developmental Dysplasia of the Hip
 Historical Features associated with specific causes of  Genu Varum
Limp  Congenital Clubfoot
 Gait abnormality  Conclusion
 Physical Examination
 Transient Synovitis of the Hip

DEFINITION OF LIMP
 Uneven, jerky or laborious gait
 Accounts for 4 per 1000 visits in the pedia ER

EPIDEMIOLOGY OF ATRAUMATIC LIMP


 Males:Female ratio of 2:1
 Median Age of 4
 Limp painful in 80% of patients (hip in 34% and knee in 19%)
 Transient synovitis most common (40% of cases in United Kingdom)
 Most patients (77%) had a benign cause that could be managed without subspecialty follow-up or admission

HISTORICAL FEATURES ASSOCIATED WITH SPECIFIC CAUSES OF LIMP


Feature Cause
Fever, Weight Loss, Anorexia Malignancy, Osteomyelitis, Septic Arthritis, Chronic Rheumatoid Disease
History of trivial trauma Fracture, contusion, SCFE, Osteochondritis Dessicans
Increased pain on activity Stress Fractures, overuse syndromes, compartment syndrome
Decreased pain on activity Juvenile Idiopathic Arthritis, Complex Regional Pain Syndrome
Recent viral illness or antibiotic Transient Synovitis, Henoch Schonlein Purpura, Serum Sickness, Chikungunya
use Arthralgia
Nocturnal Pattern Osteoid Osteoma, Malignant BoneTumor, Musculoskeletal TB
Morning Stiffness Juvenile Idiopathic Arthritis
New or Increased Sport Activity Stress Fractures, Overuse Syndrome, Osgood-Schlatter Disease, Chondromalacia
Polyarthritis Acute Rheumatic Fever, Gonoccoccal Arthritis
Back Pain Discitis, Meningitis, Vertebral Osteomyelitis, Limb Length Discrepancy
Endocrinopathies SCFE
Abdominal Pain Appendicitis

GAIT ABNORMALITY/ETIOLOGY
TYPE OF GAIT CAUSE
Fracture, Soft Tissue Injury, Transient Synovitis, Infections, Foreign Body in Forefoot, Tumor,
Antalgic Gait
Osteochondritis Dissecans
Trendelenberg
Legg-Calve-Perthes Disease (LCPD), SCFE, Developmental Dysplasia of the hip (DDH)
Gait
Steppage Gait Neurologic Conditions with Loss of Ankle Dorsiflexion
Toe Walking Gait CP, Sever Disease, Foreign Body in Heel, Toddler Calcaneal Fracture, Idiopathic
Vaulting Gait Leg Length Discrepancy (LLD) or abnormal knee mobility
Stooping Gait Appendicitis, PID, Psoas Muscle Abscess

PHYSICAL EXAMINATION FINDINGS WITH Limb Length Discrepancy


Galeazzi Test (secondary to dislocated hip in
SPECIFIC CAUSES OF A LIMP DDH, SCFE or LCPD)
Weak Gluteus Medius Muscle
PE Finding Cause Trendelenburg
(shortened in DDH, SCFE and
Internal Rotation Inflammation, Infection, Trauma Test
LCPD)
of Hip
FABERE Test Sacroiliitis

Transcriber/s: Paolo M. Ramirez


Formatting: Paolo M. Ramirez D L S H S I M e d i c i n e B a t c h 2 0 1 6 | 1 of 18
Editor/s: Leanne Gale Ramos & Jan Cynric Cacao
TRANSIENT SYNOVITIS OF THE HIP BIGGEST NOTES:
 Most common cause of non-traumatic hip pain Legg-Calve-Perthes Disease/ “Coxa Plana”
in children - Self-limiting hip disorder caused by a varying degree of
ischemia and subsequent necrosis of the femoral head
 Pain and limitation of motion without clear
- Avascular necrosis of nucleus of proximal femoral
precipitant epiphysis, abnormal growth of the physis, and eventual
 Resolves with conservative therapy remodeling of regenerated bone are the key features of
 Ages 3 to 8 years, mean of 6 years this disorder
 Male:Female of 2:1
 Fever is absent, patient non-toxic looking
 Antalgic gait SLIPPED CAPITAL FEMORAL EPIPHYSIS
 Fracture through the epiphyseal plate
EDITOR’S NOTES: What is an antalgic gait?  Common in adolescents M (10-16 yrs) and F (12-
 A form of gait abnormality, or a limp in which a 14 yrs); patients younger than 10 is associated
phase of the gait is shortened on the injured side with endocrinopathy
to alleviate the pain experienced when bearing  M:F is 2.4:1
weight on that side.  Left hip > right hip
 Obesity is a risk factor
 Effusion present, hip in abduction and external  May present with referred pain to knee
rotation  Prodromal symptoms (hip/knee pain, limp,
 Log roll of extremity may reveal muscle guarding decreased range of motion) may be:
 Must differentiate from infection  Acute- less than 3 weeks
 Chronic- more than 3 weeks
 Acute on chronic- more than 3 weeks with
an acute exacerbation with or without a
traumatic episode

Number of Risk Probability of Septic


Factors Arthritis
0 16.9
1 36.7
2 62.4  Stable SCFE- allows patient to ambulate [able to
3 82.6 bear weight]
4 93.1  Gait may be antalgic
5 97.5  Out-toeing
 Unstable SCFE- patient unable to ambulate at all,
 Procalcitonin is a sensitive and specific marker in at higher risk for AVN [compromised femoral blood
the diagnosis of septic arthritis and acute supply]
osteomyelitis and can be used to distinguish  Internal rotation painful for all forms of SCFE
inflammation from infection  Differentiate from
 Patients with inability of bear weight, T>38.5,  Femoral head AVN (LCPD)
CRP>20mg/L (2mg/dL) or ESR>20mm/hr should  Femoral neck fracture
undergo hip ultrasound and diagnostic  Femoral neck stress fracture
arthrocentesis  Osteitis pubis
 Ultrasound can identify joint effusion and  Other femoral or groin injury
facilitate aspiration (aspiration can decompress a  Endocrine/renal/ trisomy work up for patients
joint and provide pain relief) with atypical presentation
 Dynamic Contrast Enhanced MRI can help  <10yrs
distinguish septic Arthritis from transient  >16 yrs
synovitis  Short stature
 Treatment consists of pain management and  Non-obese patients
return to full activity as tolerated  Request for AP and frog-leg lateral (latter not
 Full recovery in 4 weeks advisable in acute or unstable slips; might further
 Follow-up up to 6 months to make sure that it is dislodge the capital femoral epiphysis)
transient synovitis and not early LCPD  ALWAYS check both hips, high incidence of
bilateral involvement

Transcriber/s: Paolo M. Ramirez


Formatting: Paolo M. Ramirez D L S H S I M e d i c i n e B a t c h 2 0 1 6 | 2 of 18
Editor/s: Leanne Gale Ramos & Jan Cynric Cacao
 Bone Scan- may show evidence of AVN (poor
EDITOR’S NOTES: uptake of tracer)
 Slipped capital femoral epiphysis (SCFE) is a well
 CT scan- best used to evaluate posterior slip, also
known disorder affecting adolescent hips with the
potential of serious long-term sequalae. identify mild contralateral slips
 SCFE is characterized by a deformity of the  MRI- no ionizing radiation, early detection of AVN
proximal femur at the level of the growth plate and asymptomatic mild contralateral slips
with a postero-inferior position of the capital  Ultrasound-limited utility, effusion indicates
femoral epiphysis relative to the metaphysis. unstable slip, remodelling indicates stable slip
 The term SCFE is a misnomer since it is not the
epiphysis that “slips”, but the proximal femoral
metaphysis that externally rotates with antero- SLIDE NOTE:
superior translation, thus creating a three- Natural History
dimensional deformity. - Untreated the femoral head can either lose cartilage
 Occasionally, the metaphysis can translate (chondrolysis) or develop avascular necrosis both can
postero-inferiorly, leading to a valgus SCFE. lead to degenerative joint disease

 Degree of Slippage
 Type I - less than 33% displacement
 Type II - 33 to 50%
 Type III - more than 50% displacement

AP view

*Treatment of Choice: In-situ Pinning

Frog-leg view (not advisable in acute or unstable slips)


*SCFE looks like an ice cream falling off the cone

SLIDE NOTE: A Klein's line is defined as a straight line drawn SLIDE NOTES:
along the superior basal margin of the femoral neck on the  Treatment consists of a single screw placed across the
anteroposterior radiograph. Normally this line intersects the physis. The contralateral asymptomatic slip may also
lateral aspect of the epiphysis. As progressive displacement of be fixed at the same time.
the epiphysis occurs the amount of Klein line that intersects  A systematic review of the literature recommends on
the epiphysis decreases, compared with the univolved hip. the basis of level of evidence that the best treatment
Eventually, the line completely misses intersection with the for a stable SCFE is single screw in situ fixation and for
proximal femoral epiphysis. unstable SCFEs urgent gentle reduction,
decompression, and internal fixation.

Transcriber/s: Paolo M. Ramirez


Formatting: Paolo M. Ramirez D L S H S I M e d i c i n e B a t c h 2 0 1 6 | 3 of 18
Editor/s: Leanne Gale Ramos & Jan Cynric Cacao
ETIOLOGY

Exact etiology is unknown

Disruption of vscularity of
the capital femoral
epiphysis resulting in
necrosis and subsequent
revascularization

Interruption of blood Thrombophilia or various


supply to the femoral head coagullopathies (Protein C
(excessive hip abduction in and S deficiency/activated
Pavlik harness Protein C resistance)

EVALUATION

Clinical Presentation Modality Findings


Most commonly in children from age 4 to 8 years (range Can predate changes on plain
from 2 to late teens) Bone Scan radiograph; decreased uptake or cold
Insidious onset, present as limp or pain in groin, hip, thigh lesion suggests decreased blood flow
or knee Shows areas of decreased signal
Sometimes with history of recent or remote viral illness MRI intensity in the capital femoral epiphysis
and alterations in the physis
Physical Examination Can accurately assess coverage and
Antalgic gait or Trendelenburg gait containment of the femoral head. Used
Decreased abduction and internal rotation Arthrography at the time of surgery to confirm the
Limb length inequality if present may be due to femoral degree of correction needed for
head collapse osteotomies

 Herring (Lateral Pillar) Classification

 Herring Classification

Lateral Pillar Height Compared with the


Group
Normal Side
A No change in height
B >50% maintained
B/C 50% maintained
C <50% maintained

Transcriber/s: Paolo M. Ramirez


Formatting: Paolo M. Ramirez D L S H S I M e d i c i n e B a t c h 2 0 1 6 | 4 of 18
Editor/s: Leanne Gale Ramos & Jan Cynric Cacao
From left to right: Group A, Group B, Group B/C, Group C

SLIDE NOTES:
- Lateral pillar group-A, this anteroposterior view of the right hip showed central lesion without involvement of the lateral pillar. There
were no density changes and no loss of the height of the lateral pillar.
- Lateral pillar group-B, this anteroposterior view of the right hip showed lucency in the lateral pillar with some loss of height, but not
exceeding 50% of the original height.
- Lateral pillar group-B/C, this anteroposterior view of the right hip showed a very narrow lateral pillar (2 to 3 mm wide) that was 50% of
the original height with some lucency and depressed relative to the central pillar.
- Lateral pillar group-C, anteroposterior view of right hip showed more lucency in the lateral pillar and >50% loss of height.

PROGNOSIS SURGICAL TREATMENT


 Containment
Good Poor
Age <8 Age>8
Herring Group A Herring Group B or C

NONSURGICAL TREATMENT
 Containment

SLIDE NOTE:
 Containment is achieved from either the acetabular
side or the femoral side. Current evidence favors
pelvic procedures.
 Figure above: Surgical containment methods for
LCP (A) Femoral varus osteotomy, (B) Salter
osteotomy.

DEVELOPMENTAL DYSPLACIA OF THE HIP

SLIDE NOTE:
 The femoral head revascularizes and repairs
spontaneously.
 The role of the physician is to make sure that the
mismatched femur doesn’t slip out of the acetabulum
during the healing process.
 Long term benefit is still in question.
 Modern fiberglass cast used for the nonoperative
treatment of LCP disease.

Transcriber/s: Paolo M. Ramirez


Formatting: Paolo M. Ramirez D L S H S I M e d i c i n e B a t c h 2 0 1 6 | 5 of 18
Editor/s: Leanne Gale Ramos & Jan Cynric Cacao
ETIOLOGY: PREDISPOSING FACTOR
EDITOR’S NOTES:  Wynne-Davies' criteria for ligamentous laxity
 Developmental dysplasia of the hip (DDH)
comprises a spectrum of disorders:
o An unusually shallow acetabulum
without actual displacement of the
joint
o A shallow acetabulum with
subluxation (partial displacement) of
the femoral head; or
o A frank dislocation during the
neonatal period
 Whether the instability comes first and then affects
acetabular development because of imperfect
seating of the femoral head, or is the result of a
primary acetabular dysplasia, is still not known for
sure. Both mechanisms might be important.
 Girls are much more commonly affected than boys A: Flexion of the thumb to touch the forearm
(a ratio of about 7:1), and the left hip more often B: Extension of the fingers parallel to the forearm
than the right; in 1 in 5 cases the condition is
C: Hyperextension of the elbow 15 degrees or more
bilateral.
D: Hyperextension of the knee 15 degrees
 The ideal, still unrealized, is to diagnose every case
at birth. For this reason, every newborn child E: Dorsiflexion of the ankle 60 degreee
should be examined for signs of hip instability.
Where there is a family history of congenital
dislocation, and with breech presentations, extra
care is taken and the infant may have to be
examined more than once.

 Prenatal Positioning: The breech position, associated with developmental dysplasia of the hip (DDH).

 A: A double breech position is associated with a low incidence of DDH.


 B: A single footling breech is associated with a 2% risk of DDH.
 C: A frank breech, especially with the knee(s) extended, is associated with a 20% risk of DDH

 Postnatal positioning in extension, as in this child PHYSICAL EXAMINATION


on a Native American cradleboard, contributes to  Inspection
developmental dysplasia of the hip  Asymmetry of Gluteal Folds/Apparent
Limb Length Discrepancy

SLIDE NOTE: Certain conditions such as torticollis and metatarsus


adductus have been associated with DDH

Transcriber/s: Paolo M. Ramirez


Formatting: Paolo M. Ramirez D L S H S I M e d i c i n e B a t c h 2 0 1 6 | 6 of 18
Editor/s: Leanne Gale Ramos & Jan Cynric Cacao
 LOM

EDITOR’S NOTES:
Galeazzi’s Sign: The child lies supine with the knees flexed
and the hips flexed to 90°. A positive test is indicated by one
knee being higher than the other. Limited abduction of the involved hip

PATHOLOGY
 Normal Hip vs. Subluxable But Not Dislocatable Hip

 Dislocatable Hip – reducible but unstable, hypertrophic changes (arrows) mark the Neolimbus

 Irreducible Hip – because of Intraarticular Obstruction (Inverted Limbus, Ligamentum Teres and Pulvinar)

Transcriber/s: Paolo M. Ramirez


Formatting: Paolo M. Ramirez D L S H S I M e d i c i n e B a t c h 2 0 1 6 | 7 of 18
Editor/s: Leanne Gale Ramos & Jan Cynric Cacao
 The iliopsoas tendon as an obstacle to closed gently adducts one hip and pushes
reduction. posteriorly.
 A, Anterior view. The iliopsoas tendon  B, When the examination is positive,
traverses the anteromedial aspect of the the examiner will feel the femoral head
hip joint before inserting into the lesser make a small jump (arrow) out of the
trochanter. With lateral and superior acetabulum (Barlow’s sign). When the
displacement of the femoral head and pressure is released, the head is felt to slip
lesser trochanter, the iliopsoas tendon is back into place.
stretched taut across the medial and
anterior aspect of the hip capsule.
 B, Lateral view showing external pressure
and indentation of the capsule by the
iliopsoas tendon. This hourglass
constriction of the capsule with formation
of the capsular isthmus markedly reduces
the diameter of the acetabular orifice and is
a barrier to closed reduction.

 Ortolani Test
 The Ortolani test for developmental dislocation
of the hip in a neonate.
 A, The examiner holds the infant’s
knees and gently abducts the hip while
lifting up on the greater trochanter
with two fingers.
 B, When the test is positive, the
dislocated femoral head will fall back
into the acetabulum (arrow) with a
palpable (but not audible) “clunk” as
the hip is abducted (Ortolani’s sign).
When the hip is adducted, the
 The pelvifemoral muscles become shortened and examiner will feel the head redislocate
contracted, with progressive upward posteriorly.
displacement of the femoral head, in long-
established developmental dysplasia of the hip.
Arrows represent the direction of muscle forces.

 Trendelendburg Sign
 A, As the child stands with the weight on
the normal side, the pelvis is maintained in
the horizontal position by contraction and
tension of the normal hip abductor
muscles.
 B, As the child shifts weight to the side of
the dislocated hip, the pelvis on the
opposite, normal side drops, owing to
weakness of the hip abductor muscles on
the affected side. The sideways lean of the
body toward the affected side is
PHYSICAL EXAMINATION Trendelenburg’s sign.
 Barlow Test [INFANTS ONLY]
 The Barlow test for developmental dislocation
of the hip in a neonate.
 A, With the infant supine, the examiner
holds both of the child’s knees and

Transcriber/s: Paolo M. Ramirez


Formatting: Paolo M. Ramirez D L S H S I M e d i c i n e B a t c h 2 0 1 6 | 8 of 18
Editor/s: Leanne Gale Ramos & Jan Cynric Cacao
EDITOR’S NOTES:
 Ultrasound has recently become the 
primary
imaging tool to assess the hip joint of the neonate
and young infant.
 The acetabulum and femoral head can, with
practice, be displayed with static and dynamic
ultrasound. Sequential assessment allows
monitoring of the hip during an initial period of
splintage. 

 According to the AAP recommendations
(Paediatrics 2000:105), ultrasound of the hip is to
be selectively performed only in high-risk infants
(e.g. female breech, positive family history etc), 
or
if abnormal physical findings such as asymmetric
hip abduction, hip clicks etc persist after 4 weeks of
age.
 Lordosis
 Ultrasound screening at birth is not recommended
 Excessive Lordosis is secondary to bilateral and the ideal age for selective ultrasound has been
flexion contracture of the hip set at 4 weeks of age so that minor varieties of hip
instability are not treated unnecessarily.
 X-rays have little role to play in the diagnosis of
DDH in the newborn and at times may be quite
misleading.

RADIOGRAPHIC LANDMARKS

SONOGRAPHY

*MUST KNOW

SLIDE NOTE:
 Radiographic measurements useful in evaluating
developmental dysplasia of the hip.
 Hilgenreiner’s line is drawn through the triradiate
cartilages.
 Perkin’s line is drawn perpendicular to Hilgenreiner’s
*Help support Dx. Since bones are not yet fully ossified yet, it would line at the margin of the bony acetabulum.
be difficult to see in a plain radiograph  Shenton’s line curves along the femoral metaphysis
and connects smoothly to the inner margin of the
pubis.
 Dimension H (height) is measured from the top of
the ossified femur to Hilgenreiner’s line.
 Dimension D (distance) is measured from the inner
border of the teardrop to the center of the upper tip of
the ossified femur.
 Dimensions H and D are measured to quantify
proximal and lateral displacement of the hip and are
most useful when the head is not ossified.

Femoral head coverage should normally be greater than 50%. The


alpha angle should be greater than 60 degrees.

Transcriber/s: Paolo M. Ramirez


Formatting: Paolo M. Ramirez D L S H S I M e d i c i n e B a t c h 2 0 1 6 | 9 of 18
Editor/s: Leanne Gale Ramos & Jan Cynric Cacao
*Note that the ossific nucleus should be at the medial lower
quadrant. The dysplastic hip would present with an ossific nucleus
located in other quadrants

TREATMENT
NON-OPERATIVE
 Pavlik Harness: The transverse chest strap should
be placed just below the nipple line. The hips
should be flexed to 120 degrees, and the posterior
straps should not produce forced abduction.

 Acetabular index and the medial gap. The


acetabular index is the angle between a line
drawn along the margin of the acetabulum and
Hilgenreiner’s line; it averages 27.5 degrees in
normal newborns and decreases with age.

EDITOR’S NOTES:
 The most commonly used device for the treatment
of DDH in the newborn is the Pavlik harness.
Though other devices are available (e.g. von Rosen
splint, Ilfeld splint, Frejka pillow), the Pavlik
harness is popular for its well-established efficacy
and ease of use. When appropriately applied, the
Pavlik harness prevents hip adduction and
extension but allows flexion and abduction which
leads to reduction and stabilization.
 Wilberg’s center–edge angle, the angle between
Perkin’s line and a line drawn from the lateral lip
of the acetabulum through the center of the
femoral head. This angle, a useful measure of hip
position in older children, is considered normal if
greater than 10 degrees in children 6 to 13 years of
age, and it increases with age

Transcriber/s: Paolo M. Ramirez


Formatting: Paolo M. Ramirez D L S H S I M e d i c i n e B a t c h 2 0 1 6 | 10 of 18
Editor/s: Leanne Gale Ramos & Jan Cynric Cacao
OPERATIVE
 Femoral osteotomy
 Pelvic osteotomy

EDITOR’S NOTES:
 In developmental dysplasia of hip, pelvic
osteotomies are required for instability, failure of
acetabular development, or progressive femoral
head subluxation after reduction.
 Osteotomies should only be done after congruent
reduction, satisfactory range of motion, and
reasonable sphericity is achieved by closed or open
methods.
 The choice of pelvic versus femoral osteotomy is
sometimes surgeon’s choice. Some surgeon’s prefer
to do pelvic osteotomies after age 4 and femoral
osteotomies prior to this age. In general, pelvic
osteotomies should be done when severe dysplasia
is accompanied by significant radiographic
changes, e.g. lateral acetabular ossification, etc.),
whereas changes on the femoral side (e.g. marked
anteversion) are best treated by femoral
osteotomies.
EDITOR’S NOTES:
(This was not really mentioned / discussed but placing it
here for those who are interested) GENU VARUM
 Children under 6 months: Every newborn child with
signs of hip instability – however slight – should
ideally be examined by ultrasonography. If this
shows any abnormality, the infant is placed in a
splint with the hips flexed and abducted and is
recalled for re-examination – in the splint – at 2
weeks and at 6 weeks. By then it should be possible
to assess whether the hip is reduced and stable,
reduced but unstable (dislocatable by Barlow’s test),
subluxated or dislocated.
 Persistent dislocation: 6–18 months: If, after early
treatment, the hip is still incompletely reduced, or if
the child presents late with a ‘missed’ dislocation,
the hip must be reduced (if necessary by operation)
and held reduced until acetabular development is
satisfactory.
 Persistent dislocation: 18 months–4 years: In the
older child, closed reduction carries an even greater
risk of causing avascular necrosis of the femoral
head, due to tightness of the soft tissues. The
preferred approach is to proceed straight to
arthrography and open reduction. An arthrogram
will show whether there is an in-turned limbus or any EDITOR’S NOTES:
marked degree of acetabular dysplasia.  Varus and valgus: Varus means the part distal to
 After the age of 4 years: For unilateral dislocation, the joint is displaced towards the midline, whereas
operative reduction is still feasible, at least up to the valgus means away from it. Genu varus means
age of 8–10 years; as in the former group, it may be
bow legs. Genu valgus means knock knees.
necessary to combine this with corrective
osteotomies of the femur and/or pelvis.

Transcriber/s: Paolo M. Ramirez


Formatting: Paolo M. Ramirez D L S H S I M e d i c i n e B a t c h 2 0 1 6 | 11 of 18
Editor/s: Leanne Gale Ramos & Jan Cynric Cacao
NORMAL DEVELOPMENT OF TIBIOFEMORAL ANGLE

ETIOLOGY

Developmental Factors
Static Varus Angulation + Increased
body weight (infantile Blount disease)
Hereditary Factors Dynamic gait variation secondary to
increased thigh girth + increased
body weight (adolescent Blount
disease)

Pathologic Genu varum

DIFFERENTIALS
 Primary Genu Varum
RICKETS VS BLOUNT
Primary Varus
Deformity of
the Knee

Pathologic Physiologic
Varus Varus

Early Blount Late Blount


Disease (</=4 Disease
yrs) (>4yrs)

 Secondary Genu Varum


 Metabolic ex. Rickets EDITOR’S NOTES:
 Tumor ex. Ollier Disease (Multiple  Rickets is the most common metabolic disease
of the bones encountered in children in
Enchondromatosis)
developing countries because of poverty and
 Infection malnutrition.
 Trauma  It results from poor mineralization of growing
bones before epiphyses are fused, due to
disturbance in calcium and phosphorus
metabolism.

Transcriber/s: Paolo M. Ramirez


Formatting: Paolo M. Ramirez D L S H S I M e d i c i n e B a t c h 2 0 1 6 | 12 of 18
Editor/s: Leanne Gale Ramos & Jan Cynric Cacao
RICKETS MNEMONIC  Langenskiold Classification
 Reaction of the periosteum (may occur)  Lower Recurrence if Procedure Done
 Indistinct cortex Before Age 4 and Prior to Langenskiold
 Coarse trabeculation Stage IV
 Knees, wrists, and ankles affected predominantly
 Epiphyseal plates, widened and irregular
 Tremendous metaphysis (cupping, fraying,
splaying)
 Spur (metaphyseal)

EVALUATION

Clinical Findings Suggestive of Pathologic Varus


Proximal tibial location of bowing
Sharply angular deformity
Asymmetric bowing of the two legs SLIDE NOTE:
Progressive deformity on serial examination  Langenskiöld’s classification of infantile tibia vara in
Lateral thrust during gait six progressive stages with increasing age. Surgery is
Very severe deformity usually indicated for patients more than 3 years of
age with at least a Stage III Radiograph.
 The prognostic implications suggested by
 Lateral Thrust
Langenskiöld are that simple osteotomy can be
 Lateral Movement of Knee During Stance
effective PRIOR to stage IV
Phase

When Do We Order Radiographs? NONOPERATIVE TREATMENT


Children >18mos with Clinical Deformity>20 degrees
Short (<5th percentile for height) patients Bracing
Family history of metabolic bone disease Efficacy is controversial
Findings suggestive of pathologic varus (as above) Indicated only in patients 2 to 3 years of age with stage 1
to 2 disease
Poor results with obesity or bilaterality
Improvements should occur within 1 year
Must be continued until bony changes resolve (1.5 to 2
years)

SURGICAL TREATMENT
 Epiphysiodesis

SLIDE NOTE: SLIDE NOTE:


Radiographic measurement of angular deformity of the lower limb.  Usually done in mild to moderate Blount disease
A, Tibiofemoral angle. B, Proximal metaphyseal–diaphyseal angle. patients with at least 15 to 18 months of growth
remaining.
 May be temporary or permanent.
 Temporary epiphysiodesis interrupts growth through the
MD
Prognosis use of implants such as staples, 8-plates or non-lagged
Angle
screws across the physeal plate.
<10 95% chance bowing will resolve  Growth through the physis resumes after removal of the
Considerable overlap between physiologic implant when the desired angular correction has been
11-16
genu varum and Blount Disease achieved.
>16 95% chance bowing will progress  Permanent epiphysiodesis involves the destruction of
the physis with the goal of preventing further growth
from the physis.

Transcriber/s: Paolo M. Ramirez


Formatting: Paolo M. Ramirez D L S H S I M e d i c i n e B a t c h 2 0 1 6 | 13 of 18
Editor/s: Leanne Gale Ramos & Jan Cynric Cacao
Jumper’s Knee (Sinding Larssen
 Corrective Osteotomy
Johanssen Disease)

NOTE: X-ray on the left shows slight dystrophic calcification over


the inferior pole of the patella which may not always appear on
radiographs. X-ray on the right is the adult version of Jumper’s
Knee patellar tendinitis

EDITOR’S NOTES:
 Patellar tendon (Jumpers' knee) – Pain at the site
of patellar tendon origin from the inferior pole of
the patella, due to chronic overstress. Usually seen
in children.
 Similarly tenderness may be at the insertion of the
patellar tendon on the tibial tuberosity (Osgood
Schlatter's disease). OSD is a traction
phenomenon resulting from repetitive quadriceps
contraction through the patellar tendon at its
insertion upon the skeletally immature tibial
tubercle. This occurs in preadolescence during a
time when the tibial tubercle is susceptible to
SLIDE NOTE: strain.
 Osteotomy is performed in the proximal tibia and the  Sever disease – a painful inflammation of the
calcaneal apophysis. It is classified with the child
fragments are put into the proper orientation.
and adolescent nonarticular osteochondroses.The
 Correction may be done acutely with internal fixation
etiology of pain in Sever disease is believed to be
or gradually with the use of a special external fixator repetitive trauma to the weaker structure of the
the Taylor Spatial Frame. apophysis, induced by the pull of the tendo
 Acute correction is faster and more comfortable calcaneus (Achilles tendon) on its insertion. This
(barring complications) for the patient. results in a clinical picture of heel pain in a growing
 Gradual correction with distraction osteogenesis active child, which worsens with activity.
achieves a more accurate angular and rotational (Medscape)
correction with fewer major complications such as
compartment syndrome and neuropraxia.
 Limb length discrepancies may also be corrected with OSGOOD SCHLATTER DISEASE
gradual correction with a Taylor Spatial frame.

APOPHYSITIS

Sever Disease

SLIDE NOTE: Overuse injury resulting in inflammation occurring


where the tendon is being pulled away from its immature (open
cartilage plate) bony attachment

Transcriber/s: Paolo M. Ramirez


Formatting: Paolo M. Ramirez D L S H S I M e d i c i n e B a t c h 2 0 1 6 | 14 of 18
Editor/s: Leanne Gale Ramos & Jan Cynric Cacao
SLIDE NOTE: X-ray findings may be normal. X-ray usually done  B, End-on view. The medial and
to rule out other conditions that require more aggressive plantar deviation of the navicular
treatment (i.e. neoplasm) articulation is apparent.
 C, Equinus of the neck in relation to the
tibiotalar articular surface is significant.
CONGENITAL CLUBFOOT

 Talonavicular Subluxation
 A, Normal foot, section of the talonavicular
joint.
 N, navicular.
EDITOR’S NOTES:  T, talus.
Congenital talipes equinovarus (Idiopathic Club-Foot) 
  B, Clubfoot section.
 In this deformity the foot is curved downwards
and inwards – the ankle in equinus, the heel in Note: The navicular (N) articulates with the medial neck of the
varus, and the forefoot adducted, flexed and dysmorphic talus (T). Because of the equinus, the tibia (P) and
supinated. fibula (L) are included in the section.
 The skin and soft tissues of the calf and the
medial side of the foot are short and under-
developed. If the condition is not corrected
early, secondary growth changes occur in the
bones and these are permanent. 

 The deformity is relatively common, with an
incidence of 1–3 per 1000 births. Boys are
affected twice as often as girls and it occurs
bilaterally in nearly one-half of the cases. A
family history increases the risk by 20–30 times.

ETIOLOGY: MULTIFACTORIAL
 However most infants with clubfoot have no
identifiable genetic, syndromal or extrinsic cause
PHYSICAL EXAMINATION
Extrinsic Intrinsic (Genetic)  Components of Clubfoot [MUST KNOW]
 Teratogenic Agent  Clubfoot has been noted in  Forefoot inversion and adduction
(e.g. sodium chromosomal deletion  Hindfoot inversion
amnimopterin) syndromes  Heel Equinus
 Oligohydramnios  Multifactorial system of  Tibial Internal (Medial) Torsion
 Congenial Constriction inheritance  Clubfoot versus Metatarsus Adductus [has
Rings  Incidence in first degree forefoot inversion only]
relatives is 2%  Bilateral mild metatarsus adductus.
 Parents of a child with  A, Dorsal view showing medial
clubfoot have a 10% chance deviation of all the metatarsals.
of having another child with  B, Plantar view showing the “bean-
clubfoot shaped” foot.
 If one monozygotic twin has  This type of foot is easily corrected
CTEV, the other has a 32% with serial casting.
chance of having CTEV

PATHOLOGY
 Dysplastic Bones
 Schematic illustration of a clubfoot talus.
 A, Top view. The neck is shortened
and deviated medially, so true
distinction from the body of the talus is Boo!
questionable. The articulation with the
navicular is on the medial side of the
misshapen talar head.

Transcriber/s: Paolo M. Ramirez


Formatting: Paolo M. Ramirez D L S H S I M e d i c i n e B a t c h 2 0 1 6 | 15 of 18
Editor/s: Leanne Gale Ramos & Jan Cynric Cacao
NATURAL HISTORY

TREATMENT
 Clubfoot vs. Pes Calcaneovalgus  Complete Subtalar Release
 Severe talipes calcaneovalgus in a
newborn. Note the foot “plastered” up
against the anterior aspect of the tibia. The
clinician should always examine the hips to
rule out congenital dislocations.

EDITOR’S NOTES:
Pes Calcaneovalgus 

 Foot is dorsiflexed, everted
 The dorsum readily touches the antero-lateral
surface of the leg
 Plantar flexion ceases at around the mid-
position

CLASSIFICATION

Correctable Resistant
 Also known as “easy”;  May be associated with
“flexible” teratologic conditions ,
 Responds well to non- neuromuscular
operative treatment disorders
(myelomeningocoele)
or syndromes
 Respond poorly to
splintage or relapse
quickly following
seemingly successful
manipulative treatment

Transcriber/s: Paolo M. Ramirez


Formatting: Paolo M. Ramirez D L S H S I M e d i c i n e B a t c h 2 0 1 6 | 16 of 18
Editor/s: Leanne Gale Ramos & Jan Cynric Cacao
 Calcaneocuboid Release
 Calcaneocuboid deformity; release from
the medial approach

 Wound Complications in Clubfoot Surgery

EDITOR’S NOTES:
 The Ponseti method is a manipulative technique
that corrects congenital clubfoot without invasive
surgery.
 The manipulative treatment of clubfoot deformity
is based on the inherent properties of the
connective tissue, cartilage, and bone, which
respond to the proper mechanical stimuli created
by the gradual reduction of the deformity.
 The ligaments, joint capsules, and tendons are
stretched under gentle manipulations.

 Ponseti Technique
 Tenotomy of the Achilles tendon performed towards
the end of the treatment

 Foot abduction orthosis (known as the Ponseti brace)


to maintain correction after nonoperative treatment

Transcriber/s: Paolo M. Ramirez


Formatting: Paolo M. Ramirez D L S H S I M e d i c i n e B a t c h 2 0 1 6 | 17 of 18
Editor/s: Leanne Gale Ramos & Jan Cynric Cacao
CONCLUSION
 The cause of limp can be determined through careful
history and physical examination
 Most causes are benign or self-limiting (TS,
apophysitis)
 It is important to identify the cases that require
emergent therapy (infections, SCFE)
 It is also possible to alter the natural history of some
conditions (DDH, LCPD, Blount disease,congenital
clubfoot) by identifying them early

-END-

TRANSCRIPTION DETAILS
BASIS Latest PPT RECORDINGS + NOTES + DEVIATIONS 8-10% CREDITS Xmark’s the Spot Recordings
This is a brand new lecture so study this eagerly well. And since this is new, no Past-E’s are available (LOL) 
Editor placed notes (additional info) and definitions coming from the following Ortho Books:
REMARKS
 Textbook of Orthopedics and Trauma (4 Vol Set) 2nd Ed. (2008)
 Apley's and Solomans Concise System of Orthopedics and Trauma 4th Ed
-BATCH 2016 Transcribers’ Guild Transcriptions. Version 2.0.0.0.0 Build 3203-

Transcriber/s: Paolo M. Ramirez


Formatting: Paolo M. Ramirez D L S H S I M e d i c i n e B a t c h 2 0 1 6 | 18 of 18
Editor/s: Leanne Gale Ramos & Jan Cynric Cacao
ADULT ORTHOPEDICS INFECTIONS
Joaquin Pandanan, MD, FPOA
October 27, 2014; 1:00 – 2:00 PM
Orthopedics

LECTURE OUTLINE
 Osteomyelitis  Septic Arthritis
 Acute osteomyelitis  Gonoccocal Arthritis
 Subacute osteomyelitis  Necrotizing Fasciitis
 Chronic osteomyelitis

OSTEOMYELITIS Transcriber’s Note:


 Infection of the bone and its marrow content Cancellous bones are areas where there is rich blood supply.
(e.g. proximal tibia, distal femur, proximal humerus, distal
tibia)
TYPES OF OSTEOMYELITIS
 Acute osteomyelitis (0-30 days)
 Subacute osteomyelitis (more than one month ACUTE OSTEOMYELITIS
but less than three months)
 Chronic osteomyelitis (more than three months) PATHOPHYSIOLOGY

WHO ARE AT RISK? Inflammation


 Poorly nourished elderly
 Immunocompromised (e.g. diabetic patients)
 Long term corticosteroid therapy Increased vascularity
 Rheumatoid arthritis

MECHANISM OF SPREAD Edema


 Direct bone contamination (open fractures and
other bone surgery) Thrombosis
 Soft tissue extension (pressure ulcers, infected
wound)
 Hematogenous spread (URTI, IV drug use) Ischemia and bone necrosis

Editor’s Note (From Medscape): Abscess


 Bone is normally resistant to infection. However, when
microorganisms are introduced into bone
hematogenously from surrounding structures or from Brodie's abscess
direct inoculation related to surgery or trauma,
osteomyelitis can occur.
 Bone infection may result from the treatment of Transcriber’s Notes:
trauma, which allows pathogens to enter bone and  The reason why cancellous bones are the site of infection is
proliferate in the traumatized tissue. because of the sluggish flow of blood in the area;
therefore, there is less phagocytosis – this due to the
tortuosity of the blood vessels in the area
MOST COMMON LOCATION  During the ischemic and bone necrosis stage, antibiotics are
 Cancellous part of the bone at the proximal part given and proper debridement to prevent further
of tibia or distal part of femur progression to abscess formation. This is the start of acute
osteomyelitis.
 If there is already abscess formation and is untreated or
partially treated, Brodie’s abscess will then form which is
the hallmark of subacute osteomyelitis

Editor’s Note:
 The earliest change is an acute inflammatory reaction.
The intraosseous pressure rises, causing intense pain and
obstruction of blood flow.
 By the second day pus appears in the medulla and forces
its way along the Volkmann canals to the surface, where
it forms a subperiosteal abscess. It then spreads along
the shaft, to re-enter the bone at another level, or bursts
out into the soft tissues.

Transcriber/s: Jan Cynric Cacao


Formatting: Jan Cynric Cacao D L S H S I M e d i c i n e B a t c h 2 0 1 6 | 1 of 7
Editor/s: Paolo Ramirez, Leanne Gale Ramos
From Apley's and Solomans Consise System Orthopedics and Trauma 4th Ed:

LABORATORY
 CBC, EST and CRP – elevated (first three days of
infection)
 Blood cultures – can isolate 70% of the bacteria
during the peaking of bacteria in the bone
 Wound culture

DIAGNOSTIC IMAGING

RADIOGRAPHY
 Soft tissue swelling
 Periosteal reaction/thickening
 Infection is within the bone
 Abscess tries to get out → thickening due to
abscess migration
 Appears late
BIGGEST NOTES:
 After 10-14 days of infection
 Because of the confined space and tension, tissue  Rarefaction – blackish discoloration of
necrosis occurs readily and an abscess may form the metaphyseal area
within the bone. The pus usually breaks out under the
periosteum, stripping it and eventually penetrating to
a point on the surface [Refer to image above]

Editor’s Note:
 The rising intraosseous pressure, vascular stasis,
infective thrombosis and periosteal stripping
increasingly compromise the blood supply; by the end
of 1 week there is usually evidence of necrosis.

MICROORGANISM
 Staphylococcus aureus (70-80%)
 Gram negative bacteria
 Pseudomonas
 E. coli
 Proteus sp. Distal forearm x-ray showing rarefaction
 Salmonella – sickle cell disease
 Pseudomonas – punctured wounds on the foot From Apley's and Solomans Consise System Orthopedics and Trauma 4th Ed:

CLINICAL MANIFESTATIONS
 Febrile, chills, body weakness
 Infected area – painful, swollen, extremely tender

BIGGEST NOTES:
Clinical features of acute osteomyelitis:
 Fever — acute onset with malaise
 Pain — localized to metaphysis classically
 Swelling
 Tenderness — localized
 Edema and pus (late sign) — pus may break
through the periosteum, causing a fluctuant mass.

Usually, the joint is not affected in Acute Osteomyelitis hence


will still produce good Range of Motion

Transcriber/s: Jan Cynric Cacao


Formatting: Jan Cynric Cacao D L S H S I M e d i c i n e B a t c h 2 0 1 6 | 2 of 7
Editor/s: Paolo Ramirez, Leanne Gale Ramos
Editor’s Note:
 In the first few days, there will be no changes in the
bone on the plain radiograph, but soft-tissue
swelling can be appreciated.
 By the end of the second week there may be early
radiographic signs of rarefaction of the
metaphysis and periosteal new bone formation.
 Later still, if treatment is delayed or ineffectual,
the bone may appear increasingly ragged.

BONE SCAN
 Technitium-99m scan Corticotomy procedure and placement of antibiotic beads
 Rapid bone formation after draining the abscess (5D)
 Increased blood flow
 High false-positive results Transcriber’s Note:
 Gallium scan  Antibiotic beads are left in the surgical site for 6 weeks
 80% specific and are removed after
 Requires 24-36 hours from onset of  The antibiotic is concentrated at the site of infection
symptoms, binds to inflamed tissue

SUBACUTE OSTEOMYELITIS
 Partially or untreated acute osteomyelitis
 Less symptomatic
 Classified according to the location of the lesion
 Metaphyseal, proximal tibia, diapheseal
 Low grade pyogenic abscess or result of increase
resistance
 Mild signs and symptoms as compared to acute
osteomyelitis

Editor’s Note:
 Osteomyelitis may present in a relatively mild form,
presumably because the organism is less virulent or the
Editor’s Note: patient more resistant.
 Isotope bone scan will show increased uptake in the  The distal femur and the proximal and distal tibia are
infected part. the favourite sites.
(From Medscape):
DIFFERENTIAL DIAGNOSIS Sub-Acute Osteomyelitis
 Cellulitis, abscess  The disease has an insidious onset, mild symptoms, and
 Septic arthritis lacks a systemic reaction, and supportive laboratory
data are inconsistent.
 Inflammatory arthritis – RA, gout
 Subacute osteomyelitis may mimic various benign and
 Osteoarthritis malignant conditions, resulting in delayed diagnosis
and treatment.
DEFINITIVE TEST
 Bone aspiration biopsy
 Large bore needle (19 or 18 gauge) RADIOGRAPHY
 Spinal needle  Brodie’s abscess on x-ray
 Needle is inserted up to the medullary canal  Accumulation of pus inside the bone where
there are no acute signs and symptoms
MANAGEMENT  Patient may have some discomfort
 Antibiotic management for 6 weeks  Hallmark of subacute osteomyelitis
 Cloxacillin – for S. aureus
 Surgical management – corticotomy
 Drain abscess

BIGGEST NOTES:
 Corticotomy involves cutting of the bone that may or
may not split it into two pieces but involves cortex
only, leaving intact the medullary vessels and
periosteum

Brodie’s abscess

Transcriber/s: Jan Cynric Cacao


Formatting: Jan Cynric Cacao D L S H S I M e d i c i n e B a t c h 2 0 1 6 | 3 of 7
Editor/s: Paolo Ramirez, Leanne Gale Ramos
CLINICAL MANIFESTATION
Editor’s Note:
 Laboratory investigations are often negative.
 Fairly well, afebrile
 The typical x-ray picture is of a small, oval cavity  ± Pain
surrounded by sclerotic bone – the classic Brodie’s  Sinus tract
abscess – but sometimes the lesion is more diffuse.

MANAGEMENT
 Antibiotic therapy for 6 weeks
 Surgical management – corticotomy

Editor’s Note:
 Treatment may be conservative if the diagnosis is not in
doubt. Immobilization and antibiotics (flucloxacillin and
fusidic acid) intravenously for 4 or 5 days and then orally
for another 6 weeks often result in healing, though this Sinus tract
may be slow.
 If the x-ray shows that there is no healing after
conservative treatment, open curettage may be Editor’s Note:
indicated; this is always followed by a further course of  Following acute bone infection, the patient returns with
antibiotics. recurrent bouts of pain, redness and tenderness at the
affected site.
 Classic signs are healed and discharging sinuses and x-
CHRONIC OSTEOMYELITIS ray features of bone rarefaction surrounded by dense
 Untreated acute osteomyelitis sclerosis and cortical thickening; within that area there
may be an obvious sequestrum.

Editor’s Note:
 Failure to eradicate infection of the bone in the acute LABORATORY
stage results in persistence of the organism in the bone.  CBC, ESR, CRP normal to slightly elevated
It is characterized by continuing destruction of the bone  Culture of the sinus tract
by the infective process.
 Antibiotic holiday
 Periosteal stripping results in parts of the bone
becoming avascular and necrotic. These dead fragments
 Not giving of antibiotics for 7 days
are called sequestra.  For bacteria isolation and culture
 The surrounding periosteum lays down new bone,
which surrounds the sequestrum. This is known as the DIAGNOSTIC IMAGING (RADIOGRAPHY)
involucrum.
 Necrotic material and pus continue to make their way
out of the bone through tracks that may eventually
traverse through the soft tissues to the skin, where they
form sinuses.

PATHOPHYSIOLOGY

Inflammation

Increased vascularity
x-ray of a sinus tract
Edema
SEQUESTRUM/INVOLUCRUM
Thrombosis

Ischemia and bone necrosis

Bone abscess

Sequestrum/involucrum

Transcriber’s Note: A: Normal bone; B: Periosteum C: Involucrum; D: Cloaca;


 If infection is still untreated after abscess formation, E: Sequestrum
sequestrum/involucrum ensues and this already
chronic osteomyelitis
Transcriber’s Note:
 Involucrum – new bone being layered to wall-off the
Sequestrum

Transcriber/s: Jan Cynric Cacao


Formatting: Jan Cynric Cacao D L S H S I M e d i c i n e B a t c h 2 0 1 6 | 4 of 7
Editor/s: Paolo Ramirez, Leanne Gale Ramos
From Apley's and Solomans Consise System Orthopedics and Trauma 4th Ed:

DIAGNOSTIC IMAGING (MRI) SEPTIC ARTHRITIS


 To know the extent of the infection  Infection of the joint usually bacterial in origin
 Monoarticular, but can present polyarticular

MECHANISM OF SPREAD
 Soft tissue extension (osteomyelitis on proximal
tibia)
 Hematogenous spread (abscess/furuncle)
 Direct inoculation (arthrotomy) [foreign body]

Editor’s Note:
 Infection of a joint may occur by blood-borne infection,
by direct penetrating injury (this may take the form of a
surgical procedure on the joint) or by spread of infection
from an osteomyelitic focus in an adjacent bone.

Editor’s Note: MOST COMMON LOCATION


 Magnetic resonance imaging (MRI) shows pathological  Knee (40-50%)
changes before x-rays and can help to distinguish
 Hip (10-20%)
between bone and soft-tissue infection. It is particularly
useful in detecting a focus of infection in the axial  Shoulder
skeleton.  Ankle
 CT and MRI are invaluable in planning operative  Elbow
treatment: together they will show the extent of bone
destruction and reactive oedema, hidden abscesses and
sequestra. Editor’s Note:
 Across all age groups, the commonest joint affected is
the knee, but in infants and young children septic
MANAGEMENT arthritis of the hip is more common.
 Antibiotic therapy
 Surgical removal – Sequestrectomy [removal of
the dead bone] CLINICAL MANIFESTATION
 Febrile, tender, signs of inflammation
 Severe limitation of involved joint usually in
Editor’s Note: flexed position/inability to weight bear
 By this stage, antibiotics cannot be expected to exert
anything other than a suppressive effect on the BIGGEST NOTES:
infection, which flourishes in a bed of avascular or  Flexion of a joint promotes less intraarticular
poorly vascularized tissue. pressure hence decreasing pain
 Treatment involves radical surgical clearance of all
non-viable material.
Editor’s Note:
 In a few cases these measures are insufficient to
Acute Septic Arthritis
eliminate infection, and amputation has to be
 The joint is swollen, painful and inflamed; this may be
considered.
accompanied with systemic features such as malaise,
 Sequestrectomy should be performed only if a
pyrexia, a high temperature and elevation of the
sequestrum is radiologically visible and surgically
white cell count, erythrocyte sedimentation rate
accessible.
(ESR) and C-reactive protein.

COMPLICATION
MICROORGANISM
 Septic arthritis
 Staphylococcus aureus
 Squamous cell CA (1-2%) [if still remained
untreated]  Neisseria gonorrhea

Transcriber/s: Jan Cynric Cacao


Formatting: Jan Cynric Cacao D L S H S I M e d i c i n e B a t c h 2 0 1 6 | 5 of 7
Editor/s: Paolo Ramirez, Leanne Gale Ramos
Editor’s Note: Editor’s Note:
 Over all age groups the com- monest organism is S.  Diagnostic imaging modalities such as radiography and
aureus. In sexually active adults, the commonest cause ultrasonography will simply confirm the presence of a
is Neisseria gonorrhoeae, the causative organism of joint effusion.
gonorrhoea.  However, the clinical picture together with abnormally
high values for the inflammatory markers should be
sufficient to make the provisional diagnosis of infection.
GONOCOCCAL ARTHRITIS
 N. gonorrhea
DEFINITIVE TEST
 Common among sexually active individual
 Arthrocentesis
 “Migratory arthralgia”
 60% involves more than 2 joints
SYNOVIAL FLUID
 Upper extremity > Lower extremity
 Sent to laboratory for quantitative and qualitative
 Difficult to culture
count
 Drug of choice: penicillin, ceftriaxone

LABORATORY EXAMINATION Editor’s Note:


 CBC, ESR, CRP – elevated  Arthrocentesis: the clinical procedure of using a syringe
to collect synovial fluid from a joint capsule
DIAGNOSTIC IMAGING (Wikepedia.com)
 It is also known as JOINT ASPIRATION
 Radiography – knee swelling [with capsular
 Aspiration reveals pus in the joint; fluid should be sent
distention] for microbiological investigation, including anaerobic
 Ultrasound – presence of fluid culture
 This should always be done before starting antibiotic
treatment!

*MEMORIZE
Degenerative Joint Disease Rheumatoid Arthritis
Normal Pyogenic Tuberculous
(DJD) (RA)
Appearance Straw Clear yellow Yellow Grayish-bloody Yellow
Clarity Transparent Transparent Cloudy Turbid/purulent Cloudy
Viscosity Normal Normal Decreased Decreased Decreased
Mucin clot Good Good Poor Poor Poor
Total WBC <200 2,000 15,000 50,000-200,000 20,000
Bacteria Negative Negative Negative Positive Positive

DIFFERENTIAL DIAGNOSIS  Signs and symptoms


 Acute osteomyelitis  Presents initially as cellulitis
 Abscess  Rapid progression of ecchymosis and bullae
 Osteoarthritis with joint effusion within 24 hours
 Rheumatoid arthritis  Crepitus on palpation
 Bursitis
 Transient synovitis of the hip Editor’s Note:
 A rapidly progressive and frequently fatal condition
MANAGEMENT caused by highly virulent Gram-positive (e.g.
 Surgical emergency staphylococcus, streptococcus) and Gram-negative (e.g.
 Open arthrotomy Escherichia coli) organisms.
 Arthoscopic arthrotomy  Often commencing in an extremity, a necrotizing
 Early antibiotic therapy should be initiated process involving not only fascia but all subcutaneous
tissues progresses rapidly in a proximal direction.
ASAP

Editor’s Note: ETIOLOGIC AGENT


 Prompt diagnosis and treatment is necessary, as  Clostridium perfringens (10%)
articular cartilage does not survive long in the presence  Polymicrobial in 75%
of a tense effusion of pus.  Streptococcus
 S. Aureus
 E. Coli
COMPLICATION
 Gas forming bacteria
 Osteoarthritis (articular cartilage damage)
DIAGNOSIS
NECROTIZING FASCIITIS  High index of suspicion
 Rapid and progressive limb threatening  Presence of gas on x-rays
 Dermis and fascia
 Polymicrobial but commonly involves
streptococcus

Transcriber/s: Jan Cynric Cacao


Formatting: Jan Cynric Cacao D L S H S I M e d i c i n e B a t c h 2 0 1 6 | 6 of 7
Editor/s: Paolo Ramirez, Leanne Gale Ramos
TREATMENT
 Emergent surgical debridement or amputation IS YOUR PATIENT IMPROVING?
 Foul smelling discharge  Repeat CBC, ESR, CRP
 High mortality  Febrile/afebrile
 Check ROM

Editor’s Note (From Medscape):


 A regimen of surgical debridement is continued until
SUMMARY
tissue necrosis ceases and the growth of fresh viable  Do proper history taking
tissue is observed.  Detailed PE
 If a limb or organ is involved, amputation may be  Request necessary laboratory exams and imaging
necessary because of irreversible necrosis and gangrene  Interpret imaging modalities
or because of overwhelming toxicity, which occasionally  Appropriate antibiotics
occurs.
 Early surgery to prevent complications
 Antibiotic therapy is a key consideration. Possible
regimens include a combination of penicillin G and an
aminoglycoside (if renal function permits), as well as
clindamycin (to cover streptococci, staphylococci, gram-
negative bacilli, and anaerobes).

-END-

TRANSCRIPTION DETAILS
Latest PPT. Apley's and Solomans Consise System 5-
BASIS RECORDINGS + NOTES + DEVIATIONS CREDITS -
Orthopedics and Trauma 4th Ed: 8%
Editor’s sources:
 Duckworth & Blundell (2010) Orthopaedics and Fractures
 Textbook of Orthopaedics, Trauma and Rheumatology 2nd Ed. (2013)
 Orthopedics - Current Essentials (Lange)
REMARKS
 Medscape

Editor’s notes: I incorporated additional information and visual aids from various orthopedic textbooks to have a better understanding of
the topic. I hope you don’t mind. Happy learning. 
-BATCH 2016 Transcribers’ Guild Transcriptions. Version 2.0.0.0.0 Build 3204-

Transcriber/s: Jan Cynric Cacao


Formatting: Jan Cynric Cacao D L S H S I M e d i c i n e B a t c h 2 0 1 6 | 7 of 7
Editor/s: Paolo Ramirez, Leanne Gale Ramos
SCOLIOSIS
Paul N. Atienza, MD, FPOA
Novemeber 4, 2014; 1:00 – 2:30 PM
Orthopedics

LECTURE OUTLINE:
 Basics  Signs & Symptoms  Treatment
 Genetics  Differential Diagnosis  Surgical Treatment
 Incidence  Physical Examination  Physical Therapy
 Causes  Laboratory Test  Medical Treatment
 Risk Factors  Pathologic Findings  Conclusion
 Classification  Imaging Procedures

BASICS
 Scoliosis is a three-dimensional curvature of the
spine, best appreciated on an anteroposterior
radiograph and physical examination. Both
thoracic and lumbar segments of the spine may
be affected.
 It is a LATERAL CURVATURE but sometimes
with kyphotic or lordotic components
 It is most commonly defined as a curve greater
than 10 degrees.
 The most common type is idiopathic scoliosis.
 Scoliosis may occur at any age. The most
common age at diagnosis of idiopathic scoliosis is
11 to 13 years.
 Small curves of idiopathic scoliosis are almost
equally prevalent in males and females. Females,
however, are three to four times more likely to
develop progression of the curve.

PPT NOTES:
 Majority of Idiopathic Infantile Scoliosis resolve
and is managed through observation.
 The Rib Vertebral Angle is used to observe and
monitor Idiopathic Infantile Scoliosis

BIGGEST NOTES:
 Rib Vertebral Angle Difference - The difference
between the angle formed by a vertical line
through the centre of the apical vertebral body on
an PA film and the rib on the convex side and the
same angle on the concave side
 20 degrees linked to high rate of progression
 < 20 degrees associated with spontaneous
recovery

Transcriber/s: Paolo M. Ramirez


Formatting: Paolo M. Ramirez D L S H S I M e d i c i n e B a t c h 2 0 1 6 | 1 of 7
Editor/s: Leanne Gale Ramos & Jan Cynric Cacao
CAUSES
EDITOR’S NOTES:
 Idiopathic Scoliosis
 The level and direction of the major curve convexity are  Theories about cause of idiopathic scoliosis
noted: e.g. ‘right thoracic’ means a curve in the thoracic
include a subtle connective tissue
spine and convex to the right; the hip juts out on the
concave side and the scapula on the convex. With
abnormality or neurohormonal defect.
thoracic scoliosis, rotation causes the rib angles to  Causes of congenital scoliosis include
protrude, thus producing a rib hump on the convex side hemivertebrae and fusions between
of the curve. vertebrae.

EDITOR’S NOTES:
 IDIOPATHIC SCOLIOSIS
 This group constitutes about 80% of all cases of
scoliosis. The deformity is often familial and the
population incidence of serious curves (over 30
degrees and therefore needing treatment) is 3 per
1000. The age at onset has been used to define
three subgroups: adolescent, juvenile and infantile.
A simpler division now in general use is early- onset
(before puberty) and late-onset scoliosis (after
puberty).

 Neuromuscular Scoliosis
 Cerebral palsy
 Traumatic paralysis
 Spina bifida
 Poliomyelitis
 Friedreich's ataxia
 Virtually any systemic neurologic condition
that affects the trunk

BIGGEST NOTES:
Since the condition is neurologic in origin, there is no
need to assess skeletal maturity.
Also, bracing cannot treat the condition

 Connective Tissue—Associated Scoliosis


 Marfan's syndrome
 Ehlers-Danlos syndrome
 Neurofibromatosis

PPT NOTES:
A curvature of more than 60 degrees would indicate a
severe curve

GENETICS
 Idiopathic scoliosis is transmitted as autosomal
RISK FACTORS
dominant with incomplete penetrance and  Progressive idiopathic scoliosis
variable expressivity.  Positive family history
 Female gender
INCIDENCE  Premenarchal status
 Paralytic scoliosis
 Prevalence of curves greater than 10 degrees is
 Severe spinal cord injury before adolescence
about 2% to 3%.
 Scoliosis in cerebral palsy including total
 Prevalence of curves requiring bracing (more than
involvement
25 degrees) is about 0.3%.
 Prevalence of curves requiring surgery is about 1
in 1,000 BIGGEST NOTES:
 4 to 5 years postmenarchal women with idiopathic
scoliosis will have a permanent curvature.
 Upon reaching menopause,
Osteoporosis + Scoliosis = multiple vertebral
fractures  DEGENERATIVE SCOLIOSIS

Transcriber/s: Paolo M. Ramirez


Formatting: Paolo M. Ramirez D L S H S I M e d i c i n e B a t c h 2 0 1 6 | 2 of 7
Editor/s: Leanne Gale Ramos & Jan Cynric Cacao
CLASSIFICATION
 Etiology
 Idiopathic
 Congenital
 Neuromuscular
 Connective tissue
 Degenerative
 Location (of the Apex or Middle of the Curve)
 Thoracic (T1-T10)
 Thoracolumbar (T11-L1)
 Lumbar (L2-L5)
*The apex vertebra is the MOST DEFORMED
 Subclassification of Idiopathic Scoliosis by Age
 Infantile (less than 3 years)
 Majority Resolve spontaneously
without treatment [observe only]
 Has the best prognosis
 Juvenile (3 to 10 years)
 These can be rapidly progressive or can
cause malignancy  Measure leg lengths.
 Hast the worst prognosis
 Adolescent (11 years or older)
*Each has different treatments

SIGNS AND SYMPTOMS


 They vary, depending on the location of the spine
affected.
 For thoracic curves, the ribs are rotated on the
convex side, producing a "rib hump."
 The scapula on the same side becomes more
prominent.
 With thoracolumbar and lumbar curves, one side
of the pelvis becomes more prominent, giving the
appearance of a "high hip." EDITOR’S NOTE:
 Symptoms are few until adulthood, when back Actual leg length is measured from the Anterior
pain and nerve root pain may develop. Superior Iliac Spine (ASIS) to the Medial Malleolus
 One shoulder blade sticks out more while Apparent leg length is measured from the
 One side of the rib cage appears higher Umbilicus to the Middle Malleolus
 One hip appears more prominent
 Waist appears uneven  Do the forward bend test with the patient's legs
 Leg discrepancy [true and apparent leg length] straight and observe the entire spine for
asymmetry between the right and left sides.
DIFFERENTIAL DIAGNOSIS [Adam’s Test]
 Isolated rib rotation may occur without scoliosis  This test is most useful and highly
 Kyphosis: a curvature in the sagittal plane only, sensitive, and it is used in school screening
which may be confused with scoliosis, clavicle programs. If there is asymmetry, measure
fracture, or Sprengel's deformity may give the the slope between the right and left sides of
appearance of a "high shoulder." the rib cage.
 Leg length inequality may cause the appearance
of a "high hip."

PHYSICAL EXAMINATION
 Examine the patient while he or she is standing, to
see shoulder, rib, and hip asymmetry.

Transcriber/s: Paolo M. Ramirez


Formatting: Paolo M. Ramirez D L S H S I M e d i c i n e B a t c h 2 0 1 6 | 3 of 7
Editor/s: Leanne Gale Ramos & Jan Cynric Cacao
 Some patients with a positive forward bend test PATHOLOGIC FINDINGS
do not have significant scoliosis. Follow-up a  The vertebrae are rotated toward the convexity
positive test with a radiograph if the rib slope is of the curve. In addition, individual vertebrae are
more than 6 degrees. misshapen because of growth while curved.
 Repeat the test if an abnormality is found.
 Quantify rib prominence or a hump by a IMAGING PROCEDURES
scoliometer.
 Standing posteroanterior radiography of the
entire spine is indicated [with no shoewear]
 Lateral films should be obtained if associated
abnormal kyphosis is present.
 Magnetic resonance imaging is indicated only if
there is possible spinal cord disease.
 Spine films usually show the iliac crests and allow
determination of the Risser stage for skeletal
maturity.
 The Risser stage (0-5) is the amount of
ossification of the iliac growth cartilage.
 Risser 0 is unossified, skeletally
immature
 Risser 5 is fully ossified, mature.

 Observe any kyphosis and lordosis.


 Inspect the skin over the entire spine for dimples,
hair, or vascular markings, which may signal an
underlying congenital anomaly.
 Rule out ligamentous laxity.
 Examine for café-au-lait spots or neurofibromas.
[these are usually found in Neuromuscular Scoliosis
indicating Neurofibromatosis]

EDITOR’S NOTES:
 About one-third of patients with neurofibromatosis EDITOR’S NOTE:
develop spinal deformity, the severity of which varies  Risser 1 – ilium is calcified at a level of 25%; it
from very mild (and not requiring any form of corresponds to puberty or early puberty
treatment) to the most marked manifestations  Risser 2 – ilium is calcified at a level of 50%; it
accompanied by skin lesions, multiple neurofibromata corresponds to the stage before or during growth spurt
and bony dystrophy affecting the vertebrae and ribs.  Risser 3 – ilium is calcified at a level of 75%; it
The scoliotic curve is typically ‘short and sharp’. Other corresponds to the slowing of growth
clues to the diagnosis lie in the appearance of the skin  RIsser 4 – ilium is calcified at a level of 100%; it
lesions and any associated skeletal abnormalities. corresponds to the almost cessation of growth
 Risser 5 – ilium is calcified at the level of 100% and the
iliac apophysis is fused to the iliac crest; it corresponds
 Perform a careful neurologic examination to the end of growth

LABORATORY TESTS
 None are needed for diagnosis or non-operative
treatment.

Transcriber/s: Paolo M. Ramirez


Formatting: Paolo M. Ramirez D L S H S I M e d i c i n e B a t c h 2 0 1 6 | 4 of 7
Editor/s: Leanne Gale Ramos & Jan Cynric Cacao
SURGICAL TREATMENT
 If a curve is to be fused, a rod is used to correct
the curve, and a bone graft is placed along this to
cause the vertebrae to fuse together.
 Only the curved region is fused.
 The neurologic risk is currently less than 1%.

EDITOR’S NOTES:
 OPERATIVE TREATMENT:
 The objectives are: (a) to halt progression of the
deformity; (b) to straighten the curve (including the
rotational component) by some form of
instrumentation; and (c) to arthrodese the entire
primary curve by bone grafting.
 Complications of surgery
 Neurological compromise: with modern techniques
the incidence of permanent paralysis has been
reduced to less than 1%. 

 Spinal decompensation: over-correction may
produce an unbalanced spine. This should be
avoided by careful preoperative planning. 

 Pseudarthrosis: incomplete fusion occurs in about
2% of cases and may require further operation and
grafting.
 Implantfailure:hooksmaycutoutandrodsmay
break. If this is associated with a symptomatic
pseudarthrosis, revision surgery will be needed. 

BIGGEST NOTES:
 Besides the pelvis and spine, the hand and
wrist can be used to determine skeletal PHYSICAL THERAPY
maturity.  Strengthening and stretching of abdominal and
 Wrist/Hand x-ray is taken on all patients extensor muscles if pain exists
under the age of 20. This is the most  Not indicated for routine cases of scoliosis
accurate among all the methods for  Does not help correct the curves
bone aging. Hand features vary with
the age of the child. In younger
 EDITOR’S NOTES (Nice to know):
children the presence or absence of
 The goal of physical therapy in the management of
certain carpal or epiphyseal ossification scoliosis is generally to prevent the progression of the
centers are often pointers for the degree of the scoliosis by prescribing exercises to stretch
physician about the skeletal age of a and strengthen the back, depending on the laterality of
child. the curve, and the level of the deformity. Again, as
mentioned above, PT will not necessarily “correct” the
skeletal deformity, however, depending on the cause
TREATMENT and the age of the patient (e.g. if the epiphyseal growth
 General Measures: plates have not yet closed, especially for children), then
 Observation for curves greater than 25 there’s still a chance to decrease (or even correct) the
angulation of the deformity. The exercises would
degrees in growing patients
address also the muscle imbalances brought about the
 Bracing for curves between 25 and 40 spinal deformity, which would sometimes contribute to
degrees in growing patients the low back pain.
 Surgery for curves greater than 45 degrees  Another goal is to relieve or alleviate back pain (if
 Physical therapy and exercise if there is present) with the use of electroanalgesic modalities
pain or stiffness [make the muscles flexible to such as Transcutaneous Electrical Nerve Stimulation
decrease load brought on the disc and prevent (TENS) or therapeutic US.
slipped disc, does not treat the scoliosis per se]  Finally, therapists would also provide patient education
on postural and proper back care, as well as emphasize
the importance of compliance of performance of the
DOC’S NOTES: prescribed exercises.
To prevent slipped disc or back problems, while sitting your
knee/s should be higher than your hip
MEDICAL TREATMENT
BIGGEST NOTES:  Patients with larger curves (greater than 45
REMEMBER! degrees) should see an orthopaedic surgeon to
 >25˚  Observe
see whether correction is indicated. Patients with
 25˚-40˚  Bracing [or Risser 2]
 45˚  Surgery [60 degrees thoracic and 40 degrees
moderate curves (25 to 40 degrees) should be
lumbar] braced if significant growth remains.
*The upper limit of bracing would be until 45˚!!!  Patients with minor curves (less than 25 degrees)
should be observed if they are still growing, but
they can be discharged if skeletal maturity has
been reached.

Transcriber/s: Paolo M. Ramirez


Formatting: Paolo M. Ramirez D L S H S I M e d i c i n e B a t c h 2 0 1 6 | 5 of 7
Editor/s: Leanne Gale Ramos & Jan Cynric Cacao
 Encourage patients to be as active as possible. BIGGEST NOTES:
 The spine in patients with scoliosis is not unstable.  The Cobb method is used to measure the
curvature of the scoliosis. The accuracy of the
CONCLUSION: SCOLIOSIS Cobb method relies on determining the upper
 May have 1 or more of the ff: and lower end-vertebrae of the curve. The
 One shoulder higher than the other end-vertebrae at both the upper and lower
 One shoulder blade sticks out more limits are those that tilt most severely
 One side of the rib cage appears higher toward the concavity of the curve
 One hip appears more prominent  In other words, the superior end-vertebra is
 Waist appears uneven the last vertebra whose superior border
 Leg discrepancy inclines toward the concavity of the curve to
 Neurologic exam be measured and the inferior end-vertebra is
 Abnormal skin lesions: café au lait, hemangiomas the last one whose inferior border inclines
toward the concavity of the curve. Horizontal
lines are drawn at the superior border of the
superior end-vertebra and the inferior border
of the inferior end-vertebra. Perpendicular
lines are then drawn from each of the
horizontal lines and the intersecting angles
measured.
 A curvature of more than 60˚ would
compromise the pulmonary function test
hence would need surgery.

BIGGEST NOTES:
 Plumbine Test - This is a quick visual
check to see if the spine is straight. In
scoliosis, the plumb line will fall to the left
or right of the spine instead of through
the middle of the buttocks.

-END-

TRANSCRIPTION DETAILS
Latest PPT;
8- Deuteronomy’s
BASIS Netter Collection of Musculoskeletal RECORDINGS + NOTES + DEVIATIONS CREDITS
10% Recordings 
Imaging - Spine
References:
 Apley's and Solomans Consise System of Orthopedics and Trauma 4th Ed.
REMARKS
 Google Image Search
Editor: Added additional notes and Past-e. Emphasized important point to remember
-BATCH 2016 Transcribers’ Guild Transcriptions. Version 2.0.0.0.0 Build 3205-

Transcriber/s: Paolo M. Ramirez


Formatting: Paolo M. Ramirez D L S H S I M e d i c i n e B a t c h 2 0 1 6 | 6 of 7
Editor/s: Leanne Gale Ramos & Jan Cynric Cacao
PAST-E (BATCH 2015)

The simplest screening test for Rib hump is seen on the A T8 left-hemivertebra with a
detection of scoliosis is a. Thoracic concavity curve of 35 degrees and another L3
a. Adam’s b. Thoracic convexity right hemivertebra with 28
b. Stagnara c. Lumbar concavity degrees is in a female of Risser II
c. Side bending d. Lumbar convexity should be
d. None of the above a. Observed
The “end vertebra” is one whose b. Brace
To determine flexibility of the a. Superior end plate is c. Fusion in situ
curve, one should do maximally tilted towards d. Surgical correction
a. Bending concavity
b. Traction b. Superior end plate is A 5 year old polio patient has 40
c. Fulcrum maximally tilted towards degrees curve should be treated
d. All of the above convexity by
c. Inferior end plate is a. Observation
To determine a balanced spine, maximally tilted towards b. Bracing until 18 or 19
one should do convexity years old
a. Plumb line d. Vertebra is most c. Bracing until end of
b. Forward bending deformed adolescence and fused
c. Supine bending Which among the idiopathic d. Fusion at once
d. None of the above causes has the best prognosis?
a. Infantile Treatment of paralytic curve of 45
The apex vertebra is b. Juvenile degrees in a 20 year old is
a. Most deformed c. Adolescent a. Observation
b. One whose superior end d. Adult b. Bracing
plate is maximally tilted c. Exercise
towards concavity Which among the idiopathic d. Surgery
c. Whose inferior end plate causes has the worst prognosis?
is maximally towards a. Infantile Treatment of 40 degrees thoracic
convexity b. Juvenile and 25 degrees lumbar curve with
d. Not deformed c. Adolescent Risser II is
d. Adult a. Observation
The most common among the b. Exercise
functional scoliosis is What is the most common c. Bracing
a. Idiopathic congenital anomaly associated d. Surgery
b. Congenital with scoliosis?
c. Neuromuscular a. GIT
d. Marfan’s b. Heart
c. Genitourinary
What x-ray views are used to d. Pulmonary
determine skeletal maturity?
a. Spine The upper limit of a thoracic curve
b. Pelvis for bracing to be effective is
c. Left wrist a. 20 degrees
d. All of the above b. 30 degrees
c. 45 degrees
Which of the different type of d. 60 degrees
scoliosis does not make use of
skeletal maturity? The surgical indication for scoliosis
a. Idiopathic surgery is at least CONGRATULATIONS TO BATCH
b. Neuromuscular a. 60 degrees thoracic and 2016 FOR WINNING BEST IN
c. Congenital 40 degrees lumbar PRODUCTION AND BEST IN
d. Marfan’s b. 40 degrees thoracic and SHOW CHOIR & BAND IN JFT
60 degrees lumbar 2014 
Which of the following congenital c. 60 degrees thoracic and
scoliosis has the worse prognosis? 60 degrees lumbar
a. Left hemivertebra of T7 d. 40 degrees thoracic and
and right hemivertebra of 40 degrees lumbar
L2
b. Incarcerated vertebra of Treatment of infantile idiopathic
T10 scoliosis is
c. End-block vertebrate of a. Observation
T10 & T11 b. Manual stretching
d. Unilateral bar of T10-T11 c. Bracing
d. Surgical

Transcriber/s: Paolo M. Ramirez


Formatting: Paolo M. Ramirez D L S H S I M e d i c i n e B a t c h 2 0 1 6 | 7 of 7
Editor/s: Leanne Gale Ramos & Jan Cynric Cacao
POTT’S DISEASE
Leonido C. Castillo, MD, FPOA
November 11, 2014; 1:00 – 2:00 PM
Orthopedics

LECTURE OUTLINE
 Review of the spine anatomy  Imaging studies
 Pott’s disease  Treatment
 Malignant tumor  Tuberculous kyphosis
 Lab studies  Anterior spinal fusion

REVIEW OF THE SPINE ANATOMY BACKGROUND


 The spine is composed of 33 vertebra  Tuberculous spondylitis has been documented in
 7 cervical ancient mummies from Egypt and Peru and is one
 12 thoracic of the oldest demonstrated diseases of
 5 lumbar humankind.
 5 sacral  Percival Pott presented the classic description of
 1 coccyx (4 fused bones) spinal tuberculosis in 1779.
 Normal spinal curvatures  Since the advent of antituberculous drugs and
 Cervical lordosis improved public health measures, spinal
 Thoracic kyphosis tuberculosis has become rare in industrialized
 Lumbar lordosis countries, although it is still a significant cause of
 Sacral kyphosis disease in developing countries.
 Tuberculous involvement of the spine has the
potential for serious morbidity, including
NOTES:
permanent neurologic deficits and severe
 The spinal cord spans the WHOLE length of the
deformity. Medical treatment or combined
vertebral column IN UTERO while the tip is found at
medical and surgical strategies can control the
the level of L1-L2 on adults
disease in most patients.
 The reason why there is discrepancy on the length of
the cord in utero vs. adult cord is because bones grow
faster than soft tissue EDITOR’S NOTES:
 The spine is the most common site of skeletal TB,
accounting for 50% of all musculoskeletal TB.

PATHOPHYSIOLOGY
 Pott’s disease is usually secondary to an
extraspinal source of infection.
 The basic lesion is a combination of osteomyelitis
and arthritis.
 Typically, more than one vertebra is involved.
 The area usually affected is the anterior aspect of
the vertebral body adjacent to the subchondral
plate.
 Tuberculosis may spread from that area to
adjacent intervertebral disks.
 In adults, disk disease is secondary to the spread
of infection from the vertebral body.
 In children, because the disk is vascularized, it can
be a primary site.
 Progressive bone destruction leads to vertebral
collapse and kyphosis.
 The spinal canal can be narrowed by abscesses,
granulation tissue, or direct dural invasion.
 This leads to spinal cord compression and
neurologic deficits.
POTT’S DISEASE  Kyphotic deformity occurs as a consequence of
 Is a chronic granulomatous inflammation of the collapse in the anterior spine.
spine caused by mycobacterium tuberculosis and  Lesions in the thoracic spine have a greater
is almost always secondary to a lesion elsewhere. tendency for kyphosis than those in the lumbar
 It is a local manifestation of a systemic disease. spine.

Transcriber/s: Jan Cynric Cacao


Formatting: Jan Cynric Cacao D L S H S I M e d i c i n e B a t c h 2 0 1 6 | 1 of 7
Editor/s: Paolo Ramirez, Leanne Gale Ramos
 A cold abscess can occur if the infection extends  Night cries [Relaxation of muscles during
to adjacent ligaments and soft tissues. sleep permits movements which are very
 Abscesses in the lumbar region may descend painful and wake-up the patient]
down the sheath of the psoas to the femoral  Pain can be spinal or radicular.
trigone region  cold abscess and eventually  Constitutional symptoms include low grade
erode into the skin  chronic draining sinus. fever, anorexia, and weight loss.
 Neurologic abnormalities occur in 50% of cases
NOTES: and can include spinal cord compression with
 TB of the spine is almost always secondary to a paraplegia, paresis, impaired sensation, nerve
primary lesion (usually pulmonary TB) root pain, or cauda equina syndrome.
 TB bacilli migrates to other parts of the body
(including the spine) via the bloodstream; a process PHYSICAL EXAMINATION
called bacteremia  Examination should include the following:
 Once the TB bacilli lodges to another site, it may go  Careful assessment of spinal alignment
into latency and will reactivate later if the  Inspection of skin, with attention to
environment is suitable for growing which is the detection of sinus(es)
state of immunosuppression  Abdominal evaluation for subcutaneous
flank mass
 The first ten years of life, the vertebral disc is
vascularized and this is the time where the TB bacilli  Meticulous neurologic examination
goes into the area and cause discitis  Spine deformity (kyphosis) of some degree
 As the person matures and the disc becomes occurs in almost every patient.
avascular, the focus of the infection may migrate  There may be large cold abscesses of paraspinal
centrally to the vertebral body or it may be tissues or psoas muscle that protrude under the
paradiscal(above and below) inguinal ligament. They may erode into the
perineum or gluteal area.
BIGGEST NOTES:  Neurologic deficits may occur early in the course
 What’s the difference between tuberculosis of the of disease. Signs depend on the level of spinal
spine & bacterial osteomyelitis of the spine? cord or nerve root compression.
- The diagnostic clue to the presence of
tuberculosis is that the disk is preserved,
EDITOR’S NOTES:
whereas in bacterial osteomyelitis of the
There is usually a long history of ill health and backache; in
spine, the disk is destroyed. late cases a gibbus deformity is the dominant feature.
Concurrent pulmonary TB is a feature in most children under
10 years with thoracic spine involvement.

DIFFERENTIAL DIAGNOSIS
 Actinomycosis
HISTORY  Blastomycosis
 Presentation depends on the following:  Brucellosis
 Stage of disease  Candidiasis
 Site  Cryptococcosis
 Presence of complications such as  Histoplasmosis
neurologic deficits, abscesses, or sinus tracts
 Metastatic Cancer, Unknown Primary Site
 The reported average duration of symptoms at
 Miliary Tuberculosis
the time of diagnosis is 3-4 months.
 Multiple Myeloma
 Back pain is the earliest and most common
 Mycobacterium Avium-Intracellulare
symptom.
 Mycobacterium Kansasii
 Patients have usually had back pain for
 Nocardiosis
weeks prior to presentation.

Transcriber/s: Jan Cynric Cacao


Formatting: Jan Cynric Cacao D L S H S I M e d i c i n e B a t c h 2 0 1 6 | 2 of 7
Editor/s: Paolo Ramirez, Leanne Gale Ramos
 Paracoccidioidomycosis IMAGING STUDIES
 Septic Arthritis  Radiographic changes present relatively late.
 Spinal Cord Abscess However, earliest radiographic feature is
 Tuberculosis localized osteoporosis.
 Plain radiography demonstrates the following
NOTE: Other considerations: spinal tumors characteristic changes of spinal tuberculosis:
 Lytic destruction of anterior portion of
vertebral body
EDITOR’S NOTES:  Increased anterior wedging
Spinal TB must be distinguished from other causes of
 Collapse of vertebral body
vertebral pathology, particularly pyogenic and fungal
infections, malignant disease and parasitic infestations such
 Decrease or absent intervertebral disc
as hydatid disease. Disc space collapse is typical of infection; space
metastatic lesions may cause vertebral body collapse similar  Reactive sclerosis on a progressive lytic
to that seen in TB but the disc space is usually preserved. process
 Enlarged paravertebral shadow with or
without calcification [produced by extension
INFECTION VS. MALIGNANCY of tuberculous granulation tissue and the
 Progressive pain collection of an abscess in the paravertebral
 Night pain region]
 Not relieved by rest  Absent psoas shadow
 Compression fracture
 Destruction of pedicle
 Intact disc space EDITOR’S NOTES:
 Chronic intermittent low grade fever The entire spine should be x-rayed, because vertebrae distant
from the obvious site may also be affected. The earliest signs
 Gibbus
of infection are local osteoporosis of two adjacent vertebrae
 Back pain and narrowing of the intervertebral disc space, with fuzziness
 May have neurologic defect of the end-plates. Progressive disease is associated with
signs of bone destruction and collapse of adjacent vertebral
NOTE: Red flag – pain unrelieved by usual analgesics bodies into each other.

MALIGNANT TUMOR PLAIN RADIOGRAPHY


 Common Primary sites  Loss of cervical lordosis
 Breast  Soft tissue mass
 Lung  Lytic lesion
 Prostate  Disc space involvement
 Kidneys
 Thyroid
 Systemic Malignancies

LAB STUDIES
 Tuberculin skin test (purified protein derivative
[PPD]) demonstrates a positive finding in 84-95%
of patients who are non–HIV-positive.
 Erythrocyte sedimentation rate (ESR) may be
markedly elevated (>100 mm/h).
 Microbiology studies to confirm diagnosis:
Obtain bone tissue or abscess samples to stain
for acid-fast bacilli (AFB), and isolate organisms
for culture and susceptibility.
 CT-guided procedures can be used to guide
percutaneous sampling of affected bone or
soft tissue structures.
 These study findings may be positive in only
about 50% of the cases.
 CRP is always elevated and can be used as a Cervical spine is straightened due to the affectation of the
gauge of activity of the disease & response to anterior portion of the vertebral body
treatment (therapeutic trial)
 Alkaline phosphatase is normal to twice elevated
 Can be used to differentiate from
malignancy
 POLYMERASE CHAIN REACTION has a high
sensitivity & specificity to tuberculosis specially
lung, CNS & Pott’s

Transcriber/s: Jan Cynric Cacao


Formatting: Jan Cynric Cacao D L S H S I M e d i c i n e B a t c h 2 0 1 6 | 3 of 7
Editor/s: Paolo Ramirez, Leanne Gale Ramos
 Additional findings
 Vertebral end plates are osteoporotic.
 Intervertebral disks may be shrunk or
destroyed.
 Vertebral bodies show variable degrees of
destruction.
 Fusiform paravertebral shadows suggest
abscess formation.
 Bone lesions may occur at more than one
level
 Chest x-ray maybe positive.

CT SCANNING
 CT scanning provides much better bony detail of
irregular lytic lesions, sclerosis, disk collapse, and
disruption of bone circumference.
 Low-contrast resolution provides a better
assessment of soft tissue, particularly in epidural
The picture shows disc space involvement and lytic lesion and paraspinal areas.
 It detects early lesions and is more effective for
MYELOGRAM defining the shape and calcification of soft tissue
 Dye is used to see if there is an obstruction of the abscesses.
spinal fluid  In contrast to pyogenic disease, calcification is
 Not used anymore common in tuberculous lesions.

MRI
 Can visualize both soft tissues, spinal cord and
abscess
 With contrast, can differentiate infection from
RADIOLOGIC FINDINGS SUGGESTIVE OF: malignancy
Pott’s disease Malignancy  With serial MRI, can detect the earliest change in
Pedicle destruction (-) (+) response to treatment
 MRI with gadolinium can differentiate infection
Disc involvement (+) (-) from tumor

*These radiologic findings are suggestive and are NOT


pathognomonic findings.

Transcriber/s: Jan Cynric Cacao


Formatting: Jan Cynric Cacao D L S H S I M e d i c i n e B a t c h 2 0 1 6 | 4 of 7
Editor/s: Paolo Ramirez, Leanne Gale Ramos
TREATMENT HALO-PELVIC TRACTION
 Medical
 Medical therapy requires combination
regimens with at least 3 anti-tuberculous
drugs.
 In regions with less than 4% isoniazid (INH)
resistance, a 4-drug regimen should be used
empirically.
 Treatment can be adjusted when
susceptibility information becomes
available.
 INH and rifampicin should be administered
during the whole course of therapy.
 Additional drugs are administered during
the first 2 months of therapy. These are
generally chosen among the first-line drugs,
which include pyrazinamide, ethambutol,
and streptomycin.
BIGGEST NOTES:
 The use of second-line drugs is indicated in
 Halo-Pelvic Traction is a device used to correct
cases of drug resistance.
spine deformities by placing a “Halo” (a metal
 Liver function test should be regularly
ring secured with four screws into the outer
monitored.
dipole of the skull) and a Pelvic ring (secured to
 CRP & ESR should be regularly monitored.
the two halves of the pelvis by two threaded
 Surgical
pins that traverse the whole length of the two
 Indications:
halves of the pelvis)
 Neurologic deficit (autonomic
 Extension bars fitted to the halo and pelvic rings
dysfunction, acute neurologic
provide fixation and traction when the bars are
deterioration, paraparesis, paraplegia)
gradually elongated
 Spinal deformity with instability
 No response to medical therapy
 Nondiagnostic percutaneous needle COMPLICATIONS OF HALO-PELVIC TRACTION
biopsy sample  Traction nerve injuries
 Resources and experience are key factors in the  Abducent nerve palsy
decision to use a surgical approach.  Hypoglossal nerve palsy
 The most appropriate method of reconstruction
depends on the level of vertebral spine involved TUBERCULOUS KYPHOSIS
and the extent of bony destruction.  3- or 4-stage surgery, nearly or more than 1 year
 The lesion site, extent of vertebral destruction,
and presence of cord compression or spinal
deformity determine the specific operative
approach.
 Vertebral damage is considered significant if
more than 50% of the vertebral body is
collapsed or destroyed or a spinal deformity
of more than 5° exists.
 The most conventional approaches include
anterior radical focal debridement and posterior
stabilization with instrumentation.
ANTERIOR SPINAL FUSION
EDITOR’S NOTES:  For lumbar degenerative intervertebral disc
The objectives are: (1) to eradicate or arrest the infection; (2)
to prevent or correct deformity; and (3) to prevent or treat the
major complication – paraplegia.

Chest x-ray showing vertebral shadow and the psoas line

Transcriber/s: Jan Cynric Cacao


Formatting: Jan Cynric Cacao D L S H S I M e d i c i n e B a t c h 2 0 1 6 | 5 of 7
Editor/s: Paolo Ramirez, Leanne Gale Ramos
-END-

TRANSCRIPTION DETAILS
BASIS Latest PPT RECORDINGS + NOTES + DEVIATIONS 8-10% CREDITS -
REMARKS -
-BATCH 2016 Transcribers’ Guild Transcriptions. Version 2.0.0.0.0 Build 3206-

PAST-E BATCH 2015

All the probable sites of implantation of the spinal The earliest radiologic finding in Pott’s disease is
column in adults, EXCEPT a. Paravertebral shadow
a. Vertebral body b. Disc collapse
b. Intervertebral disc c. Localized osteoporosis
c. Lamina d. Lytic lesion in the spine
d. Pedicle
In the antero-posterior view of the spine, the
All are probable sites of implantation of the spinal “paravertebral shadow” is seen at the region of
column at 3 years of age, EXCEPT a. Cervical
a. Vertebral body b. Thoracic
b. Intervertebral disc c. Lumbar
c. Pedicles d. Sacral
d. None of the above
Absent psoas shadow or line is PATHOGNOMONIC of
The intervertebral disc in adult Pott’s collapsed due to a. Pott’s disease
a. Compression of two adjacent vertebrae b. Psoas abscess
b. Direct involvement of TB bacilli c. Psoas hematoma
c. Lack of nutrition d. All of the above
d. All of the above
The alkaline phosphates in Pott’s disease is NOT
Pott’s disease emanates from a. Normal
a. PTB b. One and a half elevated
b. GI TB c. Twice elevated
c. Genitourinary TB d. Thrice elevated
d. TB meningitis
The single most specific and sensitive exam for Pott’s
Majority of musculoskeletal TB involves the disease is
a. Hip a. ESR
b. Knee b. CRP
c. Wrist c. Alkaline phosphatase
d. Spine d. PCR

“Night cry” is due to Which among the various test should be used to
a. Pressure on the gibbus when lying supine determine the activity in Pott’s
b. Increased activity of TB bacilli at night a. ESR
c. Increased symptomatology at night b. CRP
d. Release of muscle spasm at night c. Alkaline phosphatase
d. PCR
The earliest complaint of Pott’s is
a. Back pain In the so called “therapeutic trial” to diagnose Pott’s, one
b. Fever should do serial determination of
c. Paraparesis a. ESR
d. Gibbus b. CRP
c. Alkaline phosphatase
“Gibbus” is early seen and exaggerated in d. Aspiration and culture
a. Cervical area
b. Thoracic area To determine efficacy of anti-koch’s treatment, the most
c. Lumbar area sensitive test signifying successful treatment is
d. Sacral area a. ESR
b. CRP
c. Alkaline phosphatase
d. CBC

Transcriber/s: Jan Cynric Cacao


Formatting: Jan Cynric Cacao D L S H S I M e d i c i n e B a t c h 2 0 1 6 | 6 of 7
Editor/s: Paolo Ramirez, Leanne Gale Ramos
The mainstay for the treatment of Pott’s disease is An 8 year old previously treated case of Pott’s disease of
a. Anti-koch’s T7, T8 and T9 3 years ago is complaining of progressive
b. Immobilization kyphosis without neurologic signs and symptoms and
c. Bed rest had twice ESR and CRP within normal limits. He should
d. Surgery be treated by
a. Resume quadruple anti-koch’s
The absolute indication for surgical treatment of Pott’s is b. Start with secondary anti-koch’s
a. Paraparesis c. Body casting
b. Paraplegia d. Surgery
c. Kyphosis
d. Autonomic dysfunction Which surgical procedure is a big NO in Pott’s disease of
vertebral bodies of T10, T11, T12 with marked gibbus
a. Anterior decompression and bone grafting from
T9-T11
b. Anterior decompression, bone grafting and
plate-screw construct from T8-L2
c. Laminectomy from T10-T12
d. Laminectomy from T-10-T12 with pedicle screw
construct from T8-L3

Transcriber/s: Jan Cynric Cacao


Formatting: Jan Cynric Cacao D L S H S I M e d i c i n e B a t c h 2 0 1 6 | 7 of 7
Editor/s: Paolo Ramirez, Leanne Gale Ramos
COLD ORTHOPEDICS OF THE HAND
Joaquin C. Pandanan, MD, FPOA
November 25, 2014; 1:00 – 2:30 PM
Orthopedics

LECTURE OUTLINE:
 Introduction  Bites
 Infectious  Office Orthopedics
 Felon  Carpal Tunnel Syndrome
 Paronychia  Trigger finger/ Trigger thumb
 Herpetic Whitlow  De quervain’s tenosynovitis
 Pyogenic Flexor tenosynovitis  Ganglion cyst
 Interdigital Space Infection  Summary

INTRODUCTION
 Background
 “These are the usual cases seen in the clinics and  Radiographs
sometimes in the emergency room”  Hand x-ray – AP & Oblique
 So why is it called “COLD” Orthopedics?  X-ray to rule out osteomyelitis or presence
 It is termed “COLD” because it usually of a foreign body
deals with infections and most are  If the lesion is present for more than 2
done in an office setting weeks, you will see an osteomyelitis lesion
 Summary of Conditions  Aspiration
INFECTIOUS OFFICE ORTHOPEDIC  If there are doubts regarding whether the
Felon Carpal Tunnel Syndrome mass is solid or fluid
Paronychia Trigger finger/ Trigger thumb  CT-scan/MRI
Herpetic Whitlow De Quervain’s tenosynovitis  Unnecessary
Flexor tenosynovitis Ganglion cyst
Deep palmar abscess INFECTIOUS
 Usual Etiologic Agents
 Pertinent History  Staphylococcus aureus (80%-90%)
 Age  presence yellowish discharge, fluctuant
 Handedness mass, erythema
 Vaccination history  β-hemolytic streptococcus
 Occupation  (+) streaks ascending arm with
 Laborer, Dentist, Medical personnel, Office lymphadenopathy or lymphangitis
personnel  Candida albicans
 Immunologic status  Chronic paronychia; patients who are
 DM, under chronic steroids,HIV in wet-related work (dishwashing, etc.)
 These patients are immunocompromised,  Polymicrobial
hence, the chances of complicated  immunocompromised patients (DM,
aggressive infections, polymicrobial HIV, etc.)
infections, repeat-surgeries, and
amputation are high. Editor’s Note:
 Mechanism and timing  It is important to identify the etiologic agent of infection to
 Bite be able to give the proper antibiotic to the patient
 Injection  Aggressive treatment is indicated for immunocompromised
 Penetrating trauma [80% of the hand patients and empiric treatment should be given
infections are mostly penetrating trauma]
 Physical Examination
FELON
 Observation
 Pathology
 Swelling
 Deep infection of the finger pulp usually
 Fluctuance or Mass on Hand
secondary to punctured wound (80%-90%)
 Focal tenderness – know location
 Usually caused by S. aureus
 ROM of involved finger or wrist [sometimes
 Symptoms
the elbow]
 Throbbing pain on the distal aspect of the
 Special Studies
finger
 Laboratories
 Redness or swelling
 CBC, ESR, CRP
 Visible pus or yellowish discoloration on
These are baseline or guide for you to
distal pulp
know if you will continue the antibiotic
 ROM is still good
treatment or proceed with surgery, if the
patient is not improving.

Transcriber/s: Paolo M. Ramirez


Formatting: Paolo M. Ramirez D L S H S I M e d i c i n e B a t c h 2 0 1 6 | 1 of 14
Editor/s: Chrismark Reyes & Jan Cynric Cacao
PARONYCHIA
 Pathology
 Nail-fold infection
 Infection of the soft tissue around the
fingernail
 Aka “Run around abscess or infection”
 Most common bacterial infection of the
hand
 Seen most often in children, or in older
people after rough nail-trimming
 Caused usually by S. aureus
 Associated with hangnail, nail biting, finger
sucking and manicures
 Chronic cases: usually caused by Candida
 Complications if Untreated albicans
 Osteomyelitis of the distal phalanx, skin  Symptoms
necrosis, septic arthritis of the DIP  Pus around the nail plate
 Higher chance of amputation  Edge of the nail-fold becomes red and
 Treatment swollen and increasingly tender
st
 Antibiotics (Oxacillin or 1 gen
cephalosporin)
 Early Incision and Drainage
 Midlateral incision on the pulp, squeeze it,
then pus will come out

 Treatment
 Antibiotics (1st-gen Cephalosporin)
 Chronic Candida albicans cases: antifungal
 Do I and D
 Incision at the corner of the nail-fold in line
with the edge of the nail
 A pledget of paraffin gauze is used to keep
the nail-fold open
 Sometimes you have to cut the nail plate
and elevate to evacuate the pus
 Usually done if the pus spreads under the
nail

HERPETIC WHITLOW
 Pathology
 Painful infection caused by HSV 1 or 2
 Possibly due to auto-inoculation from
patient’s own mouth or genitalia or by cross
infection during dental surgery
 Occurs 2-14 days after exposure
 Common among dental workers and
BIGGEST NOTES: medical personnel exposed to oral
 A felon is often confused with acute paronychia, secretions
but it is a soft tissue infection of the fingertip pulp. It  Symptoms
may arise secondary to an acute paronychia. The  lymphadenitis, malaise, fever
clinical findings are those of a swollen, red, painful  Vesicular formation [initially] then bullae
finger pad. The treatment is surgical incision and formation
drainage together with oral antibiotics to cover S.
aureus and Streptococcus species.

Editor’s Note:
 In closed tissue compartments, (e.g. pulp space in
case of a felon) pressures may rise to levels where
the local blood supply is threatened, with the risk of
tissue necrosis

Transcriber/s: Paolo M. Ramirez


Formatting: Paolo M. Ramirez D L S H S I M e d i c i n e B a t c h 2 0 1 6 | 2 of 14
Editor/s: Chrismark Reyes & Jan Cynric Cacao
 Treatment
 Conservative; It is self-limiting to 7-10 days
 Paracetamol may be given for the fever
 I and D is contraindicated! This will only
cause spread of the virus. [important to
differentiate HW with Paronychia]
 You can give Acyclovir to relieve the
symptoms, but the disease will still run its
course. Antibiotics are given only in the
presence of secondary bacterial infection.

DOC’S NOTES:
 Diagnosing the case can be done clinically.
Laboratory procedures can also be done: Tzanck
Preparation
 Aspirate the part the fluid and assess for
presence of Tzanck cells [three multinucleated
giant cells]

BIGGEST NOTES:
 Herpetic whitlow is the name given to a specific
form of infection that is most commonly seen in
medical laboratory workers and health care
providers. It occurs from accidental inoculation of
the herpesvirus into the skin. The finger is the area
most commonly involved, because of accidental
needle sticks.

PYOGENIC FLEXOR TENOSYNOVITIS (PFT)


 Anatomy of the Hand Bursa
 Bursa
 Radial Bursa – Thumb
 Ulnar Bursa – Fifth Digit

 Pathology
 Common infection of the hand
 Bacterial infection of the tendon sheath
 Cause by S. aureus, and Strep
 Occurs due to penetrating trauma
 Key Symptoms
 Kanavel’s 4 Cardinal sign
 Fusiform swelling
 Flexed position/posture of the digit
 Pain on passive finger extension (Early
onset)
 Tenderness along the tendon sheath
(MOST IMPORTANT SIGN)

BIGGEST NOTES:
 1 & 5 infection will spread or drain to their
st th
nd rd th
respective bursa. Tenosynovitis of 2 , 3 & 4 will
limit infection on the same finger.

 Parona’s space
 Where the bursa drains Editor’s Note:
 Located above the pronator quadrates According to Apley’s, the Kanavel’s Signs of Flexor Sheath
and superiorly by your flexor tendon. Infection are as follows:
 Presence of abscess here is referred to as  Flexed position of digit
“Parona’s Abscess”  Tenderness along the course of the tendon
 Pain on passive finger extension
 Pain on active flexion

Transcriber/s: Paolo M. Ramirez


Formatting: Paolo M. Ramirez D L S H S I M e d i c i n e B a t c h 2 0 1 6 | 3 of 14
Editor/s: Chrismark Reyes & Jan Cynric Cacao
 Treatment  Presentation
 IV antibiotics  Palmar and dorsal
 Early I and D for persistent pus  Dorsal swelling more pronounced
 Late complications  Collar button abscess
 Skin necrosis, tendon lysis, contracted  Abducted position
finger, amputation in severe cases (usually in  Spread
DM patients)  Dorsally to the subcutaneous web space
 Horse shoe abscess (Parona’s Abcess)  Palmar fascia prevents spread beneath the
 A connection between the affected ulnar skin
and radial bursa forms a horse shoe shape.
This is a complication of flexor
tenosynovitis wherein an abscess of the
thumb may cause an abscess on the fifth
digit.

BIGGEST NOTES:
By the time the horseshoe abscess occurs, irrevocable
damage to the delicate gliding tissues of the
tenosynovial sheath may have occurred. Avascular
necrosis of the tendons follows quickly from vincular DOC’S NOTES:
occlusion and intracompartmental pressure How do you differentiate between interdigital web
space and dorsal subaponeurotic space?
Look at the finger if they are abducted. In dorsal
INTERDIGITAL (WEB) SPACE INFECTION subaponeurotic space, there is NO ABDUCTED
 Five Independent (Potential/Deep) Spaces of the positioning.
Hand
 Interdigital web space
 Thenar space  Treatment
 Hypothenar space  Antibiotics (1st-gen Cephalosporin)
 Midpalmar space  Double incision (dorsal and volar) to
 Dorsal subaponeurotic space evacuate abscess

 Other hand space infections


 Pathology  Thenar Space
 In comparison to flexor tenosynovitis, the
affected fingers here are usually in the
abducted position. Likewise, there is usually
a severe swelling on the dorsal side (dorsal
subcutaneous web space) of the affected
fingers or “Collar button abscess”. It does
not cause migration or spread palmarily due
to thickness of palmar fascia.

Transcriber/s: Paolo M. Ramirez


Formatting: Paolo M. Ramirez D L S H S I M e d i c i n e B a t c h 2 0 1 6 | 4 of 14
Editor/s: Chrismark Reyes & Jan Cynric Cacao
 Midpalmar Space  Treatment
 Never close bites primarily; you actually
have to open the wound
 Thorough washing and debridement/
Arthrotomy
 Puncture wounds may require
extension
 Delayed closure for large wounds;
Don’t suture!
 Antibiotics for hand infection (1st-gen
Cephalosporin)
 Proximal I and D
 When you actually try to hit a person, your
 Dorsal Subaponeurotic space tendon actually snaps back proximally, so I
and D is done on a more proximal site. You
need to extend the wound proximally

OFFICE ORTHPEDICS
CARPAL TUNNEL SYNROME
 Review of the Brachial Plexus

BITES
 Pathology
 Human
 “fight bite” often communicates with
Metacarpophalangeal joint
 Clenched fist tend to blow to the victim’s
teeth
 A clenched-fist injury typically is
characterized by a 3- to 5-mm laceration
on the dorsum of the hand or overlying an  The Brachial plexus is composed of C5-T1.
MCP joint  All the roots will unite to form trunks - the
 Usually caused by Staphylococcus aureus superior, medial, and lateral trunks.
or Eikenella corrodens (gram negative,  C5 and C6 = Superior Trunk
30%of the time, sensitive to penicillin)  C7 = Medial Trunk
 Because of the innocent appearance of  C8 and T1 = Inferior Trunk
this injury, patients may not seek medical  These three trunks will give rise to three
attention and commonly present with anterior and three posterior divisions, which
advanced infection will then form cords.
 Animal  The two anterior divisions will unite to form
 Domestic cat and dog can cause cellulitis the Lateral cord, which will give out the
 Usually caused by Pasteurella multocida Musculocutaneous nerve and contribute to
the Median nerve.
 The other anterior division will give rise to
the Medial cord, which will branch out as the
Ulnar nerve, and contribute to the Median
nerve.
 MEDIAL CORD + LATERAL CORD =
MEDIAN NERVE
 All the posterior divisions will unite and
become your Posterior cord, which will
become your Radial nerve and Axillary
nerves.
 Median Nerve (C5-C8)
 Provides motor to:
 Abductor pollicis brevis (APB)
 Flexor pollicis brevis (FPB)
 Muscle bulk to thenar
 Provides sensory to
 Lateral 3 ½ fingers (thumb, 2nd digit, 3rd
digit, and radial half of 4th digit)

Transcriber/s: Paolo M. Ramirez


Formatting: Paolo M. Ramirez D L S H S I M e d i c i n e B a t c h 2 0 1 6 | 5 of 14
Editor/s: Chrismark Reyes & Jan Cynric Cacao
 Confirmatory
 Electromyelogram & Nerve Conduction
Studies
 How do you locate your carpal tunnel? - You
try to oppose your thumb and little finger.
There’s a tendon (Palmaris longus) that
becomes stretched out. You go medially
from that and that’s where your median
nerve is located. Actually, it’s under the
Palmaris longus.
 Management
 NSAIDS
 Splinting
 Pathology  Night splinting with the wrist in neutral
 Compression of the median nerve at the  If pregnant, let the pregnancy finish
wrist by the transverse carpal ligament. first and put a splint for the mean time
 Result of any decrease in compartment size  Steroid injection (Cortisone)
or increase in compartment content  if the previous 2 fail
 Increase in compartment content is  injected in the carpal canal
noted in previous distal radius fracture,  Surgical release
and lipoma on the wrist.  If the previous 3 modalities fail; You
 Diagnosis will cut the transverse carpal ligament.
 Signs and Symptoms Be aware and be careful of the palmar
 Typical history of paresthesia along median cutaneous branch that provides
nerve distribution sensation to the thumb.
 Weakness (Cannot flex thumb and  Surgery can be performed either open
second digit will flex to or endoscopically with the shared goal
accommodate when forming a of complete release of the transverse
ring) carpal ligament and distal antebrachial
 Up to the 3rd digit only. It will fascia
NEVER INVOLVE the 5th digit.
 More pronounced at night (Pain and
numbness)
 Aggravated by grasping activities
 Physical examination
 Phalen’s test [Reverse Prayer Position]
 Flex the wrist for 2-3 minutes, and
then let the patient extend the
hand. Positive if there is a tingling
sensation in involved fingers.

TRIGGER FINGER/THUMB
 Anatomy of Hand Pulley
 5 annular pulleys (A1, A2, A3 A4, A5), 3
Cruciate pulleys
 Role of pulleys
 Reverse Phalen’s  Maintain flexor tendons at a fixed
 Extend first then flex. This is distance
MORE SENSITIVE.  Prevent bowstringing
 Tinel’s sign  A1– indispensable; this is where the
 Palpate for the median nerve pathology arises
origin and the patient will feel a
tingling sensation.

 Direct nerve compression test

Transcriber/s: Paolo M. Ramirez


Formatting: Paolo M. Ramirez D L S H S I M e d i c i n e B a t c h 2 0 1 6 | 6 of 14
Editor/s: Chrismark Reyes & Jan Cynric Cacao
 Pathology
 Most common stenosing tenosynovitis of
the hand
 Usually it is after repeated motion (like in
laborers like typewriters and secretaries)
 Most common in middle finger and ring
finger
 Thumb—common in postmenopausal
women
 Associated with carpal tunnel, inflammatory
conditions (gout, Rheumatoid arthritis,
amyloid deposition )
 Pathogenesis:
 Percutaneous release
 No need to open up. Use a gauge 19
needle and scrape the A1. Just be careful
because of the nerves.

 Clinical findings
DE QUERVAIN’S TENOSYNOVITIS
 Locked finger in flexion; Snapping upon
 Six Compartments of the Hand
finger extension
 When the finger is in the flexed position, it
is not painful. However, upon extension,
there is pain. The pain is common usually
in the morning because you tend to extend
your fingers upon waking up.
 Palpable nodule(A1 is usually above the
metacarpals; distal to that is where you can
locate the nodule)
MNEMONIC: 2-2-1-2-1-1 (lateral to medial)
 1st compartment (APL + EPB)
 Abductor policis longus
 Extensor policis brevis
 2nd compartment (ECRL + ECRB)
 Extensor carpi radialis longus
 Extensor carpi radialis brevis
 3rd compartment (EPL)
 Extensor policis longus
 4th compartment (EIP + EDC)
 Extensor indicis proprius
 Extensor digitorum communis
 5th compartment (EDM)
 Extensor digitorum minimis
 Management  6th compartment (ECU)
 NSAIDS  Extensor carpi ulnaris
 Splinting  Pathology
 Steroid injection  Stenosing synovitis of the FIRST DORSAL
 Open surgical release COMPARTMENT
 Open a small incision, just over the  AKA “ washer woman’s sprain”
metacarpal head, volarly, visualize A1,  It is most common in women between 30
then cut it vertically. and 50 years of age
 The patient can now actively flex and  The cause remains uncertain but may be
extend the finger freely and comfortably, related to friction between the tendons,
and the pulley heals again but has a larger their fibrous sheath, and the underlying
diameter. bony groove caused by movement of the
thumb and wrist

Transcriber/s: Paolo M. Ramirez


Formatting: Paolo M. Ramirez D L S H S I M e d i c i n e B a t c h 2 0 1 6 | 7 of 14
Editor/s: Chrismark Reyes & Jan Cynric Cacao
 Examination shows a sharp tenderness over thickened sheath is opened with a
the styloid process of the radius, and a longitudinal incision through the first
visible swelling and palpable thickening of compartment, freeing the involved
the fibrous sheath tendons
 Diagnosis
 Finkelstein’s test GANGLION CYST
 You clench you fingers into a fist and  Pathology
insinuate your thumb into it (in the palm).  Most common benign soft tissue tumor of
 You then try to exaggerate or stretch the the hand.
tendon by ULNARLY deviating the hand  Etiology
and by doing this you are stretching the  Synovial herniation
first dorsal compartment, thus producing  Mucoid degeneration
pain at that area.  Arise from the capsule of scapholunate
 When you have pain = (+) for DeQuervain  Arise de novo
 60-70% dorsally located
 From scapholunate capsule
 10-20% volarly located
 From scaphotrapezial joint

 B. Treatment
 Close rupture - 33% recurrence
 Management  Aspiration (uses Gauge 19 needle) - 15%
 Conservative recurrence
 Splinting  Excision - 10% recurrence
 NSAID
 Physiotherapy SUMMARY
 Steroid injection  Know the anatomy of the hand
 Surgical release [if the pain recurs or  Know the offending organism
persists]
 Institute appropriate antibiotics
 With the use of local anesthesia, a
 Monitor with laboratory exams/ clinical findings
short transverse incision is made over
 Immediate surgery when conservative management
the sheath on the lateral aspect of the
fails.
wrist; care must be taken to avoid the
sensory branches of the superficial
branch of the radial nerve. The

-END-

TRANSCRIPTION DETAILS
Xmark’s the Spot
Latest PPT, previous 8-
BASIS RECORDINGS + NOTES + DEVIATIONS CREDITS Recordings
transcriptions 10%
Mitchie’s tables
References:
Images taken from the internet
REMARKS The Netter Collection of Medical Illustration, 2nd edition, Musculoskeletal – Upper Limb
Editor’s reference: Apley’s System of Orthopedics and Fractures 9th ed.
#Brorthopedics
-BATCH 2016 Transcribers’ Guild Transcriptions. Version 2.0.0.0.0 Build 3207-

Transcriber/s: Paolo M. Ramirez


Formatting: Paolo M. Ramirez D L S H S I M e d i c i n e B a t c h 2 0 1 6 | 8 of 14
Editor/s: Chrismark Reyes & Jan Cynric Cacao
PAST-E 2015
Pathognomonic skin lesion The following is true for De Quervain’s tenosynovitis
seen in herpetic whitlow: Parona’s space EXCEPT: involves the compartment of:
a. Papule a. Pronator quadratus serve as a. Abductor pollicis longus and
b. Pustule its floor extensor pollicis brevis
c. Vesicle b. Bounded superiorly by the b. Extensor pollicis longus
d. Macule extensor tendon c. Extensor digitorum indicis
e. None of the above c. Receives drainage from and extensor digitorum
radial bursa communis
Management for herpetic d. Receives drainage from d. Abductor pollicis brevis and
whitlow includes the following ulnar bursa extensor pollicis longus
EXCEPT: e.Tenderness distal to e. None of the above
a. Antibiotic transverse carpal ligament
b. Acyclovir Most common location of
c. Incision and drainage It is also known as collar ganglion cyst at the dorsal
d. NSAIDs button abscess: aspect of the hand:
e. None of the above a. Deep palmar abscess a. Scaphotrapezial
b. Thenar abscess b. Scapholunate
Dental workers have a high c. Hypothenar abscess c. Triquestrium
risk for developing: d. Dorsal web space d. Trapezium
a. Felon e.Interdigital web space Horseshoe abscess may result
b. Paronychia abscess from infection of the thumb
c. Herpetic whitlow communicating with:
d. Flexor tenosynovitis Interdigital web space abscess a. index finger
swelling is more prominent b. middle finger
A 5 year old boy was seen ___________: c. ring finger
thumb sucking for the past a. Dorsally d. little finger
few days. He is most likely to b. Volarly
develop: c. Ulnarly An infection of the hand with
a. Felon d. Radially erythematous streaks and
b. Paronychia e. Interdigital web space lymphadenopathy would
c. Herpetic whitlow suspect ___________:
d. Flexor tenosynovitis Median nerve arises from the a. Staph aureus
e. None of the above brachial plexus as ___________: b. Pasteurella multocida
a.Union of lateral and medial c. Eikinella corrodens
Most important sign to illicit cord d. Beta hemolytic
in diagnosing flexor b.Union of lateral and Streptococcus
tenosynovitis is ________: posterior cord
a. Fusiform swelling c.Continuation of posterior cord Untreated human fight bite
b. Pain on passive extension d. Continuation of lateral cord may cause septic arthritis of
c. Tenderness along the e. None of the above __________:
tendon shaft a. DIP joint
d. Finger flexion The median nerve sensor b. TIP joint
distribution includes: c. MCP joint
Earliest sign to appear in a. Medial 2-­­1/2 of the hand d. wrist joint
Kanavel’s sign: b. Medial 3-­­1/2 of the hand
a. Fusiform swelling c. Lateral 2-­­1/2 of the hand Most common location of
b. Pain on passive extension d. Lateral 3-­­1/2 of the hand stenosing tenosynovitis among
c. Tenderness along the e. None of the above postmenopausal women:
tendon shaft a. Thumb
d. Finger flexion Reverse Phalen’s test is b. Index finger
performed by __________: c. Middle finger
A 30-­­year old male involved a. Flexing both hands at the d. Ring finger
in a fist fight sustained a wrist e. Little finger
0.5cm on his 3rd metacarpal b. Hyperextending both hands
right hand. Initial management at the wrist Tenderness of trigger finger
should include the following c. Ulnarly deviating the hands can be palpated over the
EXCEPT: d. Radially deviating the hands ____________:
a. Antibiotic a. Head of the proximal
b. Debridement phalanx
c. Extend wound incision b. Shaft of the proximal
d. Primary wound closure phalanx
e. None of the above c. Metacarpal head
d. Metacarpal shaft

Transcriber/s: Paolo M. Ramirez


Formatting: Paolo M. Ramirez D L S H S I M e d i c i n e B a t c h 2 0 1 6 | 9 of 14
Editor/s: Chrismark Reyes & Jan Cynric Cacao
PAST-E 2014
1. Felon is an infection of: 9. Human bite infection of the 17. The pathology of trigger finger
a. Proximal nail fold hand should be given antibiotics is the passage of inflamed tendon
b. Lateral nail fold with coverage from: on a tight ______ pulley
c. Nail bed a. Gram (+) a. A1
d. Distal pulp b. Gram (-) b. A2
c. Anaerobic c. A3
2. Most common etiologic agent d. All of the above d. A4
of felon: e. A5
a. Streptococcus 10. Human bite infection with
b. Staphylococcus transected tendon should be 18. The involved dorsal
c. E. coli repaired: compartment in de Quervain’s
d. Eikinella Corrodes (human bite) a. Immediately tenosynovitis is:
b. On the 2nd day a. 1st dorsal compartment
3. Herpetic whitlow should be c. On the 5th day b. 2nd dorsal compartment
treated with: d. 2 weeks after injury c. 3rd dorsal compartment
a. Antibiotics d. 4th dorsal compartment
b. Incision and drainage 11. Human bite infection most
c. Aspiration commonly cause septic arthritis 19. Finkelstein’s test is performed
d. Would left open of: by grasping the thumb and:
e. NSAIDS a. DIP joint a. Volarly flexed the wrist
b. PIP joint b. Dorsally extend the wrist
4. Cardinal signs of Kanavel c. MCP joint c. Ulnarly deviating the wrist
includes the following EXCEPT: d. Wrist joint d. Radially deviating the wrist
a. Fusiform swelling of the
involved finger 12. Sensory innervations of the 20. Trigger thumb is most
b. Pain on active extension median nerve includes: commonly seen in:
c. Tenderness along the tendon a. Dorsal web space a. Manual laborer
sheath b. Lateral 3 ½ of the finger b. Young adult
d. Flexed position of the affected c. Medial 1 ½ of the finger c. Pregnant women
finger d. Palmar aspect of the hand d. Post-menopausal women

5. Horseshoe abscess can be 13. Surgical management of carpal


formed as a result of tunnel includes surgically cutting
tenosynovitis of the: the:
a. Thumb (1st digit/thenar) a. Flexor retinaculum
b. 2nd digit b. Root of the carpal tunnel
c. 3rd digit c. Transverse carpal tunnel
d. 4th digit ligament
e. None of the above d. All of the above
e. None of the above
6. Pyogenic tenosynovitis of the
thumb drains specifically to the: 14. Phalen’s test is performed by:
a. Ulnar bursa a. Fully extending both wrist
b. Radial bursa (refer to #25) b. Fully flexing both wrist
c. Midpalmar space c. Lightly tapping the median
nerve
7. Parona’s abscess must have d. Compressing median nerve at
come from infected tenosynovitis the wrist
of the:
a. 2nd digit 15. In steroid injection of the
b. 3rd digit median nerve, one has to locate
c. 4th digit this tendon by opposing the
d. 5th digit thumb and the fifth digit:
e. None of the above a. Flexor pollicislongus
b. Flexor carpi radialis
8. In collar button abscess, pus c. Flexor carpi ulnaris
accumulates: d. Palmaris longus
a. Volarly e. None of the above
b. Dorsally
c. Laterally 16. Pain of snapping finger can be
d. Parona’s space palpated at:
e. None of the above a. DIP joint
b. PIP joint
c. MCP joint
d. Wrist joint
e. None of the above

Transcriber/s: Paolo M. Ramirez


Formatting: Paolo M. Ramirez D L S H S I M e d i c i n e B a t c h 2 0 1 6 | 10 of 14
Editor/s: Chrismark Reyes & Jan Cynric Cacao
INFECTIOUS
PYOGENIC FLEXON INTERDIGITAL (WEB)
FELON PARONYCHIA HERPETIC WHITLOW BITES
TENOSYNOVITIS SPACE INFECTION
Pathology -Deep infection of - Infection of the 
- Occurs 2-14 days after -Common infection of the hand
 - affected fingers here are 1. Human
the finger pulp soft tissue around exposure
to HSV -Bacterial infection of the tendon usually in the abducted - “fight bite” often
usually secondary to the fingernail - Common among dental sheath position. communicates with MP joint

punctured wound - Aka “Run around workers and medical -Occurs due to penetrating trauma - Likewise, there is usually a - Clenched fist tend to blow to
abscess or personnel exposed to oral severe swelling on the the victim’s teeth

infection” secretions dorsal side (dorsal
- Most common subcutaneous web space) of 2. Animal
bacterial infection the affected fingers or - Domestic cat and dog can
of the hand “Collar button abscess”. cause cellulitis
- Associated with -It does not cause migration
hangnail, nail or spread palmarily due to
biting, finger thickness of palmar fascia.
sucking and
manicures

Etiology S. aureus S. aureus HSV 1 & 2 S. aureus, Strep Human:


S.aureus
Eikenellacorrodens(gram
negative, 30% of the time,
sensitive to penicillin)
Animal:
Pasteurellamultocida
Symptoms 1) throbbing pain on Pus around the nail 1) lymphadenitis, malaise, Kanavel’s 4 Cardinal signs: - Palmar and dorsal

/ Presentation the distal aspect of plate fever 1) Fusiform swelling
 - Dorsal swelling more
the finger 2) Vesicular formation 2) Flexed position
 pronounced
2) Redness or 3) Pain on passive extension (Early - Collar button abcess

swelling
 onset)
 - Abducted position

3) Visible pus or 4) Tenderness along the tendon - Spread
yellowish sheath (Most important sign) >Dorsally to the
discoloration on subcutaneous web space
distal pulp >Palmar fascia prevents
4) ROM is still good spread beneath the skin

Complications -osteomyelitis -skin necrosis


-skin necrosis
 -tendon lysis
- higher chance of -contracted finger

Transcriber/s: Paolo M. Ramirez


Formatting: Paolo M. Ramirez D L S H S I M e d i c i n e B a t c h 2 0 1 6 | 11 of 14
Editor/s: Chrismark Reyes & Jan Cynric Cacao
amputation -amputation in severe cases (usually
in DM patients)

- Horse shoe abscess
(Parona’sAbcess) - a connection
between the affected ulnar and radial
bursa forms a horse shoe shape. This is
a complication of flexor tenosynovitis
wherein an abscess of the thumb may
cause an abscess on the fifth digit.

st st st
Treatment 1) Antibiotics (1 1) Antibiotics (1 Conservative; It is self- 1) Antibiotics (1 gen Cephalosporin) 1) Antibiotics (1st-gen 1) Never close bites primarily;
gen Cephalosporin) gen Cephalosporin) limiting to 7-10 days 2) Early I and D (for pus information) Cephalosporin) you actually have to open the
2) Early I and D 2) I and D (Incision 2) Double incision (dorsal wound

(Incision and and Drainage) Paracetamol may be given and volar) to evacuate 2) Puncture wounds may
Drainage) for the fever. (NSAIDS) abscess require extension
- Delayed
closure for large wounds; Don’t
suture!
3) Antibiotics for hand
st
infection (1 gen
Cephalosporin)
4) Proximal I and D

Contraindications I and D - only cause spread


to treatment of the virus.


- You can give Acyclovir to


relieve the symptoms, but
the disease will still run its
course. Antibiotics are
given only in the presence
of secondary bacterial
infection.

Transcriber/s: Paolo M. Ramirez


Formatting: Paolo M. Ramirez D L S H S I M e d i c i n e B a t c h 2 0 1 6 | 12 of 14
Editor/s: Chrismark Reyes & Jan Cynric Cacao
OFFICE ORTHOPEDICS
DE QUERVAIN’S
CARPAL TUNNEL SYNDROME TRIGGER FINGER/THUMB GANGLION CYST
TENOSYNOVITIS
Pathology - Compression of the median nerve at the Most common stenosing tenosynovitis of the hand - Stenosing synovitis of the first - Most common benign soft tissue
wrist by the transverse carpal ligament.
 - Usually it is after repeated motion (like in laborarers like dorsal compartment - AKA “ tumor of the hand. - Etiology
- Result of any decrease in compartment typewriters and secretaries)
- Most common in middle finger washer woman’s sprain” >Synovial herniation
>Mucoid
size or increase in compartment content. and ring finger
- Thumb—common in post-menopausal degeneration
>Arise from the capsule
-Increase in compartment content is noted women of scapholunate>Arise de novo
in previous distal radius fracture, and - Associated with carpal tunnel, inflammatory conditions (gout, - 60-70% dorsally located
From
lipoma on the wrist. Rheumatoid arthritis, amyloid deposition ) scapholunate capsule
- 10-20% volarly located
From
scaphotrapezial joint

Clinical - Typical history of paresthesia along - Locked finger in flexion; Snapping upon finger extension
Findings / median nerve distribution *When the finger is in the flexed position, it is not painful.
Symptoms/ -Weakness (Cannot flex thumb and However, upon extension, there is pain. The pain is common
Presentation second digit will flex to accommodate usually in the morning because you tend to extend your fingers
when forming a ring)
 upon waking up.
-Up to the 3rddigit only. It will NEVER - Palpable nodule (A1 is usually above the metacarpals; distal
th
INVOLVE the 5 digit. to that is where you can locate the nodule)
-More pronounce at night (Pain and
numbness)
- Aggravated by grasping activities

Diagnosis 1. Signs and Symptoms - Finkelstein’s test


2. Physical examination >You clench you fingers into a
a. Phalen’s test fist and insinuate your thumb

Transcriber/s: Paolo M. Ramirez


Formatting: Paolo M. Ramirez D L S H S I M e d i c i n e B a t c h 2 0 1 6 | 13 of 14
Editor/s: Chrismark Reyes & Jan Cynric Cacao
- Flex the wrist for 2-3 minutes, and then into it (in the palm).
>You then
let the patient extend the hand. Positive if try to exaggerate or stretch the
there is a tingling sensation n involved tendon by ULNARLY deviating
fingers. the hand and by doing this you
b Reverse Phalen’s are stretching the first dorsal
- Extend first then flex. This is more compartment, thus producing
sensitive. pain at that area.
c. Tinel’s sign >When you have pain = (+) for
- Palpate for the median nerve origin and DeQuervain
the patient will feel a tingling sensation.
d Direct nerve compression test
3. Confirmatory - EMG- NCV
4. How do you locate your carpal
tunnel?
-You try to oppose your thumb and little
finger. There’s a tendon (Palmaris longus)
that becomes stretched out. You go
medially from that and that’s where your
median nerve is located. Actually, it’sunder
the Palmaris longus.
Treatment / 1) NSAIDS
 1) NSAIDS
 1) Conservative
 1) Close rupture - 33% recurrence

Management 2) Splinting 2) Splinting
 2) Splinting
 2) Aspiration (uses Gauge 19 needle) -
- If pregnant, let the pregnancy finish first 3) Steroid injection
 3) NSAID
 15% recurrence
and 4) Open surgical release -open a small incision, just over the 4) Physiotherapy
 3) Excision - 10% recurrence
put a splint for the mean time. metacarpal head,volarly, visualize A1, then cut it vertically. 5) Steroid injection
3) Steroid injection (Cortisone) 5) Percutaneous release
- no need to open up. Use a gauge 19 6) Surgical release
- if the previous 2 fail needle and scrape the A1. Just be careful because of the nerves.
4) Surgical release
- if the previous 3
modalities fail; You will cut the transverse
carpal ligament. Be aware and be careful of
the palmar cutaneous branch that provides
sensation to the thumb.

Transcriber/s: Paolo M. Ramirez


Formatting: Paolo M. Ramirez D L S H S I M e d i c i n e B a t c h 2 0 1 6 | 14 of 14
Editor/s: Chrismark Reyes & Jan Cynric Cacao
BENIGN BONE TUMORS
Michael L. Marfori, MD, FPOA
December 2, 2014; 1:00 – 2:00 PM
Orthopedics

LECTURE OUTLINE
 Bone tumors  Dahlin modification of Lichenstein classification
 Benign bone tumors  Management

BONE TUMORS BASIC GUIDELINES IN X-RAY INTERPRETATION


 Most uncommon neoplasm  Pay attention to:
 Rare condition (0.8/100,000)  Anatomic site of the lesion
 2,900 (Sarcomas/year) vs. 169,500 (lung  Zone of transition between the lesion and
CA/year) and 193,700 (breast CA/year) the host bone [sclerotic margin]
 Low incidence/ lack of awareness results in lesions  Specific internal radiological characteristic
frequently missed. of the lesion
 Delays/mistakes in diagnosis lead to inadequate,  e.g. enchondroma is usually seen on
insufficient, and inappropriate treatment the hand and there would be
“popcorn” lesions
 “Fallen fragment sign” for unicameral
BENIGN BONE TUMORS bone cyst
 Bone loses its regulatory
mechanismunregulated growth leading to 7 X-RAY QUESTIONS
tumor formation  Flow in describing the radiographs
 Growth/destruction of bone (osteoclast  Allows educated guess of differential diagnosis
stimulation)
 Contained by intact capsule (compressed tissue) 1. WHERE IS THE LESION? PART OF THE BONE
 (-) spread to other sites INVOLVED?
 40% of primary bone tumors  Epiphysis (Chondroblastoma)
 Metaphysis (Fibrous Cortical Defect)
SIGNS AND SYMPTOMS  Diaphysis (Osteofibrous dysplasia)
 Pain and/or mass
 Pathologic fracture 2. HOW BIG IS THE LESION? AND HOW MANY
 Incidental findings BONES ARE AFFECTED?
 Larger lesion, more aggressive
DIAGNOSTIC STRATEGY  Unicameral bone cyst, calcaneus
 Patient’s age key information [since majority  Non-ossifying fibroma, distal tibia
st nd
are young patients; 1 to 2 decade]  Osteochondromatosis – multiple bone
 Pain (unremitting, progressive, pain at rest or at affectation
night
 Physical Examination is rarely diagnostic 3. WHAT IS THE TUMOR DOING TO THE BONE?
(hard/fixed soft tissue mass, vessel dilatation)  Interface between tumor and bone
 Geographic border
RADIOGRAPHS  Seen on benign lesions
 Most effective/ specific diagnostic tool  Distinct interface
 Conventional biplane/2 perpendicular planes  (+) sclerotic rim
(tangential view)  As seen in Unicameral bone cyst of the
 Non-invasive, low cost, “gross pathologic” image proximal femur
of the bone tumor
 Provide 80% correct diagnosis 4. WHAT IS THE BONE DOING TO THE TUMOR?
 Cortical Thickening
PLAIN RADIOLOGY  Slow growing tumor
 Well-defined transition between normal bone to  Host bone lay down thick wall around the
lesion (sclerotic margin) tumor quickly
 Uninterrupted periosteal reaction  Osteoblastoma of the distal humerus
 Bone expansion, endosteal scalloping, cortical
thinning
 Mineralization (osteoid or chondroid)
 No matrix formation or mineralization for
giant cell tumors

Transcriber/s: Arianne Cortes, Jan Cynric Cacao


Formatting: Jan Cynric Cacao D L S H S I M e d i c i n e B a t c h 2 0 1 6 | 1 of 11
Editor/s: Paolo Ramirez, Chrismark Reyes
6. IS THERE A SOFT TISSUE MASS?
 Aggressive tumors break out of the cortex,
produce soft tissue mass
 Giant cell tumor of the distal radius

 Endosteal Expansion
 Less aggressive tumors
 Host lays down bone as tumor erodes
endosteum
7. WHAT IS THE MATRIX?
 Bone expansion
 Osteoid matrix:
 Aneurysmal bone cyst of the distal
 Bone forming
fibula
 Cloud-like/fluffy
 e.g. Osteoblastoma, proximal tibia

 Neocorticalization  Chondroid matrix


 More aggressive tumor  Punctate, flocculent, popcorn, arc-ring
 Lysis of cortex pattern
 (+) periosteum – compensates by producing  e.g.enchondroma of metacarpal and
minimum amount of bone phalanx
 Still (+) bone formation
 Giant cell tumor of the distal ulna

 Ground glass
 Seen in fibrous dysplasia

5. IS THERE A CORTICAL BREAK?


 Very aggressive lesions break out of the cortex
 Patient may have a soft tissue mass around the
tumor
 Giant cell tumor of the proximal tibia

Transcriber/s: Arianne Cortes, Jan Cynric Cacao


Formatting: Jan Cynric Cacao D L S H S I M e d i c i n e B a t c h 2 0 1 6 | 2 of 11
Editor/s: Paolo Ramirez, Chrismark Reyes
 (-) matrix NOTE:
 e.g. giant cell tumor of the distal femur  Question 1: Location of the tumor
 Question 2-6 refers to tumor aggressiveness
 Question 7: ide of the histology (bone or cartilage
forming)

STAGING SYSTEM
 Enneking’s System for Staging Benign Bone Tumors

Stage Definition Biological Behavior Bone Tumor


Fibrous cortical defect,
1 Latent Static or heal spontaneously
osteochondroma
Unicameral bone cyst,
2 Active Progressive growth, limited by natural barriers
chondroblastoma
Progressive growth, invasive, not limited by natural
3 Aggressive Giant cell tumor, aneurysmal bone cyst
barriers

LATENT (STAGE 1) BONE FORMING


 “Inactive”
 Asymptomatic, incidental finding OSTEOID OSTEOMA
 Grows slowlyreach quiescent state  Oval/round mass – “nidus”
 Well marginated (within compartment)  Limited growth potential (5-15mm or <1cm)
 (-) deformity/ (-/+) pathologic fracture  10-12% of benign tumors
 5-24 y/o 76%, M>F (3:1)
ACTIVE (STAGE 2)  Affects the long bones at the diaphyseal region
 Symptomatic, bone destruction, active growth
 (+) pathologic fracture AREAS OF INVOLVMENT
 (+/-) soft tissue extension (limited)  Predilection for long bones(femur and tibia) as
 Moderate Tenderness well as the vertebrae (usually posterior elements)
 May occur in any bone and may be cortical,
AGGRESSIVE (STAGE 3) medullary, subperiosteal, or periarticular
 Bone destruction, soft tissue extension, (intracapsular)
pathologic fracture
 Difficult tumor removal/reconstruction CLINICAL MANIFESTATIONS
 Soft tissue extension (permeate natural barriers)  Pain/tenderness (weeks to years)
 Pain/enlarging mass, rapid growth  Pain not activity related, severe night pain
 25% recurrence rate  Relieved by aspirin and NSAIDS [due to presence
of nerve fibers/vessels]
 Nidus – nerve fibers/vessels, (+) prostaglandins
DAHLIN MODIFICATION OF  Tumor – round/ oval red soft tissue mass
LICHENSTEIN CLASSIFICATION

Cell Type Benign Malignant


Osteoid osteoma
Bone Osteosarcoma
Osteblastoma
Enchondroma
Chondroblastoma Chondrosarcoma –
Cartilage Chondromycoid mesenchymal, clear
fibroma cell
Osteochondroma
Fibrous cortical
Fibrosarcoma
defect
Fibrous Malignat fibrous
Non-ossifying
histiocytoma
fibroma
Ewing Sarcoma
Unknown Giant Cell Tumor
Adamantinoma

Transcriber/s: Arianne Cortes, Jan Cynric Cacao


Formatting: Jan Cynric Cacao D L S H S I M e d i c i n e B a t c h 2 0 1 6 | 3 of 11
Editor/s: Paolo Ramirez, Chrismark Reyes
Periosteal chondroma

ENCHONDROMA (CENTRAL LESIONS)


 Hyaline cartilage proliferation
 Disorder in enchondral ossification (cartilage
OSTEOBLASTOMA island formation)
 “Giant osteoid osteoma”  15.6% of benign tumor
 Nidus >2cm (ave. size 3.18cm)  4.7% of bone tumors
 3.5% benign tumors  Female predominance
 Rare (1% of bone tumors)
 10-25 years old (80%)
 Male predilection (78%)

AREAS OF INVOLVMENT
 Spine (40%), femur/tibia
 Cortex (65%), metaphysis (42%), diaphyseal (36%)

CLINICAL MANIFESTATIONS AREAS OF INVOLVEMENT


 Neurologic symptoms, scoliosis  Hand/foot (41%) [this is the most common primary
 Progressive pain (87%), abnormal gait bone tumor of the hands]
 Muscle atrophy (2 years duration)  Long bones (humerus, femur, tibia)
 Not relieved by aspirin intake  Metaphysis, meta-diaphysis

SIGNS AND SYMPTOMS


 Asymptomatic/incidental findings
 Pain (+) pathologic fracture of the hand

RADIOLOGIC FEATURES
 Cartilaginous matrix mineralization without
cortical disruption or endosteal erosion
 "Popcorn" calcifications
 Well defined, sharply marginated, and lobulated
*Gross picture
border
 A change to lucent regions indicate possible
malignant degeneration
 Extension into soft tissue heralds malignancy

OLLIER’S DISEASE
 Multiple enchondroma, unilateral involvement
 Shortening/bowing deformity

CARTILAGE FORMING

CHONDROMA
 Tumor from mature hyaline cartilage
 Periosteal chondroma (eccentric lesions) –
involves the surface of the bone, away from
the medulary canal
 Enchondroma (central lesions)

Transcriber/s: Arianne Cortes, Jan Cynric Cacao


Formatting: Jan Cynric Cacao D L S H S I M e d i c i n e B a t c h 2 0 1 6 | 4 of 11
Editor/s: Paolo Ramirez, Chrismark Reyes
MAFFUCCI SYNDROME
 Multiply enchondroma with soft tissue angiomas

Pedunculated osteochondroma

NOTE:
OSTEOCHONDROMA  Usually, this lesion does not need any treatment and
 “Osteocaritilaginous exostosis” only needs observation.
 Bony outgrowth (physis) + cartilage cap  If there is tenderness and progressive pain, then we
 Developmental defect (aberrant growth plate) can consider surgery, otherwise, the lesion usually
 Osteochondromatosis or hereditary multiple spontaneously regress after skeletal maturity.
exostosis in 15%
 Autosomal dominant disorder
CHONDROMYXOID FIBROMA
 Developmental abnormalities:
 Rare type of cartilage-forming tumor
 Angulation of radius
 Shortening of ulna  (+) chondroid, fibrous and myxoid tissue
(Jaffee&Lichenstein, 1948)
 Lack of trabeculation
 Remnants of cartilage (epiphyseal growth plate)
 Spontaneous regression
 1.6% of benign bone tumors; 0.5% of bone
 Asymptomatic, non-tender fixed/hard swelling or
tumors
pain with impingement of structures (bursitis),
 2nd-3rd decade; male predilection
weight bearing; limitation of joint motion
AREAS OF INVOLVEMENT
EPIDEMIOLOGY
 Pelvis (most common), long bone (2/3), knee (1/3)
 33.4% of benign bone tumors – most common
 Metaphyseal; eccentric (fusiform expansion) well-
primary benign bone tumor
circumscribed, rare calcification
 10.1% of bone tumors
 < 20 years old (60%); 2nd decade
 65% male predilection

AREAS OF INVOLVEMENT
 Distal femur/proximal tibia (knee in 36%)
 Solitary (metaphysis)
 Uncommon in small bones
 Types:
 Sessile (flat)
 Pedunculated (long/ cauliflower)

SIGNS AND SYMPTOMS


 Pain/tenderness, soft tissue mass, pathologic
fracture
 Curette fragments::
 Firm, fibrous and semi-translucent (myxoid)
 Rarely resemble hyaline cartilage
Sessile osteochondroma
CHONDROBLASTOMA
 Epiphysis
 4.79% of benign tumors; 1-1.45% of bone tumors
 <25 years old (80%); 2nd decade (60%)
 M > F (2:1)

AREAS OF INVOLVEMENT
 Knee (34%) – proximal tibia > distal femur
 Proximal humerus [called Codman’s Tumor]
 Minimal mineralization (1/3)
 Rarely cortical involvement

Transcriber/s: Arianne Cortes, Jan Cynric Cacao


Formatting: Jan Cynric Cacao D L S H S I M e d i c i n e B a t c h 2 0 1 6 | 5 of 11
Editor/s: Paolo Ramirez, Chrismark Reyes
AREAS OF INVOLVEMENT
 Long bone: femur/tibia (metaphysis →diaphysis)
 Enlarging mass (1-7 cm); average size: 5 cm

SIGNS AND SYMPTOMS


 Mild-moderate pain (months-years) – average of
20 months
 Join effusion/stiffness, minimal atrophy,
tenderness
 Limitation of Motion (“sports injury”)
 Often mistaken as a sports injury and is
usually referred to a sport medicine
specialist OSTEOFIBROUS DYSPLASIA
 Ossifying fibroma, Kempson-Campanacci
NON-NEOPLASTIC BENIGN BONE LESION  Kempson in1966 – aggressive
 Developmental  Campanacci – spontaneous regression after
 Non-ossifying fibroma/Fibrous cortical skeletal maturity
defect
 Fibrous dysplasia EPIDEMIOLOGY
 Osteofibrous dysplasia  Fibro-osseous cortical tumor
 Reactive  1st-2nd decade
 Aneurysmal bone cyst
 Unicameral bone cyst AREAS OF INVOLVEMENT
 Anterior cortex of tibia (bowing)
 Multiple radiolucencies and sclerosis
NOTE:
 Soap-like or bubble-like appearance
 Developmental abnormalities: developmental
proliferation of fibrous tissue, resolve as patient
matures
 Reactive process: not true neoplasm, cystic lesions
(blood or serous fluid) cause substantial bone
destruction

FIBROUS CORTICAL DEFECT AND NON-OSSIFYING


FIBROMA
 FCD and NOF or metaphyseal fibrous defect
 Fibrous tissue in medullary cavity

PATHOPHYSIOLOGY
 Faulty ossification/remodeling during skeletal
growth SIGNS AND SYMPTOMS
 Spectrum starting as FCD (abutting the physis)  Asymptomatic, incidental finding; pain if with
then becoming NOF (away from physis or fracture
diaphysis)  Self-limiting, regress (adolescence)
 Observation only and no need for surgical
EPIDEMIOLOGY intervention
 30-40% in children (1st-2nd decade); male
predominance FIBROUS DYSPLASIA
 Asymptomatic/incidental finding, spontaneously  Aberration in bone development
resolve  Fibro-osseous proliferation
 Bowing (weakened bone)
NOTE:  671 cases in Mayo clinic
 These lesions are usually under reported.
 Patients are seen because they have other problems EPIDEMIOLOGY
like sprains, muscle injuries  2nd to 3rd decade: females predominated
 Monostotic vs. polystotic (Albright’s syndrome)

Transcriber/s: Arianne Cortes, Jan Cynric Cacao


Formatting: Jan Cynric Cacao D L S H S I M e d i c i n e B a t c h 2 0 1 6 | 6 of 11
Editor/s: Paolo Ramirez, Chrismark Reyes
SIGNS AND SYMPTOMS SIGNS AND SYMPTOMS
 “Shepherd’s crook deformity” (femur)/tibia  Asymptomatic or pain if pathologic fracture →
 Asymptomatic/incidental findings; limping, heals at maturity
pathologic fracture, leg-length discrepancy  Metaphysis, medullary, radiolucent/fine
 70% bone pain trabeculation, thinned out cortex, “fallen
fragment sign/fallen leaf sign”
 Cortex fell down due to gravity
 Pathognomonic of unicameral bone cyst

Shepherd’s crook deformity

ANEURYSMAL BONE CYST


 ABC
 Circulatory disturbance → increase venous UBC: “fallen fragment sign”
pressure →HEMORRHAGE→ bone destruction
 Pseudo tumors GIANT CELL TUMOR
 < 20 y/o; female  GCT or Osteoclastoma
 “Aggressive” neoplasm
AREAS OF INVOLVEMENT  Pseudosarcoma
 Proximal humerus, proximal tibia or distal femur  Uniform multinucleated giant cells (osteoclasts)
(knee) and ilium from mononuclear cell →lytic
 Metaphysis/meta-diaphysis, medullary, lytic,  Similarly seen on patients with Pott’s
well defined margin disease

DIAGNOSIS EPIDEMIOLOGY
 CT/MRI: “fluid-fluid level”  Unknown cause/unknown cell of origin
 Separation of serum and blood in the  21.87% of benign bone tumor; 6.60% of bone
internal septation tumors
 20-24 years old; 85% peak incidence in 3rd
decade
 Females predominated

AREAS OF INVOLVEMENT
 Knee (most common)
 Proximal humerus, distal radius
 Epiphyseal/epi-metaphyseal

CT scan: fluid-fluid level SIGNS AND SYMPTOMS


 Eccentric, radiolucent, well circumscribed
SIGNS AND SYMPTOMS  “Fleshy”/meaty material
 Mild pain/swelling (weeks-years), pathologic  Dark brown/chocolate
fracture  Pain variable intensity (2-3 months)
 Enlarging mass/swelling (3/4 of cases)
UNICAMERAL BONE CYST  Weakness/limitation of motion, pathologic
 UBC or “simple bone cyst” fracture
 Septated, fluid-filled cyst
 Unknown cause: interstitial fluid accumulation →
increased pressure →bone necrosis
 1st 2 decades (85%)

AREAS OF INVOLVEMENT
 Proximal humerus and proximal femur
(metaphysis), proximal tibia
 Abutting physis

Transcriber/s: Arianne Cortes, Jan Cynric Cacao


Formatting: Jan Cynric Cacao D L S H S I M e d i c i n e B a t c h 2 0 1 6 | 7 of 11
Editor/s: Paolo Ramirez, Chrismark Reyes
MANAGEMENT SURGICAL MARGINS
 Observation (latent/inactive lesions) or aspiration  Intralesional excision
and injection of methylprednisolone acetate  Excision within tumor
(UBC)  25-50% local recurrence (stage 2/3)
 Serial radiographs every 3 months  Marginal excision
 Limited surgical intervention for UBC  Expendable bone: proximal fibula, distal ulna
 Limited curettage or bone grafting and scapular body
 Surgical intervention (active/aggressive)  Reactive zone
 Reserved for stage 2 and 3 tumors  5-10% local recurrence (stage 2 and 3)
 Biopsy (confirm diagnosis)
 Tumor extirpation EXTENDED CURETTAGE
 Curettage + burring: 10% recurrence
BIOPSY  Curettage + burring + phenol: 6-7%recurrence
 Removal/examination of sample tissue from living  2-3 mm additional margin
body for diagnostic purpose
 Done to rule out the possibility of a malignancy LIQUID NITROGEN
 7-20 mm margin/3-7 weeks
GENERAL TYPES  Freeze-thaw cycle (2-3 cycles)
 Closes biopsy (needles)
 Open biopsy (surgical) POLYMETHYL METHACRYLATE (PMMA)
 Add 1.3-2.8 mm from bone cement interface
CLOSES BIOPSY (NEEDLES)  Thermal effect enhances coverage and reduces
 (-) incision, stab incision the number of microscopic tumors in the area
 Specimen with needle puncture
 Types: NOTE:
 Fine needle biopsy  100% local control – Liquid nitrogen and PMMA
 Gauge 22 needle
 Reliable in soft tissue tumor diagnosis
BENIGN BONE TUMORS STAGE, TYPE AND
 Cell type are homogenous
TREATMENT OPTIONS
 (-) matrix formation
 No use for bone tumors; usually
Stage Bone tumor Treatment
indicated for soft tissue tumors
 Core needle biopsy Non-ossifying
 Gauge 14 needle fibroma
Stage 1 Observation or
 (+) core tissue (20 mm) Osteochondroma
(latent) excision
 90% accurate in differentiating Fibrous dysplasia
malignant vs. benign bone tumors Enchondroma
Osteoid osteoma
OPEN BIOPSY (SURGICAL) Chondroblastoma
Intralesional
 (+) incision Stage 2 Chondromyxoid
excision +
(active) fibroma
 Types: adjuvant
Aneurysmal bone
 Intralesional biopsy
cyst
 (+) incision
 Representative tissue taken (1x1x1 cm) Intralesional
Giant cell tumor
 Special instruments are used to excision +
Stage 3 ( Osteoblastoma
retrieve the specimen adjuvant or
aggressive) Aneurysmal bone
 Sufficient materials for cellular marginal
cyst
morphology evaluation/tissue resection
architectural study
 Excisional biopsy METHYLPREDNISOLONE INJECTION FOR UBC
 Lesion is excised en-bloc  Incision → needle insertion → irrigation with
 Both diagnostic and definitive PNSS → methylprednisolone injection
surgery (therapeutic)
TECHNIQUE
 Biopsy → excision/burr →cryoablation→ bone
grafting or cementing → (+/-) implant fixation
 Cryogablation (cryogun) and harvesting of iliac
bone graft

BONE DEFECT FILLER (BONE GRAFT)


 Role
 Structural support (articular surfaces)
 Void filler/fill defects
 Types
 Autograft
 Allograft
 Bone substitutes – synthetic bone

Transcriber/s: Arianne Cortes, Jan Cynric Cacao


Formatting: Jan Cynric Cacao D L S H S I M e d i c i n e B a t c h 2 0 1 6 | 8 of 11
Editor/s: Paolo Ramirez, Chrismark Reyes
-END-

TRANSCRIPTION DETAILS
BASIS Latest PPT RECORDINGS + NOTES + DEVIATIONS 8-10% CREDITS -
References:
REMARKS Images from the internet
http://www.orthopaedicsone.com
-BATCH 2016 Transcribers’ Guild Transcriptions. Version 2.0.0.0.0 Build 3208-

PAST-E (2015)
True for bone tumors: True for Ollier’s disease:
a. These are common neoplasms that can be a. Multiple enchondromas involving 2 or more
managed optimally, even by ordinary limbs
physicians. b. Multiple enchondromas with associated soft
b. The yearly incidences of bone sarcomas are tissue angiomas
much less compared to lung and breast c. Multiple enchondromas in the unilateral limb
carcinomas only
c. Any delays or mistakes in the diagnosis will d. Solitary enchondroma involving the small bones
seldom affect the management and of these of the hands or the feet
patients
d. Bone tumors are always detected by physical True for Maffucci syndrome:
examination and plain radiographs even by a. Multiple enchondromas in the unilateral limb
inexperienced doctors b. Multiple enchondromas with associated soft
tissue angiomas
True for benign bone tumors: c. Solitary enchondroma involving the small bones
a. 40% of all primary bone tumors of the hands or the feet
b. 60% of all primary bone tumors d. Solitary enchondroma that are discovered
c. They tend to spread early to other sites of the incidentally on radiographic studies
body
d. They produce collagenase that reabsorbs the The presence of cartilaginous cap and stalk in the knee
host bone will lead to the diagnosis of:
a. Osteoid osteoma
The radiographic picture of osteoid osteoma (OO): b. Fibrous dysplasia
a. Popcorn-like matrix in the medullary canal c. Osteochondroma
b. The lesion is >2 cm in diameter d. Osteoblastoma
c. The lesion is 5-15 cm in diameter
d. The lesion is mostly located in the spine Pathologic fractures are seen in enchondroma of:
a. Humerus
True for osteoblastoma: b. Femur
a. The lesion is 1cm in diameter and is located in c. Phalanges
the diaphysis of long bones d. Tibia
b. The occasional pain is always relieved by the
intake of aspirin True regarding the nidus in osteoid osteoma:
c. If the lesion involves the spine, this will seldom a. >2cm in diameter
lead to the development of scoliosis or b. Releases enzymes that erode the surrounding
neurologic symptoms bone causing constant pain
d. Also known as giant osteoid osteoma c. Releases prostaglandin that causes pain
d. Nerve endings and blood vessels are absent in
True for chondroma: the nidus
a. The lesion in periosteal chondromas are located
in the medullary canal A sixteen year old patient brought to the clinic with a
b. The pathophysiology is the excessive hyaline palpable mass on the left knee with a radiographic
cartilage production in the affected bone finding of a pedunculated mass on the proximal tibia.
c. Enchondromas are often symptomatic and Your impression is?
leads to pathologic fractures particularly if it a. Enchondroma
involves larger long bones b. Chondromyxoid fibroma
d. The lesion in enchondromas are eccentrically c. Osteochondroma
located in the long bones d. Osteoid osteoma

Transcriber/s: Arianne Cortes, Jan Cynric Cacao


Formatting: Jan Cynric Cacao D L S H S I M e d i c i n e B a t c h 2 0 1 6 | 9 of 11
Editor/s: Paolo Ramirez, Chrismark Reyes
True for chondromyxoid fibroma: Accumulation of interstitial fluid in the metaphyseal
a. Radiographically characterized as sessile of region of the long bone leading to bone necrosis and
pedunculated outgrowth with a cartilage cap in thinning of the cortex is the proposed pathophysiology
the metaphyseal region of this condition:
b. Radiographically characterized as eccentric, a. Aneurysmal bone cyst
fusiform expansion and few chondroid matrix b. Unicameral bone cyst
in the metaphyseal region c. Non‐ossifying fibroma
c. Radiographically characterized as endosteal d. Ossifying fibroma
expansion of the medullary canal with popcorn-
like matrix in the diaphyseal region A developmental proliferation of fibrous tissue that
d. Radiographically characterized as a lesion with starts in the medullary canal in the metaphyseal region
few chondroid matrix in the epiphyseal region of long bone and resolves with skeletal maturity:
a. Aneurysmal bone cyst
True for osteochondromatosis: b. Unicameral bone cyst
a. Autosomal recessive disorder c. Non-ossifying fibroma
b. The lesion is solitary d. Ossifying fibroma
c. Presence of multiple lesions
d. No developmental abnormalities A non‐neoplastic benign bone lesion characterized by
proliferation of fibro-osseous tissue that is exclusively
The symptom of pain, swelling, and stiffness that is seen in the anterior cortex of the tibia:
often mistaken for a sports injury is seen in this a. Fibrous dysplasia
condition: b. Non-ossifying fibroma
a. Osteochondroma c. Kempson-Campanacci
b. Enchondroma d. Fibrous cortical defect
c. Chondromyxoid fibroma
d. Chondroblastoma Fibro‐osseous proliferation in the medullary canal that
cause progressive bowing of the long bone and seen as
A non-neoplastic benign bone lesion characterized by Shepherd’s crook deformity on radiographs:
self-limiting proliferation of fibrous tissues in the long a. Fibrous dysplasia
bones: b. Non-ossifying fibroma
a. Aneurysmal bone cyst c. Kempson-Campanacci
b. Unicameral bone cyst d. Fibrous cortical defect
c. Non‐ossifying fibroma
d. Chondroblastoma Presence of “fluid-fluid” level accumulation upon doing
an MRI study of a lesion in the metaphyseal region of the
long bone is seen in what bone tumor?
a. Aneurysmal bone cyst
b. Unicameral bone cyst
c. Non--‐ossifying fibroma
d. Ossifying fibroma

PAST-E (2014)

An example of a benign osteoid forming bone tumor: An example of a reactive non-neoplastic benign bone
a. Enchondroma lesion:
b. Giant cell tumor a. Aneurysmal bone cyst
c. Osteoblastoma b. Giant cell tumor
d. Chondromyxoid fibroma c. non ossifying fibroma
d. fibrous dysplasia
An example of benign cartilage forming bone tumor:
a. Enchondroma A characteristic of a benign latent bone tumor:
b. Osteoid Osteoma a. An enlarging mass that lead to significant bone
c. Osteoblastoma destruction
d. Fibrous dysplasia b. Pathologic fracture is initial presentation
c. Lesion that remain static and heal
An example of a developmental non-neoplastic benign spontaneously
bone lesion: d. Lesion that is not contained by natural barriers
a. Aneurysmal bone cyst
b. Giant cell tumor A characteristic of a benign active bone tumor:
c. Osteoblastoma a. Progressive tumor growth that is limited by
d. Fibrous dysplasia natural barriers
b. Lesion that remain static and heal
spontaneously
c. Lesion that has high recurrence rate after
surgical resection
d. Patient is asymptomatic and the lesion is an
incidental finding

Transcriber/s: Arianne Cortes, Jan Cynric Cacao


Formatting: Jan Cynric Cacao D L S H S I M e d i c i n e B a t c h 2 0 1 6 | 10 of 11
Editor/s: Paolo Ramirez, Chrismark Reyes
A characteristic of a benign aggressive bone tumor: A benign aggressive bone tumor that is composed of a
a. Progressive tumor growth that is limited by multinucleated giant cell that activate osteoclast activity
natural barriers resulting to bone loss
b. Lesion that remain static and heal a. Osteochondroma
spontaneously b. Osteoid osteoma
c. Lesion that has high recurrence rate after c. Giant cell tumor
surgical resection d. Unicameral bone cyst
d. Patient is asymptomatic and the lesion is an
incidental finding A reactive non-neoplastic bone lesion that is caused by
circulatory disturbance leading to local
An example of a benign latent bone tumor: hemorrhages and eventually bone erosion
a. Osteoblastoma a. Aneurysmal bone cyst
b. Aneurysmal bone cyst b. Giant cell tumor
c. Osteochondroma c. Chondroblastoma
d. Giant cell tumor d. Osteoid osteoma

An example of a benign aggressive bone tumor: Shepherd’s crook deformity


a. Non ossifying fibroma a. Non-ossifying fibroma
b. Fibrous dysplasia b. ossifying fibroma
c. Enchondroma c. fibrous cortical defect
d. Giant cell tumor d. fibrous dysplasia

An example of benign active bone tumor: A developmental non-neoplastic bone lesion that always
a. Chondromyxoid fibroma involve the anterior tibial cortex leading to progressive
b. Fibrous dysplasia bowing
c. Enchondroma a. Non ossifying fibroma
d. Osteochondroma b. ossifying fibroma
c. fibrous cortical defect
It has a nidus that contain nerve endings, vessels and d. fibrous dysplasia
prostaglandins
a. Osteochondroma Osteoid matrix on radiographic examination is seen in
b. Osteoid osteoma this benign bone tumor
c. Giant cell tumor a. Non ossifying fibroma
d. Unicameral bone cyst b. osteoblastoma
c. Giant cell tumor
A benign osteoid forming bone tumor that often involve d. Aneurysmal bone cysts
the spine leading to development of scoliosis
a. Aneurysmal bone cyst
b. Chondromyxoid fibroma
c. Osteoblastoma
d. Enchondroma

A benign cartilage forming bone tumor that usually


involve the hand or the foot
a. Giant cell tumor
b. Enchondroma
c. Chondromyxoid fibroma
d. Chondroblastoma

A benign surface bone lesion that is characterized by a


bony outgrowth and a cartilaginous cap
a. Osteochondroma
b. Osteoid osteoma
c. Giant cell tumor
d. Unicameral bone cyst

A rare benign cartilage forming bone tumor that is


located in the epiphysis of a growing bone
a. Aneurysmal bone cyst
b. Giant cell tumor
c. Chondroblastoma
d. Osteoid osteoma

Transcriber/s: Arianne Cortes, Jan Cynric Cacao


Formatting: Jan Cynric Cacao D L S H S I M e d i c i n e B a t c h 2 0 1 6 | 11 of 11
Editor/s: Paolo Ramirez, Chrismark Reyes
MALIGNANT BONE TUMORS
Michael L. Marfori, MD, FPOA
January 5 & 6, 2015; 1:00 – 2:30 PM
Orthopedics

LECTURE OUTLINE
 Introduction
 Bone forming tumors  Other skeletal sarcomas
 Osteosarcoma  Malignant Fibrous Histiocytoma
 Cartilage forming tumors  Fibrosarcoma
 Chondrosarcoma  Ewing’s Sarcoma
 Mesenchymal chondrosarcoma  Malignant Giant Cell Tumor
 Clear cell chondrosarcoma  Management of Bone Sarcomas
 Chondrosarcoma secondary to  Summary
osteosarcoma

INTRODUCTION QUESTION #3 WHAT IS THE TUMOR DOING TO THE


 Sarkomas/sarcoma (means “fleshy growth”) BONE?
 Derived from the mesoderm  Permeative Border
 Spread/metastasize to distant sites  Indistinct interface (no distinction between the
 Lungs host bone and the tumor)
 Other bones  Hazy
 Lymph nodes  Tumor tends to permeate through host bone
 Locally aggressive, infiltrates tissues [due to the actions
of collagenase]
 COLLAGENASE is the enzyme produced by the
tumor that resorbs bone
 60% of primary bone tumors
 Remember that Benign bone tumors comprise
the 40%

PRESENTING SIGNS AND SYMPTOMS


 Pain [progressive type and is relieved by intake of
medication]
 Mass [grows rapidly]
 Pathologic fracture
 Incidental findings are rarely seen

7 X-RAY QUESTIONS
 Allows educated guess of differential diagnosis REMEMBER!
Benign – GEOGRAPHIC
 Basically the same as Benign Bone Tumors
Malignant – PERMEATIVE
 Questions:
 Question 1: LOCATION OF THE TUMOR
 Question 2-6: TUMOR AGGRESSIVENESS  Moth-eaten
 Question 7: IDEA OF HISTOLOGY  Indistinct interface
 Hazy and indistinct
QUESTION #1 IN WHICH BONE IS THE LESION  “eaten away”/scalloping
LOCATED? WHICH PART OF THE BONE IS
INVOLVED?
 Osteosarcoma
 Usually present in the Metaphysis (most of
these diseases are seen in the knee area)
 Chondrosarcoma
 Usually present in the Scapular Body and Pelvis
 These are more common in the flat bones and
the proximal parts of long bones

QUESTION #2 HOW LARGE IS THE LESION?


 The larger the lesion, the more aggressive it is
(Chondrosarcoma  Proximal Femur)

Transcriber/s: Arianne Cortes & Nimrod Gabornes


Formatting: Paolo M. Ramirez D L S H S I M e d i c i n e B a t c h 2 0 1 6 | 1of 11
Editor/s: Chrismark Reyes & Jan Cynric Cacao
QUESTION #4 WHAT IS THE BONE DOING TO THE
BIGGEST NOTES:
TUMOR?
BENIGN MALIGNANT
 These periosteal reactions are disorganized,
Cortical thickening Onion-peel appearance
interrupted
Endosteal Expansion Sunburst appearance
 Host bone unable to contain the tumor
Neocorticalization Codman’s Triangle
 These occur due to rapidly enlarging masses that the
body cannot cope up with.
 Previously these 3 were considered to be QUESTION #5 IS THERE A CORTICAL BREAK?
pathognomonic  Lesion cannot be contained
 Breakout of the cortex
ONION-PEEL
SUNBURST APPEARANCE QUESTION #6 IS THERE A SOFT TISSUE MASS?
APPEARANCE
 VERTICAL  HORIZONTAL laying down  Tumor expands find its way out of the cortex
lamellations of bony spicules
QUESTION #7 WHAT IS THE MATRIX?
 Osteoid Matrix
 Cloud-like, fluffy wisps of cotton,
 Both intramedullary soft tissue mass

CODMAN’S TRIANGLE
 Cuff of periosteal new bone forming at the boundary of
mass  Chondroid matrix
 Rapidly ELEVATING PERIOSTEUM (due to enlarging  Punctuation marks, flocculent, “arcs-rings”
mass causing a gradual lifting of the periosteum) pattern, stippled calcification,
 POPCORN-LIKE

Transcriber/s: Arianne Cortes & Nimrod Gabornes


Formatting: Paolo M. Ramirez D L S H S I M e d i c i n e B a t c h 2 0 1 6 | 2of 11
Editor/s: Chrismark Reyes & Jan Cynric Cacao
 No Matrix
 No mineralization MALIGNANT BONE TUMOR CLASSIFICATION
 Hollow lytic lesions involving also the
metaphyseal area of the long bones Bone-forming Osteosarcoma
Cartilage-forming Chondrosarcoma
STAGING SYSTEM Other Skeletal Malignant fibrous
ENNEKING SYSTEM FOR STAGING OF BONE Sarcomas histiocytoma,
SARCOMAS fibrosarcoma, Ewing’s
STAGE GRADE SITE METASTASIS Sarcoma, Malignant
IA G1 T1 M0 giant cell tumor
IB G1 T2 M0
IIA G2 T1 M0
IIB G2 T2 M0
III G1-G2 T1-T2 M1
 Grade (G):
 G1=Low (Less cellularity, limited mitotic figures,
and no pleomorphism)
 G2=High (Multiple cellularity, undifferentiated,
increased pleomorphism)
 Site (T):
 T1= INTRAcompartmental(inside the bone)
 T2=EXTRAcompartmental (outside the bone)
 Metastases (M):
“20 items only? Think 20 thousand bone
 M0=No regional or distant metastasis
 M1=Regional or distant metastasis present cancer patients you’ll be able to save” 

BONE FORMING TUMORS


OSTEOSARCOMA OSTEOGENIC SARCOMA IN PAGET’S DISEASE
 AKA Osteogenic Sarcoma  Aka OSTEITIS DEFORMANS
 Neoplastic cells (malignant osteoblasts)  Progressive osteoclastic activity; resorption and
synthesize/deposit bone matrix formation leading to sclerosis
 MOST COMMON MALIGNANT BONE TUMOR  Second peak (>60 y/o)
 Bimodal peak (<20 y/o and >60 y/o)  1/500 to 1/12,000; M = F; indolent, asymptomatic
 Higly malignant tumor arising within the bone and  Common in Europe, Australia/New Zealand
spread rapidly outwards to the periosteum and  <10% familial pattern (Autosomal Dominant
surrounding soft tissue pattern)
 Induced by viral infection? (paramyxovirus)
TYPES
 Primary (de novo)
 Secondary (Fibrous dysplasia, bone infarctions, Paget’s
disease)

SIGNS AND SYMPTOMS


 Pain, mass, night pains, (-) systemic symptoms

LABORATORY
 Elevated Alkaline phosphatase/ Lactic
dehydrogenase
 Used for prognostication
 3-5x increase indicates poor prognosis

LOCATION
 Commonly found in the KNEE (50%) OSA arising in Paget’s disease
 Distal femur (49%)
SUBTYPES OF OSTEOSARCOMA
 Proximal tibia (22%)
 Proximal humerus (11%)
CONVENTIONAL OSTEOSARCOMA
 Classic/ Intramedullary
NOTE:  High grade lesion
 Oscteosarcoma affects the METAPHYSEAL region  70% of malignant tumors
 M>F (3:2)
 2 decade (66%)
nd

Transcriber/s: Arianne Cortes & Nimrod Gabornes


Formatting: Paolo M. Ramirez D L S H S I M e d i c i n e B a t c h 2 0 1 6 | 3of 11
Editor/s: Chrismark Reyes & Jan Cynric Cacao
 Presentation
 Pain/Swelling (3-6 mos) over the knee (2/3)
[commonly affects the KNEE]
 Firm, immobile, warm mass, tenderness
 Neglected tumors have dilated veins, limitation
of motion
 Managed by SURGERY AND
CHEMOTHERAPY
(NEOADJUVANT CHEMO, THEN SURGERY, THEN ADJUVANT CHEMO)

*Telangiectatic variant
SURFACE OSTEOSARCOMA
 Low-intermediate grade surface OSA

PAROSTEAL
 Low grade surface OSA
 Arises from the OUTER LAYER OF THE
PERIOSTEUM or the periosseous tissues adjacent to
the cortex
 Rare (1.8% of malignant bone tumors)
OSTEABLASTIC VARIANT  F>M
 Intense osteoid matrix  Mass of posterior aspect of the KNEE
 Prominent sclerosis  Managed with SURGERY ONLY
 Matrix fill up the marrow cavity, entraps the pre-
existing bony trabeculation PERIOSTEAL
 Intermediate grade surface OSA
TELANGIECTATIC VARIANT  Arises from the INNER LAYER OF THE
 Purely lytic PERIOSTEUM
 “BAG OF BLOOD” TUMOR (PURELY BLOOD)  Rare (1.8% of malignant bone tumors)
 No flesh-like tumor tissue  M>F
 Septated spaces  Mass extend to DIAPHYSIS (remember that usually
 Minimal osteoid matrix osteosarcomas are located in the metaphysis)
 3.4% of all OSA  Managed with SURGERY & CHEMOTHERAPY
 Typically presents as a mass “sitting on the cortex”
 SCALLOPED CORTEX

CARTILAGE FORMING TUMORS


CHONDROSARCOMA PRESENTATION
 Cartilage-forming malignant cells  Typically, the bone lesion has already existed for a few
 UNRESPONSIVE TO CHEMOTHERAPY OR months to years and is still asymptomatic
RADIOTHERAPY (treatment is PURELY SURGICAL)  Pain few months to years; non-painful lesion
 4 -6 decades of life
th th
[INDOLENT COURSE]
 Men > women  Discovered incidental finding pathologic fracture
 Usually slow growing
LOCATION
TYPES  Pelvis, spine, shoulder/hip girdle (2/3) (typically
 Primary (de novo) involving the AXIAL bones)
 Secondary (from previous osteochondroma, or any
chondroma)

Apley’s System of Orthopedics and Fractures::


 Central chondrosarcoma – tumor develops in the
medullary cavity [either tubular or flat bones], most
commonly at the proximal end of the femur or in the
innominate bone of the pelvis
 Peripheral chondrosarcoma – usually arises in the
cartilage cap of an exostosis [osteochondroma] that has
been present since childhood

Transcriber/s: Arianne Cortes & Nimrod Gabornes


Formatting: Paolo M. Ramirez D L S H S I M e d i c i n e B a t c h 2 0 1 6 | 4of 11
Editor/s: Chrismark Reyes & Jan Cynric Cacao
MESENCHYMAL CHONDROSARCOMA PRESENTATION
 Small round cells  Slow growing, >5 years.
 3 -4 decade (2/3)  Osteolytic expansion, little chondroid matrix
rd th

 0.7% of malignant tumors  Typically occurs in the ENDS OF LONG BONES,


articular cartilage (EPIPHYSEAL) [like
PRESENTATION chondroblastoma]
 Pain/swelling (days-years)  Low-grade neoplasm; surgery alone would save the
 Lytic, STIPPLING CALCIFICATION, large soft tissue patient
mass
 VERY AGGRESSIVE type of tumor SECONDARY CHONDROSARCOMA FROM
 Managed by CHEMOTHERAPY OSTEOCHONDROMA
 Risk for malignant transformation:
CLEAR CELL CHONDROSARCOMA  Solitary Osteochondroma: 1-2%
 Rare, 1290 cases  Multiple/familial type: 5-25%
 M>F  Onset of 2° malignancy: 30 years after diagnosis
 4 -5 decade
th th
 SUDDEN INCREASE IN SIZE/PAIN

REMEMBER!
All that were discussed under chondrosarcoma are treated
by surgery EXCEPT for Mesenchymal chondrosarcoma

OTHER SKELETAL SARCOMAS


MALIGNANT FIBROUS HISTIOCYTOMA
 Fibroblastic/ myofibroblastic spindle/ pleomorphic
cells
 High grade neoplasm
 >50 y/o (50%), 1-5 % of sarcomas

PRESENTATION
 (-) mineralization/matrix [no matrix]
 Pain, swelling (chronic, indolent)
 Mild elevated alkaline phosphatase
 Management: CHEMOTHERAPY AND SURGERY

LOCATION
 DISTAL FEMUR/PROXIMAL TIBIA, PROXIMAL
HUMERUS [KNEE], PELVIS (METAPHYSIS,
METADIAPHYSIS)
EWING’S SARCOMA
FIBROSARCOMA  Small, round-cell tumors
 Malignant spindle-cell neoplasm with fibroblastic  1% childhood tumors
 2 to Osteosarcoma (3.56% malignant bone
nd
differentiation ND
 (+) collagen-rich matrix tumors)[2 MOST COMMON PRIMARY
 >50 y/o (50%0, <5% sarcomas MALIGNANT BONE TUMOR]
 10-20 y/0; M=F, rare in Asians/Blacks [usually
PRESENTATION Europeans]
 Fleshy/rubbery mass  Lytic/moth-eaten destruction with patch of
 (-) osteoid/mineralization[no matrix] sclerosis
 MOTH-EATEN, eccentric, pain, swelling, LOM,  Arises from endothelial cells in the bone marrow
pathologic fracture
LOCATION
LOCATION  Involves the DIAPHYSIS of long bones (femur, tibia,
 DISTAL FEMUR/PROXIMAL TIBIA humerus) and is commonly seen in FLAT BONES or
(METAPHYSIS) AXIAL BONES (vertebral body, ilium (pelvis), ribs,
clavicle)

Transcriber/s: Arianne Cortes & Nimrod Gabornes


Formatting: Paolo M. Ramirez D L S H S I M e d i c i n e B a t c h 2 0 1 6 | 5of 11
Editor/s: Chrismark Reyes & Jan Cynric Cacao
LABORATORY FINDINGS
 Leukocytosis/anemia, increased ESR/CRP (vs.
osteomyelitis since it is similar to an infection
due to cytokine activity)
 Elevated alkaline phosphatase

MALIGNANT GIANT CELL TUMOR


 History of PREVIOUS DIAGNOSED CASE OF GIANT
CELL TUMOR, HISTORY OF SEVERAL PRIOR
SURGERIES of the benign precursor, intralesional
Humeral Ewing’s Sarcoma
curettage followed by tumor recurrence

 [NIMMIE’S NOTES] a bone cement was placed in the site


of lesion removed, then a gap or cavity that was left will
be a new site of tumor growth, then a repeat intralesional
curettage with a bone graft or cement will be done again,
to which another cycle happens until the Giant Cell Tumor
that is repeatedly excised will be malignant

 1% of malignant tumor (rare)


 5.8% of giant cell tumors, male are more predisposed
than females to have this disease
 Common sites: KNEE (distal femur to proximal tibia)
 Occurs at an average of 12 years after diagnosis of
Hemipelvis, involved the entire iliac body and pubic bone Giant Cell Tumor
and can affect up to acetabular area  Rapidly growing sarcoma
 Lytic, articular surface involvement
 Managed by RESECTING THE ENTIRE AREA AND
PRESENTATION
REPLACING IT WITH A METALLIC PROSTHESIS
 Onion-peel, Codman’s triangle, “TRIMMED
WHISKER”/”BRUSH-LIKE” [Typical appearance
of disorganized periosteal reaction]

GROSS APPEARANCE
 Liquid consistency (pus-like)

HISTOLOGIC APPEARANCE
 Bluish color of the tumor
 Small round blue cells/oval nuclei

SIGNS & SYMPTOMS


 Tends to have systemic manifestations
 Back pain; fever, fatigue, MALAISE, weight loss
(due to cytokine activity) [can be misdiagnosed as
having osteomyelitis or infection of the bone
because when we do a biopsy, the tissue that is
harvested from the patient tends to be “milky” or
“pus-like”]

MANAGEMENT OF BONE SARCOMAS


 Critical steps in management of life-and-limb-  Multi-disciplinary approach: involves the
threatening disease pathologists, oncologists, radiologists,
 Any break in the protocol may cause the life-and-limb sometimes the radio-oncologists,
of the patient nutritionists, dieticians, and psychiatrists
 Pre-operative staging studies for patients who underwent amputations
 History and Physical Examinations  Well-planned biopsies
 Additional exams like MRI, CT-Scan,  Attainment of adequate surgical margins
Chest x-rays, blood exams, Chest CT-Scan  Some patients are required to do
 Patients also are usually referred to chemotherapy then a formal surgery
medical oncologists for adults and  Usual surgical management is wide
pediatric oncologists for children resection thus adequate surgical margins is
important

Transcriber/s: Arianne Cortes & Nimrod Gabornes


Formatting: Paolo M. Ramirez D L S H S I M e d i c i n e B a t c h 2 0 1 6 | 6of 11
Editor/s: Chrismark Reyes & Jan Cynric Cacao
HISTORICAL BACKGROUND: LIMB-SAVING SURGERY
 In the past (before 1970’s) Amputation is the treatment  Technically more complex than amputation
of choice for bone sarcomas (once diagnosed,  Composed of 2 procedures:
amputation is being considered immediately)  Tumor resection (surgical margins)
 However, 6 months later after amputation, local  Skeletal reconstruction
recurrence occurs at the site of the amputation
 Pulmonary metastases happens as well; if x-rays
will be requested, massive metastases are
now observed at the lungs with collapse of the
lung parenchyma
 Patient’s during those times have a 5-20% of 5
year survival rate
 During the 1970’s: Adjuvant chemotherapy was
introduced (CISPLATIN)
 Patients have a 60-70% chance of a 5 year
survival rate
 Limb saving surgeries (LSS) is viable option
because we can now SHRINK THE TUMOR
(due to chemotherapy) before doing the
surgical procedure
 During the 1980’s: MRI as part of the diagnostics LIMB-SAVING SURGERY (LSS) VERSUS
 Mapping of bone/soft tissue tumors AMPUTATION:
 Metallic implants are being produced  227 patients with osteosarcoma
 1981: 1 International Symposium on Limb Salvage
st
 Local recurrence is equivalent between amputation
(ISOLS) (8% recurrence) vs. limb salvage (10% recurrence)
 Use of prosthetic replacement for limb  Both present with 40-70% chance of 5 year survival
reconstruction rate
 Surgical Staging System was proposed  Better functional score for LSS
 Different Surgical margins were also proposed  Less expensive long term cost for LSS
 Drafting of the protocol for the management
of malignant bone tumors SURGICAL MARGINS

Old practice New practice


Biopsy without Evaluation of x-rays
imaging studies with or without
Multiple Incisions; further imaging
contamination studies
happens PROPER Biopsy
Wrong biopsy technique:
technique UTILITARIAN
Amputation with INCISION
questionable margins TECHNIQUE

DOC’S NOTES:
UTILITARIAN INCISION TECHNIQUE - standard
incision: usually after the biopsy, the same incisional margin
is being followed for future tumor resection procedures so
we do not contaminate the wound

NEOADJUVANT CHEMOTHERAPY
 Induction chemotherapy
WIDE RESECTION
 Patient receives 3 cycles
 Given preoperatively after establishing the  Remove 3-5 cm normal bone together with the
diagnosis after biopsy, but before tumor tumor
resection  IDEAL PROCEDURE FOR MALIGNANT BONE
 Decrease tumor size, tumor TUMORS RESECTION
consolidation/produce calcified rim in reactive  Desired margin FROM A BIOPSY TRACK (sarcoma
zone resection)
 Increases chances for Limb Saving Surgery  Tumor + pseudocapsule + cuff of normal tissue
 Target micrometastases (usually patient has removed en bloc in all direction’
micrometastases at initial check-up)  Cut through bone
 Evaluate tumor’s susceptibility to chemotherapy  Recommend in most low-and-high-grade bone/soft
(the higher the % of tumor necrosis the better tissue sarcomas with +/- adjuvant treatment
the prognosis)
 We can have time for planning, tumor prosthesis
fabrication/allograft procurement

Transcriber/s: Arianne Cortes & Nimrod Gabornes


Formatting: Paolo M. Ramirez D L S H S I M e d i c i n e B a t c h 2 0 1 6 | 7of 11
Editor/s: Chrismark Reyes & Jan Cynric Cacao
 TYPES:
 RESECTION WITHOUT RECONSTRUCTION
 Some are expendable bones (proximal
fibula, scapula)
 Resection without reconstruction
involving the pubis for
chondrosarcoma

 AMPUTATION
 Amputation of metatarsals for
chondosarcoma of 3rd metatarsal

 Resection without reconstruction of


the distal right femur and proximal
fibula

BONE RECONSTRUCTION
 Biological
 Use viable bone
 large segment of bones and/or tissues
stored in tissue or bone banks [in the
Philippines, we have one facility in PGH
and is cheaper than metallic implants]

 TOTAL SCAPULECTOMY
(intracompartment); [after the
procedure, the bicep and the tricep
tendons are attached to the clavicles.
The patient can move his shoulder a
little but could cannot perform a 180
degree shoulder abduction or flexion]

Transcriber/s: Arianne Cortes & Nimrod Gabornes


Formatting: Paolo M. Ramirez D L S H S I M e d i c i n e B a t c h 2 0 1 6 | 8of 11
Editor/s: Chrismark Reyes & Jan Cynric Cacao
 Allograft (other person ) vs. Autograft (self)
 Vascularized fibular graft/VFG- After  Stanmore Extendible Endoprosthesis – for
tumor resection, a bone and a vascular growing children, usually placed on the proximal
supply network is being harvested from the humerus for chondrosarcoma; magnetized,
same patient and attach to the site; the initially and gradually there’s destruction of the
grafted vascular network is being surrounding bone because of the telescoping
anastomosed with the remaining vascular effect so as the child grows, the prosthesis
network from the area where a resection expands/extends. It is very expensive, amounting
was done so as the patient *child* grows, to millions.
the graft also grows creating a “pseudo
bone/joint”. Usually, this procedure is being
done with a microvascular surgeon
 Non-vascularized fibular graft - takes
several years for a complete uptake of a
graft and may serve as a local bone already;
initially a metal implant may be placed as
guide for graft placement
 Intercalary: large segmental allograft -
for larger bones: grafts used in this
procedure are from bone banks in PGH for
Philippine setting. The allograft is first
bathed in Betadine then measurements are
done before actual resection and insertion of
the allograft. Inside the allograft there’s a RADICAL RESECTION
metal prosthesis that will hold the graft in  Entire anatomical compartment
place. Eventually, there will be an uptake of removed/extracompartment
the allograft and it will join the local bone as  Shoulder/hip disarticulation
a new part of it.  Forequarter amputation – shoulder + scapula +
 Intercalary: allograft + fibular graft part of the clavicle is removed
(autograft) - combination  Hindquarter amputation – part of thepelvis +
tumor is removed
 Endoprosthesis – for patients who can afford an  Recommended in:
implant itself  Recurrent sarcomas/failed Limb Saving
 Use of metallic implants Surgeries (LSS)
 Commercially available  Displaces pathologic fracture of bone sarcomas
 Stable, easy to match with bone defect  Sarcomas that cannot be adequately seen on
imaging studies/non-salvageable limb
[particularly those biopsies done by other
surgeons; the affected area of the concerned bone
is “contaminated” or the area involved presents
with recurrence, in which these would only
warrant radical resection straightaway

WIDE RESECTION RADICAL RESECTION


Removal of 3-5 cm normal Removal of entire
bone together with the anatomical compartment
tumor, and
pseudocapsule
Ideal surgical margin for Ideal for:
malignant bone tumor - Recurrent bone
sarcomas
 Tumor Endoprosthesis – bigger than any typical - Failed Limb Saving
prosthesis for osteoarthritic patients; longer stents Surgeries
and much larger. Sometimes the entire femur is - Displaced pathologic
removed and replaced with an entire prosthesis or fracture of the sarcoma
“mega” prosthesis - Sarcomas that cannot be
adequately imaged

Transcriber/s: Arianne Cortes & Nimrod Gabornes


Formatting: Paolo M. Ramirez D L S H S I M e d i c i n e B a t c h 2 0 1 6 | 9of 11
Editor/s: Chrismark Reyes & Jan Cynric Cacao
SUMMARY
 Benign vs. Malignant bone tumors: Signs and symptoms, classifications {osteoid, chondroid, (-) matrix, staging}
 Diagnostics: Radiographic characteristics and biopsy (closed vs. open)
 Surgical margins and bone reconstruction procedures

BIGGEST SUMMARY:
BENIGN MALIGNANT
BONE FORMING Osteoid Osteoma Osteosarcoma
Osteoblastoma
CARTILAGE FORMING Enchondroma Chondrosarcoma
Osteochindroma
Chondroblastoma
Chondromyxoid Fibroma
OTHER Giant Cell Tumor Malignant Giant Cell Tumor

-FIN-

TRANSCRIPTION DETAILS
Latest PPT, previous transcriptions;
BASIS Apley’s System of Orthopedics and RECORDINGS + NOTES + DEVIATIONS 8-10% CREDITS -
Fractures 9th ed.
References:
Images taken from the internet, Apley’s
REMARKS
TO JUNIOR INTERNSHIP AND BEYOND 
#Brorthopedics
-BATCH 2016 Transcribers’ Guild Transcriptions. Version 2.0.0.0.0 Build 3209-

PAST-E 2015

41. Local recurrences after several intralesional 45. A malignant lesion characterized as purely lytic
curettage of osteoclastoma (osteoclastoma = and with multiple septations and blood-filled:
giant cell tumor) involving the distal femur a. Classic osteosarcoma
might lead to what bone sarcoma? b. Parosteal osteosarcoma
a. Osteosarcoma c. Periosteal osteosarcoma
b. Chondrosarcoma d. Telangiectatic osteosarcoma
c. Malignant giant cell tumor
d. Ewing’s sarcoma 46. A type of osteosarcoma that is treated with just
surgery alone (wide resection):
42. The usual location of a chondrosarcoma lesion: a. Classic osteosarcoma
a. Small long bone of the hand b. Parosteal osteosarcoma
b. Flat and irregular axial bones c. Periosteal osteosarcoma
c. Metaphyseal region of the long bones d. Telangiectatic osteosarcoma
around the knee 47. Osteitis deformans of the pelvis might
d. Diaphyseal region of the long bones degenerate to what malignant bone lesion?
a. Osteosarcoma
43. A malignant lesion described as “scalloping of b. Ewing’s sarcoma
the cortex” in the diaphysis on radiographs: c. Chondrosarcoma
a. Chondrosarcoma d. Malignant giant cell tumor
b. Periosteal osteosarcoma
c. Ewing’s sarcoma 48. A patient with a pedunculated mass,
d. Fibrosarcoma asymptomatic for several decades that all of the
sudden started to complain of severe pain and
44. A malignant low grade lesion that is exclusively progressive swelling. Your differential
seen in the posterior aspect of the knee: diagnosis?
a. Classic osteosarcoma a. Osteosarcoma
b. Periosteal osteosarcoma b. Low grade chondrosarcoma
c. Parosteal osteosarcoma c. High grade chondrosarcoma
d. Telangiectatic osteosarcoma d. Malignant fibrous histiocytoma

Transcriber/s: Arianne Cortes & Nimrod Gabornes


Formatting: Paolo M. Ramirez D L S H S I M e d i c i n e B a t c h 2 0 1 6 | 10of 11
Editor/s: Chrismark Reyes & Jan Cynric Cacao
49. Malignant bone tumor characterized by 56. The abnormal periosteal reaction described as
fibroblastic spindle cell proliferation that lifting of periosteal new bone at the edge of the
arises after bone infarction of the long bone: tumor:
a. Osteosarcoma a. Codman’s triangle
b. Chondrosarcoma b. Sunburst appearance
c. Malignant fibrous histiocytoma c. Onion-peel appearance

50. Malignant bone tumor described on x‐ray as 57. A 68 year old male with progressively enlarging
lesion involving the metaphyseal region of the mass on the right thigh with no previous
long bone with permeative border and large trauma. Radiographs of the proximal femur
soft tissue mass but without a matrix. Your show a large lesion with deep endosteal
differential diagnosis? scalloping and containing chondroid matrix
a. Osteosarcoma (popcorn like appearance). Your management?
b. Mesenchymalchondrosarcoma a. Observation
c. Malignant fibrous histiocytoma b. Radiotherapy
d. Clear cell chondrosarcoma c. Surgery
d. Surgery and chemotherapy
51. Malignant bone tumor described on x-ray as a
large eccentric/moth-eaten mass with 58. A 40 year old female with a rapidly enlarging
permeative border involving the metaphyseal mass over the left leg. The x‐rays describe a
region of the long bone. There is a soft tissue large lesion with permeative border, stippling
mass but no matrix. Your differential calcification, and a large soft tissue mass.
diagnosis? Initial open biopsy showed multiple round cell
a. Osteosarcoma tumors. Your management?
b. Mesenchymal chondrosarcoma a. Observation
c. Malignant fibrous histiocytoma b. Radiotherapy
d. Fibrosarcoma c. Surgery
52. Malignant bone tumor described on x-ray as a d. Surgery and chemotherapy
large lesion with permeative border involving
the proximal femur. The matrix is characterized 59. Management for classic/intramedullary
as arc‐ring pattern with a large soft tissue osteosarcoma:
mass. Your differential diagnosis? a. Surgery, then followed by a
a. Osteosarcoma of the proximal femur combination of chemotherapy and
b. Chondrosarcoma of the proximal radiotherapy
femur b. Radiotherapy, then surgery, then
c. Fibrosarcoma of the proximal femur followed by radiotherapy
d. Ewing’s sarcoma of the proximal femur c. Neoadjuvant chemotherapy, then
surgery, then followed by adjuvant
53. Malignant bone tumor described on chemotherapy
radiographs as a lesion with permeative border d. Surgery, then followed by adjuvant
involving the meta‐diaphyseal region. The chemotherapy
abnormal periosteal reaction is characterized as
“trimmed whiskers”. Your differential 60. The knee is the most common location for what
diagnosis? malignant bone tumor?
a. Osteosarcoma a. Ewing’s sarcoma
b. Chondrosarcoma b. Chondrosarcoma
c. Ewing’s sarcoma c. Osteosarcoma
d. Fibrosarcoma

54. The periosteal reaction seen in malignant bone


tumors:
a. Cortical thickening
b. Codman’s triangle
c. Endosteal expansion
d. Neocorticalization

55. The abnormal periosteal reaction described a


horizontal layering of bone spicules:
a. Codman’s triangle
b. Sunburst appearance
c. Onion-peel appearance

Transcriber/s: Arianne Cortes & Nimrod Gabornes


Formatting: Paolo M. Ramirez D L S H S I M e d i c i n e B a t c h 2 0 1 6 | 11of 11
Editor/s: Chrismark Reyes & Jan Cynric Cacao

You might also like